Nachdem die Erläuterung des Zwillingsparadoxons in Teil 2 wegen seiner Länge und Unüberichtlichkeit (nicht zu unrecht) gerügt wurde, schiebe ich hier noch eine gekürzte, tabellarische Version nach, in der alle Berechnungen übersichtlich dargestellt sind und nur durch kurze Texte motiviert werden (mündchensmaß, wie man bei uns sagt). Das ging nicht ganz ohne Latex, was beispielsweise im Flipboard nicht angezeigt wird, wie ich bemerkt habe und deshalb in den ersten beiden Teilen vermieden hatte, also ggf. den Originalartikel hier auf Scienceblogs abrufen.

Zur Übersicht über die eingehenden Größen habe ich die Diagramme aus Teil 2 noch einmal mit eingefügt. Wem also Teil 2 zu anstrengend war, mag vielleicht hier noch einmal reinschauen. Das soll’s dann aber auch gewesen sein.

 

Sicht aus Bezugssystem A (Angela / Erde / Alpha Centauri)

Wir betrachten das ganze aus Sicht der ruhenden Erde. Auch Alpha Centauri (Entfernung d=4,3 Lichtjahre) wird als ruhend angenommen. Lichtlaufzeiten bei den einzelnen Beobachtungen ignorieren wir der Einfachheit halber, es sei angenommen, dass die beobachteten Ereignisse nachträglich aufgrund der bekannten Entfernungen zurückgerechnet worden seien.

Horst fliegt ab Richtung Alpha Centauri (roter Pfeil im Bild).

Sicht aus Bezugssystem A. Zeit läuft von unten nach oben, Entfernungen auf der waagerechten Achse. Blau: Entfernungen und Zeiten aus Sicht von A, rot : aus Sicht von H, Grün: aus Sicht von H'. Erklärung im Text. Bild: Autor.

Sicht aus Bezugssystem A. Zeit läuft von unten nach oben, Entfernungen auf der waagerechten Achse. Blau: Entfernungen und Zeiten aus Sicht von A, rot : aus Sicht von H, grün: aus Sicht von H’. Bild: Autor.

Horst fliegt hin:

(falls Tabelle nicht auf Handybildschirm passt, Bildschirm waagerecht kippen)

Größe Formel Berechnung Wert
Entfernung hin: dh=d 4,3 LJ
Geschwindigkeit H (Betrag): vH’ 0,8c
Lorentzfaktor von H’: \gamma (v_{H'})=1/\sqrt{1-\dfrac{v_{H'}^2}{c^2}} 1/\sqrt{1-0,8^2} 1,667=1/0,6
Flugzeit hin in A: th=dh/vH’ 4,3 LJ/0,8c 5,375 J
für H’ vergehen: t’h=th/γ(v) 5,375 J·0,6 3,225 J

 

Bei Alpha Centauri beamt Horst auf das entgegenkommende Raumschiff H” (grüner Pfeil) und kehrt damit zurück:

Größe Formel Berechnung Wert
Entfernung rück: dr=d 4,3 LJ
Geschwindigkeit H” (Betrag): vH” 0,8c
Lorentzfaktor von H”: \gamma (v_{H"})=1/\sqrt{1-\dfrac{v_{H"}^2}{c^2}} 1/\sqrt{1-0,8^2} 1,667=1/0,6
Flugzeit zurück in H: tr=dr/vH” 4,3 LJ/0,8c 5,375 J
für H” vergehen: t”r=tr/γ(vH”) 5,375 J·0,6 3,225 J

 

Hin- und Rückflugzeit Horst also in System A:

th+tr=5,375 J + 5,375 J = 10,75 J

Hinflugzeit Horst in System H’ und Rückflugzeit in H”:

t’h+t”r= 3,225 J + 3,225 J = 6,45 J.

 

Sicht aus Bezugssystem H’ (Horsts Hintransport)

Wir betrachten das Raumschiff H’, das Horst zu Alpha Centauri transportiert, als ruhend. Die Erde entfernt sich mit 0,8c und Alpha Centauri kommt mit 0,8c aus der Gegenrichtung, also erscheint ihr Abstand konstant und Lorentz-verkürzt:

Sicht aus dem Ruhesystem von H'. Erklärung siehe Text. Bild: Autor.

Sicht aus dem Ruhesystem von H’. Bild: Autor.

Hin: Warten, bis Alpha Centauri bei H’ ankommt (Rückflug-Schiff H” ebenso):

Größe Formel Berechnung Wert
Geschwindigkeit A (Betrag): v’A=vA 0,8c
Lorentzfaktor von A: \gamma (v_A)=1/\sqrt{1-\dfrac{v_A^2}{c^2}} 1/\sqrt{1-0,8^2} 1,667=1/0,6
Abstand Erde-Alpha Centauri: d’h=d/γ(v’A) 4,3 LJ·0,6 2,58 LJ
Flugzeit hin in H’: t’h=d’h/v’A 2,58 LJ/0,8c 3,225 J
für A vergehen: th=t’h/γ(v’A) 3,225 LJ·0,6 1,935 J

 

Zurück: Horst wechselt bei Ankunft Alpha Centauri auf das eintreffende Rückflug-Raumschiff H”. H” ist schneller als die Erde, die sich seit Beginn mit 0,8c entfernt hat. Aus Sicht von A (erstes Diagramm) kamen sich H’ und H” mit jeweils 0,8c entgegen, macht 1,6c Annäherungsgeschwindigkeit. Im System von H’ erscheint diese Geschwindigkeit gemäß Additionstheorem für relativistische Geschwindigkeiten kleiner als c (0,9756c).

Mit dieser Geschwindigkeit eilt H” auf dem Rückflug aus Sicht von H’ der Erde hinterher und braucht entsprechend lange, um deren Vorsprung aufzuholen (Vorsprung der Erde dividiert durch Geschwindigkeitsüberschuss von H”). Die Uhren laufen bei der hohen Geschwindigkeit von H” aber so langsam, dass für den Rückflug nur 3,225 Jahre Bordzeit vergehen:

Größe Formel Berechnung Wert
Geschwindigkeit H” (Betrag) v'_{H"} =  \dfrac {v_H'+v_{H"}}{1+\dfrac{v_H'\cdot v_{H"}}{c^2}} (0,8c+0,8c) / (1+0,82) 0,9756c
Lorentzfaktor von H”: \gamma (v'_{H"})=1/\sqrt{1-\dfrac{v_{H"}^2}{c^2}} 1/\sqrt{1-0,9756^2} 4,555=1/0,2195
Vorsprung Erde: d’r=t’h·v’A 3,225 J·0,8c 2,58 LJ
Aufholzeit =
Flugzeit zurück in H’:
t’r=d’r/(v’H”-v’A) 2,58 LJ / (0,9756c-0,8c) 14,692 J
für H” vergehen: t”r=t’r/γ(v’H”) 14,692 J·0,2195 3,225 J
für A vergehen: tr=t’r/γ(v’A) 14,692 J·0,6 8,815 J

 

Zeit für Erde/Angela also:

th+tr=1,935 J + 8,815 J = 10,75 J

Hinflugzeit Horst in System H’ und Rückflugzeit in H”:

t’h+t”r= 3,225 J + 3,225 J =6,45 J.

 

Sicht aus Bezugssystem H” (Horsts Rücktransport)

Wir betrachten das Raumschiff H”, das Horst von Alpha Centauri zurück zur Erde bringt, als ruhend. Die Erde und Alpha Centauri kommen mit 0,8c entgegen, also erscheint ihr Abstand genau wie in H’ Lorentz-verkürzt.

Sicht aus dem Ruhesystem von H". Erklärung siehe Text. Bild: Autor.

Sicht aus dem Ruhesystem von H”. Bild: Autor.

Hin: Alpha Centauri hat 2,58 LJ Vorsprung auf die Erde, den H’ von der Erde startend aufholen muss, was genau bei Ankunft am Ort von H” passiert. H’ ist relativ zu H” mit 0,9756c unterwegs. Die Aufholdauer entspricht der Distanz dividiert durch die Differenz der Geschwindigkeiten von H’ und Erde/Alpha Centauri.

Größe Formel Berechnung Wert
Geschwindigkeit A (Betrag): v”A=vA 0,8c
Lorentzfaktor von A: \gamma (v_A)=1/\sqrt{1-\dfrac{v_A^2}{c^2}} 1/\sqrt{1-0,8^2} 1,667=1/0,6
Geschwindigkeit H” (Betrag) v”H’ = v’H” 0,9756c
Lorentzfaktor von H”: γ(v”H’) = γ(v’H”) 4,555=1/0,2195
Vorsprung Alpha Centauri: d”h=d/γ(v”A) 4,3 LJ·0,6 2,58 LJ
Aufholzeit =
Flugzeit hin in H”:
t”h=d”h/(v”H’-v”A) 2,58 LJ/(0,9756c-0,8c) 14,692 J
für H’ vergehen: t’h=t”h/γ(v”H’) 14,692 J·0,2195 3,225 J
für A vergehen: th=t”h/γ(v”A) 14,692 J·0,6 8,815 J

 

Zurück: Horst beamt herüber und verbleibt in Ruhe wartend, bis die Erde vorbeikommt. Die nähert sich mit 0,8c und hat den längenkontrahierten Abstand von 2,58 LJ zu Alpha Centauri, der gerade vorbei flog.

Größe Formel Berechnung Wert
Abstand Erde: d”r=dr/γ(v”A) 4,3 LJ·0,6 2,58 LJ
Wartezeit auf Erde: t”r=d”r/v”A 2,58 LJ/0,8c 3,225 J
für A vergehen: tr=t”r/γ(v”A) 3,225 LJ·0,6 1,935 J

 

Zeit für Erde/Angela also:

th+tr= 8,815 J + 1,935 J= 10,75 J

Hinflugzeit Horst in System H’ und Rückflugzeit in H”:

t’h+t”r= 3,225 J + 3,225 J =6,45 J.

Es kommt also in allen Systemen dasselbe Ergebnis heraus. Weil Horst unterwegs das Bezugssystem wechselt und in jedem Bezugsystem nur den dort jeweils kürzeren Teil der Reise verbleibt, vergeht für ihn weniger Zeit als für Angela.

Die Systeme halten für sich betrachtet aber das Relativitätsprinzip ein: im als bewegt betrachteten System vergeht die Zeit langsamer und Strecken erscheinen verkürzt gegenüber dem als ruhend betrachteten. Das gilt für alle Kombinationen und funktioniert, weil die Zeitintervalle für Hin- und Rückflug in den Systemen ganz verschiedene Längen haben: Die Ereignisse, die in einem System auf der gleichen Waagerechten liegen (gleichzeitig sind), sind es in den anderen Systemen nicht. Und das gilt unabhängig von Lichtlaufzeiten, die hier gar nicht betrachtet wurden.

Kommentare (443)

  1. #1 Karl Mistelberger
    13. Juli 2018

    Aus https://arxiv.org/pdf/gr-qc/0504085.pdf

    The flaw in the “paradox” is the failure to comprehend what is meant by “A sees Earth’s clock run slow”. A cannot compare her clock with Earth’s clock because she is no where near Earth except at the start of the journey. Instead, an inertia l frame moving outbound with A’s velocity must be created, with a set of observers carrying clocks synchronized with hers. The readings on Earth’s clock can only be read by one of these observers who happens to be passing the Earth at that moment of time. But because of the relativity of simultaneity, the event in this outbound frame that is simultaneous with A ’s turnaround event P is not the 5-year mark on Earth, but is event X on Fig. 3, which is at Earth year 3003.2. So observers in A’s outbound frame do agree that Earth’s clock has run slow compared to hers, 3.2 years compared to 4 years. But while A decelerates and accelerates for the return journey, that outbound inertial frame continues flying off at 0.6c forever, and A must pick up a new inertial frame inbound at 0.6 c. In that frame, the event that is simultaneous with the turnaround is at event Y, Earth year 3006.8, 3.2 years before the return. Again, observers in the inbound inertial frame agree that Earth’s clock runs slow during the return journey, 3.2 years, compared to A’s 4 years. But the analysis using the two inertial frames has
    failed to account for the 3.6 years between events X and Y.

    Diese Darstellung ist sozusagen aufgeräumtest.

  2. #2 Funkionalistiker
    14. Juli 2018

    Mir fehlt als Abschluss die Darstellung auf den Zeitpfeil, einmal nach links gerichtet und einmal nach rechtsgerichtet.

  3. #3 Alderamin
    15. Juli 2018

    Verstehe nicht, was Du meinst – alle Zeitangaben sind doch in den Diagrammen eingetragen.

  4. #4 Stefan
    15. Juli 2018

    Was ich nicht verstehe ist, wie im anderen System rechnerisch die Zeit schneller ablaufen können soll als wie die Eigenzeit.
    Laut SRT kann die Zeit im anderen System nur langsamer werden, aber nie schneller als die Eigenzeit.
    Das drückt sich ja auch in der Gamma-Formel aus, deren Bereich nur gleichschnelle Zeit (wenn das andere System zu mir ruht) bis langsamere Zeit (wenn das andere System sich bewegt) umfasst.
    Wie also soll das gehen, dass die Zeit im anderen System schneller läuft? Und wie soll man diese angeblich schnellere Zeit mit der Gamma-Formel berechnen können?

  5. #5 Alderamin
    15. Juli 2018

    Tut sie ja nicht. Wie oben in der Berechnung zu sehen ist, vergeht die Zeit des bewegten Systems immer langsamer als die des ruhenden, egal welches System man als ruhend betrachtet. Ein Zeitabgleich kann aber nur am gleichen Ort erfolgen, und wenn keiner umkehrt, ist das höchstens einmal der Fall, so kann man keine Zeitabschnitte vergleichen. In den verschiedenen Bezugssystemen sind die Momente, wenn Horst von der Erde startet bzw. wenn er wieder zurückkehrt zu völlig unterschiedlichen Zeitpunkten, wenn man sich die Längen der einzelnen Abschnitte ansieht.

    Deswegen kann man eben nicht sagen “wenn ich den Zeitabschnitt x im ruhenden System betrachte und der im bewegten System langsamer vergeht, dann muss doch der selbe Abschnitt aus Sicht des bewegten Systems im ruhenden schneller vergehen”. Weil der Abschnitt in beiden Systemen nicht direkt vergleichbar ist. Sieh’ Dir die Bilder und die Zeiten auf der Zeitachse an.

  6. #6 Alderamin
    15. Juli 2018

    Beispiel: Zeit von Horsts Abflug bis Ankunft bei Alpha Centauri:

    A sagt: H’ braucht 5,375 Jahre bis Alpha Centauri, aber ich sehe bei H’ und H” jeweils nur 3,225 Jahre vergehen (1. Bild).

    H’ sagt: ich warte 3,225 Jahre bis Alpha Centauri nach Abflug der Erde vorbei kommt, aber ich sehe auf A nur 1,935 Jahre vergehen und bei H” nur 0,708 Jahre (2. Bild).

    H” sagt: es dauert 14,692 Jahre meiner Zeit, bis H’ Alpha Centauri eingeholt hat, aber ich sehe bei H’ nur 3,225 Jahre vergehen, und bei A nur 8,815 (3. Bild)

  7. #7 Funktionalistiker
    15. Juli 2018

    Mal mit drei Sätzen mehr:
    Die Darstellung sollte wie folgt ergänzt werden.
    Zeitpfeil nach rechts:
    Es sollten die „Daten von Braunschweig“ einfließen und –
    die Daten einer gleichen Uhr, die dem reisenden Zwilling mitgegeben wird.
    Zeitpfeil nach links:
    Die Multigraphen (analog Seismograph)zeichnen in geeigneter Weise alles in dem Zusammenhang relevante auf Papier auf. Aktivitäten des reisenden Zwillings und Daten der Atomuhren einschließlich des Taktes des Taktgebers und das Resultat des Zählwerkes.
    (Bewegungsrichtung des Papiers gleich Richtung des Zeitpfeils.)
    Interessant war für mich immer der Vergleich der von den Uhren angezeigten Daten, insbesondere die Jahreszahlen, auch Taktfrequenz und Zählwerksangabe!

  8. #8 Alderamin
    16. Juli 2018

    Bin mir immer noch nicht sicher, was Du da an Aktivitäten aufzeichnen willst – denk’ Dir einfach in den Diagrammen oben entsprechend der Beschriftungen mit Zeiten t entsprechend viele gleichmäßig voneinander entfernte Zeitmarken. Wenn etwa im ersten Bild am blauen Pfeil (auch Weltlinie genannt) 5,375 J steht, dann Striche, so dass die erste Hälfte des Pfeils (bis zur Ankunft von H’ bei Alpha Centauri) 5,375 Einheiten hat. Auf dem grünen Pfeil entsprechend 3,225 Einheiten. Alle Ereignisse sine entsprechend verlängert.

    Entsprechende Versuche hat es ja auch gegeben, z.B. Hafele & Keating. Ein anderer beobachteter Effekt ist die Lebensdauer von Myonen, die in der Stratosphäre beim Einschlag von Teilchen der kosmischen Strahlung entstehen und eigentlich nach ein paar hundert Metern schon zerfallen müssten. Aber wegen der Zeitdilatation schafft ein größerer Anteil von ihnen die Strecke bis zum Erdboden, siehe etwa diese Präsentation.

    Es ist vollkommen Wurst, welchen Prozess man betrachtet, durch die Zeitdilatation ist alles verlangsamt, Uhren, Pendel, Kernprozesse, chemische Reaktionen, biologische Vorgänge – einfach alles, was Zeit braucht. Deswegen verstimmt man auch die Zeitgeber in den Navigationssatelliten entsprechend (hier überwiegt aber der Effekt der Zeitdilatation auf der Erde im Schwerefeld, man muss beide Prozesse berücksichtigen, um den für die Höhe und Orbitalgeschwindigkeit richtigen Faktor zu wählen), damit sie synchron mit denen auf der Erde laufen.

  9. #9 Frank Wappler
    17. Juli 2018

    Alderamin schrieb (#8, 16. Juli 2018):
    > […] die Lebensdauer von Myonen, die in der Stratosphäre beim Einschlag von Teilchen der kosmischen Strahlung entstehen

    … also ungefähr die Lebensdauer von freien Myonen, d.h. exponentialverteilt, mit mittlerem Wert von ca. 2,2 μs …

    > und eigentlich nach ein paar hundert Metern schon zerfallen müssten.

    Von denjenigen Myonen, die sich bzgl. der Stratosphäre bzw. (einem Detektor auf) dem Erdboden mit Geschwindigkeit nahe Signalfrontgeschwindigkeit c bewegen, müssten eigentlich doch etliche am Detektor/Erdboden ankommen …

    > Aber wegen der Zeitdilatation schafft ein größerer Anteil von ihnen die Strecke bis zum Erdboden

    ???
    Wegen der Zeitdilatation, also der Möglichkeit des Vergleichs von Lebensdauern einzelner Myonen, obwohl sie nicht gegenüber einander ruhten, ist es überhaupt möglich, den Wert ihrer mittleren Lebensdauer zu ermitteln (und auch diesen wiederum Ensemble für Ensemble zu vergleichen).

    > Es ist vollkommen Wurst, welchen Prozess man betrachtet, durch die Zeitdilatation ist alles verlangsamt, Uhren, Pendel, Kernprozesse, chemische Reaktionen, biologische Vorgänge

    Die Zeitdilatation erlaubt überhaupt erst, Raten verschiedener Prozesse miteinander zu vergleichen, insbesondere sofern sie verschiedene Systeme betreffen, die nicht gegenüber einander ruhten.

    Dabei ist nicht etwa “alles verlangsamt“, also nicht etwa “jede Rate kleiner als jede (andere) Rate”; sondern es sind eigentliche (propere) vergleiche möglich: falls die beiden jeweiligen Raten nicht gleich sind, dann ist die eine kleiner als die andere, aber (deshalb auch) umgekehrt die letztere größer als die erstere.

    Deswegen lassen sich z.B. die Tickraten ausgewählter Uhren in GPS-Satelliten und auf dem Erdboden überhaupt miteinander vergleichen; vgl. https://en.wikipedia.org/wiki/Global_Positioning_System#Satellite_frequencies

    > Deswegen verstimmt man auch die Zeitgeber in den Navigationssatelliten entsprechend […], damit sie synchron mit denen auf der Erde laufen.

    Wäre dabei von gegenseitiger (properer) Synchronität die Rede, würden die betreffenden Uhren eher “eingestimmt” genannt werden, als “verstimmt“.

  10. #10 Alderamin
    17. Juli 2018

    Von denjenigen Myonen, die sich bzgl. der Stratosphäre bzw. (einem Detektor auf) dem Erdboden mit Geschwindigkeit nahe Signalfrontgeschwindigkeit c bewegen, müssten eigentlich doch etliche am Detektor/Erdboden ankommen …

    Mal unterstellt, die Halbwerts-Zerfallzeit seien 2,2 µs, dann kommen die Teilchen bei knapp c pro Halbwertszeit 660 m weit. Sie entstehen in ca. 15 km Höhe also rund 22,7 mal 660 m, macht einen verbleibenden Anteil von e-22,7 = 1,3*10-10. Es wäre also nicht ausgeschlossen, dass es auch ohne Zeitdilatation hin und wieder mal ein Teilchen bis auf den Boden schaffte, aber 99,99999999% würden es nicht schaffen. Dank Zeitdilatation schaffen es aber 8%-25%.

    Wegen der Zeitdilatation, also der Möglichkeit des Vergleichs von Lebensdauern einzelner Myonen, obwohl sie nicht gegenüber einander ruhten, ist es überhaupt möglich, den Wert ihrer mittleren Lebensdauer zu ermitteln (und auch diesen wiederum Ensemble für Ensemble zu vergleichen).

    Verstehe ich nicht. Ändert das etwas an dem Nachweis der Zeitdilatation durch die Häufigkeit der am Erdboden nachgewiesenen Myonen?

    Die Zeitdilatation erlaubt überhaupt erst, Raten verschiedener Prozesse miteinander zu vergleichen, insbesondere sofern sie verschiedene Systeme betreffen, die nicht gegenüber einander ruhten.

    Wieso brauche ich Zeitdilatation, um zwei beliebige Prozesse, etwa die Pulse eines Quarzzeitgebers und die Frequenz eines schwingenden Pendels zu vergleichen?

    Wäre dabei von gegenseitiger (properer) Synchronität die Rede, würden die betreffenden Uhren eher “eingestimmt” genannt werden, als “verstimmt“.

    Kannst Du nennen, wie Du magst – jedenfalls zählt eine Atomuhr pro Sekunde üblicherweise eine charakteristische Zahl von Schwingungen pro Sekunde, und diese Zahl muss für die Uhren auf den Satelliten gezielt modifiziert werden, für die auf der Erde verbleibenden nicht.

  11. #11 Leser
    runde Erde
    17. Juli 2018

    Der erste Teil hui, der zweite Teil ….. Sehr schade.

    Die Sicht, die uns als Sicht von Angela verkauft wird, ist ja nicht die Sicht von Angela. Sondern die Sicht eines Beobachters, senkrecht weit entfernt von der Sichtlinie Erde-Alpha Centauri. Angela selbst erlebt das auf Grund der Lichtlaufzeiten anders. Sie sieht Horst erst nach 5,375 Jahren + 4,3 Jahre = 9,675 Jahren auf Alpha Centauri ankommen. Da Horst ja sofort umkehrt und zurück reist fallen die Lichtlaufzeiten zwischen Horst und Angela bei der Ankunft von Horst bei Angela natürlich wieder heraus. Horst ist nach 2*5,375 Jahren = 10,75 Jahren wieder zurück bei Angela. Aus der Sicht von Angela dauert die Hinreise also 9,675 Jahre und die Rückreise nur
    1,075 Jahre. Das ist das, was Angela wirklich erlebt.

    Das ist wie mit der Marssonde : wenn wir das Signal erhalten, die Sonde ist gelandet, dann steht sie schon zwischen 5 und 20 Minuten auf dem Mars.

  12. #12 Alderamin
    17. Juli 2018

    Die Sicht, die uns als Sicht von Angela verkauft wird, ist ja nicht die Sicht von Angela. Sondern die Sicht eines Beobachters, senkrecht weit entfernt von der Sichtlinie Erde-Alpha Centauri. Angela selbst erlebt das auf Grund der Lichtlaufzeiten anders. Sie sieht Horst erst nach 5,375 Jahren + 4,3 Jahre = 9,675 Jahren auf Alpha Centauri ankommen.

    Ich hatte mehrfach erwähnt, dass Lichtlaufzeiten ausgeklammert werden, um es nicht unnötig kompliziert zu machen. Man kann ja die Laufzeiten, wenn man die Beobachtung macht, nachträglich rausrechnen. Es geht um die Zeitdilatation, nicht die Beobachtung selbst. Findet man in anderen Darstellungen zum Thema genau so.

    Der erste Teil hui, der zweite Teil ….. Sehr schade.

    Dafür gibt’s ja noch einen 2″. Teil. Wenn der dann auch schade sein sollte, tut’s mir leid.

    Ernsthaft, sämtliche Erklärungen mit 4D-Koordinaten oder Dopplereffekt oder was auch immer haben mir nie so eingeleuchtet wie ein Perspektivenwechsel und die Erkenntnis, wenn der Reisende aus Sicht des Systems, in dem er die Erde verlassen hat, umkehren will, dann muss er schneller und damit mit wesentlich mehr Zeitdilatation auf dem Rückweg unterwegs sein, um die Erde wieder einzuholen, so dass er deren aus seiner Sicht beim Hinflug langsamer vergangene Zeit am Ende locker unterbietet.

    Darum ging’s mir.

  13. #13 Leser
    runde Erde
    17. Juli 2018

    In dem 2″.Teil bin ich doch. Der “original” 2.Teil ist doch gar nicht mehr zugreifbar. Oder?

    Es ist ja richtig, daß man einen Sachverhalt so einfach wie möglich darstellt. Nur sollte man dann nicht mehr behaupten, das wäre das, was Angela erlebt. Eine Vereinfachung darf einen Vorgang nicht entstellen.

    Das ist vergleichbar mit der Frage, ob ein warmblütiges Tier im Vakuum verkocht. Wasser bei 37 Grad Celsius kocht im Vakuum. Die meisten Tiere haben aber einen Innendruck, z.B. den Blutdruck. Und es genügen kleine Drücke um den Siedepunkt über 37 Grad zu heben. Ich hoffe, niemand hat diesen Versuch real durchgeführt (Tierquälerei).

  14. #14 roel
    17. Juli 2018

    @Leser

    “In dem 2″.Teil bin ich doch” Ja, im aufgeräumten 2. Teil

    “Der “original” 2.Teil ist doch gar nicht mehr zugreifbar. Oder?” https://scienceblogs.de/alpha-cephei/2018/07/11/das-zwillingsparadoxon-minkowski-frei-teil-2/

  15. #15 Funktionalistiker
    17. Juli 2018

    @Alderamin nr. 8
    Das war jetzt aber ein Witz mit dem Pendel, gelle?
    Das Pendel geht bei mehr Gravitation schneller. Und die Sonnenuhr geht auch nicht langsamer.
    Und dann haben wir am Ende des Versuches mit den Zwillingen auf den Uhren zwei Jahreszahlen.
    Die eine Uhr zeigt beispielsweise 2100 an und die andere die dem reisenden Zwilling mitgegeben wurde, 2010. Und die stehen beide an gleicher Stelle des Zeitpfeils.
    Als Astronom würde mich das schon wegen der zu Grunde liegenden Kostellationen von Sternen und Planeten nachdenklich machen.
    Und das mit dem Multigraphen muss man schon mal begreifen.
    Die physische Existenz in der Bewegung (verkörpert durch den Zeiger und seine Bewegung) ist die Grundlage für die physikalische Bewegung in der Zeit (hier das auf dem Papier geschriebene).

    Das Problem für den Physiker: Er hat noch!!! keine andere Möglichkeit die Zahl 2010, die von der einen Uhr angezeigt wird, und die Deltas z. B. die 90 Jahre (2100 – 2010 = 90) zu berechnen.

  16. #16 Alderamin
    17. Juli 2018

    Wie roel schon sagt (und oben unter “Teil 2” auch verlinkt ist) ist der Artikel noch verfügbar und auch noch richtig, nur eben unübersichtlicher, weil Text und Zahlen vermengt sind.

    Kannst ja mal nach einer Darstellung des Themas suchen, wo die Lichtlaufzeiten mit eingerechnet werden. Macht eigentlich niemand.

    Ich habe aber den Text ein wenig ergänzt, damit ausdrücklich klar ist, dass Lichtlaufzeiten nicht betrachtet werden.

  17. #17 Alderamin
    17. Juli 2018

    Das war jetzt aber ein Witz mit dem Pendel, gelle?

    Nein, das hast Du nur missverstanden. Das Pendel ging an Wappler aufgrund der Bemerkung

    Die Zeitdilatation erlaubt überhaupt erst, Raten verschiedener Prozesse miteinander zu vergleichen

    die ich nicht verstanden habe. War nur ein Beispiel für einen anderen Prozess. Natürlich pendelt ein Pendel bei weniger als 1g langsamer und bei 0g gar nicht. Hat aber nichts mit der Zeitdilatation zu tun, sondern mit der geringeren Rückstellkraft. Egal wie schnell es pendelt, wieso braucht man die Zeitdilatation, um Frequenzen von Prozessen zu vergleichen?

    Und die Sonnenuhr geht auch nicht langsamer.

    Das liegt daran, dass die Sonne nicht Teil des bewegten Inertialsystems ist, sondern eine externe Referenz aus einem anderen Inertialsystem. Wenn man die Sonne mitnähme, ginge auch die Sonnenuhr langsamer.

    Die eine Uhr zeigt beispielsweise 2100 an und die andere die dem reisenden Zwilling mitgegeben wurde, 2010. Und die stehen beide an gleicher Stelle des Zeitpfeils.

    Ist halt so. Gilt für die Myonen genauso. Oder GPS-Satelliten. Die vorgerechneten Formeln lassen ja auch gar nichts anderes zu – wenn c=const.

    Und das mit dem Multigraphen muss man schon mal begreifen.

    Tut mir leid, dass ich Deine Beschreibung nicht verstanden habe. Pfeil nach links, Pfeil nach rechts, Seismograph…? Die Diagramme oben sind doch eindeutig. Oder?

    Das Problem für den Physiker: Er hat noch!!! keine andere Möglichkeit die Zahl 2010, die von der einen Uhr angezeigt wird, und die Deltas z. B. die 90 Jahre (2100 – 2010 = 90) zu berechnen.

    Er hat doch die Relativitätstheorie !?

  18. #18 Hugo
    18. Juli 2018

    Photonen fliegen bekanntlich mit Lichtgeschwindigkeit.
    Frage: altern Photonen?
    Können Photonen sich “gleichzeitig” an jedem Ort aufhalten?

  19. #19 Alderamin
    18. Juli 2018

    Frage: altern Photonen?

    Nein. Wir empfangen Photonen vom anderen Ende des Universums. Bis auf die Wellenlängenvergrößerung aufgrund der kosmischen Expansion haben sie sich nicht verändert. Dass die kosmische Expansion real ist und die Wellenlängenveränderung eben keine “Lichtermüdung” ist, kann man unter anderem damit belegen, dass alle Prozesse von einer Verlangsamung durch die Raumexpansion betroffen sind, etwa die Dauer von Supernova-Explosionen (Lichtkurve des radioaktiven Zerfalls der entstandenen Isotope in der Explosionswolke) und Gamma-Bursts (Kollaps von Hypernovae oder Verschmelzung von Neutronensternen). Wenn man zwei Pulse im expandierenden Universum nacheinander auf die Reise schickt, vergrößert sich der Raum zwischen ihnen durch die Expansion, und bei einem fernen Empfänger kommen sie mit mehr Zeitunterschied an, als mit dem sie beim Sender abgeschickt wurden. Deswegen werden alle zeitlichen Prozesse durch die Raumexpansion verlangsamt, inklusive der Frequenz von Licht.

    Können Photonen sich “gleichzeitig” an jedem Ort aufhalten?

    Meinst Du, über ihren Weg in die Länge gezogen? Nein. Es gibt zwar in der Quantenwelt Wahrscheinlichkeiten, wo man ein Teilchen finden kann und wo nicht, aber die Wahrscheinlichkeit, ein Teilchen sehr weit weg vom wahrscheinlichsten Pfad zu finden, wird schnell sehr klein. Ein Photon kann im Doppelspalt-Experiment zwei benachbarte Spalte gleichzeitig durchlaufen, aber je größer deren Abstand ist, desto schwieriger wird es (Beugungsfigur wird kleiner). Es ist weder über seinen ganzen Weg verschmiert, noch an jedem Ort auffindbar (das Beugungsbild eines Doppelspalts hat auch dunkle Streifen). Und wenn es dann registriert wird, dann immer nur an einem bestimmten Ort.

  20. #20 Frank Wappler
    https://pdglive.lbl.gov/DataBlock.action?node=S004T
    18. Juli 2018

    Alderamin schrieb (#12, 17. Juli 2018):
    > Mal unterstellt, die Halbwerts-Zerfallzeit [sei] 2,2 µs

    Vom Wert her bin ich mit dieser Annahme zum Zwecke dieser Diskussion ganz einverstanden (dass in der Praxis und wenn’s um hohe Genauigkeit geht u.a. zwischen μ- und μ+ unterschieden werden kann, brauchen wir hier wohl nicht auszuhecheln).

    Aber der Name der entsprechenden Messgröße ist richtiger Weise: Halbwerts-Lebensdauer;
    denn “Zeit” (im Sinne Einsteins) ist nicht zu verwechseln mit “Dauer” (im Sinne der CGPM).

    (Die PDG mogelt sich bekanntlich aus der Affäre, indem dort nur knapp und symbolisch von “Mean Life \tau“-Werten die Rede ist.)

    > dann kommen die Teilchen bei knapp c pro Halbwertszeit 660 m weit.

    Nein:
    eigentlich (um die Formulierung aus #8 aufzugreifen) kommen an solchen Teilchen
    pro 2,2 µs ihrer Lebensdauer dann Strecken von
    2,2\,{\rm \mu s} \times \frac{\beta}{\sqrt{1 - \beta^2}} \, c \simeq 660\,{\rm m} \times \frac{\beta}{\sqrt{1 - \beta^2}} Länge vorbei;
    also abhängig vom Wert \beta, der jeweils quantifiziert, wie “knapp [an] c” sich das betreffende Teilchen und die beiden Enden der betreffenden Strecke gegenüber einander bewegten.

    > […] Nachweis der Zeitdilatation durch die Häufigkeit der am Erdboden nachgewiesenen Myonen

    Es geht aber genau umgekehrt um den Nachweis der exponentiellen Verteilung der Lebensdauern der nachgewiesenen Myonen, und die Ermittlung des Mittelwertes dieser Lebensdauer-Verteilung, durch Anwendung des Theorems der RT, das als “Zeitdilatation” bekannt ist.

    > Wieso brauche ich Zeitdilatation, um zwei beliebige Prozesse, etwa die [Frequenz der] Pulse eines Quarzzeitgebers und die Frequenz eines schwingenden Pendels zu vergleichen?

    Wie denn sonst ?? — insbesondere, sofern die betreffenden “(Anzeigen-)Geber” voneinander getrennt waren bzw. nicht durchwegs koinzident blieben.

    > jedenfalls zählt eine Atomuhr pro Sekunde üblicherweise eine charakteristische Zahl von Schwingungen pro Sekunde

    Die charakteristische Zahl von Schwingungen, die z.B. im obigen Link zur SI-Einheit “Sekunde” angegeben ist, gilt (dort, nur) “bezogen auf 0 K, für ungestörte Cs133-Atome bzw. Atomuhren” (vgl. “Mises en pratique” der BIPM-Webseite).
    Ob und in wie fern eine gegebene Atomuhr gestört war, und wie ein “Bezug auf 0 K” Fall zu Fall herzustellen wäre, kann und muss aber Versuch für Versuch gemessen werden; und zwar unbefangen von irgendwelchen Erwartungen bzw. Modellen oder vorausgegangenen Ergebnissen (die manche “üblich” nennen mögen).

    > und diese Zahl muss für die Uhren auf den Satelliten gezielt modifiziert werden

    Einverstanden.
    Wenn z.B. von einem Instrumentalisten derartige Modifikationen während bzw. mit dem Ziel des sogenannten “Einstimmes des Orchesters” vorgenommen werden, sagt man dann üblicherweise, das Instrument würde dabei “gestimmt” oder “verstimmt” ?? …

    p.s.
    ScienceBlogs-Kommentar-HTML-Test:

    “μ<sup>+</sup>” wird dargestellt als: “μ+”.

  21. #21 Albrecht Storz
    18. Juli 2018

    “Ein Zeitabgleich kann aber nur am gleichen Ort erfolgen, und wenn keiner umkehrt, ist das höchstens einmal der Fall, so kann man keine Zeitabschnitte vergleichen.”

    Wie ist diese Behauptung begründet?
    Sogar bei den laufenden, erdgestützten Korrekturen der Uhren in den GPS-Satelliten wird die Signallaufzeit herausgerechnet.
    Und wenn man die Signallaufzeit heraus rechnen kann, kann man auch entfernte Uhren synchronisieren und auch Zeitabschnitte vergleichen. Wo bitte soll da ein Problem liegen?

    Folgendes würde mich auch interessieren:
    Wenn der Zeitverlauf in einem Raumschiff [R] anders sein solle wie zB auf der Startbasis [B], so kann auch keine Einigkeit über die Position (oder über die Geschwindigkeit) des Raumschiffes erlangt werden.
    Sendet zB das Raumschiff die Information: wir sind jetzt genau 2 Jahre [R-Zeit] lang geflogen und befinden uns genau 1 Lichtjahr [R-Entfernung] von der Basis entfernt, und die Basis diese Nachricht empfängt, welchen Wert würde die Basis für die Signallaufzeit dieser Nachricht annehmen müssen? Wann [B-Zeit] wäre die Nachricht und aus welcher Entfernung [B-Entfernung] wäre diese Nachricht gesendet worden?

  22. #22 Alderamin
    18. Juli 2018

    Wie ist diese Behauptung begründet?
    Sogar bei den laufenden, erdgestützten Korrekturen der Uhren in den GPS-Satelliten wird die Signallaufzeit herausgerechnet.

    Weil man sich nicht einigen kann, welche Ereignisse gleichzeitig sind, wenn man sich an verschiedenen Orten und mit relativistischer Geschwindigkeit zueinander bewegt. In den Beispielen hier ist ja bereits die Signallaufzeit herausgerechnet und trotzdem sind sich die Beobachter beispielsweise nicht darüber einig, wann die Diagramme oben ihren Anfangszeitpunkt haben. In jedem der drei Systeme ist die Waagerechte die instantane Jetztzeit. Im ersten Bild befindet sich H” bei Start von Horst aus seiner Sicht 2*2,58 = 5,16 LJ von der Erde entfernt und erreicht diese bei seiner Geschwindigkeit nach einer von ihm betrachteten Eigenzeit von 2*3,225 = 6,45 Jahren (grüne Schrift im Diagramm). Im letzten Bild sieht sich H” aber Start von Horst 14,33 LJ von der Erde entfernt und erreicht die Erde (bzw. diese ihn) nach 14,692 Jahren. Wenn man sich nicht darüber einigen kann, wann Horst losgeflogen ist und wo man zu dieser Zeit selbst war, hat man ein Problem, Zeitabschnitte zu vergleichen.

    Natürlich kann man mit der Relativitätstheorie ausrechnen, wie der andere seine Gleichzeitigkeit wahrnimmt. Es geht hier ja nur darum zu erklären, warum sich zwei bewegte Beobachter gegenseitig als verlangsamt wahrnehmen können und beide haben Recht. Eben weil sie die Zeit von verschiedenen Augenblicken an rechnen.

    Was GPS betrifft, da interessiert die Sicht des Satelliten auf uns überhaupt nicht, nur unsere auf den Satelliten, dessen Uhr so angepasst wird, dass sie synchron mit Uhren auf der Erde läuft. Und die Signallaufzeit wird benötigt, um aus der Differenz der aktuellen Zeit und der vom Satelliten gesendeten Zeit auf die Entfernung schließen zu können. Der Satellit hätte wohl andere Ansichten darüber, wann sein Signal beim Empfänger ankommt, wenn er das könnte, aber das braucht den Empfänger nicht zu interessieren. Und man kann die Uhren auf den Satelliten per Signal immer so einstellen, wie man es benötigt, man kann ja alle Effekte der RT ausrechnen und verwendet keine Messungen aus Sicht des Satelliten.

    Wenn der Zeitverlauf in einem Raumschiff [R] anders sein solle wie zB auf der Startbasis [B], so kann auch keine Einigkeit über die Position (oder über die Geschwindigkeit) des Raumschiffes erlangt werden.

    Stimmt. Wenn die Geschwindigkeit entsprechend hoch ist.

    Sendet zB das Raumschiff die Information: wir sind jetzt genau 2 Jahre [R-Zeit] lang geflogen und befinden uns genau 1 Lichtjahr [R-Entfernung] von der Basis entfernt, und die Basis diese Nachricht empfängt, welchen Wert würde die Basis für die Signallaufzeit dieser Nachricht annehmen müssen? Wann [B-Zeit] wäre die Nachricht und aus welcher Entfernung [B-Entfernung] wäre diese Nachricht gesendet worden?

    Aus Sicht des Raumschiffs hat man sich also mit 2 LJ/1 J = 0,5c entfernt, das werden beide Seiten so sehen, in der Relativgeschwindigkeit besteht Einigkeit. Auf der Erde weiß man, dass die Zeit auf dem Raumschiff um √(1-0,5²) langsamer verlaufen ist, also kann die Basis ausrechnen, dass nach Erdzeit 1 Jahr[R]/√(1-0,5²) = 1,1547 Jahre [B] vergangen sein müssen, als das Signal auf den Weg ging. Bei 0,5c wäre das Raumschiff also 1,1547/2 = 0,57735 Lichtjahre weit gekommen und so viele Jahre wäre dann die Lichtlaufzeit. Das Signal müsste folglich zum Zeitpunkt 1,1547 + 0,57735 = 1,732 Jahren [B] nach dem Start auf der Erde eingetroffen sein.

  23. #23 Daniel Rehbein
    Dortmund
    18. Juli 2018

    Es gibt zum Zwillingsparadoxon einen Punkt, der mich schon seit längerem umtreibt, und den ich bisher noch nirgendwo erklärt gefunden habe: Was ist, wenn Horst gar nicht umkehren muß, sondern wenn er nach langer Reise wieder am Ausgangspunkt ankommt, weil ein Pfad in der Raumzeit ihn dorthin führt?

    Wenn wir von der gängigen Theorie ausgehen, daß das Universum in einer Sigularität entstanden ist und sich dann ausgedehnt hat, dann ist es plausibel, daß das Universum (das gesamte Universum, nicht nur das sichtbare Universum) keinen Rand hat, aber doch nur endlich groß ist. Im einfachsten Fall wäre die Topologie des Universums eine Hyperkugel, es könnte aber auch eine kompliziertere Form sein.

    Wenn es nur endlich viele Raumpunkte gibt, dann muß bei jeder Reise irgendwann derselbe Raumpunkt ein zweites Mal passiert werden.

    Man könnte also Angela und Horst so platzieren, daß Horst nach endlicher (sehr langer) Zeit wieder bei Angela ankommt – und zwar, ohne dabei umzukehren, also insbesondere ohne Wechsel des Bezugssystems. Wie löst man das Zwillingsparadoxon dann?

  24. #24 Alderamin
    18. Juli 2018

    @Daniel Rehbein

    Siehe hier und den Link dort.

  25. #25 Daniel Rehbein
    Dortmund
    19. Juli 2018

    Mit den verschiedenen Artikel und den Kommentaren dazu bin ich wohl etwas durcheinandergeraten. Da gibt es “Teil 1”, “Teil 2” und “Teil 2, aufgeräumt”. Nun habe ich die verschiedenen Kommentare durchgearbeitet, aber es doch nicht so recht verstanden.

    Du schreibst in Kommentar #53 zu “Teil 1” den Satz “Die Situation ist nicht symmetrisch”. In dem verlinkten Paper steht unten auf Seite 587 dagegen die Aussage “The statement of the closed universe twin paradox is completely symmetrical. Albert and Betty travel in opposite directions from one another until they reunit.”

    Wir haben im Universum kein definiertes Inertialsystem. Daher lässt sich nicht entscheiden, wer von beiden stillsteht und wer das ganze Universum umrundet. Da ist doch auch nicht so etwas wie eine “Naht” oder eine Art “Nullmeridian” im Raum, der Raum ist für beide Beteiligten ein randloses Kontinuum.

    Zwei Ameisen auf der Oberfläche eines Luftballons können ja auch nicht entscheiden, wer von beiden stillgestanden hat und wer den Ballon umkreist hat, oder ob womöglich beide sich in gegensätzliche Richtungen bewegt haben.

    Aus der Erklärung unter der Zeichnung auf Seite 587 in dem verlinkten Paper lese ich heraus, daß beide Beteiligten jeweils auf einen Partner treffen, der weniger gealtert ist, also das Resultat tatsächlich symmetrisch ist. Laut dem letzten Absatz auf der Seite ist lediglich die Erklärung nicht symmetrisch, nämlich abhängig davon, welche der beiden Personen ich im Diagramm als ruhend einzeichne.

    Aber dann kommt der Teil der Erklärung, den ich nicht verstehe: Von der Person, die für das Diagramm als ruhend definiert wird, kommt ein zweites Exemplar (eine Kopie) ins Spiel. Wo kommt diese Kopie her?

  26. #26 Daniel Rehbein
    Dortmund
    19. Juli 2018

    Es lässt mich nicht schlafen 🙂

    Ich habe mir das mal anhand von Zahlen überlegt. Bleiben wir dabei, daß Horst mit einer Geschwindigkeit von 0,8c unterwegs ist, dann ergibt sich wieder der einfach so handhabende Lorentz-Faktor von 1/0,6.

    Wenn wir nun der Einfachheit halber mal annehmen, daß das zyklische Universum eine Ausdehnung von gerade einmal 8 Lichtjahren habe (definieren wir uns also ein sehr kleines Universum), dann ist Horst genau 10 Jahre unterwegs bis er wieder bei Angela ankommt. Für ihn sind dabei in seinem Raumschiff genau 6 Jahre vergangen (aufgrund des gut handhabbaren Lorentz-Faktors von 1/0,6).

    Nun kann Angela während des Fluges durch ein Fernrohr sehen und Horst beobachten. Da das Universum zyklisch ist, sieht sie ihn auf der einen Seite von sich wegfliegen und von der anderen Seite auf sich zukommen, letzteres allerdings erst nach 8 Jahren (aufgrund der angenommenen Ausdehnung des Universums von 8 Lichtjahren). 8 Jahre, nachdem Angela sich von Horst verabschiedet hat, schaut sie durch ihr Fernrohr und sieht, wie sie sich gerade von Horst verabschiedet.

    Dieser Horst, den sie da nach 8 Jahren durch das Fernglas sieht, ist zu dem Zeitpunkt noch gar nicht gealtert (sie sieht ja den Abflug). 2 Jahre später (dann sind die 10 Jahre Reisezeit vergangen) ist er dann aber plötzlich bei ihr, und für ihn sind dann die vorhin berechneten 6 Jahre vergangen. Der Horst, der auf Angela zukommt, wäre also in 2 Jahren um 6 Jahre gealtert. Was bedeutet das nun?

    In meinen Überlegungen habe ich dadurch allerdings eine Kopie von Horst definiert (nämlich den Horst, den Angela durch das Fernglas auf sich zukommen sieht, während sie in der anderen Richtung den Original-Horst immer noch von sich wegfliegen sieht). In dem Paper auf Seite 587 wird dagegen eine Kopie von der Person definiert, die als ruhend angenommen wird. Ich bin also noch nicht ganz konform zu dem Paper.

    P.S.: Um dieses Paper geht es:
    https://www.math.uic.edu/undergraduate/mathclub/talks/Weeks_AMM2001.pdf

  27. #27 Frank Wappler
    19. Juli 2018

    Daniel Rehbein schrieb (#23, 18. Juli 2018):
    > […] Hyperkugel […] Man könnte also Angela und Horst so platzieren, daß Horst nach endlicher (sehr langer) Zeit wieder bei Angela ankommt – und zwar, ohne dabei umzukehren, also insbesondere ohne Wechsel des Bezugssystems.

    Insbesondere so, dass sowohl Horst durchwegs Mitglied eines bestimmten Inertialsystems war und blieb, und auch Angela durchwegs Mitglied eines bestimmten Inertialsystems war und blieb.

    Das vorgeschlagene Hyperkugel-Universum ist demnach flach (ähnlich einem zylindrisch gewickelten Blatt Papier); zumindest hinsichtlich der Pfade von Horst bzw. von Angela, die sich beide zusammen als “entlang einer bestimmten räumlichen Dimension” des Hyperkugel-Universums darstellen lassen.

    Alle Paare von Ereignissen, die zum entsprechenden Raumzeit-Anteil des gesamten Hyperkugel-Universums gehören (oder der Einfachheit halber gerne: insgesamt alle Paare von Ereignissen des insgesamt flachen Hyperkugel-Universums) sind deshalb durch einen Raumzeit-Interval-Wert s^2 charakterisiert, das als verallgemeinertes Maß des (Quadrates des) Abstands der beiden Ereignisse voneinander aufgefasst werden kann.

    Dass Angela durchwegs Mitglied eines bestimmten Inertialsystems (oder in anderen Worten: “unbeschleunigt”) gewesen sein soll, lässt sich anhand von (Verhältnissen von) Intervall-Werten übrigens folgendermaßen ausdrücken:

    Für je drei verschiedene Ereignisse \varepsilon_{AJ}, \varepsilon_{AK}, \varepsilon_{AN} an denen Angela (kurz A) teilnahm (und die sich insbesondere dadurch unterscheiden, dass A dabei jeweils von J, K oder von N getroffen und passiert wurde), soll gelten, dass diese drei gegenüber einander gerade waren, d.h.:

    \left( \frac{s^2[ \, \varepsilon_{AJ}, \varepsilon_{AK} \, ]}{s^2[ \, \varepsilon_{AJ}, \varepsilon_{AN} \, ]} \right)^2 + \left( \frac{s^2[ \, \varepsilon_{AK}, \varepsilon_{AN} \, ]}{s^2[ \, \varepsilon_{AJ}, \varepsilon_{AN} \, ]} \right)^2 + 1 =
    2 \, \left( \frac{s^2[ \, \varepsilon_{AJ}, \varepsilon_{AK} \, ]}{s^2[ \, \varepsilon_{AJ}, \varepsilon_{AN} \, ]} \right) + 2 \, \left( \frac{s^2[ \, \varepsilon_{AK}, \varepsilon_{AN} \, ]}{s^2[ \, \varepsilon_{AJ}, \varepsilon_{AN} \, ]} \right) + 2 \, \left( \frac{s^2[ \, \varepsilon_{AJ}, \varepsilon_{AK} \, ]}{s^2[ \, \varepsilon_{AJ}, \varepsilon_{AN} \, ]} \right) \, \left( \frac{s^2[ \, \varepsilon_{AK}, \varepsilon_{AN} \, ]}{s^2[ \, \varepsilon_{AJ}, \varepsilon_{AN} \, ]} \right) .

    Und entsprechend ist auszudrücken, dass Horst durchwegs Mitglied eines bestimmten Inertialsystems (d.h. “unbeschleunigt”) gewesen sein soll:

    Für je drei verschiedene Ereignisse \varepsilon_{HP}, \varepsilon_{HQ}, \varepsilon_{HW} an denen Horst (kurz H) teilnahm (und die sich insbesondere dadurch unterscheiden, dass H dabei jeweils von P, Q oder von W getroffen und passiert wurde), soll gelten:

    \left( \frac{s^2[ \, \varepsilon_{HP}, \varepsilon_{HQ} \, ]}{s^2[ \, \varepsilon_{HP}, \varepsilon_{HW} \, ]} \right)^2 + \left( \frac{s^2[ \, \varepsilon_{HQ}, \varepsilon_{HW} \, ]}{s^2[ \, \varepsilon_{HP}, \varepsilon_{HW} \, ]} \right)^2 + 1 =
    2 \, \left( \frac{s^2[ \, \varepsilon_{HP}, \varepsilon_{HQ} \, ]}{s^2[ \, \varepsilon_{HP}, \varepsilon_{HW} \, ]} \right) + 2 \, \left( \frac{s^2[ \, \varepsilon_{HQ}, \varepsilon_{HW} \, ]}{s^2[ \, \varepsilon_{HP}, \varepsilon_{HW} \, ]} \right) + 2 \, \left( \frac{s^2[ \, \varepsilon_{HP}, \varepsilon_{HQ} \, ]}{s^2[ \, \varepsilon_{HP}, \varepsilon_{HW} \, ]} \right) \, \left( \frac{s^2[ \, \varepsilon_{HQ}, \varepsilon_{HW} \, ]}{s^2[ \, \varepsilon_{HP}, \varepsilon_{HW} \, ]} \right) .

    Außerdem lassen sich natürlich auch Verhältnisse wie z.B. \left( \frac{s^2[ \, \varepsilon_{HP}, \varepsilon_{HQ} \, ]}{s^2[ \, \varepsilon_{AJ}, \varepsilon_{AK} \, ]} \right) auswerten,
    also das Verhältnis eines (bzw. jedes) Intervalls zwischen zwei Ereignissen, an denen Angela beteiligt war und eines (bzw. jedes) Intervalls zwischen zwei Ereignissen, an denen Horst beteiligt war.

    Falls wir nun insbesondere aufeinanderfolgende Treffen/Passagen von Angela und Horst betrachten wollen, dann geht es dabei um Ereignisse, an denen jeweils sowohl Angela als auch Horst beteiligt waren (und die sich insbesondere dadurch unterscheiden, dass jeweils ausschließlich noch bestimmte weitere Beteiligte involviert waren); also z.B. ein Ereignis $\varepsilon_{AFHR} \equiv \varepsilon_{AFR} \equiv \varepsilon_{HFR}$, an dem sich A, H, F und R trafen und passierten, und Ereignis $\varepsilon_{AGHS} \equiv \varepsilon_{AGS} \equiv \varepsilon_{HGS}$, an dem sich A, H, G und S trafen und passierten (aber, zur Unterscheidung, ausdrücklich weder F noch R).

    Das Verhältnis von Horsts Dauer seit dem Verlassen Angelas bis zur (nächstfolgenden) Wiederbegenung mit Angela, \tau H_{(AFR \rightarrow AGS)} und Angelas Dauer seit dem Verlassen Horsts bis zur (nächstfolgenden) Wiederbegenung mit Horst, \tau A_{(HFR \rightarrow HGS)} ergibt sich als

    \left( \frac{\tau H_{(AFR \rightarrow AGS)}}{\tau A_{(HFR \rightarrow HGS)}} \right) := \sqrt{\left( \frac{s^2[ \, \varepsilon_{HP}, \varepsilon_{HQ} \, ]}{s^2[ \, \varepsilon_{AJ}, \varepsilon_{AK} \, ]} \right)}.

    Daniel Rehbein schrieb (#25, 19. Juli 2018):
    > […] lässt sich nicht entscheiden, wer von beiden stillsteht und wer das ganze Universum umrundet.

    Es lässt sich aber anhand der gegebenen Werte der (Verhältnisse der) Raumzeit-Intervalle s^2 sehr wohl entscheiden, ob das gerade gezeigte Verhältnis der “Runden”-Dauer von Horst und der “Runden”-Dauer von Angela größer, kleiner, oder gleich 1 war.
    Und falls noch weitere Beteiligte in Betracht gezogen werden, die ebenfalls jeweils ebenfalls durchwegs unbeschleunigt blieben, und die Angela und (dabei auch stets) Horst ebenfalls wiederholt (und regelmäßig) trafen/passierten, dann lässt sich auch auswerten, ob und wer von denen eventuell sogar noch längere “Runden”-Dauer aufwies als Angela oder als Horst; oder natürlich auch ob und wer von denen eventuell sogar noch kürzere “Runden”-Dauer aufwies als Angela oder als Horst.
    Wer die maximale “Runden”-Dauer aufwies ist dadurch gegenüber allen anderen ausgezeichnet.

    Daniel Rehbein schrieb (#26, 19. Juli 2018):
    > […] Bleiben wir dabei, daß Horst mit einer Geschwindigkeit von 0,8c unterwegs ist

    Wie sich die Werte des Geschwindigkeits-Parameters \beta (von Horst bzgl. Angela und allen, die zusammen mit Angela zum selben Inertialsystem gehörten, und gleichermaßen von Angela bzgl. Horst und allen, die zusammen mit Horst zum selben Inertialsystem gehörten) aus (Verhältnissen von) Raumzeit-Intervallen ermitteln lassen, kann ich kurzfristig nachreichen. (Wie sich Verhältnissen von Raumzeit-Intervallen zunächst einmal an sich überhaupt ermitteln lassen, bzw. wie sich zunächst überhaupt ermittlen ließe, ob eine in Betracht stehende Region flach wäre, oder in wie fern nicht, ist ein größeres Projekt. … &)

    Mit dem (Mess-)Wert \beta_A[ \, H \, ] \equiv \beta_H[ \, A \, ] \equiv \beta = 0,8 ergibt sich dann natürlich

    \left( \frac{\tau H_{(AFR \rightarrow AGS)}}{\tau A_{(HFR \rightarrow HGS)}} \right) = \sqrt{1 - \beta^2} = 0,6.

    > Wenn wir nun der Einfachheit halber mal annehmen, daß das zyklische Universum eine Ausdehnung von gerade einmal 8 Lichtjahren habe (definieren wir uns also ein sehr kleines Universum)

    Nehmen wir der Einfachheit halber auch an, dass Angela (sowie alle anderen Mitglieder des selben Inertialsystems, zu dem auch Angela gehört) durch maximale “Runden”-Dauer gegenüber all jenen auszeichnet ist, die sie (regelmäßig) trafen/passierten. Dann bedeutet “zyklisches Universum mit Ausdehnung von gerade einmal 8 Lichtjahren” insbesondere, dass es einen Beteiligten B gibt (der bzgl. A durchwegs ruhte, also zum selben Inertialsystem gehörte), bzgl. dem A durchwegs konstante Ping-Dauer von 8 Jahren feststellte. (A und B heißen demnach “zueinander antipodisch”.)

    > dann ist Horst genau 10 Jahre unterwegs bis er wieder bei Angela ankommt.

    Nein:
    Angelas Runden-Dauer (bzgl. Horst) betrug demnach \tau A_{(HFR \rightarrow HGS)} = (8 \, c \, {\rm {yr}}) / (0,8 \, c) = 10 \,{\rm {yr}}.

    Horsts Runden-Dauer (bzgl. Angela) betrug stattdessen \tau H_{(AFR \rightarrow AGS)} = 0,6 \, \tau A_{(HFR \rightarrow HGS)} = 6 \,{\rm {yr}}.

    > Für ihn sind dabei in seinem Raumschiff genau 6 Jahre vergangen (aufgrund des gut handhabbaren Lorentz-Faktors von 1/0,6).

    Eben.

    > Nun kann Angela während des Fluges durch ein Fernrohr sehen und Horst beobachten. Da das Universum zyklisch ist, sieht sie ihn auf der einen Seite von sich wegfliegen und von der anderen Seite auf sich zukommen, letzteres allerdings erst nach 8 Jahren (aufgrund der angenommenen Ausdehnung des Universums von 8 Lichtjahren). 8 Jahre, nachdem Angela sich von Horst verabschiedet hat, schaut sie durch ihr Fernrohr und sieht, wie sie sich gerade von Horst verabschiedet.

    Einverstanden.

    > Dieser Horst, den sie da nach 8 Jahren durch das Fernglas sieht, ist zu dem Zeitpunkt noch gar nicht gealtert (sie sieht ja den Abflug). 2 Jahre später (dann sind die 10 Jahre Reisezeit vergangen) ist er dann aber plötzlich bei ihr, und für ihn sind dann die vorhin berechneten 6 Jahre vergangen.

    Richtig; immer vorausgesetzt natürlich, dass die beiden “ein Jahr” (bzw. 1 \,{\rm {yr}}) gleiche Dauern meinen.

    > Der Horst, der auf Angela zukommt, wäre also in 2 Jahren um 6 Jahre gealtert.

    Erstens ist Vergleich bzw. Messung von “biologisch/chemisch/dynamischer Alterung” ein separates Thema;
    bleiben wir (der Einfachheit halber) bei Vergleichen von geometrisch-kinematischen Dauern.

    Zweitens wurde Horst drei Jahre nach Verlassen von Angela von deren Antipoden B getroffen/passiert.
    Und dazu gehört übrigens auch, dass Bs Dauer von seiner (d.h. Bs) Anzeige gleichzeitig zu Angelas Anzeige des Verlassens von Horst bis zu seiner (d.h. Bs) Anzeige des Treffens/passierens von Horst fünf Jahre war.

    Merke: von vorrangiger Bedeutung zur Feststellung von (Verhältnissen von) Dauern ist: wer wen traf/passierte.
    Oder wie Einstein formulierte:

    Alle unsere zeiträumlichen Konstatierungen laufen stets auf die Bestimmung zeiträumlicher Koinzidenzen hinaus.

  28. #28 Frank Wappler
    https://scienceblogs.de/alpha-cephei/2018/07/13/das-zwillingsparadoxon-minkowskifrei-teil-2-aufgeraeumt/#comment-2102
    19. Juli 2018

    p.s. — Formatkorrektur: …

    Falls wir nun insbesondere aufeinanderfolgende Treffen/Passagen von Angela und Horst betrachten wollen, dann geht es dabei um Ereignisse, an denen jeweils sowohl Angela als auch Horst beteiligt waren (und die sich insbesondere dadurch unterscheiden, dass jeweils ausschließlich noch bestimmte weitere Beteiligte involviert waren); also z.B. ein Ereignis \varepsilon_{AFHR} \equiv \varepsilon_{AFR} \equiv \varepsilon_{HFR}, an dem sich A, H, F und R trafen und passierten, und Ereignis \varepsilon_{AGHS} \equiv \varepsilon_{AGS} \equiv \varepsilon_{HGS}, an dem sich A, H, G und S trafen und passierten (aber, zur Unterscheidung, ausdrücklich weder F noch R).

  29. #29 Alderamin
    19. Juli 2018

    @Daniel Rehbein

    Ich verlinke die Arbeit nochmal hier, falls jemand reinschauen möchte.

    Aber dann kommt der Teil der Erklärung, den ich nicht verstehe: Von der Person, die für das Diagramm als ruhend definiert wird, kommt ein zweites Exemplar (eine Kopie) ins Spiel. Wo kommt diese Kopie her?

    Die Kopie ist der gleiche Albert zu einem anderen Zeitpunkt. Wenn man das Universum aufschneidet und flachrollt, dann fliegt Betty von einer Seite zur anderen und findet sowohl hinter sich als auch voraus einen Albert, der ruht. Die Linien der Gleichzeitigkeit sind bei ihr schräg mit Steigung 5/3, wie im ersten Teil für das Standard-Zwillingsparadox erklärt wird, d.h. alles, was auf dieser Linie ist, erscheint ihr (nach Abzug von Lichtlaufzeiten) gleichzeitig. Ihr erscheinen also mehrere Alberts kontemporär, der vom Abflug hinter ihr, der mit 4/5 ihrer Zeitrate altert und bei Abflug gleich alt war, sowie der von ihrer Ankunft vor ihr, der auch mit 4/5 ihrer Zeitrate altert, aber um 10 Jahre älter ist als sie, wenn sie ankommt; und wenn sie weiter in die Ferne schaut, sieht sie noch mehr Alberts zu verschiedenen Zeiten, weil ihre Gleichzeitigkeitslinie eine Schraubenlinie ist, die sich um die Raumzeit herumwindet. Unabhängig von Lichtlaufzeiten sieht sie also unzählige Kopien von Albert, immer den Albert, wo ihre Gleichzeitigkeitslinie die Weltlinie von Albert schneidet (eine senkrechte Linie entlang der Höhe des Zylinders). Sowohl voraus, in der Zukunft liegende Kopien, als auch hinter ihr, in der Vergangenheit liegende. Es ist aber immer derselbe Albert, nur zu verschiedenen Zeiten, die sie auf einmal wegen ihrer Bewegung überschauen kann.

    In dem verlinkten Paper steht unten auf Seite 587 dagegen die Aussage “The statement of the closed universe twin paradox is completely symmetrical. Albert and Betty travel in opposite directions from one another until they reunit.”

    “yet the resolution is asymmetric”. Symmetrisch ist hier nur, dass sich beide mit 4/5 der eigenen Rate altern sehen, aber bei der Umrundung macht Betty eine Art Zeitreise und trifft einen späteren Albert, eben den, der in Vorwärtsrichtung mit ihr zeitgleich war.

    Wir haben im Universum kein definiertes Inertialsystem. Daher lässt sich nicht entscheiden, wer von beiden stillsteht und wer das ganze Universum umrundet. Da ist doch auch nicht so etwas wie eine “Naht” oder eine Art “Nullmeridian” im Raum, der Raum ist für beide Beteiligten ein randloses Kontinuum.

    In dem Beispieluniversum besteht aber ein Unsymmetrie darin, dass die Richtung des Eingerolltseins eine bestimmte Orientierung relativ zur Zeitachse hat. Wenn die eigene Weltlinie parallel zum Zylinder ist, sieht man zeitgleich nur Ereignisse auf einem Ring um den Zylinder mit kleinstem Umfang (also senkrecht zur Zeitachse). Wenn man sich bewegt, sieht man zeitgleich Ereignisse, die auf einer Schraubenlinie liegen. Das bricht die Symmetrie der Situation. Wie es in unserem Universum aussieht, ist schwer zu sagen, wir kennen die Topologie ja nicht. Außerdem dehnt es sich schneller aus, als dass Licht es umrunden könnte, so dass wir keine Kopien der Milchstraße sehen können.

    Zwei Ameisen auf der Oberfläche eines Luftballons können ja auch nicht entscheiden, wer von beiden stillgestanden hat und wer den Ballon umkreist hat, oder ob womöglich beide sich in gegensätzliche Richtungen bewegt haben.

    Ameisen haben auch kein (signifikantes) Lorentzprodukt und bewegen sich nicht relativistisch. Und ein Ballon hat keine Zeitrichtung auf der Oberfläche.

    Lies Dir das Papier nochmal in Ruhe von vorne durch. Das ist die Erklärung mit Minkowski-Diagramm. Eigentlich sehr gut gemacht.

  30. #30 Alderamin
    19. Juli 2018

    @Daniel Rehbein

    Nun kann Angela während des Fluges durch ein Fernrohr sehen und Horst beobachten. Da das Universum zyklisch ist, sieht sie ihn auf der einen Seite von sich wegfliegen und von der anderen Seite auf sich zukommen

    Sie sieht (und hier lassen wir wieder die Lichtlaufzeit außen vor) Horst zweimal, aber da ihre Gleichzeitigkeitslinie ein Ring senkrecht zur Zylinderlänge ist, sieht sie Horst in beiden Richtung gleich alt. Horst hat aber eine spiralförmige Gleichzeitigkeitslinie und sieht sie jünger hinter sich und älter vor sich. Das macht die Unsymmetrie aus.

  31. #31 Albrecht Storz
    19. Juli 2018

    Albrecht Storz
    18. Juli 2018
    ” “Ein Zeitabgleich kann aber nur am gleichen Ort erfolgen, und wenn keiner umkehrt, ist das höchstens einmal der Fall, so kann man keine Zeitabschnitte vergleichen.”

    Wie ist diese Behauptung begründet?”

    Alderamin
    18. Juli 2018

    “Weil man sich nicht einigen kann, welche Ereignisse gleichzeitig sind, wenn man sich an verschiedenen Orten und mit relativistischer Geschwindigkeit zueinander bewegt.”

    Das ist für mich immer noch nicht nachvollziehbar. Angenommen zwei zueinander konstant bewegte Sender senden sich gegenseitig im Sekundentakt (jeweilige Bordzeit) Pulse. So könne doch beide miteinander Zeitdauern vergleichen, einfach indem sie die Pulse des jeweilig andern zählen, egal wie lang andauernd sie die Pausen dazwischen wahrnehmen.
    Konkretes Beispiel: die beiden Sender bewegen sich auf Tangenten eines angenähert gleichen Radius zu einem Supernova-Ereignis, sind also etwa gleich weit von dem Ereignis entfernt. Indem sie bei ihren Pulsfolgen Beginn und Ende des Ausbruchs markieren kann der jeweils andere ganz leicht durch Auszählen der Pulse feststellen, wie lange das Ereignis aus Sicht des jeweils anderen gedauert hat. Und die Gleichzeitigkeit ergibt sich aus der gleichzeitigen Beobachtung eines von beiden gleich weit entfernten Ereignisses.

    “Was GPS betrifft, da interessiert die Sicht des Satelliten auf uns überhaupt nicht, nur unsere auf den Satelliten, dessen Uhr so angepasst wird, dass sie synchron mit Uhren auf der Erde läuft. ”

    Genau diesen Punkt meine ich, nicht den, den Du weiter oben aufgegriffen hattest. Die Uhren auf den Satelliten laufen nämlich meinem Verständnis nach nicht ständig “frei”, sondern sie werden häufiger korrigiert, und zwar mit dem Signal der Bodenstation(en). Das möglichst exakte Übereinstimmen der Satelliten-Uhren ist ja das A und O der Satelliten-Navigation.
    So wie ich es verstanden habe wird den Satelliten jeweils ein Zeitdatum gesendet das genau im Augenblick des Empfangs auf dem jeweiligen Satelliten die dann aktuelle Zeit der Bodenstation angibt. Ich habe jetzt leider noch nicht herausbekommen, wie oft die Satelliten-Uhren “gestellt werden” (stündlich, täglich, wöchentlich, …?).
    Aber dieses Thema sollte jetzt vielleicht auch nicht allzu sehr in den Vordergrund rücken. Hier geht es ja um das Zwillingsparadoxon, nicht um GPS.

  32. #32 Albrecht Storz
    19. Juli 2018

    Albrecht Storz
    18. Juli 2018
    “Sendet zB das Raumschiff die Information: wir sind jetzt genau 2 Jahre [R-Zeit] lang geflogen und befinden uns genau 1 Lichtjahr [R-Entfernung] von der Basis entfernt, …”

    Alderamin
    18. Juli 2018

    “Aus Sicht des Raumschiffs hat man sich also mit 2 LJ/1 J = 0,5c entfernt, das werden beide Seiten so sehen, in der Relativgeschwindigkeit besteht Einigkeit. Auf der Erde weiß man, dass die Zeit auf dem Raumschiff um √(1-0,5²) langsamer verlaufen ist, also kann die Basis ausrechnen, dass nach Erdzeit 1 Jahr[R]/√(1-0,5²) = 1,1547 Jahre [B] vergangen sein müssen, als das Signal auf den Weg ging. Bei 0,5c wäre das Raumschiff also 1,1547/2 = 0,57735 Lichtjahre weit gekommen und so viele Jahre wäre dann die Lichtlaufzeit. Das Signal müsste folglich zum Zeitpunkt 1,1547 + 0,57735 = 1,732 Jahren [B] nach dem Start auf der Erde eingetroffen sein.”

    Das haut irgendwie nicht hin. Bzw. kann ich die Rechnung nicht nachvollziehen.

    Wie bringst Du die zwei Jahre Flugzeit [R-Zeit] in die Rechnung ein? Müsste es nicht
    2 Jahr[Raumschiff]/√(1-0,5²) = 2,309 Jahre[Basis]
    lauten? Dann wäre das Raumschiff nach Meinung der Basis 1,1547 LJ entfernt. Und das Signal käme nach
    1,1547+2,309=3,464 Jahren dort an.
    Okay. da hat einfach nur der Faktor 2 (“2 Jahre [R-Zeit]”) gefehlt.

  33. #33 Alderamin
    19. Juli 2018

    @Albrecht Storz

    Erst mal zur Rechnung: am Anfang hätte es schon mal heißen müssen 1LJ/2J = 0,5c.

    Wie bringst Du die zwei Jahre Flugzeit [R-Zeit] in die Rechnung ein?

    Ja, sorry, irgendwo ist mir der Faktor 2 unterwegs verloren gegangen und ich habe dann nur 1 Jahr Flugzeit gerechnet.

    1 LJ ist die längenverkürzte Strecke, die der bewegte Beobachter R misst. Der ruhende Beobachter B misst die nicht längenverkürzte Strecke 1 LJ * γ und γ ist 1/√(1-0,5²) = 1,1547. Macht also 1,1547 LJ zurückgelegte Strecke.

    Müsste es nicht 2 Jahr[Raumschiff]/√(1-0,5²) = 2,309 Jahre[Basis] lauten?

    Ja, oder alternativ 1,1547 LJ zurückgelegt mit 0,5c = 1,1547 LJ/0,5c = 2,309 J. Darauf dann noch die Lichtlaufzeit 1,1547 J, macht 3,4641 Jahre bis zum Eintreffen des Signals.

  34. #34 Albrecht Storz
    19. Juli 2018

    So weit so gut.
    Nehmen wir nun wieder die Zwillinge, ich nenne sie mal [R] und [B].
    In meinem Beispiel ist also der Zwilling [R] der Meinung, dass er zum Zeitpunkt des Absendens des Signals 2 Jahre [R] gealtert ist.
    Nach voriger Berechnung ist der Zwilling [B] zum Zeitpunkt des Absetzens der Nachricht um 2,309 Jahre [B] gealtert (und er erhält das Signal nach 3,464 Jahren [B]).

    Wenn wir aber nun sagen, dass sich nicht [R] sondern sich [B] bewegt, gilt auch obiges, aber mit vertauschten Rollen von [R] und [B]. Nur in einer Sache nicht:
    die Zeit [R] für das Absetzen des Signals bleibt gleich, denn das Signal wird ja entsprechend der [R]-Zeit ausgelöst: nämlich nach 2 Jahren [R]-Zeit.

    Wenn also [B] der Meinung ist, dass er sich bewegt, muss er trotzdem zu dem Ergebnis kommen, dass das Signal nach 3,464 Jahren [B] bei ihm ankommt.
    Denn das Ergebnis kann sich ja nicht durch die Wahl austauschbarer Standpunkte (welcher bewegt sich, [B] oder [R]) ändern.
    Obwohl nun [B] langsamer altert (weil bewegt) als [R] (der jetzt in Ruhe ist) müsste er dennoch das Signal nach 3,464 Jahren [B] erhalten obwohl er nicht 2,309 Jahre sondern weniger als 2 Jahre [B] gealtert ist.
    Wie geht das zusammen? Die Entfernung bei Absetzen des Signals muss dann wohl größer angesetzt werden?
    Das ist schon hirnverknotend …

  35. #35 Alderamin
    19. Juli 2018

    @Albrecht Storz

    Angenommen zwei zueinander konstant bewegte Sender senden sich gegenseitig im Sekundentakt (jeweilige Bordzeit) Pulse. So könne doch beide miteinander Zeitdauern vergleichen, einfach indem sie die Pulse des jeweilig andern zählen, egal wie lang andauernd sie die Pausen dazwischen wahrnehmen.

    Das ist nicht das Problem, sondern wenn jemand, der von beiden beobachtet wird “Stop” anzeigt (Lichtsignal), dann stoppen beide ihre Pulse nicht zur gleichen Zeit. Das ist schwer mit Worten zu erklären, auf Leifi-Physik ist es nett animiert (Play-Knöpfchen in den Bildern drücken)

    Konkretes Beispiel: die beiden Sender bewegen sich auf Tangenten eines angenähert gleichen Radius zu einem Supernova-Ereignis, sind also etwa gleich weit von dem Ereignis entfernt.

    Wenn die Situation symmetrisch konstruiert ist, also beide sich mit der gleichen Geschwindigkeit relativ zur Supernova bewegen (beide tangential), dann gibt es keine Differenz. Ruht aber einer und der andere bewegt sich, oder sind die Geschwindigkeiten verschieden, dann messen beide verschiedene Dauern für die Supernova.

    Wo das Problem liegt, ist wirklich schwierig mit Worten zu erklären, da ist ein Minkowski-Diagramm schon von Vorteil, oder eben meine Bilder oben. Mit der Erläuterung aus #22 bezüglich der Uneinigkeit von A und H” darüber, wo sich wer befand, als Horst sich auf den Weg machte, müsste doch klar sein, dass A und H” verschiedene Zeiten (und Orte) ermitteln werden, wenn sie sich auf ihre Beobachtungen stützen, selbst wenn man den Lorentzfaktor zur Umrechnung von Zeiten und Strecken korrekt anwendet. H” sieht Horst 14,692 Jahre vor seiner Ankunft bei Alpha Centauri und Horsts Umsteigen vergehen (3. Bild). A sieht nur 5,375 Jahre vergehen (1. Bild). Und A würde sagen, die Zeit von H” läuft und den Faktor 0,6 langsamer, also sollten für H” nur 5,375*0,6=3,225 Jahre vergehen. Und H” würde sagen, die Zeit von A verläuft um 0,6 langsamer, also sollten für A 14,692*0,6=8,8152 Jahre vergehen. Da sind keine zwei Zahlen identisch. Weil keine Einigkeit darüber besteht, wann Horst losgeflogen ist und wann er bei Alpha Centauri ankommt.

    Man muss nur die Rechnung oben nachvollziehen, dann ergibt sich das zwangsläufig.

    Genau diesen Punkt meine ich, nicht den, den Du weiter oben aufgegriffen hattest. Die Uhren auf den Satelliten laufen nämlich meinem Verständnis nach nicht ständig “frei”, sondern sie werden häufiger korrigiert, und zwar mit dem Signal der Bodenstation(en)

    Ja, einmal am Tag oder so, aber das dient nur dazu, um Toleranzen auszugleichen. Würden die Uhren auf der Erde alle perfekt synchron laufen und würde der Faktor, um den man sie oben langsamer laufen lässt, perfekt zur Bahn passen, dann bräuchte man sie nicht nachzustellen.

    So wie ich es verstanden habe wird den Satelliten jeweils ein Zeitdatum gesendet das genau im Augenblick des Empfangs auf dem jeweiligen Satelliten die dann aktuelle Zeit der Bodenstation angibt.

    Das macht Sinn, die Entfernung des Satelliten ist ja bekannt.

    Aber wesentlich ist bei GPS, dass man immer nur die Sicht der Erde einnimmt. Die Satelliten werden so eingestellt, dass sie Zeitticks produzieren, die trotz Zeitdilatation der Rate auf der Erde entsprechen. Das geht. Würde man auf dem Satelliten die Ticks einer Erduhr betrachten, dann wären die aber nicht synchron zu den Ticks, die der Satellit aussendet.

    Den (auf der GPS-Bahn größeren, aber auf niedriger Erdumlaufbahn kleineren) Effekt der Gravitation mal außen vor (der allein würde eine Synchronität erlauben; beide Beobachter wären sich einig, dass die Uhr auf der Erde langsamer läuft), dann würde der Satellit die Uhr der Erde langsamer laufen sehen, die Erde die Uhr des Satelliten aber auch. D.h. man würde die Satellitenuhr schneller einstellen, dass sie von der Erde aus gesehen mit der gleichen Rate wie auf der Erde vergeht; aus Satellitensicht würde seine schneller eignestellte Uhr aber noch mehr von der langsameren Erduhr abweichen, als dass eine Uhr mit originaler Geschwindigkeit täte. Obwohl also die Beobachter auf der Erde behaupten würden, die Ticks der Uhren wären die ganze Zeit supersynchron, würde ein Beobachter auf dem Satelliten sehen, wie sie immer weiter auseinander driften.

    Hier hat man wieder das Problem der Gleichzeitigkeit: für Erdbeobachter sind die Ticks der beschleunigten Satellitenuhr und der unveränderten Erduhr immer gleichzeitig; für den Satelliten driften sie immer mehr auseinander. Aber die Sicht des Satelliten interessiert niemanden. Hauptsache, von der Erde aus sieht alles synchron aus. Jegliche Differenz ist dann Lichtlaufzeit und somit Entfernung.

  36. #36 Alderamin
    19. Juli 2018

    Nehmen wir nun wieder die Zwillinge, ich nenne sie mal [R] und [B].

    “Red” und “Bull” 😀

    In meinem Beispiel ist also der Zwilling [R] der Meinung, dass er zum Zeitpunkt des Absendens des Signals 2 Jahre [R] gealtert ist.
    Nach voriger Berechnung ist der Zwilling [B] zum Zeitpunkt des Absetzens der Nachricht um 2,309 Jahre [B] gealtert (und er erhält das Signal nach 3,464 Jahren [B]).

    Richtig.

    Wenn wir aber nun sagen, dass sich nicht [R] sondern sich [B] bewegt, gilt auch obiges, aber mit vertauschten Rollen von [R] und [B]. Nur in einer Sache nicht:
    die Zeit [R] für das Absetzen des Signals bleibt gleich, denn das Signal wird ja entsprechend der [R]-Zeit ausgelöst: nämlich nach 2 Jahren [R]-Zeit.

    Symmetrisch wäre gewesen, wenn [B] das Signal aussendete.

    Wenn [R] immer noch das Signal aussendet, dann tut er das nach zwei Jahren und wähnt sich 1 LJ entfernt. Also kann er davon ausgehen, dass das Signal nach einem Jahr seiner Zeit auf der Erde ist. Dort verläuft aus seiner Sicht die Zeit um √(1-0,5²)=0,866 langsamer, ergo sollte [B] bei Aussendung des Signals nur 2*0,866=1,732 Jahre gealtert sein. Plus ein Jahr Lichtlaufzeit, macht 2,732 Jahre, bis das Signal bei B eintrifft. Das ist das, was [R] beobachtet. [B] sieht das völlig anders. Beide haben recht.

    Wenn also [B] der Meinung ist, dass er sich bewegt, muss er trotzdem zu dem Ergebnis kommen, dass das Signal nach 3,464 Jahren [B] bei ihm ankommt.

    Für [B] kommt es nur darauf an, wie sich [R] relativ zu ihm bewegt. Es macht keinen Unterschied, ob er sich als ruhend betrachtet und [R] als sich entfernend, oder ob er sich als entfernend sieht und [R] als ruhend. oder beide sich mit der halben Geschwindigkeit in Gegenrichtung entfernen. Nur die Relativgeschwindigkeit zwischen beiden zählt. In jedem Fall sieht [B] das Signal nach 3,464 Jahren ankommen.

    Denn das Ergebnis kann sich ja nicht durch die Wahl austauschbarer Standpunkte (welcher bewegt sich, [B] oder [R]) ändern.

    Doch. Es ändert sich, wenn man das Inertialsystem ändert.

    Obwohl nun [B] langsamer altert (weil bewegt) als [R] (der jetzt in Ruhe ist) müsste er dennoch das Signal nach 3,464 Jahren [B] erhalten obwohl er nicht 2,309 Jahre sondern weniger als 2 Jahre [B] gealtert ist.

    Nein, wie gesagt (und vorgerechnet), aus Sicht von [R] erhält [B] das Signal nach 2,732 Jahren, gemessen in [B]s Zeit (gemessen in [R]s Zeit nach 3 Jahren – 2 Jahre Flug + 1 Jahr Signallaufzeit). Das ist genau der Punkt, den ich anhand der Bilder oben zu erläutern versucht habe.

    Es ist letztlich Wurst, was die beiden voneinander beobachten, sie sind sich nicht darüber einig, wann das Signal eintrifft. Erst, wenn man sie am gleichen Ort wieder zusammenbringt, dann muss einer der ältere sein, da ist Gleichzeitigkeit für alle identisch. Und warum für diesen Fall derjenige, der unterwegs das Inertialsystem gewechselt hat, derjenige ist, der älter ist, habe ich oben vorgerechnet.

    Das ist schon hirnverknotend …

    Definitiv.

  37. #37 Albrecht Storz
    21. Juli 2018

    Alderamin
    19. Juli 2018
    “Es ist letztlich Wurst, was die beiden voneinander beobachten, sie sind sich nicht darüber einig, wann das Signal eintrifft.”

    Das ist der Punkt, der mir nicht eingeht. Wir machen folgendes Szenario auf:
    [R] und [B] (bitte keine Werbung für legale Kinderdrogen 😉 ) entfernen sich voneinander. Natürlich ist keiner irgendwie “privilegiert”, es gibt also keinen durch Bewegung oder Stillstand hervorgehobenen Partner.
    Die beiden hatten vereinbart, dass beide nach 2 Jahren jeweils eigener Bordzeit ein Signal senden.

    Es ist doch nun ganz offensichtlich, dass beide das Signal des jeweils anderen gleichzeitig empfangen müssen. Wenn sie (später) darüber kommunizieren, zu welcher (Eigen-)Bordzeit sie das Signal des anderen empfangen haben, so müssen deren Datumsstempel vollkommen identisch sein. Alles andere erscheint mir widersprüchlich und würde nicht mit der Symmetrie des Szenarios verträglich sein.

  38. #38 Alderamin
    21. Juli 2018

    @Albrecht Storz

    Es ist doch nun ganz offensichtlich, dass beide das Signal des jeweils anderen gleichzeitig empfangen müssen.

    Das ist nur offensichtlich unter der Galilei-Transformation, die wir im Alltag gewohnt sind. Aus der Konstanz der Lichtgeschwindigkeit, die experimentell erwiesen ist, folgt aber die Lorentztransformation, und nach der ergibt sich rechnerisch ganz klar, dass jeder der beiden folgendes beobachtet: Die Zeit beim Anderen vergeht langsamer, die 2 Jahre sind bei ihm länger, die Entfernung ist entsprechend größer als bei gleicher Flugzeit ohne Zeitdilatation, also kommt das Signal später an – sowohl wegen der Zeitdilatation selbst, als auch wegen der zusätzlichen Lichtlaufzeit. Nach der eigenen Zeit gemessen und der dafür erwarteten Entfernung und Lichtlaufzeit sollte das Signal früher eintreffen.

    Das sehen beide so, weil die Situation symmetrisch ist, solange keiner die Richtung ändert. Die Rechnung ist einfach, wie oben gesehen. Sie ist auch widerspruchsfrei, wie die Rechnung für das Zwillingsparadoxon zeigt. Die Auflösung ist der Verlust der Gleichzeitigkeit, der aber nur gilt, solange beide Beobachter in verschiedenen Inertialsystemen und sind und nicht beide gerade am gleichen Ort des Ereignisses, das sie betrachten, oder in gleicher relativer Bewegung und Entfernung dazu. Damit muss man sich einfach abfinden, auch wenn es einen Hirnknoten verursacht. Die Mathematik ist da gnadenlos und hat immer Recht (Unrecht kann sie nur haben, wenn das Modell falsch ist, aber c=const. ist eine einfache Annahme, die zigfach bestätigt wurde).

    Vielleicht hilft es, dann doch einmal Minkowski-Diagramme zu lernen und sich die Situation anhand dieser anzuschauen. Martin Bäker hat dazu schon einiges geschrieben, ich kann ja demnächst mal einen eigenen Versuch starten. In dem Papier über das zylinderförmig geschlossene Universum (Link in #29) ist es eingangs hervorragend erklärt. Könnte ich so wiedergeben. Mit etwas Erläuterungen, was ein inneres Produkt ist, das setzt das Papier voraus.

  39. #39 Funktionalistiker
    21. Juli 2018

    Alderamin nr. 17
    Alderamin schreibt: „Er hat doch die Relativitätstheorie!?“
    Ja und darin steckt versteckt eine Menge an Quantenphysik. Sonst würde es nicht funktionieren mit den unterschiedlichen Jahreszahlen.
    Unterschiedliche Jahreszahlen sind der entscheidende Grund, warum die Widerlegung erfolgreich ist. Und am Ende ist nicht „t“ sondern „n“ entscheidend. Wie jede Uhr besteht die Atomuhr aus einem Taktgeber und einem Zählwerk. Letzteres liefert das „n“ als materiellen Befund.
    Mit der Konstanz der Lichtgeschwindigkeit ist das so eine Sache. Teile der Gleichungen, auf die ich reflektiere, insbesondere der Term der RT sind auf quadratische/energetische Größen ausgerichtet und nicht auf lineare, wie die Geschwindigkeit. Das bedeutet hier nicht die Konstanz von c sondern von c².
    Aber das ist hier ein zu weites Feld – um mit Theodor Fontane zu sprechen.
    Außerdem – es interessiert niemand!!!

  40. #40 Alderamin
    21. Juli 2018

    @Funktionalistiker

    Alderamin schreibt: „Er hat doch die Relativitätstheorie!?“
    Ja und darin steckt versteckt eine Menge an Quantenphysik.

    Nö, die Relativitätstheorie ist älter als die Quantentheorie und beschreibt keinerlei Wahrscheinlichkeiten oder Energieniveaus. Tatsächlich sind die beiden Theorien komplementär und in bestimmten Fällen unvereinbar.

    Unterschiedliche Jahreszahlen sind der entscheidende Grund, warum die Widerlegung erfolgreich ist.

    Welche Widerlegung? Wovon? Der RT?

    Und am Ende ist nicht „t“ sondern „n“ entscheidend. Wie jede Uhr besteht die Atomuhr aus einem Taktgeber und einem Zählwerk. Letzteres liefert das „n“ als materiellen Befund.

    t ist aber = n*T = n/f…

    Mit der Konstanz der Lichtgeschwindigkeit ist das so eine Sache. Teile der Gleichungen, auf die ich reflektiere, insbesondere der Term der RT sind auf quadratische/energetische Größen ausgerichtet und nicht auf lineare, wie die Geschwindigkeit. Das bedeutet hier nicht die Konstanz von c sondern von c².

    Wenn c²=k, dann ist c=+/-√k. Bis auf die Richtung (die keine Rolle spielt, im Lorentzfaktor wird c ohnehin quadriert) ist c konstant genau dann wenn c² konstant. ist.

    Außerdem – es interessiert niemand!!!

    Was jetzt, die RT? Doch, schon.

  41. #41 hubert taber
    21. Juli 2018

    wenn sich zwei punkte trennen und verschiedene wege mit verschiedenen geschwindigkeiten (auch über c) zurücklegen dann können sich diese beiden punkte nur dann woanders treffen wenn für beide teilstrecken die rechnung t = s / v völlig ident ist.
    sonst gibt es kein treffen.
    dilatation und streckenverkürzung sind nur wirre annahmen.
    siehe unter:
    https://scienceblogs.de/mathlog/2016/12/07/kontruktivismus/#comment-234456
    mfg. h.t.

  42. #42 Alderamin
    22. Juli 2018

    @hubert taber

    Blödsinn. Ich hab’s ja sogar in Teil 1 vorgerechnet. Sollte jeder Mittelstüfler nachvollziehen können.

    Ich erkläre den Leuten hier, warum der Himmel blau ist und Du kommst daher und sagst, der Himmel sei ja gar nicht blau sondern grün, und wer das nicht sehe, sei ein Schwachkopf. Geh’ doch bitte woanders spielen.

  43. #43 Karl-Heinz
    22. Juli 2018

    Mal Gucken, zu welchen Ergebnissen jetzt die Fangemeinde so kommt.
    Angela besitzt eine Riesenkugel. Die Oberfläche der Riesenkugel wird während Horst nach Alpha Centauri hinfliegt für einen sehr kurzen Moment > aus Sicht von Angela gleichzeitig erleuchtet.
    Was sieht Horst?

  44. #44 Albrecht Storz
    22. Juli 2018

    Alderamin
    21. Juli 2018
    @Albrecht Storz

    “Die Zeit beim Anderen vergeht langsamer, ”

    Wieso das? Das ist doch nur dann der Fall, wenn ich mich selbst als unbewegt, den anderen als bewegt betrachte. Und aufgrund der Relativität können eben beide als ruhend oder in Bewegung betrachtet werden. Zählen tut nur die Relativgeschwindigkeit.

    Mathematik ist prima. Aber es gibt auch noch die Logik. Und die verbietet das Auftreten von “es gilt: A und nicht-A”. Wir sind bei unserer Erfassung der Realität auf die Logik angewiesen – genauso wie auf die Mathematik. Aber Formeln können falsch angewandt sein. Die Logik muss aber stimmen. Dass sich zB nicht zwei sich gegenseitig ausschließende Zustände aus den gemeinsamen Prämissen ergeben dürfen. (Die Bevorzugung von Mathe über Logik könnte man durchaus als ein “Lost in Math” -Phänomen paraphrasieren, siehe Sabine Hossenfelder, ebenda)

    Ich wiederhole mich: aufgrund der Symmetrie des von mir angegebenen Falls (zwei Raumschiffe entfernen sich voneinander mit konstanter Geschwindigkeit, sie senden beide nach 2 Jahren ein Signal: offensichtlich empfangen beide das Signal des jeweils anderen zur selben Zeit (Symmetrie!) -> die Zeit verläuft auf beiden Raumschiffen gleich) wäre jede andere Lösung inkonsistent.

    Nur wenn Du zeigst, wo dieses Szenario unsymmetrisch ist, kannst Du ernsthaft behaupten, beide Sender müssten sich uneinig sein über den Zeitverlauf.

    Tut mir Leid, wenn ich Dich da so in die Zange nehme, aber Du vertrittst hier ja (hoffentlich freiwillig 😉 ) die Mainstream-Position.

    Und bitte sieh das nicht als Böswilligkeit meinerseits an. Wenn ich falsch liege, dann möchte ich verstehen, warum. Wo mein Fehler liegt. Und mein Argument, dass in einer symmetrischen Situation nicht der eine etwas anderes wahrnehmen kann als der andere, ist, in meinen Augen, bisher nicht erschüttert. Ein Verweis auf hirnverknotende Formeln hilft da nicht. Zur Not müssen die Formeln leider falsch sein, nämlich dann, wenn nur dadurch die Logik zu ihrem Recht kommt.

    Und diese ganzen Überlegungen haben natürlich Auswirkungen auf das “Zwillings-Paradoxon”, wie ich gerne noch zeigen kann.

  45. #45 Karl-Heinz
    22. Juli 2018

    Nach dem ja alles so klar ist versuche ich ein bisschen Verwirrung zu stiften. Nehmen wir an Horst kommt auf die Idee, dass Angela innerhalb der 4,3 Lichtjahre Uhren auslegen soll, die mit ihr synchronisiert sind. Wenn Horst an einer der Uhren vorbei kommt, so will er einfach die aktuelle Zeit von Angela ablesen. Der Wechsel vom Inertialsystem von H’ in H” soll ausserdem augenblicklich erfolgen. Warum funktioniert die Idee von Horst nicht bzw wo ist sein Gedankenfehler?

  46. #46 hubert taber
    22. Juli 2018

    @ Alderamin #42
    warum uns der himmel blau erscheint das kann nur ich korrekt erklären.
    das können die theoretiker nicht.
    ich bitte dich nur diesen link zu lesen:
    https://scienceblogs.de/mathlog/2016/12/07/kontruktivismus/#comment-234456
    mfg. h.t.

  47. #47 Alderamin
    22. Juli 2018

    @Albrecht Storz

    Wieso das? Das ist doch nur dann der Fall, wenn ich mich selbst als unbewegt, den anderen als bewegt betrachte. Und aufgrund der Relativität können eben beide als ruhend oder in Bewegung betrachtet werden. Zählen tut nur die Relativgeschwindigkeit.

    Nein, so kann man das nicht sehen. Lokal sieht man sich immer als “in Ruhe”. Auch wenn man denkt und weiß, dass man sich bewegt, ändert sich deswegen lokal nicht die Physik. Auch wenn ich weiß, dass ich in einer Eisenbahn mit 250 km/h unterwegs bin, hopst mein Basketball relativ zum Zug und zu mir senkrecht auf und ab. Die Physik, die man in einem Inertialsystem betreibt, ist immer diejenige, die man in einem ruhenden System beobachtet. Für jemanden, der sich anders relativ zu mir bewegt, sehe ich gemäß RT zwingend die Zeit langsamer vergehen, auch wenn ich weiß, dass er ruht und ich mich bewege.

    Mathematik ist prima. Aber es gibt auch noch die Logik. Und die verbietet das Auftreten von “es gilt: A und nicht-A”.

    Das ist ja hier nicht der Fall, die Mathematik schließt eben jenen Fall ja aus, sie folgt streng logisch aus c=const. und ist konsistent, wie das vorgerechnete Zwillingsbeispiel zeigt.

    Aber Formeln können falsch angewandt sein.

    Abgesehen davon, dass Experimente (wie etwa die Lebensdauer von Myonen, die Funktion von GPS, das Hafele-Keating Experiment oder der tägliche Betrieb von Teilchenbeschleunigern) die Richtigkeit der Formeln beweisen – die Folgerungen sind einfach und zwingend, wie im ersten Teil vorgerechnet. Wenn c immer konstant erscheint, gibt es keine andere Möglichkeit als die Lorentztransformation. Die folgt genau so, wie bei einem rechtwinkligen Dreieck folgt a²+b²=c². Es ist die gleiche Formel.

    Dass sich zB nicht zwei sich gegenseitig ausschließende Zustände aus den gemeinsamen Prämissen ergeben dürfen.

    Die Prämisse ist c=const, und wenn die nicht erfüllbar wäre, würden die Experimente nicht dieses Ergebnis bestätigen. Die Mathematik hat sich nach den Experimenten zu richten.

    aufgrund der Symmetrie des von mir angegebenen Falls (zwei Raumschiffe entfernen sich voneinander mit konstanter Geschwindigkeit, sie senden beide nach 2 Jahren ein Signal: offensichtlich empfangen beide das Signal des jeweils anderen zur selben Zeit (Symmetrie!)

    Definiere “selbe Zeit”. Von wem aus gesehen? [B] und [R] empfangen das Signal des anderen nach derselben von ihnen jeweils gemessenen Zeit relativ zum Beginn, wo beide ihre Uhren synchronisierten. [B] empfängt das Signal von [R] nach 3,46 Jahren seiner Zeit (Zeitdilatation + Lichtlaufzeit) und [R] das Signal von [B] ebenso nach 3,46 Jahren dessen Zeit, das ist symmetrisch. [B] würde sagen, [R] habe das Signal aus seiner Sicht schon 3 Jahren erhalten, und umgekehrt genau so. Symmetrischer wird es nicht.

    “Ja, aber es können doch nicht beide Recht haben”. Doch, können sie. Solange sie nicht im gleichen Inertialsystem sind, können ihre Uhren verschieden laufen. Ein Widerspruch würde nur auftreten, wenn beide auf exakt dieselbe Weise ins gleiche Interialsystem zurückkommen und die Uhr des Einen zeigt etwas anderes an, als die Uhr des Anderen. Wenn nur einer umkehrt, ist die Symmetrie gebrochen. Kehren beide um mit den gleichen Geschwindigkeiten (4 Inertialsysteme, bei denen je 2 symmetrisch sind), dann werden die Uhren am Ende auch das gleiche anzeigen. Kehren beide nicht um, können die Uhren beliebig divergieren.

    Der Schluss “wenn in System [R] die Zeit t gilt, dann muss gleichzeitig in System [B] auch t gelten” ist hingegen unzulässig, denn die Gleichzeitigkeit lässt sich nur über den Zeitablauf definieren, und der ist eben aus anderer Perspektive verschieden. Beide beobachten nach zwei Jahren ihrer Zeit dasselbe, was der Andere nach der entsprechenden Zeit beobachtet, das ist symmetrisch. Aber es gibt nicht die universelle Gleichzeitigkeit in dem Sinne, dass alle eine gemeinsame Jetztzeit haben, auf der für jeden die 2 Jahre um sein müssen. Ein dritter Beoachter, der sich mit der halben Geschwindigkeit genau auf der Hälfte zwischen den beiden bewegt, der würde sagen, die Uhren laufen parallel – aber beide verlangsamt gegen seine Uhr.

    Das ist ein bisschen so, als wenn man sagt, “ein Metermaß muss immer gleich groß aussehen, egal wie weit es weg ist. Es ist auch in 1000 m Entfernung noch 1 m groß, wie kann es da kleiner erscheinen?” Erst wenn man das Metermaß wieder lokal heranholt, erscheint es wieder so groß wie ein Vergleichs-Metermaß. Man kann aber einen Beobachter auf der Hälfte der Entfernung fragen, und der würde bestätigen, dass beide Metermaße gleich groß, aber verkleinert erscheinen.

    Tut mir Leid, wenn ich Dich da so in die Zange nehme, aber Du vertrittst hier ja (hoffentlich freiwillig ) die Mainstream-Position.

    Bei dem Wort “Mainstream” gehen mir etwas die Nackenhaare hoch. Von Mainstream mag man bei der Dunklen Materie reden, weil es noch alternative Theorien gibt, die richtig sein könnten und die die Messungen (jedenfalls einige) erklären können. Es könnten auch alle falsch sein. Es geht hier aber um seit über hundert Jahren etablierte Physik, die nachgewiesenermaßen richtig ist. Die ist nicht mehr Mainstream als Newtons Fallgesetz. Alles, was sich hier als Nicht-Mainstream bezeichnet, muss, Entschuldigung, Kuhmist sein.

    Selbst wenn mal eine neue Theorie die Relativitätstheorie ablösen sollte, was vermutlich irgendwann der Fall sein wird, dann kann sie keine anderen Ergebnisse ergeben, als heutige Experimente es bestätigen; nur in Bereichen, die wir experimentell noch nicht ausgelotet haben könnten Abweichungen auftreten, nach denen die Physiker (bislang vergeblich) suchen, um eben einen Ansatz für neue Physik zu finden. Das betrifft etwa den Ereignishorizont Schwarzer Löcher. Aber nicht den Zeitverlauf bei annähernd c. Myonen aus der Stratosphäre beweisen jeden Tag, dass die spezielle RT hier richtig liegt.

    Wenn ich falsch liege, dann möchte ich verstehen, warum. Wo mein Fehler liegt.

    Dein Fehler ist, von einer universellen Gleichzeitigkeit zwischen bewegten Inertialsystemen auszugehen; dabei ist einer der Kernsätze der RT, dass es diese nicht gibt. Die ist bei Dir aber unanzweifelbares Axiom. Wenn Du Dich davon nicht lösen kannst und statt dessen den RT-Formeln folgen, weil es doch die Alltagserfahrung anders bestätigt, findest Du den Zugang zur RT nicht. Es tut mir leid, wenn meine Erläuterungen zum Zwillingsparadoxon das anscheinend bei Dir nicht leisten konnten, vielleicht versuchst Du es mal bei Martin oder einem populärwissenschaftlichen Buch. Ich kann’s nicht besser erklären. Aber auch wenn ich’s nicht erklären kann, ist es dennoch korrekt, weil experimentell nachgewiesen.

    Und diese ganzen Überlegungen haben natürlich Auswirkungen auf das “Zwillings-Paradoxon”, wie ich gerne noch zeigen kann.

    Ich möchte mich nicht ewig mit dem Thema auseinander setzen, ich habe auch noch weitere Artikel zu schreiben. Kannst gerne Deine Berechnungen hier ausbreiten, ob ich darauf eingehe, entscheide ich dann ggf., ansonsten sind ja noch Karl-Heinz und andere hier, die sich vielleicht dazu äußern mögen. Du kannst ja mal aus Deinen Folgerungen versuchen zu schließen, wie schnell ein bewegter Beobachter einen Lichtstrahl sieht. Wenn da was anderes als c herauskommt, bist Du nicht konform mit der empirischen Realtität. Oder wie der große Feynman sagte: “It doesn’t matter how beautiful your theory is, it doesn’t matter how smart you are. If it doesn’t agree with experiment, it’s wrong.”

    Und damit beende ich diesen Strang.

  48. #48 Funktionalistiker
    22. Juli 2018

    Alderamin nr. 40
    Mittlerweile befassen sich die Quantenphysiker mit Problemen, die sollte man nicht außer Acht lassen. Die Quantenphysik endet nicht da, wo alles zerhackt wird. Da gibt es beispielsweise auch die „spukhafte Fernwirkung“.
    Ich habe ja nicht umsonst geschrieben, in der RT steckt versteckt eine Menge Quantenphysik.
    Dass Du das nicht verstehst, weil es eben versteckt ist und man immer noch danach sucht, wohl seit über 100 Jahren,
    ist doch logo, oder!?
    Und wenn sich etwas versteckt, muss man eben danach suchen. Ich habe mir natürlich die Frechheit erlaubt, das mal zu tun. Ich wollte einfach mal wissen, wie Gravitation funktioniert. Und siehe da!!! Da war was zu finden- in der RT!!! Aber die RT war eben nur eine “Zutat“ von vielen. Aber sie hat die Mathematik geliefert.
    Das „einfache“ praktische Beispiel mit der Zahl „n“:
    Wenn ich zählen sage, dann meine ich Zählen und nicht die in Buchstaben gefasste Variante mit „f“ und „t“.
    Da kommt man ohne Umwege auf eine Zahl. Eine Zahl, die auch angezeigt, abgelesen und mit einer oder mehreren Zahlen verglichen werden kann. Ich muss eben einfach nur mal zählen, oder eben die Atomuhren so programmieren, dass sie nur den Takt zählen und die Zahlen anzeigen.
    Und das ergibt dann eine „Zutat“ zur Mathematik.
    Ich würde das gern weitergeben, um daraus eine den wissenschaftlichen Normen entsprechende Theorie zu machen, aber dafür interessierte sich bisher niemand!
    Aber das ist hier eben ein zu weites Feld – um nochmals mit Theodor Fontane zu sprechen.

  49. #49 Karl-Heinz
    22. Juli 2018

    @Funktionalistiker

    Was hat Zählen mit Quantenmechanik zu tun?
    Das verstehe ich überhaupt nicht. Wenn ich eine Spannung mit einem digitalen Messinstrument messe, so würde ich deswegen nie auf die Idee kommen, dass das zu messende Signal etwas mit Quantenmechanik zu tun hat.

  50. #50 rolak
    22. Juli 2018

    nie

    Ach das geht schon, Karl-Heinz, Du mußt halt funktionalistikeristischer denken.

  51. #51 hubert taber
    22. Juli 2018

    es gibt nur ein system.
    das UNIVERSUM.
    und es gibt KEINE singularen punkte.
    mfg. h.t.

  52. #52 Karl-Heinz
    22. Juli 2018

    @rolak

    Scherzkeks (Spaßvogel) 😉

  53. #53 rolak
    22. Juli 2018

    Scherzkeks

    Yoi, das nehme ich als Kompliment, herzlichen Dank dafür, Karl-Heinz! Darüber hinaus gilt selbstverständlich: Besser’n scherzigen Keks als nen weichen.

  54. #54 Karl-Heinz
    22. Juli 2018

    @rolak
    Ja, das war ein sehr eindeutiges Kompliment. 😉

  55. #55 Karl-Heinz
    23. Juli 2018

    @Albrecht Storz

    ansonsten sind ja andere hier, die sich vielleicht dazu äußern mögen

    Ich werde es mal versuchen. Bin selbst am meisten gespannt, ob so was Früchte trägt. Ich fange mal mit sehr einfachen Begriffen an. Füge ich zu den Raumkoordinaten noch die Zeit hinzu komme ich auf ein Koordinatensystem mit vier Parametern, nämlich (x, y, z, t). Die Physiker sprechen dann von einem Ereignis. Das Ereignis besteht aus einer Orts und Zeitangabe. Ein Beispiel dazu wäre, dass ein Blitz in einiger Entfernung von mir einschlägt und der Donner mich nach einiger Zeit erreicht. In diesem Fall haben wir zwei Ereignisse, nämlich der Blitz schlägt ein (E1) und der Donner erreicht meine Ohren (E2). Diese zwei Ereignisse kann man eindeutig Zahlenwerte im Koordinatensystem zuordnen. Ich werde dies mal versuchen.

    E1 (Blitz schlägt ein) = E1(x=1.372 m, y=0 m, z=0 m, t=0)
    E2 (Donner wird wahrgenommen) = E2(x=0, y=0, z=0, t=4)

    Wenn ich selbst im Ursprung des Koordinatensystems sitze, kann man auf Anhieb erkennen, dass das Ereignis E1 (Blitz schlägt ein) räumlich von mir entfernt ist, während das Ereignis E2 (Donner wird wahrgenommen) an meinem Platz stattfindet. Ich kann jetzt mit den Koordinatenangaben rumspielen und zum Beispiel berechnen, welchen räumlichen und zeitlichen Abstand die beiden Ereignisse E1 und E2 haben. Jedes Inertialsystem I hat sein eigenes Koordinatensystem. Glücklicherweise gibt es ja zur speziellen Relativitätstheorie (SRT) die Lorentz Transformation, mit der man von einem in das andere Koordinatensystem umrechnen kann. Auch damit kann man spielen.
    Man müsste doch mit Hilfe der Koordinatentransformation folgende Behauptung auf seine Richtigkeit prüfen können.
    Ausgangssituation: Person A und B bewegen sich relativ gegeneinander.
    (1) Jeder behauptet vom anderen, dass seine Uhr langsamer läuft.
    (2) Man müsste auch zeigen können, dass die Lichtgeschwindigkeit in beiden Inertialsystemen gleich c ist.
    Für den Anfang sind das mal meine Überlegungen.
    Fortsetzung folgt.

    \gamma=\frac{1}{\sqrt{1-v^{2}/c^{2}}}  Umrechnung von S nach S'  latex x’=\gamma\cdot(x-v\cdot t) $
    y'=y
    z'=z
    t'=\gamma\cdot(t-\frac{v}{c^{2}}\cdot x)
    Umrechnung von S’ nach S
    x=\gamma\cdot(x'+v\cdot t')
    y=y’
    z=z’
    t=\gamma\cdot(t'+\frac{v}{c^{2}}\cdot x')

  56. #56 Karl-Heinz
    23. Juli 2018

    upss… blödes Latex

    LORENTZ-Transformation
    \gamma=\frac{1}{\sqrt{1-v^{2}/c^{2}}}
    Umrechnung von S nach S’
    x'=\gamma\cdot(x-v\cdot t)
    y'=y
    z'=z
    t'=\gamma\cdot(t-\frac{v}{c^{2}}\cdot x)
    Umrechnung von S’ nach S
    x=\gamma\cdot(x'+v\cdot t')
    y=y'
    z=z'
    t=\gamma\cdot(t'+\frac{v}{c^{2}}\cdot x')

  57. #57 Karl-Heinz
    23. Juli 2018

    Ich beziehe mich jetzt auf Angela und Horst. Beide fragen sich nach einem Jahr, welche Zeit jeweils für den anderen vergangen ist. Für den Weg verwende ich als Einheit Lichtjahre und für die Zeit Jahre.

    E0 (Horst fliegt nach der Verabschiedung von Angela los)
    E1 (Angela stellt fest, dass ein Jahr vergangen ist)
    E2 (Im Koordinatensystem von Angela befindet sich Horst nach einem Jahr genau 0,8 Lichtjahre entfernt)

    E3 (Horst stellt fest, dass ein Jahr vergangen ist)
    E4 (Im Koordinatensystem von Horst befindet sich Angela nach einem Jahr genau -0,8 Lichtjahre entfernt)

    Die Geschwindigkeit v=0,8c
    Der Lorentzfaktor ɣ = 1/0,6 = 1,6666.
    Man beachte das der Lorentzfaktor ɣ immer größer oder gleich eins ist.
    Auf die y, z Angaben verzichte ich bei der Koordinatendarstellung, da sie immer 0 sind.

    E0(x=0; t=0) = E0(x’=0; t’=0)
    E1(x=0; t=1) = E1(x’= -1,333; t’=1,667)
    E2(x=0,8; t=1) = E2(x’=0; t’=0,6)

    Man beachte,dass E1 ungleich E2 ist.
    Die erste Fragestellung ist: Wenn für Angela genau ein Jahr vergangen ist, welche Zeit ist für Horst vergangen. Um das zu ermitteln müssen wir die Ereignisse E2(x’=0; t’=0,6) und E0(x’=0; t’=0) verwenden.

    Der Zeitunterschied von Angela zwischen E1 und E0
    ist Δt = 1-0 =1 Jahr

    Der Zeitunterschied von Horst zwischen E2 und E0
    Δt’ = 0,6-0 =0,6 Jahre

    Ich muss leider beenden. Fortsetzung folgt. 😉

  58. #58 Karl-Heinz
    23. Juli 2018

    Fortsetzung zu #57

    Wiederholung aus #57
    E0(x=0; t=0) = E0(x’=0; t’=0)
    E1(x=0; t=1) = E1(x’= -1,333; t’=1,667)
    E2(x=0,8; t=1) = E2(x’=0; t’=0,6)

    Wie man leicht erkennen kann ist der räumliche Abstand von Angela in ihrem Koordinatensystem für die Ereignisse E0(x=0; t=0) und E1(x=0; t=1) gleich null. Ebenso ist der räumliche Abstand von Horst in seinem Koordinatensystem für die Ereignisse E0(x’=0; t’=0) und E2(x’=0; t’=0,6) auch null. Da die räumlichen Abstände 0 sind kann man bei beiden die Eigenzeit zwischen den Ereignissen ermitteln.

    Jetzt betrachte ich das ganze aus dem Blickwinkel vom Horst, wenn aus seiner Sicht in seinem Koordinatensystem 1 Jahr vergangen ist. Bei der Koordinatendarstellung beginne ich zuerst mit Horst und transformiere nach Angela.
    E0(x’=0; t’=0) = E0(x=0; t=0)
    E3(x’=0; t’=1) = E3(x=1,333; t=1,667)
    E4(x’=- 0,8; t’=1) = E4(x=0 ; t=0,6)
    Für Horst sind die Ereignisse E0(x’=0; t’=0) und E3(x’=0; t’=1) relevant um seine Eigenzeit zu erhalten. Für Angela sind die Ereignisse E0(x=0; t=0) und E4(x=0 ; t=0,6) relevant um ihre Eigenzeit zu erhalten.

    Um die Antwort auf die Frage “Beide fragen sich nach einem Jahr, welche Zeit jeweils für den anderen vergangen ist.” zu erhalten benötigt man 5 Ereignisse. Das ganze ist also doch nicht so easy, wie ich am Anfang gedacht hatte. 😉

  59. #59 Karl-Heinz
    23. Juli 2018

    @Albrecht Storz

    Sendet zB das Raumschiff die Information: wir sind jetzt genau 2 Jahre [R-Zeit] lang geflogen und befinden uns genau 1 Lichtjahr [R-Entfernung] von der Basis entfernt, und die Basis diese Nachricht empfängt, welchen Wert würde die Basis für die Signallaufzeit dieser Nachricht annehmen müssen? Wann [B-Zeit] wäre die Nachricht und aus welcher Entfernung [B-Entfernung] wäre diese Nachricht gesendet worden?

    Das Raumschiff R hat die Koordinaten x’,t’

    E0(x’=0; t’=0) Raumschiff entfernt sich von der Basis
    E1(x’=0; t’=2) Raumschiff sendet ein Lichtsignal Richtung Basis aus.
    E2(x’=0; t’=4) Personen auf dem Raumschiff denken sich, dass das Signal auf der Basis angekommen sein müsste
    E3(x’=-4; t’=4) Signal ist auf der Basis angekommen

    Koordinatentransformation
    v=0,5c ɣ = 1,1547

    E0(x’=0; t’=0) = E0(x=0; t=0)
    E1(x’=0; t’=2) = E1(x=1,1547; t=2,3094)
    E2(x’=0; t’=4) = E2(x=2,3094; t=4,6188)
    E3(x’=-2; t’=4) = E3(x=0; t=3,4641)

    Das von der Raumstation ausgesandte Signal kommt in t=3,4641 Jahren auf der Raumstadtion an. Dafür wird die zeitliche Differenz von E3(x=0; t=3,4641) und E0(x=0; t=0) gebildet.

    Man beachte das die beiden Ereignisse
    E2(x’=0; t’=4) = E2(x=2,3094; t=4,6188)
    E3(x’=-2; t’=4) = E3(x=0; t=3,4641)
    nicht gleich sind.

    Für das Raumschiff sind die beiden Ereignisse E2 und E3 gleichzeitig haben aber unterschiedliche Entfernung. Für die Basistation sind E2 und E3 sowohl bezüglich Entfernung als auch Zeiten Zeiten verschieden.
    Ja, ja die Gleichzeitigkeitsfalle. 😉

  60. #60 hubert taber
    23. Juli 2018

    ich will es nicht noch fünfmal erklären.

    anstelle dieser “inertialsysteme” gibt es nur ein system das UNIVERSUM.
    und es gibt die physikalisch definierte sekunde.
    diese sekunden haben überall im universum gültigkeit.

    es ist völlig egal ob jemand stillsteht, sich bewegt, sich sehr schnell bewegt, sich wegbewegt oder wieder zurückkommt.
    t = s / v
    auch bei stillstand “läuft” die zeit weiter solange sich irgenwo etwas gemäss t = s / v bewegt.

    die “zeitdilatation” ist nur eine scheinlogische annahme die mit falschen rechnungen “zurechtgebogen” wird.

    hier ist noch der passende link:
    https://scienceblogs.de/mathlog/2016/12/07/kontruktivismus/#comment-234456

    auch mit gruss an die “SRT” und 2ART”.
    mfg. hubert taber

  61. #61 Karl-Heinz
    23. Juli 2018

    @hubert taber

    Es scheint so, dass du nicht mit Dingen zu tun hast, wo relativistische Effekte vorkommen. 😉

  62. #62 Alderamin
    23. Juli 2018

    @hubert taber

    ich will es nicht noch fünfmal erklären.

    Hat auch keiner verlangt. Erspar‘ es uns bitte.

    die “zeitdilatation” ist nur eine scheinlogische annahme die mit falschen rechnungen “zurechtgebogen” wird.

    Und Du bist ein Troll der nach Aufmerksamkeit lechzt. Mach‘ Dein eigenes Blog auf, statt hier rumzuspammen.

    Wie war das noch? Du kannst mit Normalintelligenten nicht kommunizieren? Dann lass‘ es doch einfach. Letzte Verwarnung.

  63. #63 hubert taber
    23. Juli 2018

    bitte ausnahmsweise noch einmal.
    @ karl-heinz
    bitte lese den link und dort findest du den beweis dass es keine “relativistischen” effekte gibt.
    mfg. h.t.
    bin schon wieder weg.

  64. #64 hubert taber
    23. Juli 2018

    noch ein p.s. an Alderamin:
    ich “lechze” nicht nach aufmerksamkeit.
    da mir diese völlig egal ist.
    ich möchte nur dass kompetente herrschaften BEWIESENE irrungen ins lot bringen.
    mit letzten freundlichen grüssen h.t.

  65. #65 Alderamin
    23. Juli 2018

    @Hubert

    „Bewiesene Irrungen“, ich lach’ mich tot, bewiesenermaßen irrst Du, sonst niemand.

    mit letzten freundlichen grüssen h.t.

    Damit‘s auch stimmt, helfe ich ein bisschen nach. Moderation aktiviert. Zeit, die Du nutzen kannst um in der Zwischenzeit mal ein gutes Buch über Relativitätstheorie zu lesen.

  66. #66 Captain E.
    24. Juli 2018

    @hubert taber:

    Sollte dir das entgangen sein, dass relativistische Effekte bereits nachgewiesen wurden? Selbst dein Smartphone oder Navigationsgerät wüssten nur sehr ungenau, wo du dich gerade befindest, wenn die Zeitsignale der Satelliten nicht relativistisch korrigiert würden.

    Anders gesagt: GPS und verwandte System, ohne die Kenntnisse der Relativitätstheorien aufgebaut, wären schrecklich ungenau, und die Praktiker könnten sich gar nicht erklären, warum eigentlich.

  67. #67 Albrecht Storz
    24. Juli 2018

    Alderamin
    22. Juli 2018
    @Albrecht Storz
    “Nein, so kann man das nicht sehen. Lokal sieht man sich immer als “in Ruhe”.”

    Tut mir Leid, wenn ich Dich anscheinend abnerve.

    Dein “lokal sieht man sich immer in Ruhe” ist im Lichte der Betrachtung zweier Inertialsysteme aber fehl am Platz. Und darum geht es hier. Natürlich ist ein Inertialsystem dadurch definiert, dass DARIN alles relativ zueinander ruht. Das ist ein Binsenweisheit, die trivial aus der Definition folgt.
    Wir haben hier aber ein Szenario dessen Beobachter auf sich selbst und auf ein anderes System schauen. Und dieser Beobachter kann sich als bewegt oder ruhend GEGENÜBER dem anderen System definieren (oder beide als bewegt ansehen).

    Wir sind jetzt an dem klassischen Punkt angelangt: “das ist halt so, und wenn Du es nicht glaubst bist Du ein Troll” oder ähnliches.
    Was ich feststellen muss ist, dass Du meine Argumentation entweder nicht nachvollziehen kannst (oder willst), oder Du massive innere Hemmnisse hast, Dich mit logischen Folgerungen auseinander zu setzen.
    Und nein, “das ist seit hundert Jahre gesichertes Wissen” ist kein physikalisches oder gar logisches Argument, sondern ein soziales Argument. “So viele können doch nicht irren!”
    Doch! Können sie. Und es gibt genug Beispiele in der Geschichte für solch unglaublichen Irrtum – bis in die neuere Zeit, ja, bis heute! Weil sie es nicht wagen, einen gesellschaftlich verfestigten Popanz anzugreifen.
    (Ich verweise gerne noch mal auf “Lost in Math” von S. Hossenfelder, die genau dies thematisiert).

    Nochmal: das von mir beschriebene System ist vollständig symmetrisch. Nirgendwo bemühe ich die Annahme einer absoluten Zeit, wie Du mir unterstellst. Wenn dem so wäre, könntest Du auf den konkreten Punkt in meinem Szenario hinweisen. Ich unterscheide aber konsequent und klar zwischen den jeweiligen Eigenzeiten und nur im logischen Schluss MUSS ich eine gemeinsame, “selbe Zeit” als Konsequenz annehmen. Das ist eine Folgerung, keine vorher hinein gesteckte Annahme. In der Logik muss man darauf streng achten, das ist völlig klar.

    Nochmal: da beide nach ihrer Eigenzeit nach 2 Jahren ein Signal absenden, und völlige Symmetrie gegeben ist, kann es nicht anders sein, als dass beide das Signal des jeweils anderen nach ihren jeweiligen Borduhren nach genau 4 Jahren erhalten.
    (2 Jahre Flugzeit, Signal wird in 1 LJ Entfernung abgesetzt, bis es das jeweils andere Schiff erreicht vergeht noch einmal zusätzlich ein Jahr da ja die Bewegung nicht stoppt).
    Es ist völlig unmöglich, dass die beiden Schiffe zu unterschiedlichen jeweiligen Bordzeiten das Signal des anderen erhalten. Oder dass sich das Ergebnis ändert je nachdem, ob man sich als bewegt oder ruhend betrachtet. Denn das Ereignis des Signaleintreffens kümmert sich nicht um meinen “Betrachtungsrahmen”.

    Übrigens sind Deine Argumente nicht einmal in sich konsistent.
    Am 19. schriebst Du noch: “Es ist letztlich Wurst, was die beiden voneinander beobachten, sie sind sich nicht darüber einig, wann das Signal eintrifft.” Am 22. ist es bei Dir aber wieder klar, dass beide nach 3,46 Jahre das Signal des anderen erhalten, aus irgendwelchen unerfindlichen Gründen sind aber beide nicht intelligent genug, dies für den anderen auszurechnen und würden über den Eintreff-Zeitpunkt beim anderen etwas anderes denken müssen.

    Was ist daran so schwer zu verstehen, dass das Eintreffen der Signale bei beiden zu jeweils identischen Bordzeiten erfolgen muss, und dass damit Gleichzeitigkeit begründet wird: wenn [R] das Signal von [B] erhält, weiß er: genau jetzt hat [B] auch mein Signal erhalten. Und vic versa. -> Genau so wird Gleichzeitigkeit begründet.
    Oder wie denn sonst?

  68. #68 Albrecht Storz
    24. Juli 2018

    PS: ich habe übrigens bei meiner Darstellung immer die Konstanz der Lichtgeschwindigkeit angenommen. Ich weiß nicht wie Du darauf kommst, ich würde diese hier in Fragestellen.

  69. #69 Albrecht Storz
    24. Juli 2018

    PSS:
    Die Raumschiffe entfernen sich voneinander mit 0,5 LJ/J. Darüber sind sie sich offensichtlich einig wie ich hier entnommen habe. Genauso könnten sie sich auch einigen, dass sie sich jeweils mit 0,25 LJ/J von ihrem Treffpunkt entfernen.
    Beide Raumschiffe unterscheiden sich in ihrem Zustand objektiv nicht. Also liegt auch kein objektiver Grund vor, warum ihre Uhren (faktisch, nicht scheinbar) unterschiedlich gehen sollten.
    Nach zwei Jahren jeweiliger Bordzeit sind sie 1 LJ voneinander entfernt (jeweils 0,5 LJ vom ehemaligen Treffpunkt).
    Bis hier sollte doch Einigkeit erzielbar sein. Das sind völlig triviale Annahmen und Folgerungen über die Symmetrie des Szenarios.
    Alle anderen Lösungen führen zu Asymmetrien die sich aus dem Szenario nicht begründen lassen. Im System muss die gleiche Konsequenz (gleichzeitiges Eintreffen der Signale) auftreten, egal ob ich es von dem einen oder von dem anderen Raumschiff aus betrachte. Und egal ob man sich dabei als bewegt oder als ruhend betrachtet (und vic versa).

  70. #70 Albrecht Storz
    24. Juli 2018

    PSSS:
    Die einzige Alternative wäre, dass im Rahmen der Relativitätstheorie die Logik außer Kraft gesetzt werden würde. Dann wüssten wir aber nicht einmal mehr, dass wir nichts wüssten. Wenn “A gleich nicht-A” gilt, ist alles denkbar.

  71. #71 Karl-Heinz
    24. Juli 2018

    @Albrecht Storz

    Es ist völlig unmöglich, dass die beiden Schiffe zu unterschiedlichen jeweiligen Bordzeiten das Signal des anderen erhalten.

    Das hat also wirklich keiner behauptet. Wenn jeder nach zwei Jahren eigener Bordzeit ein Signal ausendet, so wird das Signal vom anderen nach genau 3,4641 Jahren empfangen. Du kommst auf 4 Jahre und das ist definitiv falsch. Also irgendwas stimmt nicht mit deinem Modell. 😉

  72. #72 Karl-Heinz
    24. Juli 2018

    @Albrecht Storz

    Wie kommst du auf 4 Jahre? Ich komme nach deiner Methode nur auf 3 Jahre.

  73. #73 Alderamin
    24. Juli 2018

    @hubert

    Tschuldigung, hab‘ wohl vergessen, abschließend auf „Speichern“ zu klicken.

  74. #74 Alderamin
    24. Juli 2018

    @Albrecht Storz

    Dein “lokal sieht man sich immer in Ruhe” ist im Lichte der Betrachtung zweier Inertialsysteme aber fehl am Platz. Und darum geht es hier.

    Nein, da verstehst Du mich falsch, siehe unten.

    Natürlich ist ein Inertialsystem dadurch definiert, dass DARIN alles relativ zueinander ruht. Das ist ein Binsenweisheit, die trivial aus der Definition folgt.

    Nein, das ist nicht die Definition. Ein Intertialsystem bewegt sich unbeschleunigt, es wirken darin keine Trägheitskräfte. Ein Inertialsystem kann sich mit 99%c bewegen (eine Aussage, die immer nur Sinn macht, wenn man ein anderes System angeben kann, relativ zu dem es sich mit dieser Geschwindigkeit bewegt). Und dennoch ist die Physik darin identisch zu derjenigen in dem anderen, als ruhend betrachteten Inertialsystem. Wenn ich die Fenster schließe, kann ich nicht mehr durch Experimente entscheiden, ob ich ruhe oder mich mit 99% c relativ zu irgendwem bewege. Das ist die Aussage der Relativitätstheorie. Die sagt aus, dass das insbesondere auch für Licht gilt.

    Wir haben hier aber ein Szenario dessen Beobachter auf sich selbst und auf ein anderes System schauen. Und dieser Beobachter kann sich als bewegt oder ruhend GEGENÜBER dem anderen System definieren (oder beide als bewegt ansehen).

    Es darf für den Ausgang eines physikalischen Experiments keine Rolle spielen, wie der Experimentator darüber denkt. Es darf keinen Unterschied machen, ob er sich als bewegt sieht oder nicht. Was heißt das im Zusammenhang mit c=const.? Er wird keinen Unterschied feststellen, wenn er die Laufzeit eines Lichtsstrahls über eine feste Strecke misst. Beispiel: Lichtuhr. Die muss für ihn immer gleich ticken.

    Es hatte ein Kommentator im 1. Zwilings-Paradoxon-Artikel gefragt, ob man so was realisieren könnte, und da habe ich umständlich rumgedruckst. In Wahrheit gibt es diese Lichtuhr schon lange, mindestens seit dem 20. Juli 1969. Da haben die Apollo-11-Astronauten Ecken-Reflektoren auf dem Mond aufgestellt, die heute noch mit Laser angepingt werden, und wir messen damit die Entfernung des Mondes zentimetergenau über die Lauzeit des Lichts, und wir können da mit Fug und Recht die Vakuumlichtgeschwindigkeit von 299792459 m/s einsetzen. Obwohl die Erde mit fast 30 km/s um die Sonne rast. Obwohl die Sonne mit 250 km/s um die Milchstraße kreist. Und obwohl diese mit 600 km/s Richtung Großer Attraktor fällt. Spielt alles keine Rolle, es käme dasselbe heraus, wenn alle diese Bewegungen nicht existierten oder ganz andere Werte hätten. Das ist die Aussage der Relativitätstheorie.

    Wenn aber die Lichtuhr für mich mit der Frequenz senkrechter Spiegelabstand / c tickt, und ein externer Beobachter, der sich relativ zu mir bewegt, eine Zickzackkurve des Lichts mit längerem Spiegelabstand als ich wahrnimmt, aber das Licht auch für ihn nur mit c unterwegs ist, dann muss er meine Uhr langsamer ticken sehen, als ich das tue. Und aus Symmetriegründen muss das in umgekehrter Richtung genau so sein. Und folglich sind die Zeitabläufe wechselseitig verschieden. Wenn c immer gleich groß ist, egal wie ich mich bewege, dann kann es gar nicht anders sein, als dass Zeitabläufe in Inertialsystemen, die sich relativ zu meinem bewegen, verlangsamt erscheinen. Und umgekehrt.

    Das Zwillingsparadoxon versucht, dies in einen Widerspruch zu führen, aber es gibt keinen Widerspruch, wie ich vorgerechnet habe. Alles ist logisch konsistent. Die Formeln funktionieren. Mehr ist dazu eigentlich nicht zu sagen.

    Wir sind jetzt an dem klassischen Punkt angelangt: “das ist halt so, und wenn Du es nicht glaubst bist Du ein Troll” oder ähnliches.

    Von Troll habe ich nichts gesagt und glauben brauchst Du auch nichts, ich habe alles vorgerechnet (außer dem Additionstheorem für Geschwindigkeiten, da habe ich auf eine Herleitung verwiesen). Wenn Du einen Fehler findest, dann zeige ihn mir. Wenn Du keinen findest, dann ist die Herleitung wohl zwingend. Du kannst auch gerne mal eine alternative Herleitung versuchen, aus Deiner nichtrelativistischen Prämisse abzuleiten, wie die Lichtuhr eines bewegten Beobachters ticken würde und ob c dann noch genau so groß wäre wie aus Sicht des bewegten Beobachters. Wenn c da einen anderen Wert hat, erkläre warum man den Abstand des Mondes oder eines GPS-Satelliten mit der Vakuumlichtgeschwindigkeit ermitteln kann.

    Was ich feststellen muss ist, dass Du meine Argumentation entweder nicht nachvollziehen kannst (oder willst), oder Du massive innere Hemmnisse hast, Dich mit logischen Folgerungen auseinander zu setzen.

    Vielleicht verstehe ich Dich ja nicht, aber das, was ich zu verstehen meine, impliziert absolute Gleichzeitigkeit als Prämisse. Da diese aber im Widerspruch zu c=const. ist, kann ich da nicht mitgehen, weil das den empirischen Daten widerpricht. Ich kann aber umgekehrt auch nicht sehen, dass Du meiner Argumentation folgen kannst. Das ist dann der Punkt, wo ich irgendwann das Handtuch werfen muss. Mehr als vorrechnen kann ich’s nicht.

    Und auf die Experimente verweisen. Immerhin arbeiten am CERN ein paar tausend Leute mit Teilchen, die immer weiter beschleunigt werden und die Felder müssen im richtigen Moment geschaltet werden, um die Teilchen mit sich zu ziehen. Wenn die RT stimmt, dann kann das Teilchenbündel beliebig stark beschleunigt werden und trotzdem c niemals erreichen, weil aus Sicht der Teilchen c immer schneller als sie selbst erscheint – und damit eilt auch aus Sicht von uns auf der Erde Ruhenden der Lichtstrahl immer den Teilchen voraus. Hinge c vom Inertialsystem ab, dann gäbe es keinen Grund, warum die Teilchen nicht mit mehr als c um den Ring rasen sollten, aber das wird nicht beobachtet. Der Beschleuniger würde dann so nicht betrieben werden können. Er müsste die schneller werdenden Teilchen mit immer höherer Frequenz der Felder beschleunigen.

    Es ist völlig unmöglich, dass die beiden Schiffe zu unterschiedlichen jeweiligen Bordzeiten das Signal des anderen erhalten.

    Karl-Heinz hat schon gesagt, dass das auch nicht die Behauptung ist. Die Behauptung ist vielmehr, dass jeder das Signal des anderen nach 3,46 Jahren seiner Zeit beobachten wird. Und jeder wird das Signal nach 2 Jahren seiner Zeit absetzen und nach 3 Jahren seiner Zeit beim Anderen wähnen (wenn er die RT außer Acht ließe). Nicht mehr und nicht weniger.

    Am 19. schriebst Du noch: “Es ist letztlich Wurst, was die beiden voneinander beobachten, sie sind sich nicht darüber einig, wann das Signal eintrifft.” Am 22. ist es bei Dir aber wieder klar, dass beide nach 3,46 Jahre das Signal des anderen erhalten, aus irgendwelchen unerfindlichen Gründen sind aber beide nicht intelligent genug, dies für den anderen auszurechnen und würden über den Eintreff-Zeitpunkt beim anderen etwas anderes denken müssen.

    Sie sind sich dann nicht einig, wenn sie ihre eigene Bordzeit mit der vom anderen angegeben Zeit vergleichen und die RT außer Acht lassen. Natürlich würde echte Raumfahrer die RT kennen und wissen, warum die Uhren divergieren. Ist doch nur fürs Verständnis. Wie soll ich’s denn anders ausdrücken?

    Was ist daran so schwer zu verstehen, dass das Eintreffen der Signale bei beiden zu jeweils identischen Bordzeiten erfolgen muss, und dass damit Gleichzeitigkeit begründet wird: wenn [R] das Signal von [B] erhält, weiß er: genau jetzt hat [B] auch mein Signal erhalten. Und vic versa.

    Wenn beide nach 2 Jahren ihr Signal aussenden und nach 3,46 Jahren das Signal des anderen empfangen (aber eben nicht nach 3 Jahren), dann wissen sie in der Tat, dass der andere das Signal ebenfalls nach 3,46 Jahren erhalten hat, weil man vorher die Uhren synchronisiert hat und dann synchron das gleiche tut. Anderes habe ich auch nie behauptet.

    -> Genau so wird Gleichzeitigkeit begründet.
    Oder wie denn sonst?

    Gleichzeitigkeit wird darin begründet, dass zwei Ereignisse abzüglich Signallaufzeit eine Zeitdifferenz von 0 haben. In den Grafiken oben sind das jeweils waagerechte Linien (Licht würde sich schräg nach oben in den Bildern bewegen). Im ersten Bild ist der Abflug der roten Linie H’ vom Punkt A zeitgleich mit einer Entfernung des System H” von 8,6 LJ oder aus Sicht von H” mit einer Entfernung von 2*2,58 LJ = 5,46 LJ von A. Im letzten Bild ist der Abflug von H’ von A zeitgleich mit einer Entfernung von H” von A von 14,33 LJ. Da liegt offenbar eine verschiedene Ansicht zwischen A und H” darüber vor, wo H” relativ zu A war, als H’ losgeflogen ist.

  75. #75 Alderamin
    24. Juli 2018

    @Albrecht Storz

    PS: ich habe übrigens bei meiner Darstellung immer die Konstanz der Lichtgeschwindigkeit angenommen. Ich weiß nicht wie Du darauf kommst, ich würde diese hier in Fragestellen.

    Wenn Du verlangst, dass die Lichtuhr für den bewegten Beobachter genau so schnell tickt wie aus Sicht des ruhenden, dann muss implizit der bewegte eine langsamere Lichtgeschwindigkeit beobachten. Aus demselben Grund, aus dem ein gedribbelter Ball in einem Zug einem Zuginsassen langsamer erscheint als einem am Bahnsteig Wartenden, der die Summenbewegung von Ball (auf und ab) und Zug (vorwärts) beobachtet (Zickzacklinie). Die Geschwindigkeit des Balls ist nicht die selbe in beiden Inertialsystemen.

    Verlangst Du gleiche Zeitabläufe im bewegten und ruhenden System, dann verlangst Du, dass c nicht in allen Inertialsystemen gleich groß ist. Implizit.

  76. #76 Alderamin
    24. Juli 2018

    @Albrecht Storze

    Die Raumschiffe entfernen sich voneinander mit 0,5 LJ/J. Darüber sind sie sich offensichtlich einig wie ich hier entnommen habe. Genauso könnten sie sich auch einigen, dass sie sich jeweils mit 0,25 LJ/J von ihrem Treffpunkt entfernen.
    Beide Raumschiffe unterscheiden sich in ihrem Zustand objektiv nicht. Also liegt auch kein objektiver Grund vor, warum ihre Uhren (faktisch, nicht scheinbar) unterschiedlich gehen sollten. Nach zwei Jahren jeweiliger Bordzeit sind sie 1 LJ voneinander entfernt (jeweils 0,5 LJ vom ehemaligen Treffpunkt).

    Nach einem Jahr eigener Bordzeit wird beiden der Startpunkt 1/4 Lichtjahr weit entfernt erscheinen und das andere Raumschiff 1/2 LJ. Aber könnte der eine des anderen Uhr sehen (die er ihm vielleicht per Funkbild übermittelt, und da man die Geschwindigkeit des anderen kennt, kann man ja dessen Entfernung und damit die Signallaufzeit ermitteln und abziehen), dann wird jeder beim anderen nach einem Jahr nur 0,866 Jahre vergehen sehen haben und die Entfernung, aus der das Signal “ich bin jetzt 1/4 LJ vom Startpunkt entfernt” gekommen ist, auf 0,289 LJ Entfernung bestimmen. Auch wechselseitig. Wegen der Lichtuhr.

    Nicht gleichzeitig erscheinen hier der Moment, wo der eine den anderen (und sich selbst) in 1/4 LJ Entfernung sieht, und wann der andere dies aus seiner Sicht tut und bestätigt.

  77. #77 Alderamin
    24. Juli 2018

    @Albrecht Storz

    Die einzige Alternative wäre, dass im Rahmen der Relativitätstheorie die Logik außer Kraft gesetzt werden würde. Dann wüssten wir aber nicht einmal mehr, dass wir nichts wüssten. Wenn “A gleich nicht-A” gilt, ist alles denkbar.

    Alles ist streng mathematisch hergeleitet worden und die Formeln sind wiederspruchsfrei, von “Logik außer Kraft setzen” kann keine Rede sei. Die müsste hingegen außer Kraft gesetzt werden, wenn c aus Sicht eines bewegten Beobachters gleich schnell erscheint wie aus Sicht eines ruhenden und die Zeit solle dennoch bei beiden gleich schnell vergehen. Das ist mathematisch unmöglich herleitbar. Versuch’s mal.

  78. #78 Frank Wappler
    25. Juli 2018

    Alderamin schrieb (#47, 22. Juli 2018):
    > Lokal sieht man sich immer als “in Ruhe”.

    In der Physik erfolgt die Bestimmung, ob jemand (“Angela”, A) beschleunigt oder unbeschleunigt (also “ruhend”) gewesen wäre, nicht durch “Sehen” (oder “Fühlen”, oder sonstiges unmittelbar individuelles “Wahrnehmen”); sondern durch nachvollziehbare Messung:

    entweder ergeben die gemessenen Verhältnisse der Lorentzschen Distanzen \ell für alle je drei verschiedene Ereignisse \varepsilon_{AJ}, \varepsilon_{AK}, \varepsilon_{AN} an denen A teilgenommen hatte (und die sich insbesondere dadurch unterscheiden, dass A dabei jeweils von J, K oder von N getroffen und passiert wurde), dass (genau):

    \left( \frac{\ell^2[ \, \varepsilon_{AJ}, \varepsilon_{AK} \, ]}{\ell^2[ \, \varepsilon_{AJ}, \varepsilon_{AN} \, ]} \right)^2 + \left( \frac{\ell^2[ \, \varepsilon_{AK}, \varepsilon_{AN} \, ]}{\ell^2[ \, \varepsilon_{AJ}, \varepsilon_{AN} \, ]} \right)^2 + 1 =
    2 \, \left( \frac{\ell^2[ \, \varepsilon_{AJ}, \varepsilon_{AK} \, ]}{\ell^2[ \, \varepsilon_{AJ}, \varepsilon_{AN} \, ]} \right) + 2 \, \left( \frac{\ell^2[ \, \varepsilon_{AK}, \varepsilon_{AN} \, ]}{\ell^2[ \, \varepsilon_{AJ}, \varepsilon_{AN} \, ]} \right) + 2 \, \left( \frac{\ell^2[ \, \varepsilon_{AJ}, \varepsilon_{AK} \, ]}{\ell^2[ \, \varepsilon_{AJ}, \varepsilon_{AN} \, ]} \right) \, \left( \frac{\ell^2[ \, \varepsilon_{AK}, \varepsilon_{AN} \, ]}{\ell^2[ \, \varepsilon_{AJ}, \varepsilon_{AN} \, ]} \right) ;

    oder ansonsten eben nicht.

    > Die Physik, die man in einem Inertialsystem betreibt, ist immer diejenige, die man in einem ruhenden System beobachtet. Für jemanden, der sich anders relativ zu mir bewegt, sehe ich gemäß RT zwingend die Zeit langsamer vergehen, […]

    Auch Raten (z.B. Alterungsraten, Wachstums- oder Zerfallsraten) sind zu messen;
    und das erfordert insbesondere auch die Messung von (Verhältnissen von) Dauern.

    Alderamin schrieb (#74, 24. Juli 2018):
    > Ein [Inertialsystem] bewegt sich unbeschleunigt

    Die Mitglieder eines/jedes Inertialsystems sind unbeschleunigt; richtig; siehe oben.

    > es wirken darin keine Trägheitskräfte.

    Das ist wohl Teil der auf Geometrie bzw. Kinematik aufbauenden, darüber hinausgehenden Definition der (dynamischen) Messgröße “Trägheitskraft“.

    > […] Wenn aber die Lichtuhr für mich mit der Frequenz senkrechter Spiegelabstand / c tickt, und ein externer Beobachter, der sich relativ zu mir bewegt, eine Zickzackkurve des Lichts mit längerem Spiegelabstand als ich wahrnimmt, […]

    Bitte definiere “Spiegelabstand” als Messgröße.
    Wie vergleichen insbesondere Beteiligte, die nicht gegenüber einander ruhten (d.h. die nicht Mitglieder ein-und-des-selben Inertialsystems waren und blieben) “Spiegelabstände” untereinander ??

    > […] Wenn die RT stimmt, dann kann das Teilchenbündel beliebig stark beschleunigt werden und trotzdem c niemals erreichen, […]

    ???
    Die RT muss vorausgesetzt und eingesetzt werden, Werte von im Rahmen der RT definierten Messgrößen (Verhältnisse von Dauern, Verhältnisse von Distanzen, Geschwindigkeiten usw.) müssen schon ermittelt werden,
    um überhaupt experimentell entscheiden und feststellen zu können, ob irgendein gegebenes Teilchenbündel überhaupt beschleunigt gewesen wäre, oder nicht.

    Die RT wäre nicht deshalb falsch, falls (durch deren Anwendung überhaupt erst) festgestellt würde, dass z.B. ein bestimmtes gegebenes Teilchenbündel in einem bestimmten Versuch unbeschleunigt geblieben ist.

    Alderamin schrieb (#47, 22. Juli 2018):
    > […] dass Experimente (wie etwa die Lebensdauer von Myonen, die Funktion von GPS, das Hafele-Keating Experiment oder der tägliche Betrieb von Teilchenbeschleunigern) die Richtigkeit der Formeln beweisen

    Konkret welche Formeln wären denn “durch die genannten Experimente bewiesen” ?? — wohl kaum z.B. die (als Theorem der RT) bekannte Formel, die benutzt wird, um die individuellen Lebensdauern einzelner, nicht unbedingt gegenüber einander ruhender Myonen miteinander zu vergleichen und überhaupt erst einen bestimmten Wert der mittleren Lebensdauer von (freien) Myonen zu erhalten, nicht wahr ?

  79. #79 Spritkopf
    25. Juli 2018

    Liegt das eigentlich an der relativistischen Längenkontraktion, dass die Korinthen, die manche Leute kacken, stetig kleiner werden?

  80. #80 Albrecht Storz
    25. Juli 2018

    Alderamin
    24. Juli 2018
    @Albrecht Storz
    “Wenn Du verlangst, dass die Lichtuhr für den bewegten Beobachter genau so schnell tickt wie aus Sicht des ruhenden, dann muss implizit der bewegte eine langsamere Lichtgeschwindigkeit beobachten. ”

    Nein, das ist ein verdeckter Fehlschluss, denn er ist nicht zwingend, wie Du aber suggerierst. ZB können die Maßstäbe entsprechend verändert sein so dass beide zum gleichen Messergebnis kommen.

  81. #81 Albrecht Storz
    25. Juli 2018

    Alderamin
    24. Juli 2018
    @Albrecht Storz
    “Und jeder wird das Signal nach 2 Jahren seiner Zeit absetzen und nach 3 Jahren seiner Zeit beim Anderen wähnen …”
    Das stimmt deshalb nicht, weil natürlich auch die weiter zunehmende Entfernung während der Lichtlaufzeit zu berücksichtigen ist. Deshalb meine 4 Jahre. Das ist aber natürlich nur eine Petitesse.
    Was mich so langsam kirre macht, ist die Frage, wo genau wir eigentlich verschiedene Standpunkte vertreten.

    Inzwischen scheint klar zu sein, dass die Signale bei beiden Raumschiffen gleichzeitig ankommen. Wie dennoch Ungleichzeitigkeit behauptet werden kann ist mir allerdings, ehrlich gesagt, schleierhaft.

    Und wie sollen denn die Signale gleichzeitig eintreffen, wenn sie nicht gleichzeitig abgesendet wurden – bei völlig symmetrischen Bedingungen?

    Die Raumschiffe könnten beim Absetzen ihrer Signale verschieden weit voneinander entfernt gewesen sein, dies wäre sogar zwingend, wenn sie unterschiedliche, nicht synchrone Zeitverläufe messen.
    Aber, wenn beide als ruhend oder, gleichwertig, als bewegt angesehen werden können, welches Raumschiff sollte beim Absetzen der Signale weiter weg, welches näher dran gewesen sein?

    Was ist mein zentraler Punkt? Mein zentraler Punkt ist das Narrativ: bei einem bewegten Beobachter verginge die Zeit langsamer als bei einem ruhenden Beobachter.
    Nehmen wir das hier:
    https://de.wikipedia.org/wiki/Zeitdilatation
    Am Anfang des Artikels wird noch korrekter Weise von einem SCHEINBAREN Effekt gesprochen, doch gleich darauf und weiter unten lesen wir

    “Das bedeutet, dass auch Uhren, die sich relativ zum Beobachter bewegen, langsamer gehen als seine eigene.”

    und

    “Die allgemeine Tatsache, dass bewegte Uhren langsamer gehen, bezeichnet man als Zeitdilatation.”

    Das ist natürlich kompletter Quatsch, denn es gibt keinen (absolut) bewegten bzw. (absolut) ruhenden Beobachter.

    Aber es bleibt noch die Frage zu klären, wie es TATSÄCHLICH ist. Das Zwillings-Paradoxon macht ja aus einem scheinbaren Effekt, man könnte von einer Art optischer Täuschung sprechen, nun einen tatsächlichen Sachverhalt: der reisende Zwilling bleibe jünger.

    Mein Versuch, das Zwillings-Paradoxon in leichtere Einzelprobleme zu zerlegen, bisher nur der Auseinanderflug, führte mE bisher zu folgenden Übereinstimmungen:
    – wenn beide Raumschiffe nach jeweils gleicher Bordzeit ein Signal absenden, so erhalten beide Raumschiffe nach jeweils gleicher Bordzeit das Signal des anderen. (Zur Verdeutlichung: wenn die Raumschiffe darüber korrespondieren, wann welches Ereingis nach jeweils eigener Bordzeit eintrat, so müssen die numerischen Werte der Zeitstempel jeweils für /Absetzen des Signals/ und für /Empfang des Signals/ für beide Raumschiffe identisch sein.)

    Können wir uns auf diesen Stand einigen? Ich frage einfach noch einmal sicherheitshalber nach.

  82. #82 Karl-Heinz
    25. Juli 2018

    @Albrecht Storz

    Nach deiner Rechnung ist der Zeitpunt des Empfange beim Raumschiff A mit folgenden Zeiten zu rechnen.

    A ruht B mit 0,5c bewegt => 2+1= 3Jahre
    Zwischenrechnung:
    c=1
    c*t = 1 => t =1 Jahr

    A mit -0,5 bewegt B ruht => 2+1+1= 4 Jahre
    Zwischenrechnung:
    c=1
    c*t = 1+0,5c*t
    0,5*t = 1 => t =2 Jahre

    A mit -0,25 bewegt B mit 0,25 Bewegt = 2+1,333 =3,333 Jahre
    c=1
    c*t = 1+0,25c*t
    0,75*t = 1 => t=1 Jahr

    Das kann jetzt aber nicht dein Ernst sein oder?

  83. #83 Alderamin
    25. Juli 2018

    @Albrecht Storz

    “Wenn Du verlangst, dass die Lichtuhr für den bewegten Beobachter genau so schnell tickt wie aus Sicht des ruhenden, dann muss implizit der bewegte eine langsamere Lichtgeschwindigkeit beobachten. ”

    Nein, das ist ein verdeckter Fehlschluss, denn er ist nicht zwingend, wie Du aber suggerierst. ZB können die Maßstäbe entsprechend verändert sein so dass beide zum gleichen Messergebnis kommen.

    Ein Schrumpfen einer Länge senkrecht zur Bewegungsrichtung müsste aber erst geometrisch motiviert werden und wäre beim Michelson-Morley-Experiment aufgefallen – da interferiert ja Licht aus zwei senkrecht zueinander stehenden Strecken, ganz ähnlich wie im Großen bei LIGO (wo es ebenfalls auffallen würde). Und die Erde bewegt sich.

    “Und jeder wird das Signal nach 2 Jahren seiner Zeit absetzen und nach 3 Jahren seiner Zeit beim Anderen wähnen …”
    Das stimmt deshalb nicht, weil natürlich auch die weiter zunehmende Entfernung während der Lichtlaufzeit zu berücksichtigen ist. Deshalb meine 4 Jahre. Das ist aber natürlich nur eine Petitesse.

    Wie Karl-Heinz bemerkt hat, ist das inkonsistent mit der Annahme, dass immer dasselbe herauskommen sollte, egal wer sich bewegt, solange die Relativgeschwindigkeit 0,5c ist.

    Inzwischen scheint klar zu sein, dass die Signale bei beiden Raumschiffen gleichzeitig ankommen. Wie dennoch Ungleichzeitigkeit behauptet werden kann ist mir allerdings, ehrlich gesagt, schleierhaft.

    Es würde helfen, wenn Du einsehen würdest, dass Gleichzeitigkeit nicht in jedem Fall in der RT erhalten ist (in bestimmten Fällen ist sie das schon).

    Wenn A und B ihre Uhren wechselseitig verschieden schnell laufen sehen, dann kann bei einem Uhrenvergleich am selben Ort ja nicht sowohl die eine Uhr langsamer gelaufen sein als die andere, wie auch umgekehrt.Das ist genau das Zwillingsparadoxon. An verschiedenen Orten können die Uhren laufen, wie sie wollen, wenn es auf die Frage “wie spät ist es jetzt bei dir” keine Anwort gibt, in der beide darin übereinstimmen, wann denn jetzt genau sein soll. Wenn man die Uhren zum direkten Vergleich am selben Ort zusammenbringen will, muss man einen Symmetriebruch leisten, der die Gleichzeitigkeit wieder herstellt, und alles funktioniert wunderbar gemäß sehr einfacher Formeln, bei denen man sich fragt, warum vor Einstein nicht längst jemand anderes darauf gekommen ist (na ja, eigentlich ist Lorentz schon darauf gekommen, er hat das Ganze nur nicht zu Ende gedacht).

    “Die allgemeine Tatsache, dass bewegte Uhren langsamer gehen, bezeichnet man als Zeitdilatation.”
    Das ist natürlich kompletter Quatsch, denn es gibt keinen (absolut) bewegten bzw. (absolut) ruhenden Beobachter.

    Wenn man vor “bewegte” ein “relativ” einfügt, stimmt es aber wieder.

    Aber es bleibt noch die Frage zu klären, wie es TATSÄCHLICH ist. Das Zwillings-Paradoxon macht ja aus einem scheinbaren Effekt, man könnte von einer Art optischer Täuschung sprechen, nun einen tatsächlichen Sachverhalt: der reisende Zwilling bleibe jünger.

    Das ist kein Schein-Effekt – atmosphärische Myonen leben wirklich länger und schaffen es bis auf den Erdboden, obwohl sie im Mittel nur 660 m weit kommen dürften und in 15 km Höhe entstehen.

    – wenn beide Raumschiffe nach jeweils gleicher Bordzeit ein Signal absenden, so erhalten beide Raumschiffe nach jeweils gleicher Bordzeit das Signal des anderen. (Zur Verdeutlichung: wenn die Raumschiffe darüber korrespondieren, wann welches Ereingis nach jeweils eigener Bordzeit eintrat, so müssen die numerischen Werte der Zeitstempel jeweils für /Absetzen des Signals/ und für /Empfang des Signals/ für beide Raumschiffe identisch sein.)

    Können wir uns auf diesen Stand einigen? Ich frage einfach noch einmal sicherheitshalber nach.

    Ja, das hatten wir schon geklärt.

  84. #84 Alderamin
    25. Juli 2018

    @Frank Wappler

    nicht durch “Sehen” (oder “Fühlen”, oder sonstiges unmittelbar individuelles “Wahrnehmen”); sondern durch nachvollziehbare Messung

    Das verwende ich synonym. Wenn ich mit einem Messgerät arbeite, muss ich am Ende auch auf dessen Ausgabe schauen… Mach’ ich aber aus Faulheit. Gebe ich zu, mache ich auch weiterhin so.

    Bitte definiere “Spiegelabstand” als Messgröße.
    Wie vergleichen insbesondere Beteiligte, die nicht gegenüber einander ruhten (d.h. die nicht Mitglieder ein-und-des-selben Inertialsystems waren und blieben) “Spiegelabstände” untereinander ??

    Können sie vor dem Abflug am selben Ort tun. Oder sich vom selben Lichtuhr-Hersteller beliefern lassen.

    Senkrecht zur Bewegungsrichtung erhält man das Licht von “oben” und “unten” der Spiegel übrigens simultan, d.h. der als ruhend betrachtete Beobachter kann den Sehwinkel korrekt bestimmen. Ein Lichtsignal, das im richtigen Moment vom ruhenden Beobachter senkrecht zur Bewegungsrichtung des bewegten Beobachters losgeschickt wird, trifft diesen genau wenn er im rechten Winkel zum Beobachter ist und über die Ping-Zeit kann er auch die Entfernung bestimmen. Mit dem Sehwinkel ergibt sich der Spiegelabstand.

    Konkret welche Formeln wären denn “durch die genannten Experimente bewiesen” ?? — wohl kaum z.B. die (als Theorem der RT) bekannte Formel, die benutzt wird, um die individuellen Lebensdauern einzelner, nicht unbedingt gegenüber einander ruhender Myonen miteinander zu vergleichen und überhaupt erst einen bestimmten Wert der mittleren Lebensdauer von (freien) Myonen zu erhalten, nicht wahr ?

    Sondern? Die Lebensdauern von ruhenden Myonen wird man wohl im Labor zum Vergleich messen können, es gibt ja auch myonische Atome.

  85. #85 Frank Wappler
    26. Juli 2018

    Alderamin schrieb (#84, 25. Juli 2018):
    > […] und über die Ping-Zeit

    … Ping-Dauer! …

    > kann er auch die Entfernung bestimmen.

    Sehr gut: die chronometrische Entfernungs- bzw. Distanz-Definition!

    Es fragt sich dann eben (“nur noch”), wie die (Ping-)Dauern, die Angela und (die gegenüber Angela ruhenden Franziska, Gertrude, Roberta, Siglinde usw. untereinander feststellten, mit den (Ping-)Dauern zu vergleichen wären, die Horst und (die gegenüber Horst ruhenden) Jockl, Kilian, Paul, Quirin usw. untereinander feststellten.

    > […] vor dem Abflug am selben Ort […]

    Vergleiche zwischen denjenigen, die durchwegs beieinander (koinzident) waren und blieben, sind gewiss besonders einfach.

    In den relevanten Gedankenexperimenten der RT geht aber stattdessen um Beteiligte (so wie Angela und Horst), die nicht durchwegs beieinander waren und blieben, ja die sogar nicht durchwegs gegenüber einander ruhten, sondern die sich (stattdessen) höchstens gelegentlich trafen/passierten (momentan koinzident waren).

    Das heißt: es geht um deutlich unterscheidbare Versuche bzw. Versuchsanordnungen;
    und um die eine an die andere anzuschließen müsste mindestens einer der Beteiligten “aus dem bisherigen Bewegungszustand gerissen” und “in einen anderen Bewegungszustand versetzt” werden.
    (Grund genug, stets immer wieder zu messen bzw. zu vergleichen.)

    > [ … »nicht durch “Sehen” (oder “Fühlen”, oder sonstiges unmittelbar individuelles “Wahrnehmen”); sondern durch nachvollziehbare Messung« … ] Das verwende ich synonym.

    > Wenn ich mit einem Messgerät arbeite, muss ich am Ende auch auf dessen Ausgabe schauen

    Wenn ich mit einem Messgerät arbeite, muss ich am Ende die systematische Unsicherheit bzw. Abweichung bewerten, die zwischen dem Wert liegt, den ich nach Anschauen der Messgerätanzeige(n) zugeordnet habe, und dem wahren Wert der Messgröße (dem Eigenwert des Messgerät-Systems) im betreffenden Versuch.

    Das heißt insbesondere, ich muss (trotzdem) wissen und berücksichtigen, wie zumindest im Prinzip definitionsgemäß gemessen werden soll.

    > […] Sondern? […]

    Die relevanten Formeln sind nicht Ausdruck der Modellierung/Zusammenfassung von (schon vorhandenen) experimentellen Ergebniswerten,
    sondern
    Theoreme, die schon von vornherein aus den detaillierten Definition folgen, wie überhaupt gemessen werden soll (falls sich dazu die Gelegenheit ergeben sollte).

    > Die Lebensdauern von ruhenden Myonen wird man wohl im Labor zum Vergleich messen können

    Sicher; das erspart aber nicht den tatsächlichen Vergleich mit (ebenfalls gemessenen) Lebensdauern anderer Myonen(-Ensembles), die z.B. gegenüber dem betreffenden Labor nicht (durchwegs) ruhten.

  86. #86 Karl-Heinz
    26. Juli 2018

    @Frank Wappler

    Es fragt sich dann eben (“nur noch”), wie die (Ping-)Dauern, die Angela und (die gegenüber Angela ruhenden Franziska, Gertrude, Roberta, Siglinde usw. untereinander feststellten, mit den (Ping-)Dauern zu vergleichen wären, die Horst und (die gegenüber Horst ruhenden) Jockl, Kilian, Paul, Quirin usw. untereinander feststellten.

    Man merkt, dass du nie beim Bundesheer warst oder die Ausbildung dort verschlafen hast. Die Leute dort verwenden für die Entfernungsbestimmung die altbewährte Radarmethode und die funktioniert recht gut. Wenn deine Leute sich mit der Radarpistole gegenseitig beschießen, was für eine Entfernung (bei manchen auch eine Funktion der Zeit) würden sie wohl messen?

  87. #87 Albrecht Storz
    27. Juli 2018

    Um die Debatte nicht ausdümpeln zu lassen, und um zu vermeiden, dass weiter auf von mir aufgebrachten und auch so bezeichneten Petitessen rumgeritten wird möchte ich jetzt einfach zum Schluss kommen und dazu mein Modell des “Zwillings-Paradoxons” vorstellen. Dieses Szenario ist vollständig symmetrisch konstruiert, und ich frage mich, warum nirgendwo solche Szenarien diskutiert werden, sondern immer solche, bei denen immer zwangsläufig die Gefahr besteht, dass man (irrtümlich) von bevorzugten Koordinaten- (bzw. Inertial-) -systemen ausgeht. Sei es, weil man die Erde als Bezugspunkt wählt, sei es weil man, wie zB bei der Lichtuhr-Darstellung von Marcus Pössel, eine im Darstellungsrahmen ruhende Uhr einführt.

    Hier also mein symmetrisches “Zwillings-Paradoxon”:

    Von der Erde (oder vielleicht besser von Punkt 0 aus) aus starten zwei Raumschiffe in entgegengesetzten Richtungen. Wir beobachten von dem einen Raumschiff aus, wie die Lichtuhr des anderen Raumschiffes langsamer geht. Das heißt, die von uns beobachtete Uhr im anderen Raumschiff geht gegenüber unserer eigenen Uhr immer weiter nach.

    Nun kehren beide Raumschiffe gleichzeitig wieder um und fliegen synchron zur Erde zurück. Auch während dieser Phase beobachten wir auf dem relativ zu uns bewegten Raumschiff einen langsameren Zeitablauf. Das Nachgehen der anderen Uhr gegenüber unserer eigenen Uhr summiert sich also immer weiter auf.

    Wenn wir auf der Erde zusammen mit dem anderen Raumschiff eintreffen, sind wir also überzeugt, dass die Uhr des anderen Raumschiffes gegenüber unserer eigenen Uhr nachgehgen muss.

    ABER: auch der Reisende auf dem anderen Raumschiff hat unsere Uhr langsamer laufen gesehen wie seine eigene. Auch er hat über den gesammten Flug ein Aufsummieren der Zeitdifferenz gesehen. Auch er ist damit überzeugt, dass unsere Uhr gegenüber seiner Uhr nachgehen muss.

    Und nun vergleichen wir die beiden Uhren …

    Klar sollte wohl sein: nicht beide Uhren können gleichzeitig gegeneinander nachgehen.

    (PS: Es wird hier impliziert, dass alle Raumschiffmanöver vollkommen symmetrisch ausgeführt werden, also insbesondere auch gleichartige Beschleunigungen und symmetrischer Richtungswechsel erfolgen.)

    Und hier noch als kleiners Schmankerl für die, die das noch nicht kennen, die leider falsche Widerlegung eines, nach wahrscheinlich wirklich existierendem Vorbild geformten Kritikers bzgl. Zwilliingsparadoxon durch Albert Einstein höchstpersönlich:

    https://biblio.wiki/wiki/Dialog_über_Einwände_gegen_die_Relativitätstheorie

  88. #88 Albrecht Storz
    27. Juli 2018

    Karl-Heinz
    25. Juli 2018

    Du kannst freilich rechnen, was Du willst. Aber bitte diesen Unsinn nicht dann mir unterschieben …

  89. #89 Karl-Heinz
    27. Juli 2018

    @Albrecht Storz

    Mal schöne Grüße aus Caorle. Ich hoffe, dass der Himmel heute klar bleibt.
    Es war nicht meine Absicht jemanden etwas unterzuschieben. Auf dein Beispiel werde ich nach dem Essen eingehen. 😉

  90. #90 Alderamin
    27. Juli 2018

    @Albrecht Storz

    Derzeit etwas Stress (MoFi und anderes). Komme darauf zurück.

  91. #91 Karl-Heinz
    27. Juli 2018

    @Albrecht Storz

    Dieses Szenario ist vollständig symmetrisch konstruiert, und ich frage mich, warum nirgendwo solche Szenarien diskutiert werden, sondern immer solche, bei denen immer zwangsläufig die Gefahr besteht, dass man (irrtümlich) von bevorzugten Koordinaten- (bzw. Inertial-) -systemen ausgeht.

    Nun zu deiner Version. Wenn beide Raumschiffe umdrehen, wechseln beide das Bezugssystem. Die Zeit, die für die Raumschiffe beim Zusammentreffen inzwischen vergangen ist, ist die Summe der Zeiten, die sie in ihren Bezugssystemen verbracht haben. Aus Gründen der Symmetrie sind beide gleich langsam gealtert. Nur ich, der auf der Erde zurück geblieben bin, bin gegenüber den beiden älter geworden. Da aber der Treibstoff ausgesprochen teuer ist, wird ein Raumschiff vom Projekt gestrichen und es werden, nur die Uhren zwischen mir und dem Raumschiff verglichen. 😉

  92. #92 Albrecht Storz
    27. Juli 2018

    Karl-Heinz
    25. Juli 2018

    Wenn es Dir wichtig ist:

    Ich habe vereinfacht, abschnittsweise linear, aufsummiert:

    Flugzeit: 2 Jahre
    Absendepunkt: 1 Lichtjahr Entfernung
    Also braucht das Licht bis in 1 LJ Entfernung 1 Jahr. Damit ist das andere Raumschiff wieder 0,5 LJ weiter. Für diese 0,5 LJ braucht das Licht wiederum 0,5 J. Wenn es an diesem Punkt ankommt ist das Raumschiff wieder 0,25 LJ weiter weg, …

    2 Jahre + 1 Jahr + 0,5 Jahre + 0,25 Jahre ….

    Ergebnis: 4 Jahre

    Aber nochmal: ich halte diesen Punkt für absolut unwichtig und im Zusammenhang irrelevant. Ich habe auch nicht weiter oder vertieft darüber nachgedacht. Kann sein dass ich falsch rechne. Mir geht es um das Prinzip – nicht um das Durchrechnen eines Einzelfalles.

  93. #93 Albrecht Storz
    27. Juli 2018

    Karl-Heinz
    27. Juli 2018
    @Albrecht Storz

    Um Dich zu zitieren:

    Das kann jetzt aber nicht dein Ernst sein oder?

    Findest Du es gut so einen Wissenschaftsforum wie dies hier auf so eine Art zur Farce zu machen?

  94. #94 Karl-Heinz
    27. Juli 2018

    @Albrecht Storz

    Das kann jetzt aber nicht dein Ernst sein oder?

    Sorry, ist mir so rausgerutscht. Zumeist kommentiere ich mit dem Handy und da halte ich mich mit dem Schreiben extrem kurz und ab und zu mache ich eine Bemerkung, die andere dann nicht so witzig finden. Besser ich hätte argumentiert, dass in der Galilei-Transformation, die Geschwindigkeit c nicht c bleibt. Nochmals sorry.

  95. #95 Karl-Heinz
    28. Juli 2018

    @Albrecht Storz

    Du verwendest für deine Berechnung, so weit ich sehe, die Galilei-Transformation.
    r′ = rv t, t′ = t
    Nur bei der Geschwindigkeitsaddition von
    c+c/2 = 3/2 c schwindelst du. Du nimmst hier nur c. Überleg mal: Peter wirft einen Ball im Zug. Welche Ballgeschwindigkeit sieht Peter und welche Ballgeschwindigkeit sieht man vom Bahnhof aus. Nun nehmen wir statt des Balls einen Lichtstrahl oder wir nähern uns der Lichtgeschwindigkeit an.

    Die Schwindelei c+c/2 = c führt dazu, dass im
    Bezugsystem „A ruht und B mit 0,5c bewegt“ 3Jahre und im Bezugsystem „A mit -0,5c bewegt und B ruht“ 4 Jahre Laufzeit berechnet werden, was ein Widerspruch ist, da ja dasselbe rauskommen soll.

    Aber nochmal: ich halte diesen Punkt für absolut unwichtig und im Zusammenhang irrelevant. Ich habe auch nicht weiter oder vertieft darüber nachgedacht. Kann sein dass ich falsch rechne. Mir geht es um das Prinzip – nicht um das Durchrechnen eines Einzelfalles.

    Mit dem Durchrechnen im Einzelfall prüfe ich das ganze auf Plausibilität. Wie du gerade siehst, hat sich das in diesem Fall bewährt. 😉

  96. #96 Karl-Heinz
    28. Juli 2018

    @Albrecht Storz

    Wenn du anstatt der Galilei-Transformation die Lorentztransformation benutzt und für die Addition der Geschwindigkeit die Formel v= (v_1+v_2)/((1+ v_1*v_2)/c^2), dann löst sich alles in Wohlgefallen auf, bis auf die Tatsache, dass die Gleichzeitigkeit verloren geht und man beim Nachdenken einem Knoten im Gehirn bekommt. Gott sei dank, haben Leute vor uns diesen Knoten gelöst. Es lohnt sich deren Gedanken nachzuvollziehen und nicht einfach aus eigener Überheblichkeit abzulehnen.

  97. #97 Alderamin
    28. Juli 2018

    @Karl-Heinz

    bis auf die Tatsache, dass die Gleichzeitigkeit verloren geht und man beim Nachdenken einem Knoten im Gehirn bekommt

    Das löst sich auch wieder auf, wenn man sich für ein Inertialsystem entscheidet und es beibehält (ruhend, dann ist es wie bei den normalen Zwillingen, oder eines der beiden outbound- oder inbound-Systeme; bei Gelegenheit mache ich nochmal so ein Bildchen).

    Was auch hilft: Lichtlaufzeit und Doppler-Effekt berücksichtigen (ähnlich wie hier). Die Zwillinge sehen ihre Uhrticks wechselseitig durch den Doppler extrem verkürzt, wenn sie aufeinander zukommen, da wird dann Zeit mehr aufgeholt, als durch die Dilatation verloren ging.

  98. #98 Albrecht Storz
    28. Juli 2018

    Karl-Heinz
    28. Juli 2018

    Du unterstellst mir “Schwindelei”, also Lüge.

    Damit bist Du für mich nicht mehr als seriös zu behandeln. Es mag ja sein, dass Du es für normal ansiehst, andere Leute IN FETTBUCHSTABEN der Lüge zu bezichtigen und danach trotzdem zu erwarten, diese Leute sollten mit Dir anständig und rücksichtsvoll verkehren – in meinen Kreisen ist das nicht so.
    Wenn mich jemand durch Unterstellungen als “Lügner” oder “Schwindler” beleidigt, ist es für mich vorbei mit dem anständigen Dialog. Ganz einfach.
    Du agierst hier ganz offensichtlich als Provokateur. Scheint wohl Dein Job zu sein.

  99. #99 Albrecht Storz
    28. Juli 2018

    Alderamin
    28. Juli 2018
    “Die Zwillinge sehen ihre Uhrticks wechselseitig durch den Doppler extrem verkürzt, …”

    Jetzt ist der Doppler-Effekt für die Zeitdilatation (mit-)verantwortlich? Aber es ist schon irgendwie klar, dass der Dopplereffekt kein relativistischer Effekt ist – also insbesondere ganz gewiss nicht Uhren unterschiedlich schnell laufen lassen kann?

    Es ist übrigens ein Leichtes, ein Szenario, streng analog zum “Zwillings-Paradoxon”, aber ohne Beschleunigungen, zu entwickeln. Ich habe das Gezeigte hier vorgestellt, da es durch die enthaltene Symmetrie besonders augenfällig die Widersprüchlichkeit der Zeitdilatation durch Relativbewegung illustriert. (Keine zwei Uhren können gleichzeitig gegenüber der jeweils andern Uhr nachgehen!)

    Ich hoffe aber inzwischen darauf, dass zum symmetrischen “Zwillings-Paradoxon” noch etwas kommt, auf das man auch eingehen kann.

  100. #100 Alderamin
    28. Juli 2018

    @Albrecht Storz

    Ja, da kommt noch was, aber derzeit hat die MoFi Vorrang. Die Rechnung ist nicht grundsätzlich anders als beim Zwillingsparadoxon mit ruhendem Zwilling, auch da sehen sich beide wechselseitig verlangsamt, aber das Inertialsystem wird gewechselt, in Deinem Beispiel sogar von beiden.

    Es mag ja sein, dass Du es für normal ansiehst, andere Leute IN FETTBUCHSTABEN der Lüge zu bezichtigen

    Hat er nicht, er hat „schwindeln“ gesagt und damit nur ausgedrückt, dass Du da eine unzulässige Annahme machst, die mit in die Rechnung eingeht. c=const. funktioniert bei Galilei nicht. Karl-Heinz ist gut im Rechnen, wenn er Dir Fehler aufzeigt, solltest Du nicht eingeschnappt sein, sondern mitdenken. Seine Ausdrucksweise ist manchmal flapsig, aber nie böse gemeint.

  101. #101 Karl-Heinz
    28. Juli 2018

    @Albrecht Storz

    Ich habe das Gezeigte hier vorgestellt, da es durch die enthaltene Symmetrie besonders augenfällig die Widersprüchlichkeit der Zeitdilatation durch Relativbewegung illustriert. (Keine zwei Uhren können gleichzeitig gegenüber der jeweils andern Uhr nachgehen!) Ich hoffe aber inzwischen darauf, dass zum symmetrischen “Zwillings-Paradoxon” noch etwas kommt, auf das man auch eingehen kann.

    Das Beispiel, welches du dir da ausgedacht hast, ist kein gutes Beispiel, was überzeugt. Führen die Zwillinge exakt die gleichen Bewegungen in entgegengesetzter Richtung aus, so ist wenn sie sich wieder treffen, der Gang bzw. Zeitunterschied null. Wird von der SRT so vorausgesagt. Das haut keinem vom Hocker.

  102. #102 Albrecht Storz
    29. Juli 2018

    Karl-Heinz
    28. Juli 2018
    “Führen die Zwillinge exakt die gleichen Bewegungen in entgegengesetzter Richtung aus, so ist wenn sie sich wieder treffen, der Gang bzw. Zeitunterschied null.”

    Ah, sehr interessant. da habe ich die Bewegungsrichtungsabhängigkeit bei der Zeitdilatation doch glatt noch nicht gekannt. ;-(

    Könnte mir das jemand mal genauer erläutern:

    Wenn sich jemand weg bewegt, altert er langsamer, wenn er sich her bewegt, schneller? Ein Link auf eine seriöse Quelle zu dieser Erkenntnis wäre natürlich zu viel verlangt, klar.

    Ich hatte doch tatsächlich aus verschiedensten Quellen die Information erhalten, dass bei gegeneinander bewegten Uhren immer die Bewegte, egal in welche Richtung sie sich bewegt, nachginge. Wie konnte nur das absolut überragende Wissen des Karl-Heinz überall so fahrlässig unterschlagen werden.

    Was kommt wohl jetzt noch alles an Ad-Hoc-Erfindungen und Phantasmagorien daher?

    Aber was bleibt, trotz allem Unsinn der einem hier ins Gesicht schlägt:

    Gegeneinander bewegte Uhren (möglich) gehen jeweils gegeneinander nach (unmöglich).

  103. #103 Albrecht Storz
    29. Juli 2018

    Alderamin
    28. Juli 2018

    Einen bekannten Fakt absichtsvoll und bewusst falsch darstellen:

    Schwindeln (verharmlosender Ausdruck dafür)
    Lügen (klarer Ausdruck dafür)

    Die Definition für beide ist identisch.

    Karl-Heinz unterstellt mir hier, dass ich lügen würde. Das ist keine Kleinigkeit. Dass Du den Karl-Heinz dabei in Schutz nimmst, spricht Bände. Und dass Karl-Heinz rechnen könnte ist an der Stelle völlig irrelevant.
    Ich fürchte, ich kann daher wohl auch von Dir keine Kommunikation mehr auf der Basis des seriösen Austausches von Argumenten erwarten.

    Oder – überzeuge mich vom Gegenteil.

  104. #104 PS
    29. Juli 2018

    @Albrecht Storz #81
    “Mein zentraler Punkt ist das Narrativ: bei einem bewegten Beobachter verginge die Zeit langsamer als bei einem ruhenden Beobachter.“

    Da haben wir etwas gemeinsam. Die populärwissenschaftlichen Erklärungen der SRT propagieren diese in Bezug auf gleichförmige Bewegungen mE irreführende Metapher seit 100 Jahren (https://homepage.univie.ac.at/franz.embacher/SRT/Zeitdilatation.html); “Jill is aging more slowly because she’s moving!” (University of Virginia, M. Fowler, Galileo and Einstein, Special Relativity: What Time is it?; https://galileo.phys.virginia.edu/classes/109.mf1i.fall03/lectures09.pdf); M. Pössel, “Von der Lichtuhr zur Zeitdilatation” in: Einstein Online Vol. 04 (2010), 1101; https://www.einstein-online.info/vertiefung/LichtuhrZeitdilatation@set_language=de.html).

    Wenn sich jemand bei gleichförmigen Bewegungen ohne gravitative Einflüsse der Ausdrucksweise bedient „A sieht den B langsamer altern“, hat er meiner Meinung nach ein irreführendes Bild der vorbeibewegten Lichtuhr, mit der in der Schule die SRT erklärt wurde, in sein wissenschaftliches Denken einfließen lassen.

    Das angebliche langsamere Vergehen von Zeit, das zB bei M. Pössel in seiner reinen Form dargestellt wird, ist der transversale Dopplereffekt, der beide Beobachter wechselseitig in gleichem Maß betrifft und nichts mit einem langsameren Altern zu tun hat. Wenn sich von mir ein Folgetonhorn entfernt, so würde ich auch nicht auf die Idee kommen zu behaupten, nun würde der Fahrer langsamer altern. Viele populärwissenschaftlich engagierte Physiker machen das aber – um den Preis endloser Missverständnisse.

    Zu einem unterschiedlichen Alter bei sich wieder begegnenden Zwillingen kommt es nur, wenn die zwischen ihnen bestehende symmetrische Situation dadurch gebrochen wird, dass sich einer von ihnen nicht gleichförmig bewegt.

    Um das Zwillingsparadoxon nicht auf Beschleunigungsvorgänge, sondern nur auf Geometrie zurückführen zu können, wird gebeamt (“Bei Alpha Centauri beamt Horst auf das entgegenkommende Raumschiff H … und kehrt damit zurück”.) Mit dem Konstrukt einer sich nicht auf Materie beziehenden Geschwindigkeitsänderung wird genau das, was verstanden werden könnte, ausgeblendet.

  105. #105 Albrecht Storz
    29. Juli 2018

    PS:

    “Oder – überzeuge mich vom Gegenteil.”

    Dazu müssten wesentliche Grundlage geklärt sein, um auf eine gemeinsame Basis der Übereinstimmung zurück zu kehren.
    Bei mindestens folgenden Punkten muss dazu Klarheit herrschen:

    A) Lichtgeschwindigkeit c=const.

    B) Zeitdilatation tritt bei gegeneinander bewegten Uhren auf, die Bewegungsrichtung ist irrelevant.

    C) Der Doppler-Effekt hat keinerlei Bedeutung in dem Zusammenhang. Der Doppler-Effekt ist ein Effekt, der rein bei der Beobachtung und damit bei dem Beobachtenden auftritt. Auf einen Sender hat der beim Empfänger beobachtete Doppler-Effekt keinerlei Auswirkung. Insbesondere geht niemals durch den Doppler-Effekt eine bei einem Sender konstant gehaltene Uhren anders.

    D) Ein Wechsel im Inertialsystem kann vorher in einem System aufgetretene, tatsächlich, faktische Veränderungen, zB den Zeitablauf darin, nicht nachträglich ungeschehen machen. Einzig denkbar wäre eine Kompensation, die vorher aufgetretenen Veränderungen müssten dann wieder begründbar rückabgewickelt werden.

  106. #106 Albrecht Storz
    29. Juli 2018

    PS
    29. Juli 2018
    “Zu einem unterschiedlichen Alter bei sich wieder begegnenden Zwillingen kommt es nur, wenn die zwischen ihnen bestehende symmetrische Situation dadurch gebrochen wird, dass sich einer von ihnen nicht gleichförmig bewegt.”

    Leider ist auch das widerlegbar. Im Unterschied zu Einstein ist für uns heute die Kommunikation über weite Entfernungen Alltag. Daher finde ich es erstaunlich, dass anscheinend kein Mensch bisher auf die Idee kam, dass auch über Entfernungen, und über Inertialsysteme hinweg, eine Kommunikation (und insbesondere ein Uhrenabgleich) möglich ist, die über den Austausch von Blinklichtsignale hinaus geht.

    Ich habe darauf in meinem ersten dargestellten Szenario hingewiesen, dass jedes Raumschiff zur identischen (jeweiligen Bord-) Zeit das Signal des anderen empfangen muss, und dass darüber Einigkeit durch Kommunikation hergestellt werden kann – egal wie lange Zeit diese Kommunikation dafür erfordert. Das digitale Übertragen von Zeitstempeln unterliegt unmöglich einer Veränderungen der numerischen Werte durch irgendwelche relativistischen, Doppler- oder sonstigen Effekte.

  107. #107 Alderamin
    29. Juli 2018

    @PS

    Das angebliche langsamere Vergehen von Zeit, das zB bei M. Pössel in seiner reinen Form dargestellt wird, ist der transversale Dopplereffekt, der beide Beobachter wechselseitig in gleichem Maß betrifft und nichts mit einem langsameren Altern zu tun hat.

    Erkläre bitte die verlängerte Lebensdauer von stratosphärischen Myonen alleine auf der Basis des Dopplereffekts.

  108. #108 Albrecht Storz
    29. Juli 2018

    PS: an “PS”

    “PS” ist ein ungeschicktes Pseudonym. Mein Kommentar #106 ging an “PS”.

    Meine Postscripten mit “PS” markiert, bezogen sich jeweils auf Vorposts von mir.

  109. #109 PS
    29. Juli 2018

    @Albrecht Storz

    A) Ja. (Ich setzte die “Lichtgeschwindigkeit” immer in Anführungszeichen, denn sie ist keine Geschwindigkeit im herkömmlichen Sinn; es geht um das Prinzip der absoluten Begrenztheit der Wirkungsausbreitung, dem zB auch Gravitationswellen unterliegen).

    B) Wenn bei gleichförmiger Relativbewegung eine “bewegte” Uhr an einer Reihe von “ruhenden” Uhren vorbeikommt, so stellt sich bei einem Vergleich der Uhrenanzeigen anlässlich der fortlaufenden Begegnungen der einen Uhr mit einer der anderen Uhren heraus, dass die Zeitanzeige der “bewegten” Uhr zunehmend hinter den Zeitanzeigen der “ruhenden” Uhren zurückbleibt. Umgekehrt bleibt die Zeitanzeige einer der “ruhenden” Uhren zunehmend hinter den Zeitanzeigen einer Reihe “bewegter” Uhren zurück.

    C) Der Doppler Effekt tritt beim Beobachtenden auf. Er hat Bedeutung, ist er doch Ausdruck von B.

    D) Von einem Wechsel des Inertialsystems bleiben die vorher in einem System aufgetretenen Veränderungen (und zwar die in B beschriebenen Uhrenbegegnungen) unberührt. Das führt den Bruch der an sich symmetrischen Zwillingingssituation herbei.

  110. #110 Alderamin
    29. Juli 2018

    @Albrecht Storz

    Ich fürchte, ich kann daher wohl auch von Dir keine Kommunikation mehr auf der Basis des seriösen Austausches von Argumenten erwarten.

    Ich habe es mir angewöhnt, kleine Nickeligkeiten im Netz zu ignorieren, sonst schaukelt sich so etwas ganz schnell hoch. Wenn der eine eine aus seiner Sicht harmlose Bemerkung macht und der andere das böswillig interpretiert und einen drauf legt, ist das Klima sofort vergiftet.

    Dass Du den Karl-Heinz dabei in Schutz nimmst, spricht Bände.

    So was zum Beispiel. Wenn ich, Karl-Heinz’ Aussage in dem Sinne interpretiere, wie ich sie verstanden habe, spricht das lediglich Bände darüber, dass ich Missverständnisse ausräumen und aufkommenden Streit schlichten will.

    Dein Beispiel durchzurechnen und ein Bild dazu zu machen, erfordert zwei Stunden Arbeit. Im Moment habe ich die nicht, ich habe ein Haufen Bilder gemacht, die würde ich gerne verarbeiten, das dauert zwei, drei Tage. Beizeiten komme ich auf das Beispiel zurück.

  111. #111 PS
    29. Juli 2018

    @Aldemarin #107
    Jeder kann in kurzer Zeit bei entsprechender Koordinatengeschwindigkeit beliebige Entfernungen zurücklegen, da für den Reiseplan die (uU auch überlichtschnelle) Eigengeschwindigkeit maßgeblich ist. Auch der Erdbeobachter hat gegenüber dem System des Myons eine hohe Eigengeschwindigkeit, nur fehlt ihm ein Ziel, an dem er – wie das Myon – dedektiert würde.

  112. #112 Alderamin
    29. Juli 2018

    @PS

    Jeder kann in kurzer Zeit bei entsprechender Koordinatengeschwindigkeit beliebige Entfernungen zurücklegen, da für den Reiseplan die (uU auch überlichtschnelle) Eigengeschwindigkeit maßgeblich ist.

    Ich verstehe nicht, was Du meinst, die Myonen sollen mit rund 20-facher Lichtgeschwindigkeit unterwegs sein, um die normale mittlere Lebensdauer, die bei 2,2 µs (1σ) liegt und bei c für 660 m reicht, auf 15 km bis zum Boden auszudehnen? Das meinst Du sicher nicht, oder?

  113. #113 PS
    29. Juli 2018

    Bei 0,995c verkürzt sich die aus der Sicht der Myonen an ihnen vorbeibewegte Reisestrecke von 10 km um 1/Gamma auf 1000m. Das Vorbeikommen des Endes dieser Strecke (= Detektion auf der Erde) sollten 25% von ihnen bei einer mittleren Lebensdauer von 2,2 Mikrosekunden erleben. 6,5km in 2,2 Mikrosekunden ergeben eine Eigengeschwindigkeit V von ca 10c (V=Gamma*v=10*0,995=9,995c).

  114. #114 Alderamin
    29. Juli 2018

    @PS

    Es schaffen 95%. Wenn ihnen die Strecke verkürzt erscheint, erscheint uns ihre Eigenzeit verlangsamt, denn es ist dieselbe Geschwindigkeit aus beider Sicht. Eine Eigengeschwindigkeit von 10c gibt’s nicht. Hätte man am CERN bemerkt, die prügeln Teilchen mit brachialer Gewalt auf 99,9999991 % c. Egal wieviel Spannung sie durchlaufen, c wird niemals erreicht und schon gar nicht überschritten.

  115. #115 PS
    29. Juli 2018

    Du meinst die Koordinatengeschwindigkeit, ich sprach von der Eigengeschwindigkeit. Die ist manchmal illustrativ, weil sie zB zeigt, dass man auch innerhalb einer Lebenszeit recht weit kommen könnte (https://en.wikipedia.org/wiki/Proper_velocity).

  116. #116 Alderamin
    29. Juli 2018

    @PS

    Komische Größe. Nie davon gehört.

  117. #117 PS
    30. Juli 2018

    Aus dem zitierten Artikel: “The coordinate speed … is generally near lightspeed, whereas proper velocity tells us how rapidly they are covering ground on traveling-object clocks. This is important for example if, like some cosmic ray particles, the traveling objects have a finite lifetime.”

    Mir geht es darum, der beliebten populärwissenschaftlichen Argumentation entgegenzutreten, Myonen würden beweisen, dass bei gleichförmigen Bewegungen eine (einseitige) Zeitdilatation eintreten würde. Wie eben erwähnt, hat auch die Erde gegenüber dem System des Myons eine Eigengeschwindigkeit in gleicher Größe. Die Verhältnisse sind symmetrisch wie der longitudinale Dopplereffekt aus Sicht des Myons und aus Sicht der Erde.

  118. #118 Karl-Heinz
    30. Juli 2018

    @PS

    Mir geht es darum, der beliebten populärwissenschaftlichen Argumentation entgegenzutreten, Myonen würden beweisen, dass bei gleichförmigen Bewegungen eine (einseitige) Zeitdilatation eintreten würde.

    Vergiss das. Warum? Weil deine jetzige. Behauptung eh falsch ist. Ich schlage vor du versuchst, dass jetzige zu verstehen. Anschließend kannst du ja überlegen welche Vorteile und Nachteile die etwas andere Definition von w = dx/dτ bringt.

  119. #119 PS
    30. Juli 2018

    @Karl-Heimz
    Lieber Karl-Heinz, es fällt schwer, an diese Zurechtweisung konstruktiv anzuknüpfen.

  120. #120 Karl-Heinz
    30. Juli 2018

    @PS

    Lieber Karl-Heinz, es fällt schwer, an diese Zurechtweisung konstruktiv anzuknüpfen.

    Besteht überhaupt Interesse konstruktiv etwas zu erörtern?

  121. #121 Karl-Heinz
    30. Juli 2018

    @PS

    Mit zwei gleich lange Rohre, kann man ebenfalls ein scheinbares Paradoxon konstruieren.
    Das eine Rohr, welches vom Durchmesser etwas kleiner ist, dringt in das andere Rohr mit der Geschwindigkeit v ein. Beide behaupten nun, dass durch die relativistische Längenkontraktion, das jeweils andere Rohr kürzer sein muss. Kannst du uns eine Erklärung dafür geben, warum beider recht haben oder vermutest du etwas ganz anderes dahinter?

  122. #122 PS
    30. Juli 2018

    Entspricht es wirklich dem Niveau dieses Blogs – und sei es auch nur im Scherz – zu erzählen, es sei möglich, dass sich die beiden Enden zweier gleich langer, relativ zueinander bewegter Maßstäbe gleichzeitig gegenüberstehen?

  123. #123 PS
    30. Juli 2018

    Lieber Karl-Heinz, niemand tut Dir etwas zu Leide, ich führe auch keine Selbstgespräche (#108, #109).

  124. #124 Karl-Heinz
    30. Juli 2018

    @PS

    Dann kläre mich doch auf. Welche Meinung vertrittst du?
    a) ja, Zeitdilatation tritt auf
    b) alles Blödsinn, es gibt keine Zeitdilatation
    c) eine andere Theorie

  125. #125 PS
    30. Juli 2018

    @Karl-Heinz
    #104 und #109 müssten für eine Verurteilung durch die Inquisition reichen.

  126. #126 Karl-Heinz
    30. Juli 2018

    @PS

    Ich kann anbieten Scheiterhaufen für 2 Personen oder Weinverkostung in der Südsteiermark 😉
    Zu was bekennst dich? a), b) oder c)?

  127. #127 PS
    30. Juli 2018

    Mit dem kulinarischen Ausblick können wir es gut sein lassen. Bekenntnisse überlassen wir den Theologen.

  128. #128 Karl-Heinz
    31. Juli 2018

    @PS

    Myon:
    mittlere Lebensdauer 2,2 * 10^(-6) Sekunden
    Geschwindigkeit v=0,995c
    Höhe 10 km
    Flugzeit bis der Boden erreicht ist = 33,3* 10^(-6) Sekunden.
    Für Myon vergeht während des Fluges durch Zeitdilatation die Zeit 3,33 *10^(-6) Sekunden.
    Zusammenhang Halbwertszeit und Lebensdauer ist Faktor ln(2) => Halbwertszeit = 1,525 *10^(-6) Sekunden. Die Zerfallskonstanten λ= 1/(2,2 * 10^(-6) Sekunden)
    Es erreichen 22% den Boden.
    Gepriesen sei die Zeitdilatation. Halleluja 😉

  129. #129 Karl-Heinz
    31. Juli 2018

    @PS

    So jetz das ganze mit proper velocity.
    Komm schon, hilf mir.

  130. #130 Karl-Heinz
    31. Juli 2018

    @PS

    Entspricht es wirklich dem Niveau dieses Blogs – und sei es auch nur im Scherz – zu erzählen, es sei möglich, dass sich die beiden Enden zweier gleich langer, relativ zueinander bewegter Maßstäbe gleichzeitig gegenüberstehen?

    Rein intuitiv würde ich sagen, dass ein Beobachter, welcher sich in der Mitte in Ruhe befindet, für einen kurzen Augenblick sehen würde, dass sich die Enden der Stäbe, die aufeinander zu rasen, gegenüberstehen. Für den speziellen Beobachter sind beide Stäbe verkürzt. Aber du kannst mir ruhig das Gegenteil beweisen. Du darfst auch proper velocity verwenden, falls du dich damit leichter tust. 😉

  131. #131 Albrecht Storz
    31. Juli 2018

    Alderamin
    29. Juli 2018

    “Dein Beispiel durchzurechnen und ein Bild dazu zu machen, erfordert zwei Stunden Arbeit.”

    Warum solltest Du das durchrechnen? Wen interessiert das denn? Du lenkst hier vom Wesentlichen ab indem Du auf den Popanz von Karl-Heinz aufsattelst. Ich habe mehrfach darauf hingewiesen, dass diese Berechnung von mir nur dahin geworfen worden ist. Sie ist völlig irrelevant. Ich habe auch nur auf penetrante Nachfrage des Provokateurs Karl-Heinz hin meinen Rechnungsgedanken erläutert – mit dem klaren Hinweis, dass diese Rechnung völlig unwichtig ist. Und ohne Zweifel fehlerbehaftet sein kann da sie nur nebenher ausgeführt wurde. Da das Ergebnis nichts zum Erkenntnisgewinn beiträgt. Denn entscheidend ist die Frage, ob gegeneinander bewegte Uhren gegeneinander nachgehen können – UND NICHT um WIE VIEL in einer konkreten Situation!

    Hat eigentlich Dein “Superrechner” Karl-Heinz bei seiner Berechnung über die Ankunftszeit des Signals (ich glaube, es hieß “so wird das Signal vom anderen nach genau 3,4641 Jahren empfangen”) bei der Laufzeit des Signals die fortschreitende Entfernung berücksichtigt?
    Nein?
    Aber auf meiner Überschlagsrechnung herumhacken.
    Das hat System.
    Und natürlich gehst Du auch nicht auf mein Angebot ein, die gemeinsamen Grundlagen zu definieren. Denn klare Verhältnisse sind des Dogmatikers Tod.
    Und nein: es interessiert mich nicht, ob der Superrechner Quark zusammen rechnet. Ich weise nur auf die hier vorherrschende Arroganz und den Bias, der hier praktiziert wird, hin.

    Nochmal, hierum geht es:

    Gegeneinander bewegte Uhren (das ist offensichtlich möglich) gehen jeweils gegeneinander nach (wie soll das möglich sein?).

    Um nichts anderes!

  132. #132 Albrecht Storz
    31. Juli 2018

    Alderamin,

    natürlich war mein Kommentar #105 auf Deinen Kommentar #100 bezogen, und nicht etwa auf einen “PS” gemünzt, von dem ich vorher überhaupt nichts gewusst hatte, und der leider an dieser ungeschickten Stelle (was ein Zufall aber auch) reingegrätscht ist.

    Ich wiederhole den Wortlaut, Adressat dafür ist ausdrücklich Alderamin:

    ****
    Dazu müssten wesentliche Grundlage geklärt sein, um auf eine gemeinsame Basis der Übereinstimmung zurück zu kehren.
    Bei mindestens folgenden Punkten muss dazu Klarheit herrschen:

    A) Lichtgeschwindigkeit c=const.

    B) Zeitdilatation tritt bei gegeneinander bewegten Uhren auf, die Bewegungsrichtung ist irrelevant.

    C) Der Doppler-Effekt hat keinerlei Bedeutung in dem Zusammenhang. Der Doppler-Effekt ist ein Effekt, der rein bei der Beobachtung und damit bei dem Beobachtenden auftritt. Auf einen Sender hat der beim Empfänger beobachtete Doppler-Effekt keinerlei Auswirkung. Insbesondere geht niemals durch den Doppler-Effekt eine bei einem Sender konstant gehaltene Uhren anders.

    D) Ein Wechsel im Inertialsystem kann vorher in einem System aufgetretene, tatsächlich, faktische Veränderungen, zB den Zeitablauf darin, nicht nachträglich ungeschehen machen. Einzig denkbar wäre eine Kompensation, die vorher aufgetretenen Veränderungen müssten dann wieder begründbar rückabgewickelt werden.

    ****

    Aber natürlich hast Du jede Freiheit, weitere Kommunikation mit mir zu unterlassen oder durch Beharren auf Unwesentlichkeiten und Superrechner-Querschüsse zu sabotieren – wenn Dir nichts an der Klärung der Fragen liegt … Dein Entscheidung …

  133. #133 Karl-Heinz
    31. Juli 2018

    @Albrecht Storz

    Hat eigentlich Dein “Superrechner” Karl-Heinz bei seiner Berechnung über die Ankunftszeit des Signals (ich glaube, es hieß “so wird das Signal vom anderen nach genau 3,4641 Jahren empfangen”) bei der Laufzeit des Signals die fortschreitende Entfernung berücksichtigt?
    Nein?
    Aber auf meiner Überschlagsrechnung herumhacken.
    Das hat System.

    Ich sehe schon, dass du von mir Nachhilfe benötigst.
    Gegeben sind zwei Objekte A und B, die gegeneinander die relative Geschwindigkeit v besitzen.
    Wie du sicherlich weißt, gibt es keine absolute Bewegung im Raum.

    Objekt B möge ein Signal nach A schicken. Wenn du jetzt klassisch rechnest, bist du sofort mit diesem Problem konfrontiert.

    B in Ruhe, A in Bewegung: für die Laufzeit müsste neben der momentanen Entfernung das Entfernen von A während der Lichtlaufzeit berücksichtigt werden.
    A in Ruhe, B in Bewegung: Es muss nur die Entfernung zu A berücksichtigt werden.
    Hier entsteht ein Widerspruch, wenn man annimmt, dass es keine absolute Bewegung im Raum gibt.

  134. #134 Alderamin
    31. Juli 2018

    @Albrecht Storz

    Warum solltest Du das durchrechnen? Wen interessiert das denn? Du lenkst hier vom Wesentlichen ab indem Du auf den Popanz von Karl-Heinz aufsattelst.

    Damit Dir klar wird, dass in jedem Inertialsystem, auch in Deinem Beispiel, die Uhren der anderen, dagegen bewegten Inertialsysteme langsamer gehen. Ohne Berechnung und Diagramm, nur auf der Basis “gesunden galileischen Menschenverstands” ist das nicht möglich. Mit den drei Artikeln zum Zwillingsparadoxon hatte ich genau das beabsichtigt, an einem Beispiel vorzurechnen, dass kein Paradoxon besteht, alles ist schlüssig, wie sich mit praktischen Zahlen und Mittelstufenmathematik nachvollziehen lässt (ganz unabhängig von Karl-Heinz’ Rechnungen). Das gilt selbstverständlich auch für Dein Beispiel.

    Aber natürlich hast Du jede Freiheit, weitere Kommunikation mit mir zu unterlassen oder durch Beharren auf Unwesentlichkeiten und Superrechner-Querschüsse zu sabotieren – wenn Dir nichts an der Klärung der Fragen liegt … Dein Entscheidung …

    Die Frage ist aus Sicht der Relativitätstheorie eindeutig geklärt, es ging mir nur darum, dass Du das nachvollziehen kannst. Jemandem, der mich deswegen fortgesetzt beschimpft, noch weitere Zeit zu opfern, dazu fehlt mir allerdings in der Tat die Lust. Danke für das Gespräch.

  135. #135 Albrecht Storz
    31. Juli 2018

    ” der mich deswegen fortgesetzt beschimpft,”

    Oh, wer macht das denn? Du bist doch hoffentlich nicht mit dem Provokateur Karl-Heinz identisch, den ich wohlweislich genau so nenne wie er es verdient.

    Okay. Mit dieser Volte ist jetzt sonnenklar, dass hier nichts substantielles zu erwarten ist. Also fange ich mal an aufzuräumen, bevor der Deckel drauf ist.

    in #5 schreibt Alderamin: “Ein Zeitabgleich kann aber nur am gleichen Ort erfolgen, …”

    Auf meine Frage für eine Begründung dafür antwortet Alderamin in #22 mit: “Weil man sich nicht einigen kann, welche Ereignisse gleichzeitig sind, wenn man sich an verschiedenen Orten und mit relativistischer Geschwindigkeit zueinander bewegt.”

    Nun habe ich in meinem Szenario in #37 beschrieben, wie zwei sich relativ zueinander bewegende Raumschiffe über die Gleichzeitigkeit von Ereignissen synchronisieren können. Nach langem Geplänkel wird dann zugestanden, dass beide Raumschiffe das Signal des jeweils anderen Raumschiffes zu jeweils gleichen Bordzeiten empfangen müssen.

    Etwa #74 etwas verdruckst (“Karl-Heinz hat schon gesagt, …”) und dann: “Die Behauptung ist vielmehr, dass jeder das Signal des anderen nach 3,46 Jahren seiner Zeit beobachten wird. Und jeder wird das Signal nach 2 Jahren seiner Zeit absetzen …”

    Unabhängig von irgendwelchen Zahlenwerten ist damit meine Ausgangsthese bestätigt:

    Wenn in einem der Raumschiffe das Signal wie abgesprochen abgesendet wird, kann man dort auch davon ausgehen, dass in genau dem gleichen Moment (gleicher Wert auf beiden Borduhren) dies auch auf dem anderen Raumschiff geschieht.

    Wenn auf einem der Raumschiffe das Signal des anderen Raumschiffs empfangen wird, kann man davon ausgehen, dass im gleichen Moment (gleicher Wert auf beiden Borduhren) dies auch auf dem anderen Raumschiff geschieht.

    Da bis hierher Einigkeit herrscht kann über folgendes wohl kaum eine Meinungsdifferenz bestehen:
    Natürlich kann man ebenso vereinbaren, so ein Signal zB jede Sekunde zu schicken. Ganz offensichtlich können sich die beiden Raumschiffe damit laufend synchron halten. Sie müssen nur die sich verändernden Signallaufzeiten berücksichtigen. Sonst nichts.

    Damit ist die Behauptung von #5 im vollen Umfang widerlegt.

    Offensichtlich können sich zwei Raumschiffe, die sich zueinander mit Geschwindigkeiten nahe c bewegen bestens darüber verständigen, dass bei ihnen die Zeit gleich schnell vergeht.

    [An Alderamin persönlich: ich habe Dich bisher als netten Kerl wahrgenommen, warum machst Du Dich mit solchen Figuren wie Karl-Heinz gemein? Das musst Du nicht.]

  136. #136 Karl-Heinz
    31. Juli 2018

    @Albrecht Storz

    Sag mal Albrecht, was ist denn in dich gefahren? Alles OK mit dir.

    Wenn du willst können wir uns ja auch aus den Weg gehen. Das mache ich am Einfachsten so, dass ich deine Beiträge einfach ignoriere, sprich einfach nicht mehr lese.
    Willst du das?

  137. #137 Albrecht Storz
    31. Juli 2018

    “Sag mal Albrecht, was ist denn in dich gefahren? ”

    sprach der Provokateur (zB #95, #96, #120, #133, …) und grinste sich eins …

    Solche Leute vergiften alles. Was bleibt, trifft sich am Ende auf Kindergartenniveau.
    Eine Schande, solchen Leuten auf einem sich als wissenschaftlich orientiert darstellenden Forum Bestätigung für ihr Tun zu liefern.

  138. #138 Karl-Heinz
    31. Juli 2018

    @Albrecht Storz

    Ich versuche mal nicht beleidigend zu sein.
    Ich würde gerne deine Meinung zu dem Beispiel unten hören.

    Gegeben sind zwei Objekte A und B, die gegeneinander die relative Geschwindigkeit v besitzen.
    Wie du sicherlich weißt, gibt es keine absolute Bewegung im Raum.

    Objekt B möge ein Signal nach A schicken. Wenn du jetzt klassisch rechnest, bist du sofort mit diesem Problem konfrontiert.

    B in Ruhe, A in Bewegung: für die Laufzeit müsste neben der momentanen Entfernung das Entfernen von A während der Lichtlaufzeit berücksichtigt werden.
    A in Ruhe, B in Bewegung: Es muss nur die Entfernung zu A berücksichtigt werden.
    Hier entsteht ein Widerspruch bei der klassischen Berechnung, wenn man annimmt, dass es keine absolute Bewegung im Raum gibt.

    Was sagen sie dazu?

  139. #139 Alderamin
    31. Juli 2018

    @Albrecht

    Eine Schande, solchen Leuten auf einem sich als wissenschaftlich orientiert darstellenden Forum Bestätigung für ihr Tun zu liefern.

    Da das Beleidigen weitergeht, kleine Erholungspause für Dich. Wie sich jemand wegen einer kleinen Spitzfindigkeit so ereifern kann, übersteigt mein Nachvollziehungsvermögen.

  140. #140 Spritkopf
    31. Juli 2018

    @Storz #135

    ” der mich deswegen fortgesetzt beschimpft,”

    Oh, wer macht das denn?

    Sie! Gleich im nächsten Satz:

    Du bist doch hoffentlich nicht mit dem Provokateur Karl-Heinz identisch, den ich wohlweislich genau so nenne wie er es verdient.

    Karl-Heinz haben Sie schon in einem früheren Post (#131) einen Provokateur genannt. Wofür eigentlich? Und dafür, dass er sich in #55 ff. redlich Mühe gegeben hat, Ihnen Ihren Irrtum auseinanderzuklamüsern, betiteln Sie ihn höhnisch als “Superrechner”.

    Alderamin fängt sich im gleichen Zuge noch eine Beschimpfung als Dogmatiker ein.

    Kritik darf anscheinend auch nicht an Ihren Rechnungen geübt werden, das ist bei Ihnen “systematisches Herumhacken”.

    Es fällt nebenbei auf, dass Sie dieser Mischung aus Beschimpfung und passiv-aggressiver Opferhaltung nicht nur hier pflegen.

  141. #141 Karl-Heinz
    31. Juli 2018

    Ein gutes Beispiel ist der LHC
    Dort laufen zwei Hadronenstrahlen in entgegengesetzter Richtung um. Wenn ich mich zum Rohr stelle und ein Teilchen kommt vorbei, dann ist für das Teilchen nach einem Umlauf weniger Zeit vergangen, als für mich. Die beiden Strahlen, die entgegengesetzt umlaufen, werden so ziemlich die gleiche Geschwindigkeit haben nur halt in entgegengesetzter Richtung. Jetzt betrachte ich zwei Teilchen, eines vom Strahl A und vom eines vom entgegengesetzten Strahl B, welches gerade bei mir vorbeikommt. Nach einem Umlauf kommen die beiden Teilchen wieder bei mir vorbei, mit anderen Worten ich und die beiden Teilchen sind (fast) am gleichen Ort. Zwischen den beiden Teilchen ist während des Umlaufs die gleiche Zeit vergangen. In Bezug aber auf mich sind die beiden Teilchen jünger geblieben. Hat man das Prinzip einmal durchschaut, so kann man schwuppdiwupp ohne viel Aufwand die verstrichene Zeit für alle 3 Beteiligten berechnen. Man kann auch begründen warum Teilchen vom Strahl A gleich schnell altern, wie vom Strahl B.

  142. #142 Karl-Heinz
    31. Juli 2018

    @Spritkopf

    Danke 😉

    Mir sind da schon Gedanken durch den Kopf geschossen.
    “Was habe ich da wieder angestellt. Bin ich der Hauptschuldige?”

  143. #143 Albrecht Storz
    1. August 2018

    “kleine Erholungspause für Dich.”

    War ja klar: Deckel drauf. Was man hier offensichtlich nicht haben kann sind Argumente und Unbeugsamkeit.

    Mir wird “Beleidigen” vorgeworfen. Lächerlich. Ich wurde hier als Lügner beleidigt. Das ist die einzige objektiv stattgefundene Beleidigung hier.

  144. #144 Peter
    FFM
    1. August 2018

    @Karl-Heinz und die Myonen.
    Eine Frage dazu: Sie haben eine Zeitdilatation der Myonen-Lebenszeit berechnet, aber die Längenkontraktion ihres Weges nicht. Zeitdilatation und Längenkontraktion treten aber immer gemeinsam auf. D.h. Ihre Angabe der Höhe=10km wird sich auf wenige hundert Meter verkürzen. Richtig oder falsch?

  145. #145 Karl-Heinz
    1. August 2018

    @Peter

    Ich habe die Flugzeit des Myons zuerst in meinem Inertialsystem berechnet und dann auf die Zeit geschlossen, die für das Myon vergangen sein muss.

    Eine zweite Möglichkeit wäre, dass ich die Flugzeit aus Sicht des Myons berechne.
    Relativgeschwindigkeit: 0,995c (wird von beiden Inertialsystemen gemessen)
    Für das Myon ist die Strecke von 10km mit 10km/γ verkürzt. Das sind in etwa 998,75 Meter. Damit ergibt sich ein Flugzeit für das Myon in seinem Inertialsystem von (s=v*t) –> t = s/v = 3,33 * 10^(-6) Sekunden.

    Du kannst ruhig DU zu mir sagen.

  146. #146 Jolly
    1. August 2018

    Das Lügenparadoxon – Aristoteles-frei.

    “Ich wurde hier als Lügner beleidigt”

    Das ist gelogen.

  147. #147 Karl-Heinz
    1. August 2018

    @Albrecht Storz

    Sie haben mich gründlich missverstanden. Ich bitte sie daher, dies auch so zu sehen.

  148. #148 Alderamin
    1. August 2018

    @Albrecht Storz

    Mir wird “Beleidigen” vorgeworfen. Lächerlich. Ich wurde hier als Lügner beleidigt. Das ist die einzige objektiv stattgefundene Beleidigung hier.

    Jemandem zu sagen, er schwindle in seiner Formel ist gänzlich was anderes als ihn einen Lügner zu nennen. Nur wer außer Schwarz und Weiß keine Grautöne kennt, oder generell kritikunfähig ist, kann das so auslegen. Lies mal die Kommentare von anderen wohlbekannten Kommentatoren auf ScienceBlogs, die sind da wesentlich direkter (nicht dass ich den Ton hier gutheißen würde).

    War ja klar: Deckel drauf. Was man hier offensichtlich nicht haben kann sind Argumente und Unbeugsamkeit.

    Ich habe niemals und werde auch niemals jemanden wegen Argumenten und “Unbeugsamkeit” (was auch Beratungsresistenz einschließt) sperren, sondern nur wegen fortgesetzter Rumnöhlerei:

    Du agierst hier ganz offensichtlich als Provokateur. Scheint wohl Dein Job zu sein.

    Ein Link auf eine seriöse Quelle zu dieser Erkenntnis wäre natürlich zu viel verlangt, klar.

    Was kommt wohl jetzt noch alles an Ad-Hoc-Erfindungen und Phantasmagorien daher?

    Aber was bleibt, trotz allem Unsinn der einem hier ins Gesicht schlägt:

    Dass Du den Karl-Heinz dabei in Schutz nimmst, spricht Bände.

    Ich fürchte, ich kann daher wohl auch von Dir keine Kommunikation mehr auf der Basis des seriösen Austausches von Argumenten erwarten.

    Wen interessiert das denn? Du lenkst hier vom Wesentlichen ab indem Du auf den Popanz von Karl-Heinz aufsattelst

    Dein “Superrechner”

    Aber auf meiner Überschlagsrechnung herumhacken.
    Das hat System.

    Denn klare Verhältnisse sind des Dogmatikers Tod.

    Und nein: es interessiert mich nicht, ob der Superrechner Quark zusammen rechnet. Ich weise nur auf die hier vorherrschende Arroganz und den Bias, der hier praktiziert wird, hin.

    Du bist doch hoffentlich nicht mit dem Provokateur Karl-Heinz identisch, den ich wohlweislich genau so nenne wie er es verdient.

    Okay. Mit dieser Volte ist jetzt sonnenklar, dass hier nichts substantielles zu erwarten ist.

    Eine Schande, solchen Leuten auf einem sich als wissenschaftlich orientiert darstellenden Forum Bestätigung für ihr Tun zu liefern.

    Reicht das als Begründung? Meine Erfahrung, die sich hier wieder bestätigt hat, ist dass diejenigen, die am heftigsten austeilen, selbst am sensibelsten reagieren.

    In diesem Ton diskutiere ich aber nicht mit niemandem, egal, was er für Argumente hat oder zu haben meint. Deswegen Zeit für Dich, Adrenalin abzubauen.

  149. #149 Peter
    1. August 2018

    @Karl-Heinz
    Vielen Dank für Deine Antwort. Ich habe aber weitere Fragen 🙂
    Das Myonen-Experiment ist symmetrisch, d.h. aus Sicht der Erde vergeht die Zeit für die Myonen langsamer, aus Sicht der Myonen geht die Zeit auf der Erde langsamer. Warum wird die Sicht der Erde bevorzugt?

  150. #150 Niels
    2. August 2018

    @Alderamin @PS

    Eigengeschwindigkeit

    Komische Größe. Nie davon gehört.

    Ich auch nicht. Klasse.
    Und ich dachte, ich hätte so langsam alle Definitionen in der Relativitätstheorie gesehen. 😉
    Vielen Dank für den Link.

    Ich kenne auch keine einzige andere Größe in der Physik, für deren Definition man die Messungen mehrerer Beobachter benötigt.
    Sehr seltsame Idee. Für mich persönlich auch ziemlich das Unanschaulichste und Absurdeste, das man sich vorstellen kann. 🙂

  151. #151 Alderamin
    2. August 2018

    @Peter

    Man muss sich für eine Sicht entscheiden. Aus unserer Sicht entstehen die Myonen in 10-15 km Höhe und legen die entsprechende Strecke in 33-50 μs zurück. Für die Myonen vergehen 3-5 μs und sie legen aus ihrer Sicht 1-1,5 km zurück (Pi mal Daumen, ohne exakte Rechnung, nur aus dem Bauch).

    Aus Sicht der Myonen geht auch die Zeit der Erde langsamer, für sie zeigte eine Uhr am Erdboden aber bei ihrer Entstehung eine spätere Zeit an als für uns, weil Gleichzeitigkeit an verschiedenen Orten gemäß RT nicht erhalten ist. Erst beim Eintreffen am Erdboden sind die imaginäre Uhr der Myonen und die am Boden am gleichen Ort und direkt vergleichbar.

    Was die Erduhr aus Myonensicht zu Beginn zeigte, kann man über den Lorentzfaktor für ihre Geschwindigkeit und dem Zeitpunkt ihres Eintreffens am Boden nach Erdzeit zurückrechnen. Genau wie oben im Beispiel des Zwillingsparadoxons.

  152. #152 Peter
    2. August 2018

    @Alderamin
    “Man muss sich für eine Sicht entscheiden” – damit wäre das Relativitätsprinzip verletzt!

    Ein Gedankenexperiment:

    1. Vorbedingung
    Ein Myon (M1) im Labor wird auf v=0 gebremst. Zur gleichen Zeit passiert ein Myon (M2) die 10km-Grenze in Richtung M1.

    2. Ablauf
    Die beiden Myonen M1 und M2 bewegen sich mit einer bestimmten Geschwindigkeit aufeinander zu. Welches der beiden ist noch nicht zerfallen beim Zusammentreffen?

    3. Ergebnis
    Aus Sicht von M1 geht die Uhr von M2 langsamer, M1 ist also zerfallen, M2 “überlebt” und erreicht M1.
    Aus Sicht von M2 geht die Uhr von M1 langsamer, M2 ist also zerfallen und M1 überlebt.

    4. Fazit
    So lange Symmetrie herrscht, ist es ein Paradoxon, welches nicht aufgelöst werden kann. Das Zwillingsparadoxon, ist gar kein Paradoxon, da ein Zwilling das Bezugssystem wechselt und die Symmetrie bricht!

  153. #153 Alderamin
    2. August 2018

    @Peter

    Nein, das ist identisch mit dem Vergleich der Systeme A und H“ oben: A ist der Erdboden. Die 10 km entsprechen den 8,6 LJ, die M2 „zur gleichen Zeit“ passiert wie M1. Das Problem ist: für M2 bedeutet „zur gleichen Zeit“ etwas völlig anderes, seine Gleichzeitigkeitslinie wäre in einem Minkowski-Diagramm gegen die Waagerechte verkippt (um das deutlich zu machen, taugen sie wirklich mehr). In meinem Bild oben ist der Abflug von Horst von der Erde aus im System H“ viel früher, bei einer größeren Distanz. Entsprechend entstünde M1 aus Sicht von M2 viel früher, bei einer größeren Entfernung als 10 km. Mach‘ Dir den Unterschied zwischen Bild 1 und 3 oben klar, dann siehst Du, dass die Situation identisch ist. Weil „zur gleichen Zeit“ was anderes für beide Systeme ist, können beide konsistent den anderen langsamer altern sehen (an dieser mentalen Hürde scheiterte auch Albrecht).

    Es gibt ähnlich zu dieser Situation, aber auf die Längenmontraktion (statt wie hier der Zeitdilatation) das sogenannte „Garagenparadoxon“: Ein fast lichtschnelles Auto rast in eine Garage mit Toren an jedem Ende hinein. Aus Sicht eines ruhenden Beobachters passt das längenverkürzte Auto so eben gerade in die Garage, und die Türen können beide simultan geschlossen werden, wenn das Auto drinnen ist. Aus Sicht des Autos erscheint die Garage hingegen verkürzt und es passt nicht hinein, also können die Türen nicht gleichzeitig geschlossen werden. Wie löst man dieses Paradoxon? Indem das gleichzeitige Schließen im Ruhesystem der Garage ein Nacheinanderschließen aus Sicht des bewegten Autos ist. Gleichzeitigkeit ist eben auch relativ.

  154. #154 Peter
    2. August 2018

    @Alderamin
    Gut, dann frage ich mal einfacher: zwei Myonen im Labor bewegen sich aufeinander zu. Welches der beiden existiert noch, wenn sie (in der Raumzeit) aufeinander treffen?

  155. #155 Alderamin
    3. August 2018

    @Peter

    Lässt sich ohne weitere Angaben so nicht beantworten, dazu braucht es Zeit und Ort der Entstehung aus der Sicht eines Systems und die Geschwindigkeit der Myonen relativ zueinander (und womöglich relativ zum Beobachter, wenn dieser ein Dritter in einem eigenen System ist).

    Die Konstruktion jeglicher Paradoxa scheitert daran, dass man zwei gegeneinander bewegte Inertialsysteme nur einmal zeitlich und räumlich zusammenbringen kann. Ein Ende eines jeglichen Zeitintervalls bleibt immer offen und wird in den Inertialsystemen ganz unterschiedlich wahrgenommen. Siehe Bilder 1 und 3 oben.

  156. #156 Karl Mistelberger
    3. August 2018

    Nebenbei bemerkt: Früher (Anfang der 70er) war Physik noch nicht so langweilig wie diese endlosen Diskussionen des Zwillingsparadoxons.

    Übungsaufgabe: Die Polizei stoppt einen Autofahrer, der bei Rot über eine Kreuzung gefahren ist. Der Autofahrer redet sich damit heraus, dass er etwas schneller unterwegs gewesen sei und die Ampel ihm deswegen Grün gezeigt hätte. Diskutieren Sie den Vorgang …

  157. #157 Karl-Heinz
    3. August 2018

    @Karl Mistelberger

    Habe ich schon durch gerechnet. 😉

    Nach dem Aufstehen habe ich auf die Uhr geguckt und festgestellt, dass das Myon, welches ich heute zum Frühstück erwarte, meine 10 km Sphäre erreicht hat. Nach 33,3’µs diskutiere mit dem Myon über das Ereignis “meine 10 km Sphäre zu durchqueren”. Wir kommen zu der Erkenntnis, dass das Ereignis “meine 10 km Sphäre zu durchqueren” von meiner Sicht aus nicht das gleiche Ereignis ist, wie aus der Sicht meines Gastes, dem Myon. Würde man die beiden Ereignisse vergleiche, wäre das genau so, als wenn man zwei verschiedene Uhren vergleicht.
    Berechnung sind so um die die 4 Zeilen. Falls Interesse besteht, würde ich diese Berechnung posten.

  158. #158 Karl-Heinz
    3. August 2018

    @Karl Mistelberger
    Auch du könntest, das Beispiel mit dem Myon durchrechnen, dann könnten wir die Ergebnisse vergleichen.
    Das größte Problem bei mir waren die Vorzeichen.(+/-) 😉

  159. #159 Peter
    3. August 2018

    @Alderamin
    – Es gibt keinen weiteren Beobachter
    – Die beiden Myonen bewegen sich mit der “Standard”-Geschwindigkeit von 0,9995 aufeinander zu.
    – Die Strecke beträgt 10 km

  160. #160 Karl-Heinz
    3. August 2018

    @Peter

    Das Problem sind die 10km. Für welches Myon gilt den diese 10km Distanz. Du hast, dewegen eine Unsymmetrie. Folgerungen, die mit Symmetrie argumentieren, müssen daher nicht richtig sein. 😉

  161. #161 Peter
    3. August 2018

    @Karl-Heinz
    Die 10 km gelten für beide.
    Parallel können wir ja ein echtes Zwillingsparadoxon betrachten: zwei Raumfahrerzwillinge starten vom gleichen Punkt in der Raumzeit entgegengesetzt mit gleicher Geschwindigkeit, kehren um und treffen sich in der Raumzeit wieder am selben Punkt. Sie vergleichen ihre Uhren: Welche geht nach?

  162. #162 Karl-Heinz
    3. August 2018

    @Peter
    Bei den 10 km musst du dich entscheiden für wen dies gilt. Natürlich kannst du das ganze umdrehen und die 10km für den anderen nehme. Du musst jetzt aber beachten, dass das jetzt zwei unterschiedliche Fälle sind, die getrennt berechnet und betrachtet werden müssen.

  163. #163 Peter
    3. August 2018

    @Karl-Heinz
    es wird von einem dritten Beobachter eine Strecke von 10 km abgesteckt, an deren Enden jeweils ein Myon erzeugt werden soll. Er drückt in der Mitte einen Schalter, welches ein Signal zu den beiden 5 km entfernten Myonen-Erzeugern schickt. M1 wird ruhend (im Bezugssystem des dritten Beobachters) und M2 auf M1 zubewegt (im Bezugssystem des dritten Beobachters) erzeugt (gehen wir davon aus, dass die Erzeugung technisch bedingt jeweils genau gleich lang dauert, da dieselben Erzeuger genutzt werden). Nachdem der dritte Beobachter das Signal abgesandt hat, scheidet er aus der Betrachtung aus.

  164. #164 Spritkopf
    3. August 2018

    @Peter
    So wie du das beschreibst, brauchst du noch nicht mal einen dritten Beobachter, weil dieser sich im gleichen Inertialsystem wie das ruhende Myon M1 befindet. Heißt, dass die Strecke von 10 km für M1 gilt, aber für M2 nicht.

  165. #165 Karl-Heinz
    3. August 2018

    @Spritkopf
    Genau … 😉

    So jetzt werden wir mal ein bisschen Mathematik auspacken. Wir überlegen uns wann M1 und M2 im Inertialsystem M1 sterben werden. Die beiden werden, wenn mal von der Quantenmechanik absieht, nicht zur gleichen Zeit und am gleichen Ort sterben, sprich das Sterbe-Ereignis der beiden können nicht gleich sein. Das gilt für jedes beliebige Inertialsystem.
    Zum Beispiel können beide in einem bestimmten Inertialsystem zur gleichen Zeit sterben, da aber die Ergebnisse verschieden sind, muss dies am unterschiedlichen Ort geschehen.

    @Peter
    Das ganze ist schon verrückt, oder?

  166. #166 Karl-Heinz
    3. August 2018

    Zwei Ereignisse E1 und E2 in einem Inertialsystem sind genau dann gleich, wenn sie zur gleichen Zeit und am gleichen Ort stattfinden.

    Wenn man mit jemandem einen Treffpunkt ausmacht, benötigt man dafür eine Ortsangabe und die Zeit. Die Koordinaten dazu sind E(x0,y0,z0,t0) in Bezug auf ein Inertialsystem I.

  167. #167 Peter
    3. August 2018

    @Spritkopf
    Woher wissen denn die Myonen M1 und M2 welche Strecke für sie gilt? Dies scheint eine willkürliche Festlegung eines Beobachters (hier: Spritkopf) und damit eine Verletzung des Relativitätsprinzips zu sein!

  168. #168 Alderamin
    3. August 2018

    @Peter

    Wenn sich die Strecke relativ zum Beobachter bewegt, ist sie verkürzt. Ruht die Strecke relativ zum Beobachter, ist sie es nicht. (Ein schnell fliegendes Raumschiff erscheint dem ruhenden Beobachter wiederum verkürzt – das ist auch eine relativ zu ihm bewegte Strecke!).

    Da ja (wie in meinen Artikeln dargelegt) die Relativgeschwindigkeit beiden Beobachtern gleich groß erscheint (bis aufs Vorzeichen), muss dem gegenüber der Strecke bewegten Beobachter, für den weniger Zeit vergeht, die zurückgelegte Strecke kürzer erscheinen wegen s=v*t.

  169. #169 Peter
    3. August 2018

    @Alderamin
    Aus Sicht von M1 bewegt sich M2. Aus Sicht von M2 bewegt sich M1. Und beide “messen” aus ihrer Sicht dasselbe Vorzeichen: das jeweils andere Myon kommt entgegen! Es gibt kein ausgezeichnetes System, egal, wie man es dreht oder wendet.

    Nur noch mal zur Erinnerung:
    Zitat Karl-Heinz: “Bei den 10 km musst du dich entscheiden für wen dies gilt.”
    Zitat Spritkopf: “Heißt, dass die Strecke von 10 km für M1 gilt, aber für M2 nicht.”

    Beide verletzen das Relativitätsprinzip, denn es gibt kein ausgezeichnetes Bezugssystem!

  170. #170 Karl-Heinz
    3. August 2018

    @Peter

    Im Raum befindet sich ein Stab.
    Das vordere Ende P1 hat die Koordinaten P1( x=1, y=1, z=1). Das hintere Ende hat die Koordinaten P2( x=2, y=2, z=2). Wie groß ist seine Länge? Wenn du das beantwortest, dann kommt die nächste Frage, die ich jetzt noch nicht verraten will. 😉

  171. #171 Peter
    3. August 2018

    @Karl-Heinz
    Möchtest Du mir nun den Satz des Pythagoras beibringen? 🙂

  172. #172 Karl-Heinz
    3. August 2018

    @Peter

    Nö,
    Ich will auf was bestimmtes hinaus. 😉
    Also, wie groß ist die Länge?

  173. #173 Peter
    3. August 2018

    Wurzel(3)
    Jetzt bin ich dran:
    Gegeben sei die Grammatik G = ({0, 1, #}, {S, T, U}, P, S) mit
    P = {S → 0T1 | 1U0,
    T → 0T0 | 1T1 | #
    U → 1U0 | 0T1 | #1}
    Frage: Von welchem Typ ist die Grammatik G?

  174. #174 Karl-Heinz
    3. August 2018

    @Peter
    Tust du Compiler programmieren?
    Nix verraten. Ich muss erst mal nachsehen was eine Grammatik ist, das ganze wird aber dauern . 😉

    So jetz lasse ich den Stab in x-Richtung mit der Geschwindigkeit v wandern.
    P1(x=1+v*t,y=1,z=1)
    P2(x=2+v*t,y=2,z=2)
    Wie groß ist jetzt seine Länge?

  175. #175 Peter
    3. August 2018

    Sorry Karl-Heinz, wir haben Besuch bekommen, muss nun weg (wir wollen Essen gehen). Ich antworte Morgen – ok?

  176. #176 Karl-Heinz
    3. August 2018

    @Peter
    Kein Problem, viel Spaß und Mahlzeit. 😉

  177. #177 Alderamin
    3. August 2018

    @Peter

    #173 ist eine kontextsensitive (Typ-1-Grammatik), die überhaupt nichts zum Verständnis der Relativitätstheorie beiträgt.

    Aus Sicht von M1 bewegt sich M2. Aus Sicht von M2 bewegt sich M1. Und beide “messen” aus ihrer Sicht dasselbe Vorzeichen, das jeweils andere Myon kommt entgegen!

    Natürlich hat die Relativgeschwindigkeit aus Sicht der Inertialsysteme verschiedene Vorzeichen = Richtungen, sonst kämen sie sich nicht entgegen. Jedes Koordinatensystem hat Basisvektoren, die man vernünftigerweise in die gleichen Richtungen orientiert, wenn man sie vergleichen will (und wenn man das nicht tut, kann man sie entsprechend transformieren). Aber der Betrag der Geschwindigkeit ist gleich. Im Lorentzfaktor wird die Geschwindigkeit quadriert, das macht sie dann als Einflussfaktor richtungsunabhängig.

    Es gibt kein ausgezeichnetes System, egal, wie man es dreht oder wendet.

    Das ist richtig, das heißt aber nicht, dass beide dieselben Richtungen der Bewegung sehen und es heißt auch nicht, dass beide ein und dieselbe Strecke gleich groß sehen, sondern wechselseitig entsprechende Strecken, die entsprechend bewegt sind. Heißt: Myon 1 nimmt eine relativ zu sich unbewegte Strecke unverkürzt wahr, Myon 2 dieselbe Strecke verkürzt (das wären etwa die 10 km, über die wir hier reden). Myon 2 nimmt eine relativ zu sich unbewegte Strecke unverkürzt wahr (etwa den Durchmesser eines parallel mit ihm fliegenden Protons), Myon 1 sieht dieselbe Strecke verkürzt. Das ist absolut symmetrisch und nur genau so richtig, und folgt unvermeidlicherweise aus meiner Herleitung des Lorentzfaktors in Teil 1, und wird auch oben im Artikel bei der Berechnung benutzt, bitte nochmal lesen.

    Nur noch mal zur Erinnerung:
    Zitat Karl-Heinz: “Bei den 10 km musst du dich entscheiden für wen dies gilt.”
    Zitat Spritkopf: “Heißt, dass die Strecke von 10 km für M1 gilt, aber für M2 nicht.”

    Genau so ist das richtig und nicht anders. In welchem System ruhen die 10 km? Nur für M1. Für M2 sind sie kürzer.

    Beide verletzen das Relativitätsprinzip, denn es gibt kein ausgezeichnetes Bezugssystem!

    Der erste Halbsatz ist falsch, und wenn Dir das Verständnis dafür fehlt und Du es nicht gemäß Teil 1 nachvollziehen kannst, dann kommen wir hier nicht weiter.

  178. #178 Peter
    4. August 2018

    @Karl-Heinz
    Im Sinne Newtons ändert sich die Länge nicht, innerhalb der SRT erscheint die Länge um den Lorentzfaktor in x-Richtung verkürzt (wenn v die Relativbewegung eines Beobachters zum Stab ist).

    @Alderadmin
    Die Kenntnis von formalen Sprachen trägt grundsätzlich zum Verständnis von komplexen Systemen bei, also auch der SRT. Im Übrigen ist Deine Antwort dazu falsch.

    Nochmal: Zwei Myonen im Weltraum haben den Abstand d und kommen mit Geschwindigkeit v aufeinander zu. Wegen des Relativitätsprinzips muss für beide Sichtweisen dieselbe Betrachtung gelten.

    In einem formalen System gibt es Symbole und Regeln. Und nur nach diesen Regeln erstellte Ausdrücke sind auch Element dieses Systems. Die bloße Zugehörigkeit einzelner Teilausdrücke reicht nicht. Beispiel: 1+2=4. Obwohl Teilausdrücke der Gleichung Elemente der Mathematik sind, ist der Gesamtausdruck kein Element der Mathematik, da er sich nicht herleiten lässt aus den Regeln. Deshalb mein Ausflug in die formalen Sprachen. Deine Diagramme mögen für sich genommen korrekt sein, da Du aber die Regeln missachtest (so wie übrigens sehr viele “Lehrer” der SRT) sind sie zur Erklärung der längeren Lebensdauer von Myonen ungeeignet! Nur zur Info: Ich bin kein “Gegner” der RT (ganz im Gegenteil!), weigere mich aber, unlogische Schlüsse zu akzeptieren.

  179. #179 Karl-Heinz
    4. August 2018

    @Peter

    Nochmal: Zwei Myonen im Weltraum haben den Abstand d und kommen mit Geschwindigkeit v aufeinander zu. Wegen des Relativitätsprinzips muss für beide Sichtweisen dieselbe Betrachtung gelten.

    Das sind ja Angaben von dir.
    * Wir prüfen, ob die Angaben vollständig sind.
    ◩ geg.: zwei Myonen, relative Geschwindigkeit v, Abstand d

    v=s/t = (γs)/(γt)= s’/t’ =v’
    da v=v’ folgt auch γ =γ’
    γ … Lorentzfaktor

    s’=γs oder s=γs’ je nachdem für welches Myon der Abstand von 10km gilt. Bei deiner Angabe fehlt aber der Hinweis für welches Myon der Abstand gilt.

    Wie war das Abendessen gestern?
    Ich wollte schon schreiben, ob du ein wenig schwindelst, habe es mir dann aber verkniffen, irgendwie aber doch geschrieben. 😉

  180. #180 Peter
    4. August 2018

    @Karl-Heinz

    Bei deiner Angabe fehlt aber der Hinweis für welches Myon der Abstand gilt.

    Er gilt für beide. Und jetzt?

    Wie war das Abendessen gestern?

    Unsere selbsteinladende Gäste wollten plötzlich lediglich etwas von der Pizzeria holen.Kann man sowas “Abendessen” nennen?

    Ich wollte schon schreiben, ob du ein wenig schwindelst,….

    Ich kann Dir nicht ganz folgen.

    Tust du Compiler programmieren?

    Ich bin Informatiker. Compilerbau gehört aber nicht zu meinen aktuellen Tätigkeiten. Jedoch interessiere ich mich sehr für die Theorie komplexer Systeme, siehe z.B. https://www.mpg.de/96174/cpt08_Komplexe_Systeme.pdf
    oder
    https://de.wikipedia.org/wiki/Komplexes_System

    Die Beschreibung komplexer Systeme scheint auf alle Ebenen zu passen: Physik, Chemie, Biologie, soziale Systeme, Aktien, Mathematik, Informatik, Astronomie etc. – und sie kann damit als eine allgemeine Theorie angesehen werden. Sehr interessant in diesem Zusammenhang das Buch “A new kind of Scrience” von Stephen Wolfram (https://www.wolframscience.com/). Wolfram ist Dir bekannt? Siehe hier https://de.wikipedia.org/wiki/Stephen_Wolfram

  181. #181 Karl-Heinz
    4. August 2018

    @Peter

    Komplexer Systeme, Stephen Wolfram klingt sehr interessant. 😉

    Er gilt für beide. Und jetzt?

    Na, dann ist die Ausgangssituation der beiden Myonen M1 und M2 nicht ident, oder die relative Geschwindigkeit ist gleich null. Lässt sich relativ einfach beweisen. Da verstehe ich dich wiederum nicht, dass du das nicht verstehst.

  182. #182 Alderamin
    5. August 2018

    @Peter

    Jetzt endlich die Rechnung (derzeit Urlaub und mehr draußen als drinnen vor dem Rechner…):

    es wird von einem dritten Beobachter eine Strecke von 10 km abgesteckt, an deren Enden jeweils ein Myon erzeugt werden soll. Er drückt in der Mitte einen Schalter, welches ein Signal zu den beiden 5 km entfernten Myonen-Erzeugern schickt. M1 wird ruhend (im Bezugssystem des dritten Beobachters) und M2 auf M1 zubewegt (im Bezugssystem des dritten Beobachters) erzeugt (gehen wir davon aus, dass die Erzeugung technisch bedingt jeweils genau gleich lang dauert, da dieselben Erzeuger genutzt werden). Nachdem der dritte Beobachter das Signal abgesandt hat, scheidet er aus der Betrachtung aus.

    Also M1 ruht relativ zum Pulsgeber und M2 bewegt sich relativ zu M1 und dem Pulsgeber mit 0,995 c. Ich nehme mal eine Geschwindigkeit von 0,995 c und eine Lebensdauer beider Myonen von 3,35 µs, damit es sich mit den 10000 m besser ausgeht (bei 0,9995 c zerfällt M2 nicht am Ort von M1 am Erdboden, sondern erst viel später woanders, bzw. hätte sich der Zerfall mit dem Einschlag im Boden wohl erledigt).

    Sicht von M1 und Pulsgeber:
    Das Signal erreicht M1 am Boden und M2 in 10000 m Höhe gleichzeitig 5000 m/c = 16,667 µs nach Aussendung. M1 entsteht bei Eintreffen des Pulses und nach 3,35 µs zerfällt M1 wieder (18,867 20,017 µs nach Signalaussendung). M2 lebt 3,35 µs / √(1-0,995²) = 33,5 µs nach Eintreffen des Pulses (γ = 1/√(1-0,995²) ist fast exakt 10) und fliegt in dieser Zeit 10000 m weit bis zum Boden (33,5e-6m * 0,995 * 3e+8m/s = 10000m). Inklusive Pulslaufzeit stirbt M1 bei 20 µs und M2 bei 50,2 µs bei Ankunft am Boden. M1 schließt aus der Geschwindigkeit vom M2, dass M2 eine eigene Lebenszeit von 3,35 µs gemessen haben sollte.

    Sicht von M2:
    Pulsgeber und M1 kommen mit 0,995c entgegen. Pulsgeber ist 5000 m * √(1-0,995²) = 500 m näher als M1 (der 5-km-Abstand ist um 1/γ verkürzt). Puls bewegt sich mit c Richtung M1, M1 kommt Puls mit 0,995 c entgegen, macht 1,995 c Relativgeschwindigkeit, mit der die 500 m überbrückt werden. Zeit bis zum Eintreffen des Pulses bei M1 folglich 500m/(1,995c) = 0,835 µs. M1 entsteht und lebt 3,35 µs / √(1-0,995²) = 33,5 µs.

    Wenn der Puls im Inertialsystem von M2 an dessen Entstehungsort eintrifft, ist M2 500 m vom Pulsgeber und 1000 m von M1 entfernt (um 1/γ verkürzt). Zur Zeit der Aussendung war der Pulsgeber weiter entfernt, wir dürfen das Inertialsystem von M2 für die gesamte Betrachtung nicht wechseln. Wenn der Pulsgeber sich mit 0,995 c nähert und der Puls mit c, dann kommt er mit 0,005 c langsamer näher als der Puls selbst. Vom Aussendungsort an gerechnet hat der Puls 500 m Vorsprung herausgeholt, wenn er bei M2 in 1000 m Höhe eintrifft, damit die Eingangsbedingung “Pulsgeber in der Mitte von M1 und M2 und M2 10000 m / γ = 1000 m von M1 entfernt” zutrifft. D.h. der Puls erreicht M2 500 m / 0,005 c = 333,3 µs nach Aussendung. Von da an lebt M2 noch 3,35 µs und legt in dieser Zeit die 1000 m zum Boden zurück.

    Inklusive Pulslauffzeit stirbt M1 bei 0,835 µs + 33,5 µs = 34,335 µs und M2 bei 333,3 + 3,35 = 336,68 µs nach Signalgabe.

    Weil also das Eintreffen des Pulses bei M2 aus dessen Sicht viel länger dauert als bei M1, ist M1 längst zerfallen, wenn M2 am Boden ankommt, obwohl M1 wegen der Zeitdilatation viel länger lebt. Das ist wieder die Relativität der Gleichzeitigkeit, die alles zurecht rückt.

    Deine Diagramme mögen für sich genommen korrekt sein, da Du aber die Regeln missachtest (so wie übrigens sehr viele “Lehrer” der SRT) sind sie zur Erklärung der längeren Lebensdauer von Myonen ungeeignet!

    Wo habe ich jetzt genau welche Regel der SRT missachtet (möglichst mit Beleg)? Zeitdilatation und Längenverkürzung sind jedenfalls im Rahmen der Regeln und wie in Teil 1 dargelegt auch vollkommen unvermeidliche Schlussfolgerungen aus c=const. (und |vrel| = const).

    Die Kenntnis von formalen Sprachen trägt grundsätzlich zum Verständnis von komplexen Systemen bei, also auch der SRT. Im Übrigen ist Deine Antwort dazu falsch.

    Und in der Tat, die Grammatik ist natürlich kontextfrei, links ist kein Kontext gegeben und die Klammern aus 0en und 1en lassen sich nur mit einem Stackautomaten erkennen. Ist schon über 30 Jahre her, blöder Fehler, habe mich von den rechten Seiten verwirren lassen. Der Zusammenhang mit der SRT erschließt sich mir trotzdem nicht.

  183. #183 Karl-Heinz
    5. August 2018

    @Alderamin

    (18,867 µs nach Signalaussendung)

    Sieht so aus als hättest du statt der vereinbarten Lebensdauer von 3,35 µs bei dieser Addition die echte Lebensdauer von 2,2 µs genommen. Der Fehlerteufel schläft nie. 😉

  184. #184 Alderamin
    5. August 2018

    @Karl-Heinz

    Ja, stimmt, danke. Ursprünglich hatte ich mit 2,2 µs gerechnet, aber da das Teilchen dann nicht am Boden angekommen wäre, habe ich’s geändert, aber dort vergessen. Die 2,2 µs sind ja eh nur ein Mittelwert, es schaffen es ja gerade nicht alle Myonen bis zum Erdboden, nur erheblich mehr als erwartet. Hier rechne ich der Einfachheit halber mit einer festen Zerfallszeit. Geht ja nur ums Prinzip.

    Die Rechnung im bewegten Inertialsystem ist im Grunde immer die gleiche. Damit lässt sich auch Storzens Drillingsparadoxon auflösen. Plus: Beim Wechsel des Inertialsystems gehen dem wechselnden Beobachter durch abrupte Änderung der Jetztzeit-Linie etliche Jahre im beobachteten Inertialsystem verloren, wie in diesem Bild ersichtlich (schwarzer Bereich erscheint nie “gleichzeitig” und wird beim Wechsel überschlagen). Gilt dann auch wechselseitig bei voneinander wegstrebenden und dann simultan umkehrenden Raumschiffen.

  185. #185 Karl Mistelberger
    5. August 2018

    > Beim Wechsel des Inertialsystems gehen dem wechselnden Beobachter durch abrupte Änderung der Jetztzeit-Linie etliche Jahre im beobachteten Inertialsystem verloren …

    University Meets Public
    10. Oktober 2011, VHS Meidling

    Franz Embacher

    100 Jahre Zwillingsparadoxon
    Wie real sind die Effekte der Speziellen Relativitätstheorie?

    Abstract:

    Im Jahr 1911 schuf Paul Langevin mit dem Zwillingsparadoxon eines der faszinierendsten Gedankenexperimente der Speziellen Relativitätstheorie. In der – bis heute geltenden – Interpretation von Max von Laue half es zur Klärung der Frage, ob die von Einsteins Theorie vorhergesagte Abhängigkeit des Zeitflusses vom Bewegungszustand des Beobachters “scheinbar” oder “real” ist.

    https://homepage.univie.ac.at/franz.embacher/Rel/UMP2011/

  186. #186 Philip
    Wuppertal
    6. August 2018

    @Peter

    Zeitdilatation und Längenkontraktion treten aber immer gemeinsam auf.

    Wenn man diese Wörter unbedingt benutzen möchte*), ja.
    Allerdings nicht “Zeitdilatation” der der Myonen als Uhren und “Längenkontraktion” der Strecke.
    Die Strecke ist dann als “kontrahiert” zu interpretieren, wenn man das Myonen als ruhend und die Erde als sehr schnell bewegt betrachtet.
    Was übrigens nicht heißt, dass es für das Myon, so es denn Augen hätte, auch so aussähe. Etwas, das sich nähert, sieht nämlich weiter weg aus, als es aktuell sein muss, denn man sieht es mit schrumpfender Verzögerung. Es sieht auch schneller aus, ggf. überlichtschnell – ist es aber nicht. Etwas, das mit c startet, würde man gleichzeitig starten und ankommen sehen.
    —-
    *)
    Wenn jemand eine Salami der Länge L schräg, in einem Winkel θ, in einen Messschieber hält und deshalb nur mehr L·cos(θ) herausbekommt, würde dies niemand “Längenkontraktion” und die Tatsache, dass ein Schrägschnitt um 1/cos(θ) breiter ist als ein Querschnitt, niemand “Breitendilatation” nennen.

  187. #187 Philip
    6. August 2018

    @Albrecht Storz

    Die einzige Alternative wäre, dass im Rahmen der Relativitätstheorie die Logik außer Kraft gesetzt werden würde.

    Das ist sicherlich nicht der Fall. Allerdings muss Logik nicht zwangsläufig einfach und schon gar nicht intuitiv sein.
    Die Intuition “erzählt” uns eine Menge nachweisbaren Blödsinn – etwa auf dem Gebiet der Statistik. Sie zur Instanz zu machen, wäre also verkehrt.
    Die Logik, die zur SRT führt, ist freilich relativ simpel:

    (i) In zwei beliebigen relativ zueinander bewegten Inertialsystemen Σ und Σ’ gelten dieselben Gesetze der Physik.
    (ii) Zu den Gesetzen der Physik gehören auch die der Elektrodynamik und alles, was direkt daraus folgt, also auch die elektromagnetische Wellengleichung, die c als Ausbreitungstempo für elektromagnetische Wellen enthält.
    ⇒ Die Lichtgeschwindigkeit muss in Σ’ wie in Σ den Betrag c haben (die Richtung kann abweichen, das verletzt nicht die Wellengleichung). Umrechnungen Σ↔Σ’ müssen das berücksichtigen (und die LORENTZ–Transformation tut das).
    Auch die innere Geometrie der Raumzeit muss das berücksichtigen, und die MINKOWSKI – Metrik tut das.

  188. #188 PS
    12. August 2018

    #150 Niels 
2. August 2018
    “@PS Eigengeschwindigkeit (https://en.wikipedia.org/wiki/Proper_velocity).
    Komische Größe. Nie davon gehört. … Ich kenne auch keine einzige andere Größe in der Physik, für deren Definition man die Messungen mehrerer Beobachter benötigt. Sehr seltsame Idee. Für mich persönlich auch ziemlich das Unanschaulichste und Absurdeste, das man sich vorstellen kann. ”

    Die Eigengeschwindigkeit ist zwar eine Funktion zweier Eigen- bzw. Ruhegrößen (der Eigenzeit aus Sicht des Systems S und der Ruhelänge aus Sicht des Systems S’), sie hängt aber deswegen nicht unbedingt von den Messungen mehrerer Beobachter ab.

    Bei der Messung der Koordinatengeschwindigkeit ermittelt S den zurückgelegten Weg eines Materiepunktes und die dafür benötigte Zeit. Wir wissen ohne zusätzliche Messung, dass bis zum Ende des Prozesses beim Materiepunkt (am Ort M des Systems S’) um den inversen Lorentzfaktor weniger Eigenzeit vergangen ist.
    (Randbemerkung: Das heißt aber nicht, dass alle in S’ synchron laufenden Uhren langsamer gehen würden, das heißt nicht einmal, dass auch nur diese eine Uhr bei M im Hinblick auf die von ihr angezeigte Eigenzeit “langsamer gegangen” wäre; diese falsche Metapher, das Narrativ laut A. Storz #81, ist für alle Missverständnisse verantwortlich).

    Bei der Messung der Eigengeschwindigkeit ermittelt S die Länge einer vorbeikommenden Strecke und die für das Vorbeikommen benötigte Zeit. Wir wissen ohne zusätzliche Messung, dass die Ruhelänge der Strecke im System S’ um den Lorentzfaktor größer ist als die in S gemessene Länge.

    Die unterschiedlichen Begriffe von “Geschwindigkeit” begegnen uns im Alltag, aber bei kleinen Geschwindigkeiten fällt ihr fundamentaler Unterschied niemandem auf.
    Um Koordinatengeschwindigkeit handelt es sich bei der “Wettkampfrichtermethode”, bei der die Zeit gestoppt wird, die ein Läufer für einen ausgemessenen Weg benötigt (beides im selben System).
    Um Eigengeschwindigkeit handelt es sich bei der “Autofahrermethode”, bei der ein Autofahrer die Zeit stoppt, die zwischen zwei Kilometerstangen der Autobahn vergeht.

    Die Eigengeschwindigkeit ist weder unanschaulich noch absurd, sondern der aussagekräftigste Wert für Geschwindigkeit, der sich denken lässt. Der Autofahrer kann seine Geschwindigkeit theoretisch unbegrenzt erhöhen. Zum Schluss würde er den Autobahnkilometer mit mehrfacher “Lichtgeschwindigkeit” zurücklegen, ohne dass seine Koordinatengeschwindigkeit aus Sicht der Autobahn die “Lichtgeschwindigkeit” überschreiten würde.

    Und da sind wir beim Myon (@Karl-Heinz #129): Das Verhältnis Erde-Myon ist symmetrisch. Ein Myon (S) kommt an einem Beobachter (S’) in 10km Höhe vorbei. Beide haben relativ zueinander eine Eigengeschwindigkeit von 0.995*Gamma = 10c (10km/2,2 µs). Sie kommen zur Zeit t=0; t’=0 aneinander vorbei. Beide legen in einer Eigenzeit von 2,2µs im gegenbeteiligten System eine Strecke (Ruhelänge) von 10km zurück (die im eigenen System gemessen 600 m lang ist).
    Weder beim Myon noch beim Beobachter vergeht deswegen die Eigenzeit langsamer (den sich aus dieser falschen Metapher ergebenden Widerspruch zeigt @Peter in #152 ganz richtig auf).

    Das übliche Rechenbeispiel wird zwar verwickelter, wenn der Start des Myons in 10km Entfernung vom Beobachter erfolgt, wodurch die Startzeiten aus wechselseitiger Sicht der verlorenen Gleichzeitigkeit ausgesetzt sind, an den Eigengeschwindigkeiten und dem Phänomen, dass man auch mit kurzer Lebenszeit weit kommen kann, ohne dass “die Zeit langsamer vergeht”, ändert das nichts.

  189. #189 Karl-Heinz
    12. August 2018

    @PS

    Die Eigengeschwindigkeit ist zwar eine Funktion zweier Eigen- bzw. Ruhegrößen (der Eigenzeit aus Sicht des Systems S und der Ruhelänge aus Sicht des Systems S’), sie hängt aber deswegen nicht unbedingt von den Messungen mehrerer Beobachter ab.

    Jetz aber widersprichst du dich selbst.

  190. #190 PS
    12. August 2018

    @Karl-Heinz
    Ich schrieb:
    “Bei der Messung der Eigengeschwindigkeit ermittelt S die Länge einer vorbeikommenden Strecke und die für das Vorbeikommen benötigte Zeit. Wir wissen ohne zusätzliche Messung, dass die Ruhelänge der Strecke im System S’ um den Lorentzfaktor größer ist als die in S gemessene Länge.”
    Meinst Du, dass ein derartiger Messvorgang – eben nur aus der Sicht von S – nicht durchführbar wäre? Wenn nein, dann stimmt wohl der Satz, dass es sich “nicht unbedingt um die Messungen mehrerer Beobachter” handeln muss.

  191. #191 Karl-Heinz
    12. August 2018

    @PS
    Verständnisfrage: Ist die Eigengeschwindigkeit eindeutig oder hängt ihr Wert von S’ oder S” oder S” usw. ab?

  192. #192 PS
    12. August 2018

    @Karl-Heinz
    Die Eigengeschwindigkeit V beschreibt das Verhältnis zweier Systeme S und S’ – ebenso wie die Koordinatengeschwindigkeit v. Insofern ist die V ebenso wenig “eindeutig” (im Sinne von “absolut”) wie v.
    Oder meinst Du mit “eindeutig” etwas anderes als “absolut”?

  193. #193 Karl-Heinz
    12. August 2018

    @PS
    Ok…
    Nehmen wir ein Koordinatensystem an, in dem ein Teilchen hin und her flitzt.
    Dann ist die Koordinatengeschwindigkeit des Teilchens v=s/t.
    Die Eigengeschwindigkeit des Teilchens wäre dann v=s/τ= γ*s/t. Also ist die Eigengeschwindigkeit im Allgemeinen eigentlich nur eine Rechengröße ähnlich dem Schwarzschildradius oder?

  194. #194 Karl-Heinz
    12. August 2018

    @PS
    Die Bezeichnung Eigengeschwindigkeit suggeriert einem, dass es sich um die echte Geschwindigkeit des Teilchens handelt, was natürlich vollkommener Unsinn ist. Für mich ist sie nur eine Rechengröße, mehr aber nicht. Ob sinnvoll oder nicht, sei mal dahingestellt.

  195. #195 PS
    13. August 2018

    Es ist eine reine Definitionsfrage: sowohl v als auch V sind “nur eine Rechengröße”. Beide haben ihre Berechtigung.

    v ist nicht “echter” als V, im Gegenteil, beim relativistischen Impuls m*V läuft V der v den Rang ab. V zeigt einem Reisenden, wie weit er wirklich kommt und welchen Impuls er wirklich hat. Beides kommt bei v nur indirekt zum Ausdruck.

  196. #196 Karl-Heinz
    13. August 2018

    @PS
    Wenn es deine Intention ist mit Hilfe der Eigengeschwindigkeit zu zeigen, dass die Zeit für alle gleich schnell vergeht, dann wirst du früher oder später auf folgendes Problem stoßen. Ich bin in Ruhe, Teilchen entfernt sich von mir und kehrt wieder zurück, so ist in dieser Zeitspanne für das Teilchen weniger Zeit vergangen. Und das kannst mit der Eigengeschwindigkeit auch nicht erklären. 😉

  197. #197 Philip
    Wuppertal
    13. August 2018

    @Alderamin, #116

    Komische Größe. Nie davon gehört.

    Das mag daran liegen, dass dieses Wort in der deutschen Sprache nicht etabliert ist. Googelt man „Eigengeschwindigkeit“, so wird diese als die Geschwindigkeit eines Passagieres innerhalb eines Fahrzeuges definiert, die sich mit der des Fahrzeugs selbst zur Gesamtgeschwindigkeit zusammensetzt.

    Dies ist hier natürlich nicht gemeint.

    Vielmehr ist die Eigengeschwindigkeit oder Celerität (nicht zu verwechseln mit der Rapidität, das ist wieder etwas anderes) im Unterschied zur gewöhnlichen Geschwindigkeit

    v› = ds›/dt

    die Größe

    γ·v› := v›/√{1 – ‹v,v›/c²} = p›/m = ds›/dτ,

    die beliebig groß werden kann. Sie setzt sich aus den räumlichen Komponenten der Vierergeschwindigkeit

    v» = (γ·c | γ·v›) = (cdt/dτ | dx/dτ | dy/dτ | dz/dτ)

    zusammen, deren MINKOWSKI-Betrag

    |v»| = √{«v,v»} = √{γ²c² – 㲋v,v›} ≡ c

    ist. Mit der Celerität wächst also zwangsläufig die zeitliche Komponente der Vierergeschwindigkeit, sodass der Quotient v› immer kleiner als c bleibt und sich c asymptotisch nähert.
    —————-

    @Karl-Heinz
    Nein, eine reine Rechengröße würde ich das nicht nennen, das ist sie ebenso wenig wie v›.

    Die Idee von Geschwindigkeit war ja eigentlich immer, die Koordinaten nach einem skalaren ‘Parameter’ (im Sinne einer Laufvariablen, nicht einer Formvariablen) zu differenzieren, also nach τ. Nur dass in der NEWTON’schen Näherung t≡τ ist und man ebensogut nach t differenzieren kann.

    Außerhalb dieser Näherung geht das nicht, weil t selbst eine Koordinate ist. Somit ist v› quasi ein Maß für die Neigung der eigenen Weltlinie gegen die Zeitachse, erfüllt aber gar nicht mehr das, was wie von einer Geschwindigkeit erwarten.

    Vor allem ist aber γv› eine höchst physikalische und für Dich als Reisenden durchaus relevante Größe, nämlich ein Maß dafür, wie schnell Du eine bestimmte Strecke zurücklegst, und zwar nach Deiner eigenen Uhr.

    Du magst einwenden, „für Dich“ verkürze sich die Strecke, aber das stimmt nicht. Die Strecke ist dann als kürzer zu interpretieren, wenn Du Dich als ruhend und sowohl Start als auch Ziel als mit –v› bewegt betrachtest.

    Dies ist übrigens keine echte Verkürzung, sondern wenn Du Dich als ruhend betrachtest, interpretierst Du andere Ereignisse – deren absoluter Abstand (natürlich der MINKOWSKI-Abstand)

    Δς := √{‹Δs,Δs› – c²Δt²} ≡ √{‹Δs’,Δs’› – c²Δt’²}

    ohnehin kleiner ist als der Abstand der Ereignisse, die Du als gleichzeitig interpretierst, wenn Du Dir selbst die Geschwindigkeit v› zuschreibst.

    So oder so: Da steht nicht EINSTEIN mit der Kelle und winkt Dich raus, und Du stößt auch nicht unbedingt auf einen Widerstand, sondern Du kannst beliebig weiter beschleunigen, nur dass Du dann eben auch schneller durch die Zeit stochst.

  198. #198 Philip
    13. August 2018

    @Karl-Heinz

    …so ist in dieser Zeitspanne für das Teilchen weniger Zeit vergangen. Und das kannst mit der Eigengeschwindigkeit auch nicht erklären.

    Wohl aber mit der Vierergeschwindigkeit und damit, dass deren MINKOWSKI-Betrag immer gleich c sein muss:

    (γc)² – 㲋v,v› ≡ c².

    Der zeitliche und der räumliche Anteil von v» sind Cosinus Hyperbolicus und Sinus Hyperbolicus von einer Art Winkel, der Rapidität genannt wird.

    Die Funktionen sind im Gegensatz zum „nomalen“ Cosinus und Sinus nicht periodisch, sondern beide gehen gegen Unendlich, wenn die Rapidität dies tut.

    Der Tangens Hyperbolicus wiederum geht gegen 1 – das Tempo |v›|/c.

  199. #199 PS
    13. August 2018

    @Philip
    “Du kannst beliebig weiter beschleunigen, nur dass Du dann eben auch schneller durch die Zeit stochst.”

    Durch die Relativbewegung verzeichnet der Reisende eine verkürzte Reisestrecke, was sein rasantes Fortkommen ermöglicht (Myon).

    Zudem begegnen dem Reisenden aber synchronisierten Uhren des Systems S’, die zunehmend vorgehen. Dieses “Schneller durch die Zeit stochen” ist ein Effekt, der erst bei Systemwechsel (Beschleunigung) in den Blick kommt (Zwillingsparadoxon).

    Beides hat mit einem “langsameren Vergehen von Zeit” nichts zu tun.

  200. #200 Frank Wappler
    13. August 2018

    Karl-Heinz schrieb (#86, 26. Juli 2018):
    > Man merkt, dass du nie beim Bundesheer warst oder die Ausbildung dort verschlafen hast.

    Manch Alt-Gediente bemerken an dieser Stelle Nuancen eines bestimmten Fehlschlusses

    > verwenden für die Entfernungsbestimmung die altbewährte Radarmethode

    Um die altmodisch-koordinatenbezogene Radarmethode überhaupt einzusetzen, wie man sie u.a. dort beschrieben finden kann, müssten ja offensichtlich zunächst bestimmte T-Koordinaten-Werte festgesetzt und (den einzelnen Anzeigen der Beteiligten) zugeordnet werden; bzw. im Zusammenhang mit Bondis Anleitung müsste zunächst festgestellt worden sein, welche Paare von Beteiligten/Enden durchwegs gegenüber einander geruht hätten.

    Koordinaten-freier spricht man (deshalb eher, spätestens seit Synge diese Bezeichnungen vorgeschlagen hat) von der chronometrischen Definition von Entfernung bzw. (falls die betreffenden Beteiligten/Enden gegenüber einander ruhten) von Distanz.

    > Wenn deine Leute sich mit der Radarpistole gegenseitig beschießen, was für eine Entfernung (bei manchen auch eine Funktion der Zeit) würden sie wohl messen?

    Gute Frage! Wenn zum Beispiel, wie schon anderswo ausführlicher skizziert,

    – Jockl und Paul gegenüber einander ruhten, so dass diese beiden überhaupt durch eine bestimmte (chronometrisch definierte) Distanz voneinander charakterisiert wären, und

    – Jockls Anzeige der Passage von Franziska und Pauls Anzeige der Passage von Roberts gleichzeitig waren, und

    – Franziska und Roberta gegenüber einander ruhten, so dass diese beiden überhaupt durch eine bestimmte (chronometrisch definierte) Distanz voneinander charakterisiert wären, und

    – Franziskas Anzeige der Passage von Jockl und Robertas Anzeige der Passage von Paul gleichzeitig waren,

    dann lässt sich jedenfalls herleiten/beweisen, dass die Distanz zwischen Jockl und Paul gleich der Distanz zwischen Franziska und Roberta ist.

  201. #201 Frank Wappler
    13. August 2018

    Philip schrieb (#197, 13. August 2018):
    > Die Idee von Geschwindigkeit war ja eigentlich immer, die Koordinaten nach einem skalaren ‘Parameter’ (im Sinne einer Laufvariablen, nicht einer Formvariablen) zu differenzieren

    Nein: das wäre (“schon immer”) stattdessen eine Idee von Koordinaten-Geschwindigkeit.

    Die Idee von (Durchschnitts-)Geschwindigkeit ist dagegen:
    die Distanz zwischen zwei geeigneten (d.h. durchwegs gegenüber einander ruhenden) Enden ins Verhältnis zur (relevanten) Belegungs-Dauer der durch diese beiden Enden festgesetzten Strecke zu setzen;
    kurz: "\overline v := s / t".

    (Momentan-Geschwindigkeit, v “an einem bestimmten Ende”, ergibt sich ggf. als Grenzwert einer Folge von Durchschnitts-Geschwindigkeits-Werten im Grenzübergang zu immer kleineren Distanzen “bis zum betreffenden Ende”.)

  202. #202 Philip
    13. August 2018

    Nein: das wäre … stattdessen eine Idee von Koordinaten-Geschwindigkeit.

    Nein, die ist

    v› = ds›/dt = (dx/dt | dy/dt | dz/dt),

    wobei t die Koordinatenzeit ist, also eine Koordinate und eben kein skalarer „Parameter“ – im Unterschied zu τ, der Eigenzeit, die ja eine Weglänge durch die Raumzeit darstellt.

    In der NEWTON’schen Näherung ist dt≈dτ, weshalb man in der NEWTON’schen Mechanik Raum und Zeit getrennt behandelt und t als „Parameter“ betrachtet, nach dem man die Koordinaten (die räumlichen nämlich) abzuleiten hat.

    Hingegen ist

    γv› = ds›/dτ = (dx/dτ | dy/dτ | dz/dτ)

    mutmaßlich genau das, was @PS als Eigengeschwindigkeit bezeichnet hat, und das ist die Ableitung nach einem skalaren „Parameter“, nämlich τ.

    Übrigens: Wie bekommt Ihr LaTeX hin? Ich habe ‘[latex]\Delta s›[/latex]‘ oder auch $\Delta s›$ ausprobiert, zumindest in der Vorschau konnte ich keinen Code erkennen.

  203. #203 Karl-Heinz
    13. August 2018

    @Philip

    Latex-Vorschau funktioniert hier leider nicht nicht.
    Test:
    x^2
    $Latex Latexcode$

  204. #204 Alderamin
    13. August 2018

    @Philip

    Wie bekommt Ihr LaTeX hin?

    Kommentierungs-FAQ!

  205. #205 Philip
    13. August 2018

    @PS
    Für das Folgende ist es zweckmäßig, Start und Ziel mit A und B zu bezeichnen. Beide sollen der Einfachheit halber relativ zueinander stationär sein, und ein Koordinatensystem Σ soll so ausgerichtet sein, dass B von A aus genau in x-Richtung liegt, mit dem Abstand Δx.

    Den „reisenden“ Beobachter werde ich mit B’ bezeichnen; er (oder sie, das ist natürlich wurscht) bewegt sich (in Σ beschrieben) mit v›=(v|0|0) von A nach B und benötigt dafür nach einer in B stationären Uhr die Zeitspanne

    \Delta t = \frac{\Delta x}{v}.

    In einem anderen Koordinatensystem Σ’ ist B’ stationär, während A und B sich beide mit –v› bewegen.

    Durch die Relativbewegung verzeichnet der Reisende eine verkürzte Reisestrecke, was sein rasantes Fortkommen ermöglicht….

    Das ist ein verbreiteter Irrtum. Die Strecke zwischen A und B ist in Σ’ „verkürzt“, d.h., es ist

    \Delta x' = \frac{\Delta x}{\gamma} = \Delta x \cdot \sqrt{1-\frac{‹v,v›}{c^2}}.

    In Σ’ kommt B’ aber weder rasant noch sonstwie fort, sondern ist stationär, weswegen auch

    \Delta t'\equiv \Delta \tau

    ist. Vielmehr entfernt sich A und nähert sich B mit

    \frac{\Delta x'}{\Delta t'} = \frac{\Delta x'}{\Delta\tau} = –v›.

    Dabei sieht B für B’ natürlich weiter weg aus, als es rechnerisch aktuell sein muss (Retardierungseffekt), erscheint wegen der abnehmenden Verzögerung im Zeitraffer und scheint sich schneller zu bewegen, als es das tatsächlich tut. Das ist übrigens auch das Geheimnis sogenannter Überlichtschneller Quellen, die man aus astronomischen Beobachtungen kennt.

    In Σ hingegen ist der Abstand zwischen A und B, wie oben angegeben, Δx, also keinesfalls „verkürzt“. Da greift dann auch Deine Ausführung von der Eigen-Geschwindigkeit

    \gamma\cdot v = \frac{\Delta x}{\Delta\tau}= \frac{\Delta x}{\Delta t'}.

  206. #206 Philip
    13. August 2018

    Ich weiß schon, warum ich die LaTeX-Funktion hier eher selten nutze. Schließlich gibt es ohne Vorschau keine Vorwarnung vor „Formula does not parse“. Mann, , geht mir das auf den Koffer!

  207. #207 rolak
    13. August 2018

    ohne Vorschau keine Vorwarnung

    Es gib noch eine Welt neben den SB, Philip, Du bist also nicht ganz ‘ohne’. Als LaTₑX-Vorschau nutze ich gerne die Kommentarfunktion jenes hochinteressanten blogs.

  208. #208 Karl-Heinz
    13. August 2018

    @rolak

    Ich befürchte, dass die Vorschau für Latex bei deinem favorisieren Link nicht mehr funktioniert.

  209. #209 PS
    13. August 2018

    @Philip #205

    “In Σ’ kommt B’ aber weder rasant noch sonstwie fort, sondern ist stationär.”

    Ich habe nie behauptet, dass B’ in ∑’ rasant vorwärts kommt, sondern dass sich an B’ ruhend in ∑’ eine um 1/Gamma verkürzte Reisestrecke der Ruhelänge A-B vorbeibewegt (was eben einer Reise des B’ von A nach B entspricht).

    “Dabei sieht B für B’ natürlich weiter weg aus, als es rechnerisch aktuell sein muss (Retardierungseffekt)”

    “Weiter weg” als rechnerisch was, “aktuell” wann?

    “[B (?)] erscheint wegen der abnehmenden Verzögerung im Zeitraffer und scheint sich schneller zu bewegen, als es das tatsächlich tut. ”

    Nein, B hat aus der Sicht von ∑’ die Koordinatengeschwindigkeit v.

    “In Σ hingegen ist der Abstand zwischen A und B, wie oben angegeben, Δx, also keinesfalls „verkürzt“. ”

    Ich habe nie behauptet, dass die Ruhelänge A-B aus Sicht des ∑ verkürzt wäre.

    Ich vermag nach Deiner bisherigen Erklärung den “verbreiteten Irrtum” in der Aussage, “durch die Relativbewegung verzeichnet der Reisende eine verkürzte Reisestrecke, was sein rasantes Fortkommen ermöglicht….” noch nicht zu erkennen.

    Vielleicht liegt es aber nur an der Formulierung::

    “Durch die Relativbewegung verzeichnet der Reisende eine gegenüber ihrer Ruhelänge verkürzte Reisestrecke A-B, was sein rasantes Fortkommen von A nach B ermöglicht.”

  210. #210 Philip
    14. August 2018

    @PS

    Ich habe nie behauptet, dass B’ in ∑’ rasant vorwärts kommt,…

    Dann habe ich die Aussage…

    Durch die Relativbewegung verzeichnet der Reisende eine verkürzte Reisestrecke, was sein rasantes Fortkommen ermöglicht….

    …missverstanden, wozu die Formulierung allerdings auch ein bisschen einlädt. Da hast Du mit Deinem Schlussatz den Nagel auf den Kopf getroffen.

    ..sondern dass sich an B’ ruhend in ∑’ eine um 1/γ verkürzte Reisestrecke der Ruhelänge A-B vorbeibewegt…

    Klar, genauso ist da auch. Die Strecke ist dann als „verkürzt“ zu interpretieren, wenn sie als bewegt interpretiert wird.

    …(was eben einer Reise des B’ von A nach B entspricht).

    Das stimmt zwar, aber die Interpretation von B’ als stationär ist gleichbedeutend mit der Wahl des Koordinatensystems Σ’ zum Bezugssystem. Die Interpretationen zu vermischen, indem man z.B. sagt „B’ legt eine um 1/γ verkürzte Strecke zurück“, ist das, was ich mit dem verbreiteten Irrtum meine.

    Der Rest bezieht sich auf den Unterschied zwischen dem, was jemand misst bzw. ggf. berechnet und dem, was er sieht.

  211. #211 rolak
    14. August 2018

    befürchte

    Tja, Karl-Heinz, das ist allerdings rechts kurzfristig defunkt geworden – hab mal angefragt…

  212. #212 PS
    14. August 2018

    @Philip #210
    Das „B’ legt eine um 1/γ verkürzte Strecke zurück“, ist tatsächlich irreführend.
    Gemeint war:
    Für einen in konsequenter Relativität ruhenden Reisenden B’ sind die Orte A und B sowohl Besucher als auch Besuchte. B’ reist von A nach B, indem er eine aus seiner Sicht um 1/γ verkürzte Strecke mit den Orten (Enden) A und B an sich vorbeikommen lässt.
    —-
    “Der Rest bezieht sich …”

    Gibt es einen “Unterschied zwischen dem, was jemand misst … und dem, was er sieht”?

    [Frage: wie kommt man zu der Funktion, die du für das Zitieren verwendest?]

  213. #213 Frank Wappler
    14. August 2018

    Philip schrieb (13. August 2018):
    > [ … » … stattdessen eine Idee von Koordinaten-Geschwindigkeit.« … ] Nein, die ist […]

    Es lassen sich wohl mehrere unterscheidbare Ideen von “Koordinaten-Geschwindigkeit” ausdrücken. Eine davon ist zweifellos:

    > (dx/dt | dy/dt | dz/dt),

    > wobei t die Koordinatenzeit ist, also eine Koordinate […]

    … und wobei “(x | y | z)” natürlich ebenfalls ein Koordinaten-Tupel bezeichnet.

    > Hingegen ist […] (dx/dτ | dy/dτ | dz/dτ)

    > mutmaßlich genau das, was @PS als Eigengeschwindigkeit bezeichnet hat

    Mag schon sein.
    Jedenfalls treten auch im letzteren Ausdruck Koordinaten “(x | y | z)” auf.
    Es handelt sich deshalb ebenfalls um “eine Idee von” Koordinaten-Geschwindigkeit;
    im Unterschied zu Koordinaten-frei definierter und auszudrückender (geometrisch-kinematischer) Geschwindigkeit (“an sich”),
    also der schon genannten (Mess-)Größe

    “Verhältnis aus
    – Distanz zwischen zwei gegenüber einander ruhenden Enden und
    – Belegungs-Dauer der durch diese beiden Enden festgesetzten Strecke
    (zusammen mit der Richtungangabe: startend bei dem einen Ende der Strecke, und zielend auf das andere)”.

    > wobei t die Koordinatenzeit ist, also eine Koordinate und eben kein skalarer „Parameter“ – im Unterschied zu τ, der Eigenzeit, die ja eine Weglänge durch die Raumzeit darstellt.

    Zu unterscheiden sind (mindestens):

    – der Wert \tau A[ \, \_K, \_Q \, ] der Messgröße Dauer (je eines bestimmten Beteiligten, A, zwischen je zwei bestimmten seiner Anzeigen, A_K bzw. A_Q),

    – eine auf eine bestimmte Anzeige A_K bezogene (und bzgl. einer weiteren Anzeige A_Q orientierte) Eigenzeit-Parametrisierung der Anzeigen As

    \tau A_K : \mathcal A \rightarrow \mathbb R \times \tau A[ \, \_K, \_Q \, ] mit

    \tau A_K[ \, \_P \, ] \mapsto \tau A[ \, \_K, \_P \, ] \times \text{Max}\left[ \, \text{Sgn}[ \, \left( \frac{\tau A[ \, \_K, \_P \, ]}{\tau A[ \, \_K, \_Q \, ]} \right)  - \left( \frac{\tau A[ \, \_P, \_Q \, ]}{\tau A[ \, \_K, \_Q \, ]} \right) \, ],  \text{Sgn}[ \, \left( \frac{\tau A[ \, \_P, \_Q \, ]}{\tau A[ \, \_K, \_Q \, ]} \right) - 1 \, ] \, \right], und

    – jede Parametrisierung der Anzeigen As durch reelle Koordinaten

    t : \mathcal A \rightarrow \mathbb R.

    > Übrigens: Wie bekommt Ihr LaTeX hin?

    Im Falle des “Formula does not parse”-Falles mit der (vorläufigen?) https://scienceblogs.de/sic/2014/01/27/science-of-science-communication/#ScienceBlogs.SandBox

  214. #214 Frank Wappler
    14. August 2018

    p.s. zu #213 — angepasst an die beschränkte Zeilenlänge und dadurch hoffentlich besser lesbar:

    Zu unterscheiden sind (mindestens):

    – der Wert \tau A[ \, \_K, \_Q \, ] der Messgröße Dauer (je eines bestimmten Beteiligten, A, zwischen je zwei bestimmten seiner Anzeigen, A_K bzw. A_Q),

    – eine auf eine bestimmte Anzeige A_K bezogene (und bzgl. einer weiteren Anzeige A_Q orientierte) Eigenzeit-Parametrisierung der Anzeigen As

    \tau A_K : \mathcal A \rightarrow \mathbb R \times \tau A[ \, \_K, \_Q \, ] mit

    \tau A_K[ \, \_P \, ] \mapsto
    \tau A[ \, \_K, \_P \, ] \times \text{Max}\left[ \, \text{Sgn}[ \, \left( \frac{\tau A[ \, \_K, \_P \, ]}{\tau A[ \, \_K, \_Q \, ]} \right)  - \left( \frac{\tau A[ \, \_P, \_Q \, ]}{\tau A[ \, \_K, \_Q \, ]} \right) \, ],  \text{Sgn}[ \, \left( \frac{\tau A[ \, \_P, \_Q \, ]}{\tau A[ \, \_K, \_Q \, ]} \right) - 1 \, ] \, \right], und

    – jede Parametrisierung der Anzeigen As durch reelle Koordinaten

    t : \mathcal A \rightarrow \mathbb R.

  215. #215 Frank Wappler
    14. August 2018

    p.s. zu #213, #214 — Berichtigung:

    Zu unterscheiden sind (mindestens):

    – der Wert \tau A[ \, \_K, \_Q \, ] der Messgröße Dauer (je eines bestimmten Beteiligten, A, zwischen je zwei bestimmten seiner Anzeigen, A_K bzw. A_Q),

    – eine auf eine bestimmte Anzeige A_K bezogene (und bzgl. einer weiteren Anzeige A_Q orientierte) Eigenzeit-Parametrisierung der Anzeigen As

    \tau A_K : \mathcal A \rightarrow \mathbb R \times \tau A[ \, \_K, \_Q \, ] mit

    \tau A_K[ \, \_P \, ] \mapsto
    \tau A[ \, \_K, \_P \, ] \times \text{Max}\left[ \, \text{Sgn}[ \, \left( \frac{\tau A[ \, \_K, \_P \, ]}{\tau A[ \, \_K, \_Q \, ]} \right)  - \left( \frac{\tau A[ \, \_P, \_Q \, ]}{\tau A[ \, \_K, \_Q \, ]} \right) \, ],  \text{Sgn}[ \, \left( 1 - \frac{\tau A[ \, \_P, \_Q \, ]}{\tau A[ \, \_K, \_Q \, ]} \right) \, ] \, \right], und

    – jede Parametrisierung der Anzeigen As durch reelle Koordinaten

    t : \mathcal A \rightarrow \mathbb R.

  216. #216 Alderamin
    14. August 2018

    @PS

    [Frage: wie kommt man zu der Funktion, die du für das Zitieren verwendest?]

    Ebenso: Kommentierungs-FAQ (Link oben rechts).

  217. #217 PS
    14. August 2018

    Das populärwissenschaftliche Narrativ, aus der Sicht des Ruhesystems würde die (“gesamte”) Zeit in gleichförmig bewegten Systemen “langsamer vergehen” (vgl. @Alderamin #38, #47, #74, #75 usw”) ist mE falsch bzw. extrem irreführend. Ich beziehe mich im Folgenden ausschließlich auf gleichförmige Bewegungen.

    a) Eine “synchronisierte Uhr eines Systems” geht gleich schnell wie alle anderen Uhren (Lichtuhren gehen überall gleich schnell, weil c überall gleich ist)
    und zeigt dieselbe Zeit wie alle anderen Uhren dieses Systems.

    b) Aus der Menge der synchronisierten Uhren zweier zueinander gleichförmig bewegter Systeme wird nun von jedem System jeweils eine Uhr am selben Ort zur selben Zeit auf gleiche Zeigerstellungen gebracht (zB “Standardsituation” t, t’, x, x’=0). Damit weichen die Zeigerstellungen sämtlicher anderer sich begegnender Uhren der beiden Systeme voneinander ab (Lorentz-Transformation L-T)!

    Aus der L-T ergibt sich zB, dass die Uhr bei einem bewegten Materiepunkt M (ruhend im System S’) weniger Zeit anzeigt als die ihr begegnende Uhr ruhend im Systems S.

    Wie steht es nun mit der Behauptung, der Zeitablauf im System S’ wäre verlangsamt, die Zeit würde dort langsamer vergehen? Ist man zu dieser Behauptung berechtigt, nur weil der Stand zweier sich begegnender Uhren voneinander abweicht? Folgt aus dem Abweichen in logischer Stringenz, dass eine Uhr langsamer gegangen sein muss als die andere, vor allem wenn man bedenkt, dass sich auch die Uhrenabstände verschieben könnten?

    Nein, selbst wenn sich diese Behauptung nur auf eine einzige Uhr des Systems S’ beziehen würde (und nicht wie üblich auf alle), wäre sie falsch.

    Alle Uhren des als ruhend definierten Systems S (vgl. “Interpretation” @Philip #210), von dem aus der Vorgang betrachtet wird, zeigen die gleiche Zeit an (oben a). Die Uhren des anderen Systems S’, die diesen synchronen Uhren des Systems S gerade begegnen, zeigen nicht die gleiche Zeit an (oben b). Je nach dem Ort der Uhrenbegegnungen ist deren Stand einmal weiter fortgeschritten als der Stand der Uhr des Systems S (wie eben bei dem Materiepunkt M), an anderen Orten bleibt deren Anzeige hinter denen der betreffenden synchronisierten Uhr des Systems S zurück (L-T).

    In Bezug auf die Gesamtheit dieser – aus Sicht des zum Bezugssystem gewählten Systems S – nicht synchroner Uhren des Systems S’ (in Bezug auf eine auch dort an sich einheitlich nach dem Muster oben a) vergehenden Zeit) ist die Behauptung, die Zeit in S’ würde langsamer vergehen, sinnlos (die Uhren des betrachteten Systems S’ haben aus der Perspektive des Systems S, in dem alle Uhren den gleich Sand anzeigen, “wie ein Sack Flöhe” völlig uneinheitliche Anzeigen).

  218. #218 Philip
    Wuppertal
    14. August 2018

    @PS
    Der Tag „blockquote“ in den bekannten Tag-Zeichen öffnet das Zitat, „/blockquote“ in denselben Zeichen schließt es.

    Gibt es einen “Unterschied zwischen dem, was jemand misst … und dem, was er sieht”?

    Selbstverständlich. Du siehst ja z.B. mich, wenn ich z.B. 300m von Dir entfernt bin, nicht so, wie ich „jetzt“ bin, sondern so, wie ich vor 1μs war. Das ist natürlich fast „jetzt“, aber bei größeren Abständen ist auch die Verzögerung größer (‘n dolles Fernrohr, um überhaupt was zu sehen, setze ich mal voraus).

    Und wenn ich mich auch noch schnell entferne oder nähere (die Technik dafür setze ich wieder einfach voraus), siehst Du mich auch nicht in der Entfernung, in der ich bin.

    Retardierungseffekte

    Entferne ich mich mit v und siehst Du mich im Abstand Δs (das kann man ja messen), ist dies schon Δs/c her, und in der Zwischenzeit muss ich mich um (v/c)Δs weiter entfernt haben, insgesamt also „jetzt“ Δs(1+(v/c)) entfernt sein.

    Nähere ich mich mit v und siehst Du mich im Abstand Δs, ist dies schon Δs/c her, und in der Zwischenzeit muss ich mich um (v/c)Δs genähert haben, insgesamt also „jetzt“ nur noch Δs(1–(v/c)) entfernt sein.

    Außerdem nimmt die Verzögerung im ersten Falle zu, sodass Du meine Uhr nicht um γ, sondern um

    K := √{c+v}/√{c–v}

    langsamer gehen siehst – und ich Deine ebenfalls (und die Strahlung entsprechend rotverschoben). Im zweiten Falle nimmt die Verzögerung ab, und somit siehst Du meine Uhr mit

    K⁻¹ = √{c–v}/√{c+v}

    schneller gehen und ich Deine auch. K und K⁻¹ sind sogenannte BONDI- oder auch DOPPLER-Faktoren, und es ist

    ½(K + K⁻¹) = γ
    ½(K – K⁻¹) = γv/c.

    Uminterpretation

    Wenn Du Dich selbst als bewegt und mich als stationär interpretierst, musst Du die o.g. Retardierungseffekte zur Aberration uminterpretieren.
    Fährst Du bei Windstille durch den Regen, der deshalb eigentlich vertikal fällt, kommt der Regen für Dich mit zunehmendem Tempo mehr und mehr schräg von vorn. Mit Licht ist das ähnlich. Deshalb sieht etwas, auf das Du Dich zu bewegst, weiter weg aus als es ist.

  219. #219 Frank Wappler
    14. August 2018

    PS schrieb (14. August 2018):
    > […] Lichtuhren gehen überall gleich schnell […]

    Nein, sondern:
    gleiche Lichtuhren gehen gleich schnell;
    d.h. alle Paare von (je zwei gegenüber einander ruhenden, einander spiegelnden) Enden, die gleiche gegenseitige Ping-Perioden-Dauern aufweisen, und deshalb (entsprechend der chronometrischen Distanz-Definition) auch jeweils gleiche Distanz ihrer beiden Spiegel-Enden voneinander aufweisen, haben deshalb auch gleiche “Tick”-Raten.

    Es sind aber auch ungleiche Lichtuhren vorstellbar, und ggf. auffindbar.

  220. #220 Philip
    Wuppertal
    14. August 2018

    @Frank Wappler
    Ich fürchte, ich kann Deinen Ausführungen nicht ohne Legende folgen. Hast Du die Formelzeichen mal näher erläutert?

  221. #221 Frank Wappler
    15. August 2018

    Philip schrieb (#220, 14. August 2018):
    > @Frank Wappler Ich fürchte, ich kann Deinen Ausführungen nicht ohne Legende folgen.

    Ich kann und sollte Deinen Ausführung in #202 wohl von vornherein mit vergleichbarer Zurückhaltung begegnen, und möchte deshalb bezüglich der dort benutzten Notation nochmals ausdrücklich nachfragen … Vor allem:

    Steht in #202 der (Teil-)Ausdruck “dx/dt” als Abkürzung für die Ableitung der Funktion “x[ t ]” (bzw. die Werte dieser Ableitung an bestimmten “Stellen t”) ?

    Bezeichnet “x” dabei eine Koordinate, bzw. einen bestimmten Koordinaten-Wert, bzw. den Wert der Koordinaten-Funktion “x[ t ]” an einer bestimmten “Stelle t” ?

    Bezeichnet “(dx/dt | dy/dt | dz/dt)” dort ein (geordnetes) Tripel von entsprechenden Ableitungen ?

    Und falls so, wäre “(x | y | z)” die entsprechende Notation für ein (geordnetes) Tripel von Koordinaten (bzw. von Koordinaten-Funktionen) ?
    (Dieser letztere Ausdruck erscheint auf dieser Seite offenbar erstmals in meinem Kommentar #213.)

    p.s.
    > Hast Du die Formelzeichen mal näher erläutert?

    Ich hatte immerhin versucht, “meine Formelzeichen” u.a. im Kommentar #213 mit Namen und kurzen Erläuterungen auszustatten … Aber ich fasse das dort Relevante gern zusammen:

    – jeden unterscheidbaren, einzeln identifizierbaren Beteiligten (a.k.a. “Beobachter”, “materiellen Punkt”, “principal identifiable point” …) bezeichne ich mit einfachem Großbuchstaben: A, B, M usw. usf., genau so, wie es uns schon Einstein vorgemacht hat.

    ein Koinzidenz-Ereignis bezeichne ich mit den relevanten daran Beteiligten als Indices (und ausreichend vielen, um das betreffende Koinzidenz-Ereignis ggf. eindeutig zu identifizieren): \varepsilon_{AP}, \varepsilon_{BQ}, \varepsilon_{FGJ}, \varepsilon_{FGK}, etc. (Mir ist leider nicht bekannt, dass Einstein diese oder eine vergleichbar ausdrückliche Notation zur Bezeichnung von Koinzidenz-Ereignissen einschl. der relevanten Beteiligten benutzt hätte).

    eine Anzeige jeweils eines einzelnen Beteiligten an einem bestimmten Koinzidenz-Ereignis, also dessen Anteil am betreffenden Koinzidenz-Ereignis, bezeichne ich mit dem (Großbuchstaben-)Namen des betreffenden Beteiligten und den zur Identifikation des betreffenden Koinzidenz-Ereignisses erforderlichen relevanten weiteren Beteiligten als Indices: A_P für As Anzeige der Koinzidenz mit P im Koinzidenz-Ereignis \varepsilon_{AP}, P_A für Ps Anzeige der Koinzidenz mit A im Koinzidenz-Ereignis \varepsilon_{AP}, usw.

    die (geordnete) Menge der Anzeigen jeweils eines bestimmten Beteiligten bezeichne ich mit dem entsprechenden (Großbuchstaben-)Namen in Kursivschrift, z.B. \mathcal A \equiv \{ ... A_P ... \} für die (geordnete) Menge der Anzeigen As.

    – die Dauer jeweils eines bestimmten Beteiligten, zwischen je zwei bestimmten seiner Anzeigen, bezeichne ich mit dem (dafür allgemein üblichen) Buchstaben \tau zusammen mit den ausdrücklichen Angabe des (Großbuchstaben-)Namens des betreffenden Beteiligten, und der Angabe seiner beiden Anzeigen als Argumente ausdrücklich “verkapselt/private“; also \tau A[ \, \_P, \_Q \, ] für die Dauer As zwischen dessen Anzeigen A_P und A_Q.

  222. #222 Philip
    15. August 2018

    @Frank Wappler

    Ich kann und sollte Deinen Ausführung in #202 wohl von vornherein mit vergleichbarer Zurückhaltung begegnen,…

    Musst Du gar nicht. Für das Verständnis Deiner Ausführungen wäre halt nur eine Legende hilfreich. Vor allem die vielen Indizes können etwas verwirrend sein, ich vermutete, dass sie für Ereignisse stehen, aber genau wissen tat ich das nicht.

    …und möchte deshalb bezüglich der dort benutzten Notation nochmals ausdrücklich nachfragen …

    Gern. Die Position eines zunächst beliebigen Punktes, im konkreten Fall des Schwerpunktes eines bestimmten Körpers, relativ zu einem Referenzpunkt O beschreibe ich durch den Ortsvektor s›.
    Dessen Notation orientiert sich ein wenig an der des quantenmechanischen Zustandsvektors |ψ⟩, der als ‘Ket’-Vektor die Abstraktion eines Spaltenvektors darstellt. Dementsprechend hat s› in einem ganz bestimmten Koordinatensystem Σ die Form des Tripels

    (1.1) s› = (x | y | z) bzw. (x; y; z),

    wobei ‘|’ bzw. ‘;’ für den Zeilenwechsel steht. Der entsprechende Zeilenvektor (orientiert am ‘Bra’-Vektor) schreibt sich mit Kommata anstelle von Semikolons oder vertikalen Strichen als

    (1.2) ‹s = (x, y, z).

    Das Skalarprodukt des Ortsvektors mit sich selbst schreibe ich in Anlehnung an Matrizenprodukt und Skalarprodukt eines Zustandsvektors als

    (1.3) ‹s,s› = x² + y² + z² = |s›|².

    Sowohl die einzelnen Koordinaten als auch s› insgesamt können natürlich als Funktion einer weiteren, der zeitlichen Koordinate t auftreten,

    (2.1) s›(t) = (x(t) | y(t) | z(t))
    (2.2) ‹s(t) = (x(t), y(t), z(t)),

    und danach auch abgeleitet werden:

    (3.1) v›(t) = ds›(t)/dt = (dx(t)/dt | dy(t)/dt | dz(t)/dt)
    (3.2) ‹v(t) = ‹ds(t)/dt = (dx(t)/dt, dy(t)/dt, dz(t)/dt),

    wobei in der Tat

    (4) dx(t) = x(t+dt) – x(t)

    ist. Da es sich dabei um Ableitung der räumlichen Koordinaten nach einer weiteren, nämlich der zeitlichen Koordinate handelt, heißt v› auch die Koordinatengeschwindigkeit.

    Sowohl die einzelnen Koordinaten als auch s› insgesamt können aber auch als Funktion eines skalaren Parameters τ auftreten, der entlang der Weltlinien gemessen wird, nämlich,

    (5.1) s›(τ) = (x(τ) | y(τ) | z(τ))
    (5.2) ‹s(τ) = (x(τ), y(τ), z(τ)),

    und danach kann auch abgeleitet werden:

    (6.1) γ·v›(t) = ds›(τ)/dτ = (dx(τ)/dτ | dy(τ)/dτ | dz(τ)/dτ)
    (6.2) γ·‹v(t) = ‹ds(t)/dt = (dx(τ)/dτ, dy(τ)/dτ, dz(τ)/dτ).

    Hier werden zwar Koordinaten abgeleitet, aber eben nach der Eigenzeit, weshalb γν› Eigengeschwindigkeit heißt.

  223. #223 PS
    16. August 2018

    @Frank W #219
    Verschiedene Lichtuhren mögen unterschiedlich oft ticken, sie gehen aber immer gleich schnell.
    Zu #217 wäre nachzutragen, dass die Uhren in ihren Koordinatengittern in gleicher Ruhelänge voneinander entfernt aufgestellt werden.

    @Philip #218
    1. Danke für den Hinweis auf „blockquote“, aber wie bediene ich diesen “Tag”?

    2. Sehen – Messen:
    Ich verstehe was du meinst, ziehe aber eine andere Begrifflichkeit vor:
    Es macht keinen Unterschied, ob man das Verzeichnen von Ereignissen (“Feststellen der Koordinatenwerte eines Ereignisses”) als “Sehen (iwS)” bezeichnet oder ob man nur das Verzeichnen derjenigen Ereignisse, die darin bestehen, dass an einem bestimmten Ort eine Nachricht von einem entfernten Ereignis eintrifft, als “Sehen” (ieS)” bezeichnet: Verzeichnen ist immer Messen (beim Sehen: Verzeichnen von Ort und Zeit des Eintreffens der Nachricht, wobei man beim Menschen unterscheiden müsste: beim Auge oder beim Gehirn).
    Daher gibt es mE keinen Unterschied zwischen dem, was jemand misst und dem, was er “sieht”.

    3. “Retardierungseffekt” ist offenbar der longitudinale Dopplereffekt.
    “Aberration” ist offenbar der transversale Dopplereffekt.

    “Wenn Du Dich selbst als bewegt und mich als stationär interpretierst, musst Du die o.g. Retardierungseffekte zur Aberration uminterpretieren.”

    Diese “Uminterpretation” von einem longitudinalen in einen transversalen Dopplereffekt ist mir unklar
    (die den Dopplereffekt bestimmende Relation von Sender und Empfänger hängt nicht davon ab, wer sich als ruhend betrachtet).

    4. “Deshalb sieht etwas, auf das Du Dich zu bewegst, weiter weg aus als es ist.”

    Ersetze ich Sehen durch Verzeichnen, müsste Dein Satz, sofern ich ihn richtig verstanden habe, lauten: “Wenn sich Dein System S’ gegenüber dem System S mit v nach rechts (Osten) bewegt, dann verzeichnest Du in Deinem System S’ ein Ereignis x’=-a’, (t’=b’) und das System S mit x=-a, (t=b=0), wobei -a’ größer ist als -a.” (Das genannte Ereignis kann als Anwesenheit eines Materiepunktes interpretiert werden, der sich auf den Beobachter S im Zentrum seines Koordinatensystems zubewegt).

  224. #224 Frank Wappler
    16. August 2018

    Philip schrieb (15. August 2018):
    > Die Position eines zunächst beliebigen Punktes, im konkreten Fall des Schwerpunktes eines bestimmten Körpers, relativ zu einem Referenzpunkt O beschreibe ich durch den Ortsvektor s›.

    Tja — Vektorräume sind eine feine Sache; und erst recht diejenigen “mit zusätzlicher Struktur (wie “Norm” oder “Skalarprodukt”).

    Fragt sich aus Sicht der Physik bzw. hier insbesondere der Astronomie allerdings, ob und welchen “(Positionen von) Punkten” welche Vektoren zugeordnet werden sollten, bzw. wie die unterstellte(n) “zusätzliche(n) Struktur(en)” nachvollziehbar als Messgröße(n) definiert, d.h. zumindest im Prinzip experimentell festgestellt und bewertet werden könnte(n) …

    > […] hat s› in einem ganz bestimmten Koordinatensystem Σ die Form des Tripels

    > (1.1) s› = (x | y | z) bzw. (x; y; z),

    Natürlich wäre einem bestimmten/gegebenen Punkt bzw. Vektor in sehr, sehr vielen verschiedenen Koordinatensystemen jeweils ein bestimmtes/eindeutiges Tripel reeller Zahlen zuzuordnen. Aber:

    > Das Skalarprodukt des Ortsvektors mit sich selbst schreibe ich […] als

    > (1.3) ‹s,s› = x² + y² + z² = |s›|².

    Na, dadurch sind aber wirklich ganz, ganz bestimmte/spezielle Koordinaten bzw. Koordinatensysteme ausgewählt!
    Vermutlich (ich bin mir nicht ganz sicher, ob allein schon deshalb beweisbar-zwangsläufig) sollen die in Betracht stehenden Koordinaten sogar so gut/affin gewählt sein, dass

    – für jede reelle Zahl k der entsprechende Produkt-Vektor “(k ⊙ s)›” durch das (relle Zahlen-)Tripel (k x | k y | k z) auszudrücken wäre;

    wobei dieser Produkt-Vektor “(k ⊙ s)›” zunächst Koordinaten-frei u.a. dadurch beschrieben wäre, dass
    ‹(k ⊙ s),(k ⊙ s)› = k^2 ‹s,s› und ‹s,(k ⊙ s)› = k ‹s,s› und
    dass sich außerdem ein Vektor “((1 – k) ⊙ s)›” finden lässt, mit dem gilt
    “(((1 – k) ⊙ s)› ⊕ (k ⊙ s))› = s›”

    und (sogar)

    – für jedes Paar von Vektoren s› = (x | y | z) und r› = (n | p | q) der entsprechende Summen-Vektor “(r ⊕ s)›” durch das (relle Zahlen-)Tripel (n + x | p + y | q + z) auszudrücken wäre;

    wobei dieser Summen-Vektor “(r ⊕ s)›” zunächst Koordinaten-frei u.a. dadurch beschrieben wäre, dass
    ‹(r ⊕ s),(r ⊕ s)› = ‹r,r› + 2 ‹r,s› + ‹s,s›.

    (Aus gegebenen Norm-Werten eines Vektorraumes lassen sich solche ganz, ganz besondere, gute/affine Koordinaten(-Systeme) mit dem bekannten Verfahren konstruieren.)

    Und für solche ganz, ganz besondere, gute/affine Koordinaten(-Systeme) trifft es sicherlich zu, dass
    der Vektor, der in Form eines Tripels von Koordinaten-Ableitungen (bzgl. eines bestimmten geeigneten Parameters, i.A. θ) ausgedrückt ist,
    und Ableitung dieses Vektors (bzgl. dieses selben Parameters)
    einander gleich sind.

    Auf Koordinaten(-Systeme) im Allgemeinen, und insbesondere auf “weniger gute/affine”, trifft solche Gleichheit aber nicht zu;
    und deshalb ist im Allgemeinen “Geschwindigkeit” von “Koordinaten-Geschwindigkeit” zu unterscheiden.

  225. #225 Frank Wappler
    16. August 2018

    PS schrieb (#223, 16. August 2018):
    > Verschiedene Lichtuhren mögen unterschiedlich oft ticken, sie gehen aber immer gleich schnell.

    Nimm eine bestimmte Lichtuhr, bestehend insbesondere aus zwei voneinander getrennten, durchwegs gegenüber einander ruhenden, einander geeignet spiegelnden Enden, A und B.

    Füge einen weiteren Spiegel M (dessen Dicke nahezu vernachlässigbar gegenüber dem Abstand zwischen A und B sein soll) als “Mitte zwischen” A und B ein,
    so dass A und M eine Lichtuhr bilden.

    Die Tick- bzw. Ping-Dauer der Lichtuhr “AM” ist dann (nahezu) halb so groß wie die Tick- bzw. Ping-Dauer der Lichtuhr “AB” war.

    (Daraus schließt man bzw. deshalb sagt man auch, dass der Abstand zwischen A und M (nahezu) halb so groß war und blieb, wie der Abstand zwischen A und B.)

    Und deshalb sage ich auch, dass die Lichtuhr “AM” doppelt so schnell ging (bzw. tickte, bzw. pingte) wie die Lichtuhr “AB”. — Du nicht ?!?

  226. #226 PS
    16. August 2018

    @Frank W
    Ich verstehe, was du meinst. Wir sollten aber nicht sowohl eine hohe Frequenz des Tickens als auch eine hohe Geschwindigkeit, mit der die Zeitanzeige einer Uhr voranschreitet, mit demselben Ausdruck “schnell gehende Uhr” bezeichnen.

  227. #227 Karl-Heinz
    16. August 2018

    @Frank Wappler, @PS

    Die Pingdauer einer Lichtuhr hängt natürlich vom Abstand der Platten ab. Wenn man Uhren über ihr Ticken vergleicht sollten sie verständlicherweise den gleichen Plattenabstand haben.

  228. #228 Philip
    Wuppertal
    17. August 2018

    @PS
    1. Wenn Du ‘<blockquote>Zitat</blockquote>’ eingibst, erscheint

    Zitat

    2. Es geht mir nicht um „sehen“ oder „verzeichnen“, also ob man etwas mit den Augen oder mit einem Messgerät wahrnimmt. Mit der Messung meine ich eben auch die Ermittlung des Zeitpunktes, zu dem ein Signal emittiert wurde, und da ist es mit der „Verzeichnung“ nicht getan.
    Angenommen, wir nähern uns einander mit einer Relativgeschwindigkeit von 0,8c. Für diesen Fall sagt die SRT voraus, dass meine Uhr sozusagen entlang Deiner Zeitachse um den Faktor 4/3 langsamer geht (und umgekehrt). Wenn wir dabei Signale mit Zeitstempel austauschen, ergibt sich dies, aber eben erst durch die Rückrechnung.
    Du kannst Dich als ruhend auffassen, musst dies aber nicht. Fasst Du mich als ruhend auf, muss die Rückrechnung, dass meine Uhr schneller geht als Deine, und zwar um den Faktor ¾. Dabei ist das, was Du wann verzeichnest, selbstverständlich unabhängig von der Interpretation.

    3. Der longitudinale Dopplereffekt gehört zu den Retardierungseffekten. Die Aberration ist genau dasselbe, nur anders herum interpretiert, und mitnichten der transversale DE. Der geht mit dem einher, was man üblicherweise „Zeitdilatation“ nennt.

    4. Find’ ich etwas kompliziert ausgedrückt.
    Nehmen wir wieder konkret an, wir bewegen uns mit 0,8c aufeinander zu. Du bekommst von mir ein Signal und zugleich das Echo von einem Signal von Dir, das 20 Minuten alt ist. Das heißt:
    Wenn Du Dich als ruhend ansiehst, muss mein Signal 10 Minuten alt sein und logischerweise aus 10 Lichtminuten Entfernung stammen.
    Meine aktuelle Entfernung kann dann aber nur noch 2 Lichtminuten betragen, sofern sich meine Geschwindigkeit nicht geändert hat, und 2½min = 150s später werde ich an Dir vorbeikommen.
    Wenn Du mich als ruhend ansiehst, sieht das anders aus. In diesem Fall ist es Deine Uhr, die langsamer geht. Du wirst mich in 150s Eigenzeit erreichen, aber in meinem Ruhesystem sind das 200s, was mit einer „aktuellen“ Entfernung von 160 Lichtsekunden oder 2&frac23; Lichtminuten korrespondiert. Wobei sich dieses „aktuell“ auf einen früheren Zeitpunkt auf meiner Uhr bezieht. Misst Du meine Entfernung in diesem Moment, kommst Du aber wiederum auf 10 Lichtminuten, und das kannst Du dann nicht als Retardierungseffekt interpretieren, sondern musst es der Aberration in die Schuhe schieben.

  229. #229 PS
    17. August 2018

    Danke für das “Zitat”.
    Ich finde Deine Begrifflichkeit sehr aufschlussreich, weil es mein Verständnis der SRT erweitert.
    Im Vergleich dazu drücke ich jetzt das Gleiche mit anderen Begriffen aus, vielleicht ist das auch für Dich aufschlussreich:

    zu 2.: Oberbergriff ist das “Ereignis”. Die Koordinatenwerte eines Ereignisses werden durch Messvorgänge in einem Intertialsystem ermittelt. Die Messung erfolgt durch die Reflektion von Lichtstrahlen, die wegen c=konstant nach Halbierung ihrer Gesamtlaufzeit die zeitliche Entfernung eines Ereignisses (und dessen Ortsvektor) anzeigen.
    Die Messung kann aber auch durch regelmäßig verteilte synchronisierte (ruhende) Uhren erfolgen (siehe #217), über deren Anzeigen (beim Eintritt eines Ereignisses bei einer von ihnen) sich der Beobachter später informieren lässt.
    Die Messdaten des Ereignisses werden dann in dem dazugehörigen Koordinatensystem verzeichnet. Der ganze Vorgang ist “Verzeichnen eines Ereignisses”.

    dass meine Uhr sozusagen entlang Deiner Zeitachse um den Faktor 4/3 [richtig 3/5=0,6] langsamer geht (und umgekehrt). Wenn wir dabei Signale mit Zeitstempel austauschen, ergibt sich dies, aber eben erst durch die Rückrechnung. (…) Fasst Du mich als ruhend auf, muss die Rückrechnung, dass meine Uhr schneller geht als Deine, …

    Das beschreibt #217 mit dem “Abweichen der Anzeigen der sich begegnenden Uhren”. Die von Dir ins Spiel gebrachte “Zeitachse” (aber nur “sozusagen”), an der der im Ursprung des Systems S’ ruhende Beobachter S’ mit seiner bei ihm ruhenden Uhr entlang fährt, ist nichts anderes als die im anderen System S hintereinander in gleichen Ruheabständen aufgestellte Uhrenreihe, mit deren Stand der vorbei kommende Beobachter S’ den Stand seiner Uhr vergleicht. Signalaustausch und Rückrechnung können entfallen, was die Sache mE einsichtiger (und symmetrisch) macht.

    Dabei gehen aber alle Uhren “gleich schnell”. Das symmetrische Abweichen der Uhrenanzeigen ist Folge der begrenzten Wirkungsausbreitung c=konst. Deshalb stellt der im Ursprung des Systems S ruhende Beobachter ebenfalls fest, dass auf den Uhren der Uhrenreihe des Systems S’, an denen er vorbeikommt, mehr Zeit vergangen ist als auf der bei ihm ruhenden Uhr.

    “Dass meine Uhr schneller geht als Deine”: Diese Ausdrucksweise ist wieder ein Beispiel für das irreführende Narrativ (#217), das ein Verständnis der SRT erschwert und vernebelt. Nichts geht hier schneller oder langsamer. Außerdem ist “schneller” auch insofern verfehlt, als die Uhr bei einem Beobachter (und damit alle synchronisierten Uhren seines Systems) nie mehr Zeit anzeigt als eine der Uhren, an denen er vorbeikommt.

  230. #230 Alderamin
    17. August 2018

    @PS

    Verstehe bei Deinen Ausführungen (Uhren gehen gleich schnell, sieht nur so aus) nicht, wie Du damit die Myonenlebensdauer erklären willst. Sieht es nur so aus, als ob die Myonen (in gewisser Prozentzahl, die jedenfalls viel höher ist als ohne RT-Dilatation) den Erdboden erreichen, oder erreichen sie ihn? Und wenn sie ihn erreichen, wie lange haben sie dann existiert? Nach unserer Erdzeit?

    Die Entstehungshöhe von 10-15 km ist bekannt, die Geschwindigkeit nahe c auch.

  231. #231 PS
    17. August 2018

    @ Aldemarin
    Sie erreichen ihn. Eine Prozessdauer (die “Lebenszeit”) zu definieren macht aber mE nur dann Sinn, wenn man sie im eigenen Ruhesystem misst (gleich wie bei den Ruhelängen). Mag sein, dass dieser Prozess in anderen Systems als länger gemessen wird, deswegen hat das Myon aber keine “längere Lebenszeit”.

    Wir können natürlich den Flug eines Myons nicht nur aus der Warte dessen Eigenzeit (und dessen Eigengeschwindigkeit) beurteilen, sondern wir können unseren Blick auch darauf richten, wie die Ereignisse des Starts und des Eintreffens des Myons im System des Myons S’ und im System der Erde S nach der L-T verzeichnet werden.

    [Das Folgende Beispiel ist trotz ist vereinfachender Zahlenangaben etwas mühsam und kann auch übersprungen werden:
    Die Ursprünge der Systeme S (Erdoberfläche) und S’ (Myon) befinden sich bei x=0; t=0 und x’=0; t’=0 in der “Standardsituation”. v=0,6c.
    Start:
    S: Das Ereignis des Beginns der Reise des Myons findet aus Sicht des Systems S bei x=-1; t=0 statt. Der Erdbeobachter S ruht aus der Sicht seines Systems S “gleichzeitig” mit dem Reisebeginn im Ursprung seines Systems S, was als ein Ereignis “A” mit den Koordinaten x=0; t=0 interpretiert werden kann.
    S’: Das Ereignis des Beginns der Reise des Myons findet aus der Sicht seines Systems S’ bei x’=-1,25; t’=0,75 statt. Der Beobachter S befindet sich aus der Sicht des Systems S’ “gleichzeitig” mit dem Reisebeginn in der Position des Ereignisses x’=-0,45; t’=0,75. Das Ereignis “A” fand aber aus Sicht des Systems S’ bereits früher zu den Ereigniskoordinaten x’=0; t’=0 statt.
    Ankunft:
    S: Das Ereignis der Ankunft des Myons beim Beobachter S (“auf der Erde”) findet aus Sicht des Systems S bei t=1,25; x=0 statt.
    S’: Das Ereignis der Ankunft des Myons beim Beobachter S (“auf der Erde”) findet aus der Sicht des Systems S’ beim Myon mit den Koordinaten t’=2,0833; x=-1,25 statt.
    Das alles würde man in Minkowski-Diagrammen auf den ersten Blick nachvollziehen können.]

    Aus Sicht des Systems S’ (Myon) hat also der Prozess der Reise 1,333 Sekunden gedauert, aus der Sicht des Systems S (Erde) hat der Prozess der Reise 1,666 Sekunden (1,333*Gamma) gedauert. Aus der Sicht des Systems S der Erde standen aber zwei entfernte Ereignisse zur Beurteilung, aus der Sicht des Systems S’ des Myons nur zwei Ereignisse am selben Ort (beim Myon).

    Die unterschiedliche Prozessdauer, die ja symmetrisch wechselweise auftritt, kann aus den zu #217 genannten Gründen nicht dahin interpretiert werden, dass im System S’ (bezogen auf eine in ihm ruhende Uhr oder gar bezogen auf alle in ihm ruhenden Uhren!!!) die Zeit langsamer vergehen würde als bei einer (oder gar allen) im System S ruhenden Uhren! Der Ausdruck “S sieht die Uhren des S’ langsamer vergehen” ist eine irreführende Übernahme einer den Dopplereffekt beschreibenden Sprechweise, man sehe ein sich entfernendes Licht langsamer blinken usw. Kein vernünftiger Mensch würde argumentieren, er höre ein sich entfernendes Folgetonhorn tiefer, deshalb würde dort die Zeit langsamer vergehen. So wie dieser Schluss beim klassischen Dopplereffekt verfehlt ist, ist er es auch beim relativistischen (siehe #217).

    Beim Zwillingsparadoxon, auf das sich Dein einleitender Aufsatz bezieht, kommt allerdings ein Systemwechsel dazu, der tatsächlich einen geringeren Verbrauch an Eigenzeit beim beschleunigten Materiepunkt zur Folge hat.

  232. #232 Alderamin
    17. August 2018

    @PS

    Wir können natürlich den Flug eines Myons nicht nur aus der Warte dessen Eigenzeit (und dessen Eigengeschwindigkeit) beurteilen, sondern wir können unseren Blick auch darauf richten, wie die Ereignisse des Starts und des Eintreffens des Myons im System des Myons S’ und im System der Erde S nach der L-T verzeichnet werden.

    Die Erklärung entspricht, soweit ich das überblicke, ungefähr dem, was ich in #182 Peter zu erläutern versuchte, dass eben die unterschiedliche Gleichzeitigkeit sicherstellt, das jeder des anderen Uhr langsamer laufen sieht, ohne dass das zu einem Widerspruch führt. Aber nur das Myon sieht die Strecke verkürzt (der Erdbeobachter sieht das Myon verkürzt, aber das ändert nichts an dessen Flugzeit).

    Die unterschiedliche Prozessdauer, die ja symmetrisch wechselweise auftritt, kann aus den zu #217 genannten Gründen nicht dahin interpretiert werden, dass im System S’ (bezogen auf eine in ihm ruhende Uhr oder gar bezogen auf alle in ihm ruhenden Uhren!!!) die Zeit langsamer vergehen würde als bei einer (oder gar allen) im System S ruhenden Uhren!

    Das ist ja auch nicht die Behauptung. Die Behauptung ist, dass die Uhr in S’ aus Sicht von S langsamer läuft. Und umgekehrt. Aber wie gesagt, nur aus Sicht von S’ ist die Strecke verkürzt, d.h. S’ schafft sie aus seiner Sicht in einer geringfügig verlängerten mittleren Myonenlebensdauer und S sieht S’ die größere Strecke in einer verlangsamten Zeit zurücklegen, so dass beide darüber einig sind, dass das Myon die Strecke schafft.

    Dass das Myon die Strecke schafft, zeigt aber, dass es sich nicht nur um ein Phänomen der Beobachtung aufgrund von Lichtlaufzeiten handelt, sondern durchaus real ist. Das Myon lebt wirklich länger und schafft eine Strecke, die ihm normalerweise und ohne Zeitdilatation unmöglich wäre.

    Aus Sicht des Myons würde man allerdings sagen: die entgegenkommende Strecke ist wirklich verkürzt, deswegen ist sie in einer geringfügig überdurchschnittlichen Myonenlebensdauer zurücklegbar.

    Realität ist das, was man messen kann. Hier hat man zwei Ansichten desselben realen Sachverhalts, der sich nur verschieden in Raum und Zeit manifestiert.

    Dass eine Uhr in einem beliebigen Inertialsystem aus dessen Innenansicht immer gleich schnell geht, ist hingegen offensichtlich, schließlich ist die Geschwindigkeit des Zeitablaufs über die Geschwindigkeit einer Uhr (die auf physikalisch festliegenden Schwingungsprozessen beruht) definiert.

  233. #233 Frank Wappler
    17. August 2018

    Alderamin schrieb (#232, 17. August 2018):
    > Das Myon lebt wirklich länger und […]

    Länger als (!) … welches andere Myon, zum Beispiel ??

    Nein! — Das betreffende Myon hatte schlicht und einfach und proper seine bestimmte Lebensdauer;
    und die Belegungsdauer der (Enden der) Flugstrecke war wirklich (um den Faktor 1 / \sqrt{1 - \beta^2} > 1) länger als die Lebensdauer des Myons.

    > Aus Sicht des Myons würde man allerdings sagen: die entgegenkommende Strecke ist wirklich verkürzt, [..]

    Nein! — Die betreffende Flugstrecke hatte schlicht und einfach und proper ihre bestimmte Länge (z.B. die Distanz zwischen einem bestimmten Atomkern der Erdatmosphäre und einem bestimmten Detektor am Boden, sofern diese beiden gegenüber einander ruhten und deshalb überhaupt durch eine bestimmte Distanz voneinander charakterisiert gewesen wären);
    und ebenso schlicht und einfach und proper und wirklich ist die Distanz zwischen dem betreffenden Myon und einem geeigneten Beteiligten (z.B. einem bestimmten Neutrino), der gegenüber dem Myon durchwegs ruhte und dessen Anzeige der Passage des Detektors gleichzeitig zu Geburtsanzeige des Myons war, um den Faktor \sqrt{1 - \beta^2} < 1 kürzer als die Distanz der beiden Enden der Flugstrecke voneinander.

    > Realität ist das, was man messen kann.

    Stimmt. Und dazu gehört nicht zuletzt: Messwerte denjenigen zuzuschreiben, die dadurch (eigentlich, proper) charakterisiert sind.

  234. #234 Alderamin
    17. August 2018

    @Frank

    Länger als (!) … welches andere Myon, zum Beispiel ??

    Wir reden hier über grundsätzliche Prozesse. Jedes Myon hat seine eigene individuelle Lebensdauer, die lediglich einer bestimmten Verteilung gehorcht. Damit man das nicht ständig erklären muss, gehen wir hier davon aus, dass das betrachete Myon eben eines sei, das die Strecke vom Entstehungsort bis zum Erdboden gerade eben schafft und vergleichen eines, das der RT unterliegt mit seinem hypothetischen Pendant in einem Galileischen Raum. Und jetzt hundertmal aufschreiben:

    Die “Tyrannei der Präzision” behindert die Wissenschaftskommunikation

    Nein! — Das betreffende Myon hatte schlicht und einfach und proper seine bestimmte Lebensdauer

    Aber nicht im Vergleich mit einem ruhenden Myon. Ruhende Myonen leben 2,2 µs (Mittelwert). Die am Boden nachgewiesenen müssen mindestens 33 µs gelebt haben (was einzelnen Myonen in Ruhe auch gelingt, aber nicht einem so hohen Anteil, wie es beobachtet wird).

    Nein! — Die betreffende Flugstrecke hatte schlicht und einfach und proper ihre bestimmte Länge

    Ja, und die ist halt proper aus der Sicht des Erdlings 10 km, aber proper aus der Sicht des Myons nur ein paar hundert Meter. Also im Vergleich, wenn Myon und Beobachter sich darüber unterhalten würden, aus Sicht des Myons wirklich kürzer.

    “Myonen können sich gar nicht mit Menschen unterhalten” in 3 – 2 – 1 …

  235. #235 PS
    18. August 2018

    @Aldermin #232

    Das Myon lebt wirklich länger und schafft eine Strecke, die ihm normalerweise und ohne Zeitdilatation unmöglich wäre

    Du sprichst mit anderen Worten aus, dass die Prozessdauer (das “Leben”) nach der L-T aus Sicht des Systems S länger ist als aus der Sicht des Systems S’ (#231).
    Für mich hat das “wirklich” einen irreführenden Beigeschmack (“wirklich” = “absolut wahr”). Statt “wirklich” ist besser “aus Sicht anderen Systems” zu setzen, wie Du es selbst oft betonst. In Deinem folgenden Satz “Aus Sicht des Myons ist die entgegenkommende Strecke wirklich verkürzt” ist dann nur mehr das “wirklich” zu viel.
    Am ehesten könnte man in einer Unterhaltung als “wirkliche Lebensdauer” noch die im Ruhesystem S’ gemessene, kürzeste Prozessdauer bezeichnen, so wie man in einer Unterhaltung mit “wirklicher Länge” eines Maßstabs wohl seine längste Länge, nämlich seine Ruhelänge (und nicht die aus Sicht irgendeines bewegten Beobachters) meint.

    Die Behauptung ist, dass die Uhr in S’ aus Sicht von S langsamer läuft. Und umgekehrt.

    Deine Behauptung bezog sich – soweit ich das verfolgt habe – immer auf die “gesamte Zeit” bzw. alle Uhren des anderen Systems, nicht bloß auf eine bestimmte in diesem anderen System ruhende Uhr.
    Aber auch mit dieser Einschränkung ist die Aussage mE unrichtig und irreführend.
    “Aus Sicht von S” heißt “im Koordinatensystem S verzeichnet” (#229). Man kann die Anzeigen der Uhren (als “Ereignisse”) so verzeichnen, wie sie an Ort und Stelle eintreten, oder so, wie sie an einem anderen Ort des betreffenden Systems nach Zwischenschaltung von Lichtsignalen eintreten (Dopplereffekt).
    Wenn wir den Dopplereffekt als störend ausklammern, dann ergeben sich die Koordinatenwerte der Ereignisse (zB des Tickens am immer gleichen Ort eines Systems) durch L-T: Die Anzeige der Uhr, die im Ursprung des Systems S’ ruht, bleibt hinter den Anzeigen der Uhren des Systems S, an denen sie vorbeikommt, zurück, so wie umgekehrt die Anzeige der Uhr, die im Ursprung des Systems S ruht, hinter den Anzeigen der Uhren des Systems S’, denen sie begegnet, zurück bleibt (#217; dieses wechselseitige Zurückbleiben ist für den wechselseitigen transversalen Dopplereffekt verantwortlich).

    Mit diesen Anzeigen der Uhren des Systems S’ ist der Beweis erbracht, dass es – außer der Uhr S’, die gegenüber einer Uhr S nachgeht, zumindest eine Uhr S’ gibt, die gegenüber einer Uhr S vorgeht (dabei zeigen alle Uhren des Bezugssystems S die gleiche Zeit an, weil wir uns in dieses hineinversetzen).
    Dies widerlegt schon einmal die Behauptung, dass aus Sicht des einen Systems alle Uhren des anderen Systems langsamer laufen würden bzw. dass aus Sicht des einen Systems “die Zeit” des anderen Systems (die von allen dort ruhenden Uhren gleich angezeigt wird) langsamer vergehen würde.
    Aber auch die Behauptung, zumindest eine bestimmte Uhr in S’ würde aus Sicht von S langsamer laufen, kann nicht stimmen, weil es ausgeschlossen ist, dass eine der im System S’ ruhenden Uhren ein anderes Gangverhalten hat als alle anderen.
    Die “anders laufenden Uhren” sind nur eine irreführende Redensart, die auf #104 zurückgeht.

  236. #236 Karl-Heinz
    18. August 2018

    @PS
    In der Einfachheit liegt die wahre Schönheit. Darum folgendes Beispiel, welches es zu beantworten gilt um deine Überlegungen zu testen, derer ich nicht mehr ganz folgen kann.

    Entlang der Flugstrecke der Rakete werden Bojen mit einer eingebauten Atomuhr angebracht. Fliegt die Rakete an einer Boje vorbei, so übermittelt die Rakete ihre Bordzeit den Bojen. Nach Abschluss des Experiments werde die Daten (Bordzeit der Rakete im Augenblick des Vorbeifluges und Systemzeit der Boje) eingesammelt und ausgewertet. Die Bojen wurden mit einer so langsamen Geschwindigkeit zum jeweiligen Ort gebracht, dass ihre Systemzeit sich relativistisch nicht verändert hat.
    Frage: Wird ein Gangunterschied zwischen der Uhr der Rakete und unserer Uhr gemessen?

  237. #237 Philip
    Wuppertal
    18. August 2018

    “Dass meine Uhr schneller geht als Deine”: Diese Ausdrucksweise ist wieder ein Beispiel für das irreführende Narrativ (#217), das ein Verständnis der SRT erschwert und vernebelt.

    Völlig richtig, und ich entschuldige mich für meine unpräzise Ausdrucksweise. Das gilt allerdings auch für…

    …dass auf den Uhren der Uhrenreihe des Systems S’, an denen er vorbeikommt, mehr Zeit vergangen ist als auf der bei ihm ruhenden Uhr.

    …(Hervorhebung von mir), weil man nicht vernünftig RT betreiben kann, wenn man zugleich noch immer den Raum als das aktuell-Vorfindliche und die Zeit als etwas im Kopf hat, das „vergeht“.
    Raum und Zeit getrennt zu behandeln und gleichzeitig sauber und verständlich SRT zu betreiben ist einfach unmöglich.
    Nun zu den vorpräparierten im Raum verteilten Uhren: Da Geschwindigkeit relativ ist, kann man nicht sagen, dass sie ruhen, sondern nur, dass sie dieselbe Geschwindigkeit haben. Diese Eigenschaft ist nämlich bezugssystemunabhängig.
    Natürlich können sie auch synchronisiert worden sein und laufen dann synchron – in genau demjenigen Koordinatensystem, nennen wir es Σ in dem ihre gemeinsame Geschwindigkeit 0 ist.
    In einem anderen Koordinatensystem Σ’, dessen räumlicher Ursprung O’ sich in Σ mit v› bewegt, in dem also diese gemeinsame Geschwindigkeit der Uhren –v› ist, laufen die Uhren isochron, aber nicht synchron, und das liegt an der Relativität der Gleichzeitigkeit.
    Angenommen, in O’ sitzt ein Beobachter, der um 11:00Uhr (nach beiden Uhren, die er in diesem Moment so synchronisiert) mit v›=(0,8c|0|0) an einer bestimmten Uhr vorbeikommt. Die nächste Uhr befinde sich (Σ-mäßig) von dort aus bei Δs›=(1Lmin|0|0), also Δx=1Lmin*) und zeigt natürlich, da synchronisiert, ebenfalls 11:00Uhr an, wobei dies natürlich nur weiß (mit einem dollen Fernrohr würde man in diesem Augenblick 10:59 ablesen).
    Um nach Σ’ umzuinterpretieren, müssen wir LORENTZ-transformieren, und erhalten
    Δt’ = γ(Δt – (v/c)(Δx/c)) = 11:00Uhr + (5/3)·(0 – 0.8·60s) = 11:00Uhr – 80s = 10:58:40Uhr. Sie geht also um 80s vor. Schon deshalb wird die Uhr von O’ beim Vorbeiflug an der nächsten Uhr eine frühere Zeit anzeigen als diese.
    Was ich gern noch weiter ausführen werde.
    —-
    *)Wehe, jemand komme auf die Idee, das jetzt noch in m oder km umrechnen zu wollen und v› in m/s oder gar in km/h! Solche künstlichen Komplizierungen haben einen Erkenntniswert von nullinger, und wer das tut, gehört über’s Knie gelegt!

  238. #238 Karl-Heinz
    18. August 2018

    Philip

    Und ich glaubte immer, dass folgenden Aussagen stimmen. 😉

    •Relativistische Effekte sind real
    •Bewegte Uhren gehen ganz real langsamer!
    •Bewegte Körper sind ganz real kürzer!

  239. #239 Karl-Heinz
    18. August 2018

    @Philip

    •Relativistische Effekte sind real
    •Bewegte Uhren gehen ganz real langsamer!
    •Bewegte Körper sind ganz real kürzer!

    Stimmt das Ganze etwa nicht?

  240. #240 PS
    18. August 2018

    @Karl-Heinz #236

    Wird ein Gangunterschied zwischen der Uhr der Rakete und unserer Uhr gemessen?

    Ja, die Anzeige der Raketenuhr weicht von den Anzeigen unserer synchronisierter Bojenuhren ab (L-T).
    Aber es gibt keinen “Gangunterschied”, und zwar weder zwischen einzelnen Uhren der beiden Systeme, und schon gar nicht zwischen allen synchronisierten Uhren des Raketensystems gegenüber allen synchronisierten Uhren des Bojensystems.
    Die Zeitanzeige einer bestimmten Uhr ist eine Relation, so wie die Ausbreitung eines Lichtstrahls (Zeit wird mit Lichtuhren gemessen). Die Ausbreitung von Lichtstrahlen wird in verschiedenen Systemen unterschiedlich verzeichnet, dennoch würde niemand behaupten, zwischen den Lichtstrahlen bestünde ein “Gangunterschied”.

    Zu “real” #238 f: beschränkte Aussagekraft, ähnlich wie “wirklich” (#235).

  241. #241 Philip
    Wuppertal
    18. August 2018

    @Karlheinz
    real im Sinne von tatsächlich messbar und nicht nur scheinbar sind die Effekte, aber in einem ähnlichen Sinne wie eine 30cm lange Salami in eine nur 24cm lange Schublade passt, wenn man sie in einem 37°-Winkel hinlegt, und wie ein Schnitt in diesem Winkel durch eine Salami von 4cm Dicke eine elliptische Schnittfläche von 5cm großer Achse ergibt.

    Die Vorstellung einer bewegungsbedingten mechanischen Verformung in dem Sinne, dass man theoretisch zwei baugleiche Körper nebeneinander legen und anhand dessen, welcher kürzer sei, diesen als „den bewegten“ identifizieren könnte, ist definitiv Unfug.

    Es widerspricht dem wesentlichsten Grundsatz der RT: Bewegung ist relativ.

  242. #242 Albrecht Storz
    18. August 2018

    Alderamin
    1. August 2018 #148

    [Edit Alderamin] Ich bin diesen Kindergarten bald leid! OT gelöscht. [/Edit]

  243. #243 PS
    18. August 2018

    @Philip 237
    Bewegte Uhren laufen nicht synchron, aber isochron? Isochron wird mE nie verwendet und scheint synonym mit synchron zu sein. Auf welchen Unterschied willst Du mit diesen beiden Begriffen abstellen bzw. in welchem Sinn soll jetzt isochron richtig sein?

    [Zu traditionellen Maßeinheiten:] Solche künstlichen Komplizierungen haben einen Erkenntniswert von Null …

    Da sprichst Du mir aus dem Herzen, daher habe ich versucht, Dein Beispiel auf der Basis Zeiteinheit = 1 Lichtsekunde, Raumeinheit = 1 Lichtsekunde sowie der einer Zeitachse und einer Raumachse zu übersetzen, bin aber gescheitert: In O’ des Systems ∑’ soll ein Beobachter sitzen, der “nach beiden von ihm synchronisierten Uhren” [das macht nur Sinn, wenn diese beiden Uhren mit ihm aus Sicht des Systems ∑ “die Geschwindigkeit 0 haben”, also dort ruhen] zu einem bestimmten Zeitpunkt an einer “bestimmten Uhr” [offenbar an einer in ∑ ruhenden Uhr] vorbeikommt. Die “nächste Uhr” (der beiden genannten, in ∑’ ruhenden Uhren?) soll sich aus Sicht des Systems ∑ (“Σ-mäßig”) bei Δx=1Lmin befinden und “zeigt natürlich, da synchronisiert, ebenfalls 11:00 Uhr an”, jetzt also offenbar wieder aus Sicht des Systems ∑’. Jetzt soll diese Mischung aus Koordinatenangaben zweier Systeme über die L-T nach ∑’ uminterpretiert werden. Das kann ich nicht mehr nachvollziehen.

    Vielleicht möchtest Du aber ohnehin eine Situation darstellen, wie sie in #231 geschildert wird (Myon = “bewegte Uhr”, ruhend im System S’ (bzw. ∑’), aber -1 vom Ursprung des Systems S’ entfernt; Erdoberfläche = “die nächste ruhende Uhr”, ruhend im Ursprung des System S (bzw. ∑).

    Sie geht also um 80s vor. Schon deshalb wird die Uhr von O’ beim Vorbeiflug an der nächsten Uhr eine frühere Zeit anzeigen als diese.

    Im Beispiel #231 geht beim Start die Uhr des im Systems S’ ruhenden Myons um 0,75 vor. Das ist aber mE nicht der eigentliche Grund, warum sie “beim Vorbeiflug an der nächsten Uhr” [gemeint: bei der Begegnung mit der im Ursprung des Systems S ruhenden Uhr] eine frühere Zeit [eine kürzere Prozessdauer] anzeigt. Die an einer ruhenden Uhrenreihe vorbeibewegte Uhr würde auch dann eine kürzere Prozessdauer anzeigen, wenn sie beim Start die gleiche Zeit anzeigt wie die ihr gerade begegnende ruhende Uhr.

  244. #244 Karl-Heinz
    18. August 2018

    @Philip

    Die Vorstellung einer bewegungsbedingten mechanischen Verformung in dem Sinne, dass man theoretisch zwei baugleiche Körper nebeneinander legen und anhand dessen, welcher kürzer sei, diesen als „den bewegten“ identifizieren könnte, ist definitiv Unfug.

    Wenn sich ein Körper relativ zu mir bewegt, kann ich sehr wohl seine Relativgeschwindigkeit zu mir und seine Länge messen. Wie kommst du auf die Idee, dass sich der Körper dabei mechanisch verformt. Natürlich ist oder erscheint die gemessene Länge eines bewegten Körper für uns kürzer als seine Ruhelänge. Was die Ursache dafür ist, sollte zum jetzigen Zeitpunkt eigentlich schon klar sein, oder?

  245. #245 Karl-Heinz
    18. August 2018

    [Edit Alderamin] Der Fairness halber auch gelöscht. [/Edit]

  246. #246 Philip
    Wuppertal
    18. August 2018

    @Karl Heinz

    Wie kommst du auf die Idee, dass sich der Körper dabei mechanisch verformt?

    Das traditionelle Wording legt dies nahe, nicht ich – im Gegenteil. Ich lehne dieses Wording ja ab.

    Natürlich ist oder erscheint die gemessene Länge eines bewegten Körper für uns kürzer als seine Ruhelänge.

    Weil es so etwas wie ein Schrägschnitt ist. Ein Schrägschnitt durch die „Weltwurst“.

    Nicht der in Σ’ ruhende Körper an sich hat sich „zusammengezogen“, sondern für den in Σ ruhenden Beobachter sind zwei Ereignisse am Anfang und Ende des Körpers gleichzeitig, die es in Σ’ gar nicht sind und deren absoluter Abstand
    Δς = √{Δs’² – c²Δt’²}
    ohnehin kleiner ist als Δs'(t’=const.).

  247. #247 Philip
    18. August 2018

    @PS

    Bewegte Uhren laufen nicht synchron, aber isochron?

    Das kann man nicht so pauschal sagen. Uhren, deren Geschwindigkeit (einschließlich Richtung!) gleich ist, laufen stets isochron (vorausgesetzt, sie sind alle o.k.).

    Isochron wird mE nie verwendet und scheint synonym mit synchron zu sein.

    Ist es nicht. Synchron ist immer auch isochron, aber im Allgemeinen nicht umgekehrt: https://de.m.wikipedia.org/wiki/Synchronität#Verwandte_Bezeichnung_zur_Charakterisierung_zeitlicher_Vorgänge

  248. #248 Philip
    Wuppertal
    19. August 2018

    @PS, #229

    Die von Dir ins Spiel gebrachte „Zeitachse“ (aber nur „sozusagen“), an der der im Ursprung des Systems S‘ ruhende Beobachter S‘ mit seiner bei ihm ruhenden Uhr entlang fährt, ist nichts anderes als die im anderen System S hintereinander in gleichen Ruheabständen aufgestellte Uhrenreihe,…

    Nein, eine Zeitachse ist nicht nur sogenannt, sondern eine Weltlinie. Die Zeitachse von Σ ist die Weltlinie des Referenzpunktes O, die von Σ’ ist die des Punktes O’.
    Ich bezeichne mit Σ ein Koordinatensystem mit 4 Achsen, das mir die gesamte Raumzeit kartographiert. Sein Ursprung ist (O, t=0), was entweder ein ganz bestimmtes Ereignis ist oder die Position des Referenzpunktes O „jetzt“ (in diesem Fall hätte man einen beweglichen Zeitnullpunkt, sodass t0 die Zukunft bedeutete).

    Mit S, genauer S[t], würde ich ein räumliches Koordinatensystem bezeichnen, das nur den 3D-Raum zum Zeitpunkt t (sozusagen eine „Ebene“ in der Raumzeit) kartographiert.

    S'[t’] übrigens kartographiert nicht denselben 3D-Raum wie S[t], sondern einen gewissermaßen gegen S[t] „gekippten“ Raum – und das ist das Neue in der Relativitätstheorie.

  249. #249 Alderamin
    19. August 2018

    @Philip

    Das Bild mit der schrägen Salami lass‘ ich gelten, denn die passt ja auch wirklich in die Schublade. Und ein längenverkürztes Raumschiff passt wirklich in eine Garage, die kürzer ist, als das Raumschiff in seiner eigenen Wahrnehmung. Man kann ganz kurz die vordere und hintere Türe gleichzeitig schließen und das Raumschiff ist im Durchflug kurz vollständig drin (Garagenparadoxon). Das ist so real, wie‘s werden kann. Wie man‘s dann nennt, ist Kosmetik.

  250. #250 Karl-Heinz
    19. August 2018

    @Alderamin

    Und wenn man die Längenverkürzung im Minkowski-Diagramm nachvollziehen will, so muss man im Hinterkopf haben, dass die x und x’-Achse unterschiedlich skaliert sind, denn sonst steht man wie ein Esel vor dem Tor und man checkt es einfach nicht. Die Position von Spitze und Ende eines bewegten Körpers wird dann zur gleichen Zeit gemessen und daraus die Länge ermittelt.

  251. #251 Philip
    Wuppertal
    19. August 2018

    Hallo Alderamin,
    bitte missverstehe mich nicht: Das Salami-Bild versteht sich als eines, das mutatis mutandis auf die Maße bewegter Körper angewandt werden soll.

    Dabei steht die Salami nicht für einen Körper als solchen, sondern für die „Weltwurst“-Abschnitt eines Körpers, und ihre Länge L nicht für seine Länge, sondern für die Dauer T dieses „Weltwurst“-Abschnittes respektive eines Vorgangs, der an einen Körper gebunden ist.

    Für die Ausdehnung des Körpers – auch in seiner potentiellen Bewegungsrichtung – steht der Durchmesser d.

    Mutatis mutandis deshalb, weil die Abmessungen der Salami der EUKLIDISCHen Metrik

    (1.0) Δs = √{Δz² + Δx²}

    unterliegt, wobei Δs in diesem Fall L und

    (1.1) Δz = L·cos(θ)
    (1.2) Δx = L·sin(θ)

    ist, während der „Weltwurst“-Abschnitt der MINKOWSKI-Metrik

    (2.0) Δτ = √{Δt² – Δs²/c²}

    unterliegt, wobei Δτ hier T und

    (2.1) Δt = T·cosh(ζ)
    (2.2) Δs = c·T·sinh(ζ)

    mit der Rapidität ζ=artanh(v/c) ist.

    Und ein längenverkürztes Raumschiff passt wirklich in eine Garage, die kürzer ist, als das Raumschiff in seiner eigenen Wahrnehmung. Man kann ganz kurz die vordere und hintere Türe gleichzeitig schließen…

    …was aber nur in Σ – dem Ruhesystem der Garage oder besser eines zweitürigen Hangars – gleichzeitig ist. Im Ruhesystem Σ’ des Raumschiffes erfolgt die Schließung (und hoffentlich wieder Öffnung, wenn man eine Katastrophe vermeiden will) der vorderen Türe jedoch früher als die der hinteren.
    Das Raumschiff ist niemals eindeutig und unbestreitbar (unabhängig vom Bezugssystem) kürzer als der eigentlich gleich lange Hangar.

  252. #252 Karl-Heinz
    19. August 2018

    @Philip

    Doch, doch lieber Philip. Man kann die Länge eines bewegten Körper messen. Diese Länge L ist dann meine Realität.
    Wie misst man eigentlich eine Länge?
    L=x2-x1
    Wie misst man eigentlich eine Länge eines bewegten Körpers?
    L(t)=L=x2(t)-x1(t)
    Das ist ja überhaupt nicht kompliziert, oder? 😉

  253. #253 Alderamin
    19. August 2018

    @Philip

    Das Raumschiff ist niemals eindeutig und unbestreitbar (unabhängig vom Bezugssystem) kürzer als der eigentlich gleich lange Hangar.

    Unabhängig vom Bezugssytem natürlich nicht, aber was interessieren mich Beobachtenden anderer dahergeflogener Leute Bezugssysteme? 😀

  254. #254 Karl-Heinz
    19. August 2018

    @Philip

    Ein ruhendes Elektron hat ein kugelsymmetrisches elektrisches Feld. Bei einem bewegten Elektron ist sein Feld nicht mehr kugelsymmetrisch. Das ist die Realität für mich. Jemand der sich mit dem Elektron bewegt sieht halt eine andere Realität.

  255. #255 Karl-Heinz
    19. August 2018

    @Philip

    Frage an Philip. Darf ich den Magnetismus behalten? Ich habe gerade erfahren, dass der
    Magnetismus ebenso wie die Längenverkürzung ein relativistisches Phänomen ist. 🙂

  256. #256 Philip
    Wuppertal
    19. August 2018

    @Karl-Heinz
    ich habe ja auch nicht geschrieben, das Raumschiff sei nicht real kürzer als der eigentlich gleich lange Hangar, sondern es ist dies nicht eindeutig und unbestreitbar im Sinne von bezugssystemunabhängig.

    Wobei: Die Länge des Raumschiffs entlang der t=const. – Ebene ist definitiv kürzer. Dies freilich ist keine Kontraktion – ebenso wie die Tatsache, dass der Schrägschnitt durch eine Salami der Dicke d im Winkel θ die größere Breite d/cos(θ) hat, bedeuten würde, dass die Salami insgesamt in die Breite gezogen würde.

  257. #257 Karl-Heinz
    19. August 2018

    @Philip

    Dafür gibt’s dann ja eh die beiden Größen Eigenzeit und Eigenlänge. 😉

  258. #258 Philip
    Wuppertal
    19. August 2018

    @Alderamin
    Was heißt hier „andererleuts Bezugssysteme“?
    Es gibt nicht Dein oder mein Bezugssystem, es gibt unterschiedliche Koordinatensysteme, aus denen man eines als Bezugssystem auswählen kann.

    Wenn ich in einem Zug sitze, kann ich natürlich den Erdboden als riesiges Laufband ansehen, auf dem der Zug auf der Stelle rollt – das muss ich aber nicht.

    Man könnte sagen, ein Bezugssystem, das an den Erdboden gebunden ist, sei ja nicht „meines“, was also kümmert das mich? Es ist aber schon üblich, so ein Koordinatensystem als Bezugssystem zu verwenden, auch wenn man sich relativ zum Erdboden bewegt.

    Im Übrigen ist die Betrachtung ein und desselben Geschehens in verschiedenen möglichen Bezugssystemen und das herauskristallisieren von Größen, die vom Bezugssystem unabhängig sind, das A und O einer Theorie mit dem Wort „Relativität“ im Namen.
    That’s what Relativity is all about.

  259. #259 Karl-Heinz
    19. August 2018

    @Philip

    Man könnte sagen, ein Bezugssystem, das an den Erdboden gebunden ist, sei ja nicht „meines“, was also kümmert das mich? Es ist aber schon üblich, so ein Koordinatensystem als Bezugssystem zu verwenden, auch wenn man sich relativ zum Erdboden bewegt.

    Na ja, wenn ich im Zug sitze, dann hätte ich schon gerne ein Bezugssystem, das an den Zug gebunden ist. Und wenn’s so richtig relativistisch wird, dann sowieso.

  260. #260 Philip
    Wuppertal
    19. August 2018

    Hallo Karl-Heinz,
    Du kannst jedwedes Bezugssystem haben, das Dein Herz begehrt. Das ist ja das Schöne an der Relativitätstheorie.

    Üblicherweise verhält man sich allerdings inkonsistent und verwendet ein am Zug orientiertes Bezugssystem Σ’ für alles, was sich im Zug abspielt, und ein am Erdboden orientiertes, Σ, als „Außensystem“, das alles Äußere und die Bewegung des Zuges selbst beschreibt.
    Das darf man gern machen, solange man weiß, was man tut, und solange man nicht so einen Käse redet wie „sich innerhalb oder außerhalb eines Bezugssystems befinden“, als sei der Zug selbst Σ’.

  261. #261 Alderamin
    19. August 2018

    @Philip

    Es gibt nicht Dein oder mein Bezugssystem, es gibt unterschiedliche Koordinatensysteme, aus denen man eines als Bezugssystem auswählen kann.

    Nein, aus der Sicht eines Experimentators kann man sich kein Bezugssystem aussuchen. In jedem Bezugssystem liefern alle Experimente das gleiche Ergebnis, nämlich dasjenige, das einem ruhenden System entspricht. Im fahrenden Zug hopst ein gedribbelter Ball senkrecht auf und ab. Wie das aus anderen Bezugssystemen aussieht, kann man nur indirekt schließen/ausrechnen, aber nicht direkt messen.

    Die objektive Realität ist aber genau das, was man messen kann. Deswegen ist das eigene Bezugssystem immer das für die Realität relevante. Dass die Realität in anderen Bezugssystemen anders aussieht, liegt in der Natur der RT. Eine Realität über alle Bezugssysteme gibt es spätestens seit der RT nicht mehr. Bestimmte objektive Größen sind eben vom Bezugssystem abhängig.

  262. #262 Philip
    Wuppertal
    19. August 2018

    Hallo Alderamin,
    Relativität bedeutet m.E. nicht, dass es mehrere Realitäten gibt – nicht in dem Sinne, wie ich das Wort verstehe.
    Real ist, was gemessen wird, das ist schon richtig. Viele Messungen sind jedoch nicht interpretationsfrei. Eine Ausnahme ist die entlang einer Weltlinie – von einer mitgeführten Uhr – gemessene Eigenzeit, auf die ich später zurückkomme.

    Nein, aus der Sicht eines Experimentators kann man sich kein Bezugssystem aussuchen.

    Aber sicher! Relativität bedeutet, dass er sich jede beliebige aktuelle Geschwindigkeit v› zuschreiben können muss, einschließlich 0. Seine Messergebnisse werden mit jeder solchen Interpretation konsistent sein.

    Wie das aus anderen Bezugssystemen aussieht, kann man nur indirekt schließen/ausrechnen, aber nicht direkt messen.

    Man kann vieles nicht direkt messen, so etwa alles, das sich woanders ereignet. Man kann es nur ex post beobachten, und der genaue Zeitpunkt eines solchen Ereignisses ist Interpretation und hängt vom Bezugssystem ab, die Messergebnisse selbst jedoch nicht.

    Eine Realität über alle Bezugssysteme gibt es spätestens seit der RT nicht mehr.

    Das ist genau falsch. Ich nannte eben die Eigenzeit als Beispiel für genau dies. Wenn Du in einer Launge einen Kaffee trinkst und Deine Armbanduhr beim letzten Schluck 5 min später anzeigt als beim ersten (z.B. 09:05 und 09:10), ist das eindeutig und für jeden Beobachter so, in jedem Bezugssystem.
    Und natürlich sämtliche Naturgesetze, d.h. grundlegenden Beziehungen zwischen physikalischen Größen. Die meisten Größen selbst mögen in unterschiedlichen Koordinatensystemen unterschiedliche Werte haben, die Beziehungen jedoch sind dieselben. Der Betrag c der Lichtgeschwindigkeit zum Beispiel.
    Von daher könnte die Relativitätstheorie ebensogut ‘Absolutheitstheorie’ heißen, weil sie die Absolutheit solcher Beziehungen unterstreicht.

  263. #263 Karl-Heinz
    20. August 2018

    @Philip

    Ohne Bezugssystem kann man die Realität schwer abbilden. Die Realität ist deswegen schon an ein Bezugssystem gekoppelt.

    Es gilt, dass grundlegenden physikalischen Gesetze gegenüber dem Wechsel zwischen den Bezugssystemen invariant sein müssen. Zum Beispiel stimmt die Lichtgeschwindigkeit in allen Bezugssystemen überein.

  264. #264 Karl-Heinz
    20. August 2018

    @Philip

    Um die Realität zu beschreiben, benötigt man immer ein Bezugssytem. Natürlich gibt es Größen, die in allen Bezugsystemen gleich sind. Aber auch hier ist es sinnvoll das Bezugsystem anzugeben um deren Berechnungen später nachvollziehen zu können. Wenn nach einer Eigenlänge gefragt ist, weiß ich sehr wohl, dass es nicht die Länge ist, die ich vom bewegten Körper gemessen habe, sondern es sich um jene Länge handelt, die der Körper einnimmt, wenn er in Ruhe ist. Jetzt zu argumentieren, dass meine gemessene Länge vom bewegten Körper nicht real ist, sei mir bitte nicht böse, ist grober Unfug.

  265. #265 Philip
    Wuppertal
    20. August 2018

    Hallo Karl-Heinz,

    Um die Realität zu beschreiben, benötigt man immer ein Bezugssytem.

    Das bestreite ich überhaupt nicht. Zumindest auf bestimmte Größen trifft das zu, auf andere weniger. Ich lege nur gesteigerten Wert drauf, dass man sich und seinen Zuhörern stets klar macht, dass dieses Bezugssystem Σ, wie ich es nennen möchte, eine von mehreren möglichen Interpretationen der Realität darstellt und die Wahl eines anderen Koordinatensystems Σ’ zum Bezugssystem mit demselben Recht möglich ist.
    Das ist umso wichtiger, wenn es um Relativität geht, denn da geht’s ja gerade um die Freiheit der Bezugssystem-Auswahl.

    Natürlich gibt es Größen, die in allen Bezugsystemen gleich sind. Aber auch hier ist es sinnvoll das Bezugsystem anzugeben um deren Berechnungen später nachvollziehen zu können.

    In praxi macht man das oft nicht zuerst. Die Länge L=30cm einer Salami auf einem Tisch wird man eher direkt messen und nicht erst ein Koordinatensystem aufzeichnen und dann die z- und x- Positionen der Enden vergleichen, um dann erst durch die Anwendung des PYTHAGORAS

    √{(z₂-z₁)² + (x₂-x₁)²} = L

    zu berechnen.

    Wenn nach einer Eigenlänge gefragt ist, weiß ich sehr wohl, dass es … sich um jene Länge handelt, die der Körper einnimmt, wenn er in Ruhe ist.

    Mit dem „wenn“ ist das keine ‘Relativity’-Argumentation, sondern eine im Sinne der LORENTZ’schen Äthertheorie. Da gibt er einen Äther, und der definiert, was stationär ist – eigentlich.
    Denn dank einer Art Verschwörung dieses Äther, zu der diese „Längenkontraktion“ gehört, merkt der Depp auf der Brücke des Raumschiffs nix davon, dass er sich relativ zu diesem Äther bewegt.
    Eine Argumentation im Sinne der Relativitätstheorie würde nicht „wenn“, sondern „in dem Koordinatensystem, in dem“ lauten. Wobei natürlich ein (1+3)D- Koordinatensystem Σ’ gemeint ist. Es ist Unfug, beim Betreiben der Relativitätstheorie Raum und Zeit getrennt halten zu wollen.

    Jetzt zu argumentieren, dass meine gemessene Länge vom bewegten Körper nicht real ist, sei mir bitte nicht böse, ist grober Unfug.

    Ich bin Dir nicht böse, sondern nur etwas genervt, andauernd falsch verstanden zu werden.
    Ich sage nicht, die Messung sei nicht real, sondern, dass sie indirekt und nicht interpretationsfrei ist. Du kannst halt nicht in aller Ruhe ein Lineal an einen Körper halten, der sich relativ zu ihm bewegt, geschweige denn einen Messschieber benutzen. Was Du benutzen kannst, ist eine Lichtschranke, aber dann verwendest Du dabei eine Uhr, und in Σ’ ist dies eine mit -v› bewegte Uhr, die somit um den Faktor γ langsamer geht, sodass Du die gemessene/berechnete Länge mit γ multiplizieren musst, um die Σ’-Länge herauszufinden. In Σ ist das natürlich anders.
    Das Raumschiff ist nicht per se kürzer, sondern entlang einer t=const.-„Ebene“ von Σ. Schrägschnitt halt.

  266. #266 PS
    20. August 2018

    @Karl-Heinz #264
    Das Gemessene ist “real” oder “wirklich”. Es gibt nicht ein richtig Gemessenes, das mit der Wirklichkeit nicht übereinstimmen würde), aber die Relativität führt zu verschiedenen Wirklichkeiten, sodass der Begriff vermieden werden sollte. Würde man sich daran halten, (u “real” #238 f: beschränkte Aussagekraft, ähnlich wie “wirklich” (#235), könnte man sich ein aneinander Vorbeireden ersparen.

  267. #267 Peter
    20. August 2018

    Das symmetrische Myonenexperiment führt zu inkonsistenten Interpretationen: Zwei Myonen kommen mit v aufeinander zu und treffen sich in einem Raumzeitpunkt. Je nach Wahl des Bezugssytems überlebt mal das eine, mal das andere. Deshalb lässt sich die lange Lebensdauer der Myonen nicht mit der SRT erklären. Das ZP hingegen ist gar nicht paradox, denn nur einer der Zwillinge wechselt das Koordinatensystem, der andere nicht. Hier gibt es keine Inkonsistenz.

  268. #268 Alderamin
    20. August 2018

    @Peter

    Ich hatte doch in #182 vorgerechnet, dass es keine Inkonsistenzen bei den Myonenlebensdauern gibt. Über das Überleben besteht beim Zusammentreffen Konsenz zwischen den Beobachtern, nur nicht darüber, wann die Uhren zuvor gestartet wurden. In der SRT ist die Gleichzeitigkeit relativ.

  269. #269 PS
    20. August 2018

    @ Philip #265

    Was Du [für die Ermittlung der Länge eines in ∑’ ruhenden Maßstabs aus der Sicht von ∑] benutzen kannst, ist eine Lichtschranke, aber dann verwendest Du dabei eine Uhr, und in Σ’ ist dies eine mit -v› bewegte Uhr, die somit um den Faktor γ langsamer geht, sodass Du die gemessene/berechnete Länge mit γ multiplizieren musst, um die Σ’-Länge herauszufinden.

    Wieder ein Beispiel, wie irreführend das Narrativ von der “langsamer gehenden Uhr” ist.
    Die in ∑ aufgestellte Lichtschranke benötigt zwei dort ruhende Uhren. Die messen sowohl die Koordinatengeschwindigkeit des Maßstabs v = 0,6 (Bewegung nach rechts, Osten) als auch dessen Länge “1”aus Sicht von ∑ (“Start” des rechten Endes des Maßstabs” bei der linken Uhr zur Zeit t=0, Ankunft des linken Endes des Maßtabs bei der linken Uhr zur Zeit t= 1,666, “gleichzeitige Position der Maßstabsenden” zur Zeit t=1,666 in einer Entfernung von “1”).

    Aus der Sicht von ∑’ ist das also nicht “eine” bewegte Uhr, sondern es sind zwei.
    Die in ∑ gleichzeitige Anwesenheit der beiden Enden des bewegten Maßstabs bei den beiden Uhren der Lichtschranke findet aus der Sicht von ∑’ zu verschieden Zeitpunkten (im Beispiel t’ links = 1,33 bzw. t’ rechts = 2,083) statt, wobei diese Ereignisse 1,25 (= “Ruhelänge des Maßstabs”) voneinander entfernt stattfinden.

    Die rechte Uhr in ∑’ zeigt weniger Zeit an als die Uhr von ∑.
    Die linke Uhr in ∑’ zeigt mehr Zeit an als die Uhr von ∑.
    Schon deshalb ist es – wie oftmals betont – verfehlt, davon zu sprechen, dass in einem System die Zeit langsamer vergehen würde als im anderen.
    Dieses Auseinanderfallen der beiden Anzeigen war schon zu Beginn des Prozesses (rechtes Ende bei linker Uhr zur Zeit t=0, t’=0) vorhanden: rechte Uhr in ∑’ t’=0,75; linke Uhr in ∑’ t’ = 0.

    Die größere räumliche Entfernung der Ereignisse (1,25) ist also nicht darauf zurückzuführen, dass “eine mit -v› bewegte Uhr um den Faktor γ langsamer geht”, sondern die größere Entfernung ist von Anfang des Prozesses an da, so wie die Abweichung der Anzeige der beiden Uhren aus Sicht ∑ von Anfang an da war.

    Selbst wenn diese beiden Uhren mit verschieden Anzeigen im Zuge der Bewegung “schneller” oder “langsamer” gehen würden (diese Ausdrucksweise ist aber wie gesagt sinnlos), dann hätte das nicht den geringsten Einfluss auf die Länge des Maßstabs (am Abstand der beiden Uhren aus Sicht ∑).

    Natürlich kann man sich aus alter Gewohnheit immer wieder vorsagen “aber dann verwendest Du dabei eine Uhr, und in Σ’ ist dies eine mit -v› bewegte Uhr, die langsamer geht” usw., aber man legt sich keine Rechenschaft mehr ab, was man mit diesen Worten eigentlich meint, und man realisiert nicht, dass man sich von einem echten Mitdenken verabschiedet hat.

    Keiner traut dem eigenen kritischen Verstand, jeder möchte beklatscht werden und mit der Mehrheit auf die Unbelehrbaren herabschauen.

    Seit Einstein (auf Basis SRT) sagte, “am Äquator gehen Uhren langsamer”, sind wir verzaubert. Stellen wir uns auf einer rotierenden Scheibe (mit sehr großem Durchmesser, zB ein Lichtjahr) einander gegenüber auf. Jetzt – sagt der Jargon – sieht jeder die Uhr des anderen langsamer gehen. Wer altert schneller?

  270. #270 Alderamin
    20. August 2018

    @Philip

    Du kannst halt nicht in aller Ruhe ein Lineal an einen Körper halten, der sich relativ zu ihm bewegt

    Warum nicht? Das Garagenparadoxon ist doch genau das: die Länge des ruhenden Hangars mit derjenigen des Raumschiffs zu vergleichen. Natürlich praktisch so nicht durchführbar, aber man könnte das Raumschiff beispielsweise mit hellem Blitzlicht (Picosekunden-Laserpuls) fotografieren und auf dem Foto den Sehwinkel und das Verhältnis Höhe zu Länge bestimmen. Bei bekannter Entfernung hast Du auch die absoluten Maße (also in Metern statt °).

  271. #271 Frank Wappler
    https://pdglive.lbl.gov/DataBlock.action?node=S004T
    20. August 2018

    Alderamin (#234, 17. August 2018):
    > […] gehen wir hier davon aus, dass das betrachete Myon eben eines sei, das die Strecke vom Entstehungsort bis zum Erdboden gerade eben schafft

    Das war bisher durchwegs so, und sollte auch weiterhin nicht schwerfallen.

    > und vergleichen eines, das der RT unterliegt mit seinem hypothetischen Pendant in einem Galileischen Raum.

    Am Aufstellen von Hypothesen über ein “Pendant in einem Galileischen Raum“, was auch immer das sein soll, beteilige ich mich jedenfalls nicht absichtlich.

    Mir geht’s allein um die Messgrößen, die im Rahmen der RT nachvollziehbar definiert sind; und wie sich über deren Anwendung und über die damit ermittelten Messwerte unmissverständlich sprechen lässt.

    Dazu gehört nun mal unter anderem, die Lebensdauer eines Myons nicht mit der Aufzeichnungsdauer eines Detektorsystems zu verwechseln, das den Lebensverlauf des Myons überwachte;
    und die Distanz zwischen den beiden Enden eines Zuges nicht mit der Distanz zwischen zwei Eisenbahnschwellen zu verwechseln, die gleichzeitig anzeigten, dass sie das eine bzw. das andere Zugende passierte.

    > […] Die “Tyrannei der Präzision” behindert die Wissenschaftskommunikation

    We must be able to communicate what we have asked and what we have done an what we have found.

  272. #272 Alderamin
    20. August 2018

    @Frank Wappler

    Am Aufstellen von Hypothesen über ein “Pendant in einem Galileischen Raum“, was auch immer das sein soll, beteilige ich mich jedenfalls nicht absichtlich.

    Das ist die zu falsifizierende Antithese! (Wenn die RT nicht gelten würde, würden die Myonen den Erdboden nicht – in entsprechender Anzahl – erreichen können).

  273. #273 Frank Wappler
    20. August 2018

    Alderamin schrieb (#249, 19. August 2018):
    > Man kann ganz kurz die vordere und hintere Türe gleichzeitig schließen und das Raumschiff ist im Durchflug kurz vollständig drin (Garagenparadoxon).
    > […] eine Garage, die kürzer ist, als das Raumschiff […]

    Ganz recht: die Garage kann kürzer sein als das Raumschiff.
    Um die beschriebene Versuchsanordnung zu erfüllen, reicht es aus, dass die Länge der Garage (die Distanz der beiden Garagentüren voneinander) nur ein Wenig mehr als das \sqrt{1 - \beta^2}-fache der Länge des Raumschiffs (Distanz der beiden Raumschiff-Enden voneinander) ist.

    > […] ein längenverkürztes Raumschiff

    Das betreffende Raumschiff dabei als “längenverkürzt” zu bezeichnen, obwohl es doch schlicht und proper seine bestimmte Länge hat und behält, ist aber bestenfalls unnötig …

  274. #274 Frank Wappler
    20. August 2018

    Alderamin schrieb (#272, 20. August 2018):
    > Wenn die RT nicht gelten würde […]

    Wenn nicht die im Rahmen der RT nachvollziehbar definierten Messgrößen (Lorentzsche Distanz, Dauer, chronometrische Distanz, Geschwindigkeit, …) untersucht und die entsprechenden Messoperatoren angewandt würden, um Versuch für Versuch ggf. Messwerte dieser Messgrößen zu ermitteln, dann —
    welche denn sonst ??

  275. #275 Alderamin
    20. August 2018

    @Frank

    Hier ging es darum, Belege für die Richtigkeit (und Widerspruchsfreiheit) der RT vorzulegen, da manche in der Diskussion diese anzweifelten. Die Zeitdilatation lässt sich anhand von Prozessen belegen, die in Ruhe schneller ablaufen als bei relativistischer Geschwindigkeit (relativ zum Beobachter). Die Zerfallsdauer von Myonen ist ein Beispiel für so einen Prozess. Man vergleicht die Strecke, die ein Myon durchschnittlicher Lebensdauer ohne Zeitdilatation höchstens hätte zurücklegen können (maximal mit c) mit derjenigen, die tatsächlich beobachtet wird, und zeigt dann, dass nur mit dem Effekt der Zeitdilatation die Ankunftsrate der Myonen am Erdboden erklärbar ist.

    Wenn die Betrachtung der zu wiederlegenden Antithese nicht zulässig sein soll, wie soll man belegen, dass die Zeitdilatation die Lebensdauer vergrößert hat? Wenn man bei einem Widerspruchsbeweis die falsche Prämisse nicht voraussetzen darf, wie soll man diese dann zu einem Widerspruch führen?

  276. #276 Alderamin
    20. August 2018

    @Frank

    Das betreffende Raumschiff dabei als “längenverkürzt” zu bezeichnen, obwohl es doch schlicht und proper seine bestimmte Länge hat und behält, ist aber bestenfalls unnötig …

    Wenn es in die Garage passt, die kürzer ist als das Raumschiff in relativer Ruhe, dann ist es längenverkürzt. Im Bezugssytem der Garage. Wohlgemerkt.

  277. #277 Philip
    Wuppertal
    20. August 2018

    Hallo PS (zu #243),
    ich sehe Klärungsbedarf, denn in #237 habe ich mich unklar ausgedrückt.
    Die Koordinatensysteme Σ und Σ’ (ich würde da nicht das Summenzeichen ‘∑’ verwenden) sind natürlich (1+3)D, sie kartographieren die gesamte Raumzeit, und ihre Zeitachsen sind dabei nichts anderes als die Weltlinien der Punkte O und O’. Das `O` steht hier für origin, könnte aber auch für observer oder object stehen.

    In O’ des soll ein Beobachter sitzen,…

    …respektive sich mit v›=(0,8c|0|0) mitbewegen, in Σ betrachtet. Der Einfachheit halber haben wir die v› -Richtung als x- Richtung identifiziert.
    Entlang der x-Achse sind Uhren Ui positioniert, jede Lichtminute eine, beginnend mit U0 in O. Und die sind untereinander Σ – synchronisiert.
    Auch der relativ zu O bewegte Beobachter hat natürlich eine Uhr U’, die natürlich nicht synchron und auch nicht isochron mit Ui läuft. Wenn ich geschrieben habe…

    …nach beiden Uhren, die er in diesem Moment so synchronisiert

    …ist das natürlich inakkurat ausgedrückt. Gemeint ist nur, dass U’ in dem Moment, in dem sie an U₀ vorbei kommt, dieselbe Zeit t’=0 anzeigt wie diese (t=0).
    Die 11:00:00Uhr habe ich willkürlich gesetzt, weil wir Menschen mit einer Uhrzeit besser vertraut sind als mit einem abstrakten „t=soundsoviel Sekunden“. Dies ist der bestimmte Zeitpunkt, zu dem U’

    …an einer „bestimmten Uhr“ vorbeikommt…

    …, nämlich U₀.

    Die „nächste Uhr“ (der beiden genannten, in Σ‘ ruhenden Uhren?)…

    Richtig, und ich nenne die nächste Uhr U₁.

    …soll sich aus Sicht des Systems Σ („Σ-mäßig“) bei Δx=1Lmin befinden und „zeigt natürlich, da synchronisiert, ebenfalls 11:00 Uhr an“, jetzt also offenbar wieder aus Sicht des Systems Σ.

    Eben nicht. U₀ und U₁ zeigen in der t=0 – „Ebene“ 11:00Uhr an, wenn U’ an U₀ vorbeikommt, also „aus Sicht von Σ“ oder besser in Σ ausgedrückt. Auch wird U’ U₁ in 01:15min erreichen, dabei aber nur Δt/γ=00:45min anzeigen.
    In Σ’ ausgedrückt kommt um 11:00:00Uhr (t’=0) natürlich U₀ an U’ vorbei, und U₁ nähert sich.
    Der Zeitpunkt und Ort in Σ’, zu dem U₁ 11:00:00Uhr angezeigt hat, wird durch LORENTZ-Transformation Σ→Σ’ ermittelt und liegen bei
    t’ = –γ(v/c)(x/c) = –(5/3)(4/5)60s = –80s, also 10:58:40Uhr und
    x’ = γx = (5/3)Lmin = 100Ls.
    Das ist weiter weg, liegt aber in Σ’ auch in der (relativen) Vergangenheit. In den seitdem auf U’ vergangenen 80s muss U₁ 64Ls näher gekommen sein (also x'(t’=0)=36Ls) – sodass sie U’ mit v=–0,8c in 45s passieren wird – und t=+(3/5)80s=+48s anzeigen muss.
    In Σ’ geht also U₁ langsamer, gegenüber U₀ aber um satte 48s vor!
    Durch den langsameren Takt wird die Zeitanzeige von U₁ in den 45min bis zum Passieren von U’ nur um (3/5)45min=27min weiter vorrücken, dann also t=+75s anzeigen, also 01:15min. Das ist genau das, was in Σ auch herauskommt, wie es ja auch sein muss.

  278. #278 PS
    20. August 2018

    @Alderamin #270

    Wären eine Garage A und eine Leiter B – jeweils in ihren Ruhesystemen gemessen – gleich lang und bewegen sich beide aufeinander zu (A von links bzw. von Westen, B von rechts bzw. von Osten), dann trifft

    aus der Sicht der Garage A zuerst das hintere (rechte) Ende der kürzeren Leiter auf das vordere (rechte) Ende der Garage (Ereignis E2) und dann das vordere (linke) Ende der Leiter auf das hintere (linke) Ende der Garage (Ereignis E1),

    während

    aus der Sicht der Leiter B zuerst das hintere (linke) Ende der kürzeren Garage A auf das vordere (linke) Ende der Leiter (Ereignis E1) und dann das vordere (rechte) Ende der Garage auf das hintere (rechte) Ende der Leiter (Ereignis E2) trifft.

    Zum ärgerlichen Umstand, dass die Ereignisse E1 und E2 ihre Reihenfolge vertauschten, kommt der ärgerliche Widerspruch, dass die Garage in die Leiter, und die Leiter in die Garage passt.

    Was lernen wir daraus? Man muss sich vom Glauben verabschieden, man könnte die bewegte Leiter zu irgendeinem Zeitpunkt an die Abmessungen der Garage “anlegen”, so wie man das von ruhenden Gegenständen her gewohnt ist. Die Frage, ob eine bewegte Leiter in die Garage “passt”, ist sinnlos.

  279. #279 PS
    20. August 2018

    @Philip #277
    Danke für die Klarstellung. Aber du wirst zugeben, es ist mühsam zu folgen, wenn man nur Worte (und keine Diagramme) verwenden kann und mit traditionellen Uhrzeiten rechnet. Soweit ich folgen kann, meinen wir dasselbe.

    Ich finde auf meiner Tastatur nur ∑, überhaupt verwende ich lieber S und S’.

  280. #280 Karl-Heinz
    20. August 2018

    @PS

    Dein Beispiels mit der Garage und Leiter ist einfach grauslich ausformuliert. Das kann man ja keinem zumuten.

  281. #281 Philip
    Wuppertal
    21. August 2018

    Hallo Alderamin,

    Warum nicht?

    Weil Du keine Zeit hast. Körper und Lineal liegen nur einen Augenblick lang (theoretisch nur eine infinitesimale Zeit) nebeneinander.
    Hinzu kommt aber folgendes: Die Zeitgleichheit, zu der einerseits das erste Ende des Körpers und das des Lineals uns andererseits das zweite Ende des Körpers und das zweite Ende des Lineals (respektive eine bestimmte Marke) genau nebeneinander zu liegen kommen, ist im Ruhesystem des Lineals gegeben, im Ruhesystem des Körpers jedoch nicht. Gleichzeitigkeit ist relativ.

    Das Garagenparadoxon ist doch genau das: die Länge des ruhenden Hangars mit derjenigen des Raumschiffs zu vergleichen.

    Das ist doch noch kein Paradoxon! Dieses kommt erst dadurch zustande, dass man diese Situation in zwei Koordinatensystemen vergleicht und zu dem Schluss kommt, dass man ebensogut die Garage als verkürzt müsse beschreiben können, das Raumschiff müsse also gleichzeitig kürzer und länger als die Garage sein.

    …man könnte das Raumschiff beispielsweise mit hellem Blitzlicht (Picosekunden-Laserpuls) fotografieren und auf dem Foto den Sehwinkel und das Verhältnis Höhe zu Länge bestimmen. Bei bekannter Entfernung hast Du auch die absoluten Maße (also in Metern statt °).

    Das könnte zwar funktionieren (wobei noch immer gilt, dass der Körper keineswegs verkürzt aussieht, solange er sich nicht schon wieder entfernt), aber man muss sich klar machen, dass der in Σ betrachtet an mehreren Stellen gleichzeitig ausgelöste Blitz im Ruhesystem Σ’ des Körpers nicht gleichzeitig, sondern in einer Art überlichtschnellen Kaskade auslöst, die von vorn nach hinten wandert. In der Zwischenzeit bewegt sich der Körper weiter und wird natürlich kürzer gemessen.
    Ich bestreite gar nicht, dass der Körper in Σ real kürzer ist als in Σ’, aber es bleibt durch seine Σ-Gebundenheit eine Interpretation. Tatsächlich sind ja Σ und Σ’ selbst Interpetationen dessen, was wann und wo stattfindet.
    Wenn man will, kann man sich sogar auf den Standpunkt stellen, das eigentliche Objekt sei gar nicht der 3D-Körper, sondern die (1+3)D-„Weltwurst“ (oder „Weltstrang“), deren Gestalt sich durch Bezugssystemwechsel nicht verändert. Der „3D-Körper zum Zeitpunkt t’“ in Σ’ ist lediglich der Quer- der „Körper zum Zeitpunkt t“ in Σ der Schrägschitt eines und desselben Objekts.

  282. #282 Karl-Heinz
    21. August 2018

    @Philip

    Die gemessene Länge L des bewegten Objektes ist ….
    Daraus folgt, dass Objekt hat die Ruhelänge (Eigenlänge) L0 von ….

    Und jetzt? Rüge von dir, dass es die gemessene Länge L des bewegten Objektes eigentlich gar nicht gibt?

  283. #283 Karl-Heinz
    21. August 2018

    @Philip

    Anstatt Alderamin zu belehren könntes du ja @PS ein bisschen Nachhilfe geben. 😉

  284. #284 Philip
    Wuppertal
    21. August 2018

    Hallo Karl-Heinz,
    ich belehre Alderamin nicht, ich streite mit ihm um’s Wording, ebenso wie ja auch schon mit Dir.

    Ich bestreite auch nicht, dass es die gemessene Länge des Körpers gibt, aber ich betrachte das als Koordinatendifferenz – und damit eine Interpretation – und weigere mich inzwischen, das als Kontraktion zu bezeichnen, weil dieses Wort etwas Falsches suggeriert und die Relativität von Bewegung vernebelt.

    Man fährt besser, wenn man einen Körper eben nicht als 3D-Objekt betrachtet, sondern als (1+3)D-Objekt (Weltwurst, Weltstrang), den man in unterschiedliche Richtungen messen kann.
    Wenn ich da einen Messschieber dran halte, bekomme ich natürlich unterschiedliche Ergebnisse, je nachdem, ob ich ihn gerade oder schräg dran halte.

    Jeder Mensch wird sofort einsehen, dass dies völlig normal und gar nicht paradox ist. Das wäre vielleicht anders, wenn man hier von einer geheimnisvollen Breitendehnung sprechen würde.

  285. #285 Karl-Heinz
    21. August 2018

    @Philip

    War ja nur Spaß.
    Aber was ist so Schlimm daran, wenn bewegte Objekte in der Länge verkürzt erscheinen? Das mit der Salami ist eigentlich nur eine Analogie. Erklärt wird dadurch aber nichts. 😉

  286. #286 Alderamin
    21. August 2018

    @Philip

    Weil Du keine Zeit hast. Körper und Lineal liegen nur einen Augenblick lang (theoretisch nur eine infinitesimale Zeit) nebeneinander.

    Infinitesimal nur dann, wenn das Raumschiff exakt die Länge der Garage hat. Verlangt ja keiner, ein bisschen mehr Tempo und es wird ein wenig kürzer, dann hat man mehr Zeit für die Messung. Man kann jedenfalls eine obere Schranke angeben und die kann immer noch kleiner sein als die Länge des Raumschiffs in dessen Bezugssystem.

    Hinzu kommt aber folgendes: Die Zeitgleichheit, zu der einerseits das erste Ende des Körpers und das des Lineals uns andererseits das zweite Ende des Körpers und das zweite Ende des Lineals (respektive eine bestimmte Marke) genau nebeneinander zu liegen kommen, ist im Ruhesystem des Lineals gegeben, im Ruhesystem des Körpers jedoch nicht. Gleichzeitigkeit ist relativ.

    Schon klar, aber wie gesagt, relevant für die Messung ist das Bezugssytem, in dem die Messung stattfindet. In einem anderen Bezugssystem misst man etwas anderes Man kann es nicht allen Bezugssystemen in gleicher Weise recht machen. In verschiedener Weise schon, die Ergebnisse müssen insgesamt konsistent sein. Es kann nicht sein, dass das Raumschiff in einem System die Garagentore lädiert und im anderen nicht.

    Das ist doch noch kein Paradoxon!

    Ich habe den Begriff “Garagenparadoxon” nur synonym mit dem Gedankenexperiment verwendet. Es gibt natürlich keine Paradoxa in der RT, nur Schein-Paradoxa, die auf falsche Anwendung der RT zurückzuführen sind.

    aber man muss sich klar machen, dass der in Σ betrachtet an mehreren Stellen gleichzeitig ausgelöste Blitz im Ruhesystem Σ’ des Körpers nicht gleichzeitig, sondern in einer Art überlichtschnellen Kaskade auslöst, die von vorn nach hinten wandert.

    Oder von der Mitte nach außen, wenn man das Laserlicht über einen Kollimator in eine ebene Wellenfront verwandelt. Der Weg der Strahlen außen ist dann ein paar cm länger und das Raumschiff entsprechend weit in Flugrichtung versetzt. Aber wirklich nur ein paar Zentimeter. Wenn man das Raumschiff genau im rechten Winkel zur Bewegungsrichtung erwischt, ist der Messfehler sehr klein und man würde die Längenverkürzung eines makroskopischen Objekts (sagen wir, 10 m lang und bei γ = 1/0,6 auf 6 m verkürzt) mit guter Approximation messen.

    aber es bleibt durch seine Σ-Gebundenheit eine Interpretation.

    Das ist auch genau meine Auffassung und ein schönes Schlusswort für unseren Austausch, würde ich sagen. Belassen wir’s doch dabei.

  287. #287 Philip
    Wuppertal
    21. August 2018

    Hallo Karl-Heinz,

    Aber was ist so Schlimm daran, wenn bewegte Objekte…

    Natürlich weiß jeder, der ernsthaft Relativitätstheorie betreibt, wenn er von „dem ruhenden“ und „dem bewegten“ Objekt spricht, dass Fortbewegung relativ ist.
    Das Wording „atmet“ allerdings nicht dieses Relativitätsprinzp und ist deshalb zutiefst unbefriedigend. Für mich. Für interessierte Laien, die mitlesen oder zuhören, ist es irreführend, weil sie denken könnten, hier sei eindeutig, wer oder was hier ruht und wer oder was nicht.
    Ein und dasselbe Objekt O’ ist bewegt bzw. in Ruhe, je nachdem, ob man Σ oder Σ’ als Bezugssystem verwendet.

    …in der Länge verkürzt erscheinen?

    Verkürzt erscheinen ist komplett irreführend, weil das nach „verkürzt aussehen“ klingt (was im Allgemeinen einfach falsch ist), aber auch, weil es die Aussagen in unzulässiger Weise entschärft (nach dem Motto „is ja gar nicht so, sieht nur so aus“).
    Auch das stimmt nämlich nicht. Dass das in Σ’ ruhende Objekt – entlang der x-Achse gemessen – kürzer ist als entlang der x’-Achse gemessen, ist ja sehr wohl real.
    Man muss sich aber darüber im Klaren sein, dass die x-Achse und die x’-Achse nicht parallel sind.

    Das mit der Salami ist eigentlich nur eine Analogie.

    Natürlich ist das eine Analogie (auch ohne „eigentlich nur“). Die Salami wird in einem EUKLIDischen Raum gemessen, steht aber für etwas, das eine andere Geometrie hat, nämlich einen „Weltwurst“-Abschnitt in der MINKOWSKIschen Raumzeit.

    Erklärt wird dadurch aber nichts.

    Das sehe ich komplett anders. Analogien sind dazu da, etwas zu erklären, indem sie es anschaulich machen. Hier geht es darum, dass nicht Raum und Zeit für sich getrennt betrachtet werden können und weder das Zwillingsparadoxon noch das Garagenparadoxon wirklich paradox (im Sinne von echt widersprüchlich) ist.
    Das Gezerre und Gequetsche im traditionellen Wording erklärt jedenfalls weit weniger und wird dem Namen Relativitätstheorie nicht gerecht.

  288. #288 PS
    21. August 2018

    @Philip #287

    Stimme allem zu, nur der Satz

    Man muss sich aber darüber im Klaren sein, dass die x-Achse und die x’-Achse nicht parallel sind.

    ist missverständlich.
    Die Verkippung der Achsen ist der Kunstgriff, der dem geometrischen Rechenschieber des Minkoski-Diagramms für die L-T zu Grunde liegt. In den Bezugssystemen selbst sind die x-Achse und die x’-Achse aber parallel, ebenso die y/y’-Achsen und die z/z’-Achsen, bei denen keine “Längenkontraktion auftritt.

  289. #289 Frank Wappler
    21. August 2018

    Alderamin schrieb (#275, 20. August 2018):
    > […] ein Myon durchschnittlicher Lebensdauer ohne Zeitdilatation […]

    Wie wäre denn (zunächst) ein bestimmter Wert der “durchschnittlichen Lebensdauer ohne Zeitdilatation” zu ermitteln,
    insbesondere hinsichtlich mehrerer Ensembles von Myonen, die sich (u.a.) dadurch unterscheiden, dass nicht alle durchwegs “auf einem Haufen” blieben ? …

    > Die Zeitdilatation lässt sich anhand von Prozessen belegen, die in Ruhe schneller ablaufen als bei relativistischer Geschwindigkeit (relativ zum Beobachter).

    Die Zeitdilatation ist ein Theorem der (Messgrößen der) RT, das sich mit dem Vergleich von Dauern (bzw. dem Vergleich der Kehrwerte, also von Frequenzen bzw. von Raten) beschäftigt. Es kann deshalb zur Feststellung benutzt werden, ob bestimmte beobachtbare Prozesse gleich “schnell abliefen“, oder ansonsten dafür, Verhältnisse von Dauern (wie z.B. von individuellen Lebensdauern) bzw. Verhältnisse von Prozess-Raten zu ermitteln.

    Dieses Theorem bzw. die damit definierte Messmethode kann durch eventuelle Befunde, die durch deren Anwendung überhaupt erst erhalten würden, offensichtlich nicht widerlegt werden;
    und es ist deshalb auch nicht nötig zu sagen, dass sie durch solche Befunde “belegt” wäre, die durch deren Anwendung überhaupt erst erhalten wurden.

    Ein Beispiel für eine experimentell prüfbare These (Erwartung, Modell) in diesem Zusammenhang ist:
    “das Ensemble der im nächsten Versuch erzeugten und überwachten Myonen hat die (nahezu, bis auf statistische Unsicherheit) gleiche (durch Anwendung des Zeitdilatations-Theorems festgestellte) mittlere Lebensdauer wie das Ensemble des vorhergehenden Versuchs.”

    Ein Beispiel für eine (ebenfalls experimentell prüfbare) entsprechende Antithese ist:
    “das Ensemble der im nächsten Versuch erzeugten und überwachten Myonen hat eine wesentlich kleinere (durch Anwendung des Zeitdilatations-Theorems festgestellte) mittlere Lebensdauer als das Ensemble des vorhergehenden Versuchs.”

  290. #290 Alderamin
    21. August 2018

    @Frank Wappler

    Wie wäre denn (zunächst) ein bestimmter Wert der “durchschnittlichen Lebensdauer ohne Zeitdilatation” zu ermitteln,

    Weiß nicht genau, aber es gibt ja z.B. myonische Atome, in denen die Myonen die Stelle von Elektronen einnehmen (mit solchen hat man schon kalte Fusionsprozesse hinbekommen, myonisch katalysierte Fusion; leider nicht ergiebig genug, um praktisch relevant zu sein). Die sind dann im Prinzip in Ruhe, sie halten sich ja in quantenmechanischen Orbitalen auf und kreisen nicht etwa auf Bahnen. Da Myonen sich wie Elektronen verhalten, wird man sie aber auch in relative Ruhe manövrieren können und dann ihre Zerfallsdauer in Ruhe messen, denke ich.

    Dieses Theorem bzw. die damit definierte Messmethode kann durch eventuelle Befunde, die durch deren Anwendung überhaupt erst erhalten würden, offensichtlich nicht widerlegt werden;

    Warum nicht? Wenn ruhende Myonen doch schneller zerfallen? Und man sogar quantitativ mit den Formeln für die Zeitdilatation und die für die Verteilung der Myonenzerfallsdauern im Ruhezustand auf die beobachtete Rate schneller Myonen am Boden kommt? Bestätigt doch, dass das verwendete Modell offenbar funktioniert.

    Ein Beispiel für eine experimentell prüfbare These (Erwartung, Modell) in diesem Zusammenhang ist: […]

    Man kann die Zeitdilatation leider nicht zwischen zwei Versuchen abschalten und alles andere unverändert lassen. Man kann nur die Lebensdauern schneller und langsamer Myonen vergleichen. Dem am nächsten kommen wohl Beschleunigerversuche, bei denen die Teilchenenergien und -zerfallsdauern exakt gemessen werden, und das in großer Zahl. Wäre da etwas unerwartetes in Bezug auf die Zeitdilatation herausgekommen, hätten wir davon gehört. Ganz sicher. Die Leute da arbeiten den ganzen Tag mit der Relativitätstheorie.

  291. #291 Frank Wappler
    21. August 2018

    Alderamin schrieb (#276, 20. August 2018):
    > Wenn es in die Garage passt, die kürzer ist als das Raumschiff in relativer Ruhe, dann ist es längenverkürzt.

    Wenn mit “(hinein-)passen” genau die beschriebene, als “Garagenparadoxon” bezeichnete Versuchsanordnung gemeint ist, bei der das Raumschiff durchwegs seine Länge behielt, weil seine beiden Enden durchwegs gegenüber einander ruhten, dann stiftet der zitierte Sprachgebrauch sicherlich Verwirrung. …

    (Für einen Versuch, in dessen Verlauf sich die Länge des Raumschiffs z.B. bis gleich zur Länge der Garage verringern sollte, müssten stattdessen die beiden Raumschiffenden zunächst aus ihrer gegenseitigen Ruhe gerissen und anschließend wieder geeignet in gegenseitige Ruhe gesetzt werden.)

  292. #292 Karl-Heinz
    21. August 2018

    @Frank Wappler

    Für einen Versuch, in dessen Verlauf sich die Länge des Raumschiffs z.B. bis gleich zur Länge der Garage verringern sollte, müssten stattdessen die beiden Raumschiffenden zunächst aus ihrer gegenseitigen Ruhe gerissen und anschließend wieder geeignet in gegenseitige Ruhe gesetzt werden.

    Das verstehe ich nicht. Kannst du es für Dummies übersetzen?

  293. #293 Philip
    Wuppertal
    21. August 2018

    Hallo PS,

    ich kann #288 nicht zustimmen:

    Die Verkippung der Achsen ist der Kunstgriff, der dem geometrischen Rechenschieber des MINKOWSKI-Diagramms für die L-T zu Grunde liegt.

    Nein. Nicht jedenfalls, wenn man die Relativität ernst nimmt, denn dazu gehört die Relativität der Gleichzeitigkeit.

    In den Bezugssystemen selbst sind die x-Achse und die x’-Achse aber parallel, ebenso die y/y’-Achsen und die z/z’-Achsen, bei denen keine “Längenkontraktion auftritt.

    Bei Letzteren hast Du Recht, bei Ersterem keinesfalls. Parallel (oder übereinstimmend) sind die x-t-Ebene und die x’-t’-Ebene.
    Unter der x-Achse verstehe ich die Raumzeit-Gerade mit

    (t=0 | {x} | y=0 | z=0),

    unter der x’-Achse eine mit

    (t’=0 | {x’} | y’=0 | z’=0).

    In Σ verläuft die x’-Achse nun mal auch in zeitlicher Richtung, und in Σ’ die x-Achse.

    Du kannst entweder die Relativität ernst nehmen oder Raum und Zeit als zwei voneinander unabhängige Größen (und somit Zeit nicht als Richtung) behandeln, nicht beides.

  294. #294 Frank Wappler
    22. August 2018

    Alderamin schrieb (#290, 21. August 2018):
    > […] Warum nicht?

    Warum ein bestimmter Messwert nicht der Messoperation widersprechen kann, durch deren Anwendung er ermittelt wurde ? —
    Weil, um das vermeintlich Offensichtliche in Worte zu fassen, so ein Messwert zwangsläufig im Wertebereich der betreffenden Messoperation liegt, und nicht außerhalb.

    Als ein konkretes und betont überschaubares Beispiel, bei dem es zur Abwechslung nicht um RT geht, schauen wir mal zum Schachspiel, mit seinen bekanntlich sorgfältig/unmissverständlich formulierten Regeln (“Fide Laws of Chess”), durch deren Anwendung, Partie für Partie, der jeweilige Ausgang eines regulären Spiels ermittelt wird: Weiß gewinnt, Schwarz gewinnt, oder Remis/Patt.

    Könnte man etwa von irgendeinem dieser regulären Schach-Spielausgänge sagen, er widerspräche den Schach-Regeln ?? …

    Wenn eine konkrete Partie von zwei konkreten Spielern begonnen wird, dann mögen diese beiden ja z.B. verschiedene Erwartungen hinsichtlich des bevorstehenden Ausgangs der Partie haben; und der letztlich ermittelte tatsächliche Ausgang würde zwangsläufig mindestens einer dieser Erwartungen widersprechen.

    Aber die Schach-Regeln selbst beinhalten keine derartigen Erwartungen; sie beschäfigen sich nicht mit den individuellen Fähigkeiten oder gar der Tagesform bestimmter Spieler, im Rahmen dieser Regeln gegen andere Spieler zu gewinnen.

    Entsprechend beinhaltet die RT-Messgrößen-Definition, -Messoperation bzw. -Spielregel zum Messen/Vergleichen von Dauern keinerlei Erwartungen zu Lebensdauer-Verhältnissen bestimmter Elementarteilchen, oder gar hinsichtlich “jeweils herrschender Versuchsbedingungen” im Einzelnen …

    > Beschleunigerversuche […] Wäre da etwas unerwartetes in Bezug auf die Zeitdilatation herausgekommen, hätten wir davon gehört.

    Sollte bei einem experimentellen Versuch etwas Unerwartetes gefunden werden, so wäre dies zwangsläufig etwas Unerwartetes in Bezug auf bestimmte gegenteilige Erwartungen;
    und nicht etwas Unerwartetes in Bezug auf die angewandte Messmethode, die gar keine bestimmten Erwartungen enthält.

  295. #295 Peter
    22. August 2018

    @Alderarmin
    Myonen haben nach der Erzeugung eine ganz spezifische Geschwindigkeit und Halbwertszeit. Werden sie im Labor gebremst, so ändert sich ihre Halbwertszeit. Was im Übrigen die Hauptaufgabe von Teilchenbeschleunigern ist: Die Halbwertszeit von Strukturen durch Bremsen(Beschleunigen) stark reduzieren, so dass die Zerfallsprodukte detektiert werden können.Die SRT wird also für die unterschiedkichen Halbwertszeiten der Myonen gar nicht benötigt.

  296. #296 Alderamin
    22. August 2018

    @Peter

    Werden sie im Labor gebremst, so ändert sich ihre Halbwertszeit.

    Aufgrund welchen Naturgesetzes soll sich denn die Halbwertszeit eines Quantenteilchens durch Abbremsung ändern? Abgesehen von der RT?

    Was im Übrigen die Hauptaufgabe von Teilchenbeschleunigern ist: Die Halbwertszeit von Strukturen durch Bremsen(Beschleunigen) stark reduzieren, so dass die Zerfallsprodukte detektiert werden können.

    Das erste was ich höre. Quelle dazu?

  297. #297 Alderamin
    22. August 2018

    @Frank Wappler

    Warum ein bestimmter Messwert nicht der Messoperation widersprechen kann, durch deren Anwendung er ermittelt wurde ? —

    Nein, es ging darum, warum es kein Beleg für die Zeitdilatation der RT sein soll, dass bei einem Prozess mit schnellen Myonen eine längere Halbwertszeit (geschlossen aus der mit v < c zurückgelegten Strecke) gemessen wird als bei annähernd ruhenden Myonen. Kann sein, dass ich Deinen Absatz

    Dieses Theorem bzw. die damit definierte Messmethode kann durch eventuelle Befunde, die durch deren Anwendung überhaupt erst erhalten würden, offensichtlich nicht widerlegt werden;
    und es ist deshalb auch nicht nötig zu sagen, dass sie durch solche Befunde “belegt” wäre, die durch deren Anwendung überhaupt erst erhalten wurden.

    nicht richtig verstanden habe.Jedenfalls kam er als Antwort auf meine Aussage

    Die Zeitdilatation lässt sich anhand von Prozessen belegen, die in Ruhe schneller ablaufen als bei relativistischer Geschwindigkeit (relativ zum Beobachter).

    und ich habe versucht, ihn als Gegenargument zu meinem Satz zu interpretieren.

  298. #298 Frank Wappler
    22. August 2018

    Karl-Heinz schrieb (#292, 21. August 2018):
    > [… »Für einen Versuch, in dessen Verlauf sich die Länge des Raumschiffs z.B. bis gleich zur Länge der Garage verringern sollte, müssten stattdessen die beiden Raumschiffenden zunächst aus ihrer gegenseitigen Ruhe gerissen und anschließend wieder geeignet in gegenseitige Ruhe gesetzt werden.« …] Das verstehe ich nicht.

    Das tut mir leid.
    Besonders, weil ich nicht mal erkennen kann, ob vielleicht nur bestimmte Teile dieses Zitats aus #291 unverständlich blieben.

    > Kannst du es für Dummies übersetzen?

    Die Frage unterstellt, dass wir Dummies eine gemeinsame, all uns Dummies gleichermaßen verständliche Sprache hätten …

    Aber immerhin kann ich zunächst einmal darauf hinweisen, wie gewisse andere Dummies etwas beschrieben haben, das sie offenbar dem “Ladder paradox” gegenüberstellen wollten (womöglich ähnlich wie das Zitierte dem weiter oben beschriebenen “Garagenparadoxon” gegenüberstellt war):

    What if the back door (the door the ladder exits out of) is closed permanently and does not open? Suppose that the door is so solid that the ladder will not penetrate it when it collides [… The] front of the ladder collides with the back door, the back of the ladder does not know it yet, so it keeps moving forwards (and the ladder “compresses”).

  299. #299 Peter
    22. August 2018

    @Alderarmin
    Die Naturgesetze, welche den Zerfall von Strukturen durch Beschleunigung beschreiben, werden in der QM beschrieben, nicht der RT.
    Deine Anmerkung bzgl. Teilchenbeschleuniger kann ich nicht ernst nehmen! Oder glaubst du etwa nicht, dass dort “Teilchen” zur Kollision gebracht werden? Kollision = starke Beschleunigung entgegen der Flugrichtung.

  300. #300 Karl-Heinz
    22. August 2018

    @Peter

    Und was ist mit den Teilchen, die keiner starken Beschleunigung ausgesetzt sind? Erklär uns mal warum diese Teilchen zerfallen!

  301. #301 Alderamin
    22. August 2018

    @Peter

    Die Naturgesetze, welche den Zerfall von Strukturen durch Beschleunigung beschreiben, werden in der QM beschrieben, nicht der RT.

    Ich bitte um ein Stichwort, nach dem ich suchen kann. Ich kenne kein Gesetz der QM, das die Zerfallszeit über die Geschwindigkeit beeinflussen könnte.

    Deine Anmerkung bzgl. Teilchenbeschleuniger kann ich nicht ernst nehmen! Oder glaubst du etwa nicht, dass dort “Teilchen” zur Kollision gebracht werden? Kollision = starke Beschleunigung entgegen der Flugrichtung.

    Doch, natürlich glaube ich, dass in Beschleunigern Teilchen zur Kollision gebracht werden, aber mit dem Zweck, die Bewegungsenergie in Strahlung umzuwandeln, aus der dann völlig neue Teilchen entstehen, die man nachweisen möchte. Du sagtest aber etwas ganz anderes:

    Die Halbwertszeit von Strukturen durch Bremsen(Beschleunigen) stark reduzieren, so dass die Zerfallsprodukte detektiert werden können.

    Dass durch Abbremsen die Halbwertszeit reduziert wird, dafür habe ich nach einer Quelle gefragt. Diese Behauptung habe ich noch nirgends gehört.

  302. #302 Frank Wappler
    22. August 2018

    Alderamin schrieb (#297, 22. August 2018):
    > […] Kann sein, dass ich Deinen Absatz [#289] nicht richtig verstanden habe. Jedenfalls kam er als Antwort auf meine Aussage [#275]

    Mein Versuch der Gegenargumentation bezog sich genauer betrachtet nur auf den ersten Teil der genannten Aussage (den ich schon an sich offenbar nicht verstehe, geschweige denn die daraus gebildete Aussage insgesamt), nämlich:

    Die Zeitdilatation lässt sich anhand von Prozessen belegen, […]

    Ich kann nochmals zwei knappe Formulierungen anbieten (und etwas ausführlicher erläutern), die ich (jeweils für sich genommen) stattdessen für verständlich und richtig halte:

    1. Die Zeitdilatation lässt sich als Theorem der RT beweisen.
    (Unter bestimmten Voraussetzungen an eine gedachte Versuchsanordnung folgt ein bestimmter Formel-Zusammenhang zwischen dem Wert β der normierten gegenseitigen Geschwindigkeit zweier Inertialsysteme, und dem Verhältniswert bestimmter Dauern von bestimmten Mitgliedern dieser beiden Inertialsysteme)

    2. Die Zeitdilatation lässt sich auf geeignete experimentelle Prozesse anwenden.
    (Falls die betreffende experimentelle Versuchsanordnung die Voraussetzungen des Theorems erfüllt, lässt sich der bewiesene Formel-Zusammenhang benutzen, um aus dem dafür schon ermittelten Messwert β den zutreffenden Wert des Verhältnisses der betreffenden Dauern zu errechnen.)

    An den zweiten Teil der genannten Aussage scheint allerdings weder die eine noch die andere dieser beiden vorgeschlagenen Formulierungen zu passen …

    [… Prozesse,] die in Ruhe schneller ablaufen als bei relativistischer Geschwindigkeit (relativ zum Beobachter).

    Mir erschiene in diesem Zusammenhang jedenfalls eine Formulierung passender und ggf. verständlicher, die sich anstatt mit dem Vergleich bzw. dem Verhältniswert von “Ablauf-Schnelligkeiten” ausdrücklich mit dem Vergleich bzw. dem Verhältniswert von bestimmten Dauern befasst; z.B. mit dem Verhältniswert der Lebensdauern zweier bestimmter Myonen.

    > Halbwertszeit (geschlossen aus der mit v < c zurückgelegten Strecke)

    In Anwendung des Zeitdilatations-Theorems auf eine geeignete Versuchsanordnung (und mit den erforderlichen Messwerten der entsprechenden RT-Messgrößen) können wir jedenfalls die Lebensdauer eines bestimmten einzelnen Muons M folgendermaßen schließen:

    \tau M^{\text{life}} := \sqrt{ 1 - \left( \frac{v}{c}\right)^2 } \, \frac{PQ}{v},

    wobei die Streckenlänge PQ (Distanz zwischen zwei gegenüber einander ruhenden Streckenenden, P bzw. Q, die an Erzeugung bzw. Zerfall des Myons M teilgenommen hatten) und die Geschwindigkeit v_{PQ}[ \, M \, ] := \beta_{PQ}[ \, M \, ] \, c := \beta \, c := v des (unbeschleunigt gedachten) Myons gegenüber diesen beiden Streckenenden bekanntlich mit der Belegungsdauer \tau P_M Q^M der beiden Streckenden durch das Myon zusammenhängt:

    v := v_{PQ}[ \, M \, ] := \frac{PQ}{\tau P_M Q^M} := \frac{c}{2} \, \left( \frac{\tau P^{\text{ping_}Q}}{\tau P_M Q^M} \right) = \frac{c}{2} \, \left( \frac{\tau Q^{\text{ping_}P}}{\tau P_M Q^M} \right) .

    Oder sollte die Lebensdauer des Muons etwa durch andere Schlüsse ermittelt werden ? …

    p.s.
    Alderamin schrieb (#10, 17. Juli 2018):
    > Mal unterstellt, die Halbwerts-Zerfallzeit seien 2,2 µs […]

    Die Halbwerts-Zerfallsdauer und die mittlerer Lebensdauer eines (geeignet zahlreichen und statistisch gleichmäßigen) Ensembles bei exponentiellem Zerfall unterscheiden sich übrigens ca. um den Faktor 0,6931.

  303. #303 Frank Wappler
    22. August 2018

    p.s. — \LaTeX-Formatierung zu #302:

    > Halbwertszeit (geschlossen aus der mit v < c zurückgelegten Strecke)

    In Anwendung des Zeitdilatations-Theorems auf eine geeignete Versuchsanordnung (und mit den erforderlichen Messwerten der entsprechenden RT-Messgrößen) können wir jedenfalls die Lebensdauer eines bestimmten einzelnen Muons M folgendermaßen schließen:

    \tau M^{\text{life}} := \sqrt{ 1 - \left( \frac{v}{c}\right)^2 } \, \frac{PQ}{v},

    wobei die Streckenlänge PQ (Distanz zwischen zwei gegenüber einander ruhenden Streckenenden, P bzw. Q, die an Erzeugung bzw. Zerfall des Myons M teilgenommen hatten) und die Geschwindigkeit v_{PQ}[ \, M \, ] := \beta_{PQ}[ \, M \, ] \, c := \beta \, c := v des (unbeschleunigt gedachten) Myons gegenüber diesen beiden Streckenenden bekanntlich mit der Belegungsdauer \tau P_M Q^M der beiden Streckenden durch das Myon zusammenhängt:

    v := v_{PQ}[ \, M \, ] := \frac{PQ}{\tau P_M Q^M} := \frac{c}{2} \, \left( \frac{\tau P^{(\text{ping}\_Q)}}{\tau P_M Q^M} \right) = \frac{c}{2} \, \left( \frac{\tau Q^{(\text{ping}\_P)}}{\tau P_M Q^M} \right) .

    Oder sollte die Lebensdauer des Muons etwa durch andere Schlüsse ermittelt werden ? …

  304. #304 Karl-Heinz
    23. August 2018

    @Frank Wappler

    Also ich komme mit den Begriffen Lebensdauer und Halbwertszeit aus, wobei die Halbwertszeit gleich der Lebensdauer * ln(2) ist. Bewegt sich das Myon muss noch den Lorentzfaktor berücksichtigen. Damit kann ich eigentlich alles berechnen.

    Kannst du uns Dummies mal erklären was jetzt genau eine Belegungsdauer ist und wie sie definiert ist. 😉

  305. #305 Karl-Heinz
    23. August 2018

    @Frank Wappler

    Bitte die Erläuterungen in einer verständlichen Sprache abfassen. Danke.

  306. #306 rolak
    23. August 2018

    Belegungsdauer?

    Ganz einfach, Karl-Heinz:
     - Hotelfachwesen: Wie lange belegt Gast G Zimmer Z?
     - Gastronomie: Wer braucht wie lange, ein Schinkenbrot zuzubereiten?
     - Medizin: Diagnostik Zunge→Krankheit
     - …

    in einer verständlichen Sprache

    Unmöglich, Karl-Heinz.

  307. #307 Philip
    23. August 2018

    Hallo Karl-Heinz,

    Bewegt sich das Myon, muss man noch den Lorentzfaktor berücksichtigen.

    Ich würde die Formulierung…

    Rechnet man in einem Koordinatensystem, in dem sich das Myon bewegt, muss man noch den Lorentzfaktor berücksichtigen.

    …bevorzugen, da es dem Relativitätsprinzip viel besser Rechnung trägt.

  308. #308 Karl-Heinz
    23. August 2018

    @Philip

    Danke Philip für die Korrektur. Vielleicht kannst du mir erklären, was eine Belegungsdauer (für eine bestimmte Strecke?) ist.

  309. #309 Peter
    23. August 2018

    @Alderamin
    Die Halbwertszeit ist die Zeit, in der eine Struktur zur Hälfte zerfallen ist bzw. umgewandelt wurde. Diese Zeit wird durch Kollision (= Beschleunigung) verringert. Falls du mit der Begrifflichkeit nicht einverstanden bist, dann nutze “Lebensdauer”. Die Lebensdauer von Strukturen wird durch Beschleunigung verringert. Das geschieht ebenso mit dem Myon: Bei starker Bremsung von fast c auf 0 wird seine Lebensdauer verkürzt. Die SRT wird hierbei nicht benötigt. Deshalb kann die “normale” Lebensdauer auch nicht als Bestätigung der SRT genutzt werden.

  310. #310 Philip
    23. August 2018

    Hallo rolak,
    alles Beispiele, die nichts mit Teilchen zu tun haben.
    Unter einer Belegungsdauer könnte man sich natürlich die Dauer der Belegung eines bestimmten Zustandes durch ein Teilchen vorstellen, aber welcher sollte das in diesem Zusammenhang sein?

  311. #311 Karl-Heinz
    23. August 2018

    @Peter

    Ein bisschen Mathematik und du wirst erkenne, das es zwischen Halbwertszeit und Lebensdauer einen Zusammenhang gibt. 😉

  312. #312 Alderamin
    23. August 2018

    @Peter

    Diese Zeit wird durch Kollision (= Beschleunigung) verringert.[…] Die Lebensdauer von Strukturen wird durch Beschleunigung verringert.

    Ich bat Dich um einen Beleg für diese Behauptung.

    Ich finde zumindest in der Herleitung der Myonen-Zerfallsdauer (vorgerechnet hier in #1…#8) keinen Parameter für die Beschleunigung.

  313. #313 Karl-Heinz
    23. August 2018

    @Peter

    Frage: Bist du ein Einsteinleugner?
    Das ist keine bös gemeinte Frage. Ich will nur wissen in welche Richtung deine Überlegungen gehen.
    Danke für die Antwort im Voraus.

  314. #314 Frank Wappler
    23. August 2018

    Karl-Heinz schrieb (23. August 2018):
    > […] mit den Begriffen Lebensdauer und Halbwertszeit

    Für die letztere Größe benutze und empfehle ich den Namen “Halbwertsdauer“,
    weil es sich dabei um eine Größe handelt, die kommensurat sowohl zu (individueller) Lebensdauer eines einzelnen instabilen Beteiligten als auch zu mittlerer Lebensdauer eines entsprechenden Ensembles ist;
    und weil das Wort “Zeit” historisch in zahlreichen verschiedenen Bedeutungen benutzt wurde, wobei insbesondere die von Einstein hervorgehobene Bedeutung,

    dass ich an Stelle der “Zeit” die “Stellung des kleinen Zeigers meiner Uhr” setze,


    sicherlich nicht als die von “Dauer” aufzufassen ist.

    > Bewegt sich das Myon […]

    Kannst du uns Dummies bitte bestätigen,

    – dass wir uns dabei zusammen mit dem betreffenden Myon außerdem zwei weitere Beteiligte vorstellen sollen,
    – die voneinander getrennt waren und blieben und
    – die gegenüber einander durchwegs ruhten,

    und

    – dass sich diese beiden Beteiligten einerseits und das betreffende Myon andererseits gegenüber einander bewegten, also nicht alle drei gegenüber einander ruhten ?

    Falls so, kannst Du uns Dummies bitte zugestehen,

    – dass wir den einen dieser beiden gegenüber einander ruhenden Beteiligten P nennen, und den anderen Q, und falls so:
    – dass wir uns als P insbesondere einen Beteiligten vorstellen, der zusammen mit dem Myon an dem Ereignis teilgenommen hatte, das die Erzeugung dieses Myons beinhaltete, und
    – dass wir uns als Q insbesondere einen Beteiligten vorstellen, der zusammen mit dem Myon an dem Ereignis teilgenommen hatte, das den Zerfall dieses Myons beinhaltete ?

    Falls so …

    > Kannst du uns Dummies mal erklären was jetzt genau eine Belegungsdauer ist und wie sie definiert ist.

    … meine ich mit der relevanten Belegungsdauer (der Strecke PQ durch das Myon):

    – sowohl die Dauer Ps von dessen Anzeige der Teilnahme am Ereignis, das die Erzeugung des Myons beinhaltete, bis zu dessen Anzeige gleichzeitig zu Qs Anzeige der Teilnahme am Ereignis, das den Zerfall des Myons beinhaltete

    – als auch gleichermaßen die Dauer Qs von dessen Anzeige gleichzeitig zu Ps Anzeige der Teilnahme am Ereignis, das die Erzeugung des Myons beinhaltete, bis zu Qs Anzeige der Teilnahme am Ereignis, das den Zerfall des Myons beinhaltete.

    (Die Belegungsdauer einer bestimmten Strecke durch jemanden bzw. durch etwas kann insbesondere als ein Maß der Streckenbelegung dienen.)

    > […] muss noch den Lorentzfaktor berücksichtigen.

    Kannst du uns Dummies bitte bestätigen, dass “der Lorentzfaktor” in diesem Fall den reellen Zahlenwert meint, den man erhalten kann indem

    – der relle Zahlenwert des Verhältnisses der Pingdauer von P und Q untereinander und der genannten
    Belegungsdauer bestimmt wird,

    – ein Viertel dieses Verhältniswerts von der Zahl 1 abgezogen wird, und schließlich

    – die Wurzel aus dem so erhaltenen Differenzwert gezogen wird ?

    Danke im Voraus!

  315. #315 Frank Wappler
    23. August 2018

    p.s. — Korrektur zu #314:

    > […] indem
    >
    – der relle Zahlenwert des Verhältnisses der Pingdauer von P und Q untereinander und der genannten Belegungsdauer bestimmt wird,
    > – ein Viertel

    … des Quadrates …

    > dieses Verhältniswerts von der Zahl 1 abgezogen wird, und schließlich
    > – die Wurzel aus dem so erhaltenen Differenzwert gezogen wird ?

  316. #316 Karl-Heinz
    23. August 2018

    @Frank Wappler

    Geh Wappler kannst du nicht für uns ein Beispiel mit richtigen Zahlenwerten berechnen?
    Bitte Erläuterungen bei der Berechnung auf ein Minimum reduzieren . Die stören nur. 😉

  317. #317 Philip
    Wuppertal
    23. August 2018

    Hallo Karl-Heinz,
    #308: Wat weeß icke? Ich kann nur Vermutungen anstellen, und das habe ich #310 getan. Konkreter würde ich sagen, vermutlich ist das die Zeit, die sich das Myon zwischen P und Q befindet.
    #313: Das Wort „Einsteinleugner“ ist irreführend. Selbst der hartnäckigste GdRT leugnet ja nicht EINSTEIN, sondern widerspricht ihm.

  318. #318 Karl-Heinz
    23. August 2018

    @Frank Wappler

    Was sagst zur ausdruckstarken und klaren Sprache von Philip? 😉

  319. #319 Karl-Heinz
    23. August 2018

    So das mit dem Frank Wappler habe ich gecheckt.
    Kommentar folgt später.

  320. #320 Karl Mistelberger
    23. August 2018

    Richard Feynman erklärt den Sachverhalt ganz ohne Wapplerei:

    A very interesting example of the slowing of time with motion is furnished by muons, which are particles that disintegrate spontaneously after an average lifetime of 2.2×10^−6 sec. They come to the earth in cosmic rays, and can also be produced artificially in the laboratory. Some of them disintegrate in midair, but the remainder disintegrate only after they encounter a piece of material and stop. It is clear that in its short lifetime a muon cannot travel, even at the speed of light, much more than 600 meters. But although the muons are created at the top of the atmosphere, some 10 kilometers up, yet they are actually found in a laboratory down here, in cosmic rays. How can that be? The answer is that different muons move at various speeds, some of which are very close to the speed of light. While from their own point of view they live only about 2 μsec, from our point of view they live considerably longer—enough longer that they may reach the earth. The factor by which the time is increased has already been given as 1/sqrt(1−u^2/c^2). The average life has been measured quite accurately for muons of different velocities, and the values agree closely with the formula.

    https://www.feynmanlectures.caltech.edu/I_15.html

    In seinen posthum veröffentlichten Briefen (“Absolut vernünftige Abweichungen vom ausgetretenen Pfad”) beschreibt er detailliert den Fall, wo er leichtsinnigerweise das Angebot annahm, bei der Sendung “About Time” der NBC als wissenschaftlicher Berater zu agieren und auf sein Betreiben hin in dieser das Zwillingsparadoxon erläutert wurde.

    Feynmans Briefwechsel mit Mr. Y ist sehr lesenswert. Aus seinen mit Engelsgeduld formulierten höflichen Antworten:

    Um festzustellen, ob wir hinsichtlich der mit dem Uhrenparadoxon zusammenhängenden Phänomene tatsächlich unterschiedlicher Meinung sind, möchte ich Sie daher fragen, wie Ihrer Ansicht nach das Ergebnis des folgenden Experiments aussehen würde:

    [Es folgt eine Skizze von Mesonen auf einer Kreisbahn im Magnetfeld. Deren eine Hälfte wird abgebremst und zerfällt in Ruhe, die andere Hälfte fliegt ungebremst weiter und zerfällt in rascher Bewegung auf der Kreisbahn.]

    Ihren Briefen konnte ich nicht entnehmen, was Sie unter diesen Bedingungen erwarten. Lassen Sie mich das bitte wissen, damit wir die Sache eingehend erörtern können.

    Bitte [Mr. Y], beantworten sie diese Frage. Erzählen Sie mir nicht immer wieder, “meine These” zeige, dass ich nichts von Relativität verstehe, sprechen Sie nicht dauern von “meiner Philosophie” und unterlassen Sie die Behauptung, ich wisse nicht, dass es sich beim Meson um ein Elementarteilchen handle. All das mag zutreffen oder auch nicht. … Sagen Sie mir, welches Verhältnis von Treffern in den Zählern Sie erwarten.

    [in den archivierten Unterlagen Feynmans befindet sich kein weiterer Brief von Mr. Y]

  321. #321 Frank Wappler
    24. August 2018

    Karl Mistelberger schrieb (#320, 23. August 2018):
    > Richard Feynman erklärt den Sachverhalt ganz ohne Wapplerei:

    Ich freue mich über die Gelegenheit, öffentlich auffindbar zu Feynmans Erklärung Stellung nehmen zu können:

    […] muons, which are particles that disintegrate spontaneously after an average life[…] of 2.2×10^−6 sec [duration].

    Richtig; d.h. ausdrücklich (nur) für Myonen, deren Zerfälle spontan erfolgten, und nicht etwa diebezügilch “gestört” (gehemmt, oder gefördert bzw. induziert).

    […] It is clear that in its short lifetime a muon cannot travel, even at the speed of light, much more than 600 meters.

    Damit wird behauptet und unterstellt, es sei “clear”, dass die im Rahmen der Relativitätstheorie beschriebenen Messgrößen (bzw. Messmethoden zu Feststellung von) Lebensdauer, Geschwindigkeit (bzgl. eines bestimmten geeigneten “Streckensystems”) und Distanz nicht benutzt und zugrundegelegt würden.

    Das ist zu bestreiten; besonders weil Feynman (zumindest im oben vorgelegten Zitat) eine Erklärung schuldig bleibt, wie diese relevanten Messgrößen denn ansonsten anders definiert und deren Messwerte festgestellt werden sollten.

    […] While from their own point of view they live only about 2 μsec, from our point of view they live considerably longer [by a factor of] 1/sqrt(1−u^2/c^2).

    Mit dieser “point of view”-Formulierung wird die Belegungsdauer (der Erdatmosphären- bzw. Labor-Strecke mit dem betreffenden Myon) als Lebensdauer des Myons (miss-)interpretiert;
    anstatt deutlich zu unterscheiden und konkret zu benennen, um wessen Dauer es jeweils eigentlich (properly) geht.

    The average life [duration] has been measured quite accurately for muons of different velocities, and the values agree closely with the formula.

    Die erwähnte Übereinstimmung (der Verhältniswert von Streckenbelegungsdauer und ebensdauer) “mit der Formel” beruht vermutlich auf der Modell-Annahme bzw. -Erwartung, dass die mittlere Lebensdauer jedes der untersuchten Myonen-Ensembles von (hinreichend genau) bestimmter Geschwindigkeit (bzgl. Erdatmosphäre bzw. “Labor”) nahezu gleich den o.g. Wert von 2.2×10^−6 sec gewesen wäre, also dass die betreffenden Myonen in Wesentlichen spontan und ungestört zerfallen wären.

    Aus gemessenen Streckenbelegungsdauern und Werten “u/c” ließe und lässt sich dagegen in Anwendung der betreffenden Formel überhaupt erst Modell-unabhängig messen, ob die untersuchten Myonen-Ensembles tatsächlich alle nahezu gleichermaßen die bekannte mittlere Lebensdauer gehabt hatten, oder in wie fern nicht.

  322. #322 Karl-Heinz
    24. August 2018

    @Frank Wappler

    Deine Pingzeit. Warum doppelt?
    Pingzeit =Pingzeit von P nach Q + Pingzeit von Q nach P.
    Frage: warum 2 mal?

  323. #323 Karl Mistelberger
    24. August 2018

    > #321 Frank Wappler, 24. August 2018
    > Ich freue mich über die Gelegenheit, öffentlich auffindbar zu Feynmans Erklärung Stellung nehmen zu können.

    Schön, aber wie lautet Ihre Antwort auf die Frage Feynmans?

    Feynman schreibt in diesem Zusammenhang:

    Viele Jahre hindurch habe ich mich in meinen Arbeiten zur Physik auf grundlegende Weise dieser Vorstellungen bedient und weiß nichts von einem direkten Verstoß – vielmehr sind sie offenbar sehr nützlich, um neue Phänomene vorherzusagen. Ich wäre überrascht, aber sehr froh, wenn man mir eine andere Betrachtungsweise darlegte, die zweierlei leistet:

    (1) Korrekt alle jene Auswirkungen vorhersagt, die man bislang experimentell beobachtet hat.

    (2) Ein abweichendes Ergebnis für den reisenden Zwilling vorhersagt.

    Mit freundlichen Grüßen
    R.P. Feynman

    In Anlehnung an meinen Landsmann Wolfgang Pauli nehme ich wie folgt Stellung: Wappler sagt nicht nur nichts, der wappelt bloß.

  324. #324 Philip
    Wuppertal
    24. August 2018

    Hallo Karl Mistelberger,
    FEYNMAN schreibt also

    Ich wäre überrascht, aber sehr froh, wenn man mir eine andere Betrachtungsweise darlegte, die zweierlei leistet:…

    Wenn ich könnte, würde ich ihn fragen, warum er so froh darüber wäre. Weil es wieder einmal geeignet wäre, die Physik zu revolutionieren?
    Natürlich ist es irgendwie spannender im Sinne von Sensation, wenn schon wieder etwas komplett Neues ans Licht kommt, das die Physik revolutioniert, als wenn eine bestehende Theorie bestätigt wird.
    Allein, “Revolutionarität” ist alles andere als ein Wert an sich. Physik muss
    in sich konsistent und
    mit experimentellen Befunden konsistent
    sein.

    Die eigentliche Revolution der Physik wurde schon von GALILEI eingeleitet, der etliche der wichtigsten Prinzipien aufgestellt hat, auch wenn sie nun als ‘NEWTON’sche Gesetze’ oder ‘N. Axiome’ bezeichnet werden.
    EINSTEIN hat nicht gegen GALILEI revoltiert, sondern sich dessen Revolution angeschlossen, indem er MAXWELL in GALILEIs Welt integrierte, in der Fortbewegung relativ ist.
    Soweit es die Klassische Physik betrifft, wurde diese erst durch EINSTEIN vollendet, nämlich mit der ART, die aus GALILEIs Erkenntnis, dass Körper unterschiedlicher Masse im Gravitationsfeld dieselbe Beschleunigung erfahren, das Äquivalenzprinzip gemacht hat, auf dem sie aufbaut.

    Soweit ich weiß, ging es auch EINSTEIN nie darum, um jeden Preis “was Neues” zu bringen und Revolution um der Revolution willen zu machen, sondern um die Formulierung einer eleganten Physik, die allein auf klaren Prinzipien aufgebaut ist (die sich, wie wir dank Emmy NOETHER seit 100 Jahren wissen, aus Symmetrien herleiten lassen).

    Sch*** was auf die Myonen, um es mal ganz salopp zu sagen.
    Die RT wurde nicht aufgestellt, um zu erklären, wieso so viele von denen die Erdoberfläche erreichen, sondern um die neuen Erkenntnisse MAXWELLs mit GALILEIs Relativitätsprinzip zu verheiraten.

    Umgekehrt bestätigen uns die Myonen – vielleicht – die Relativitätstheorie, indem sie deren Vorhersagen bestätigt und die der NEWTON’schen Näherung nicht.

  325. #325 Captain E.
    24. August 2018

    Übrigens: Ernst Mach hat sich nicht nur mit Überschallgeschwindigkeit beschäftigt, sondern auch bereits mit Relativität. Albert Einstein dürfte dessen Überlegungen bereits gekannt haben. Und natürlich hat Hermann Minkowski den “Nachhilfelehrer” gegeben und Einstein die nötige Mathematik beigebracht.

    Die Relativitätstheorien haben vielleicht nur einen Vater, aber ein paar (Ur-) Großväter und Onkel. Und wohl auch mindestens eine Tante. 😉

  326. #326 Karl-Heinz
    24. August 2018

    @Captain E.

    Das sehe ich auch so. Den Paradigmenwechsel aber zu vollziehen, da kann ich nur sagen „Hut ab“. 😉

  327. #327 Frank Wappler
    24. August 2018

    Karl-Heinz schrieb (24. August 2018):
    > Deine Pingzeit.

    Ich habe in meinen Kommentaren auf dieser ScienceBlogs-Seite durchwegs von “Pingdauer” geschrieben;
    und insbesondere schon oben (#314) meine Gründe dafür genannt.

    Wie der Name “ping” nahelegen soll, geht es dabei (jeweils) um die Dauer eines bestimmten Beteiligten (nennen wir diesen wie schon oben “P“), von einer bestimmten Signalanzeige Ps bis zu Ps Anzeige der (allerersten) Wahrnehmung, dass ein bestimmter anderer Beteiligter (wie oben: “Q“) angezeigt hatte, die Signalanzeige Ps wahrgenommen zu haben;
    symbolisch:

    \tau P_{\sigma}^{(\text{ping}\_Q)} := \tau P[ \, \_\sigma, \_\text{sah}\_Q\_\text{sah}\_P\_\sigma \, ].

    Falls P und Q durchwegs (oder zumindest während ein paar gegenseitigen Pings) gegenüber einander ruhten, haben sie dabei definitionsgemäß gegenseitig gleiche Pingdauern:

    \tau P_{\sigma}^{(\text{ping}\_Q)} = \tau Q_{\sigma}^{(\text{ping}\_P)}.

    > Pingzeit =Pingzeit von P nach Q + Pingzeit von Q nach P.

    Vermutlich soll damit angedeutet werden, dass sich die Pingdauer Ps bzgl. Q als Summe zweier bestimmter Dauern Ps auffassen und ausdrücken lässt. Für gegenüber einander ruhende Beteiligte P und Q (die deshalb Gleichzeitigkeit oder Ungleichzeitigkeit ihrer Anzeigen gegenüber einander bewerten können) insbesondere:

    \tau P[ \, \_\sigma, \_\text{sah}\_Q\_\text{sah}\_P\_\sigma \, ] = \tau P[ \, \_\sigma, \_\text{gleichzeitig zu}\_Q\_\text{sah}\_P\_\sigma \, ] + \tau P[ \, \_\text{gleichzeitig zu}\_Q\_\text{sah}\_P\_\sigma,  \_\text{sah}\_Q\_\text{sah}\_P\_\sigma \, ].

    Und definitionsgemäß dann auch:

    \tau P[ \, \_\sigma, \_\text{gleichzeitig zu}\_Q\_\text{sah}\_P\_\sigma \, ] = \tau P[ \, \_\text{gleichzeitig zu}\_Q\_\text{sah}\_P\_\sigma,  \_\text{sah}\_Q\_\text{sah}\_P\_\sigma \, ],

    und deshalb

    \tau P[ \, \_\sigma, \_\text{sah}\_Q\_\text{sah}\_P\_\sigma \, ] = 2 \, \tau P[ \, \_\sigma, \_\text{gleichzeitig zu}\_Q\_\text{sah}\_P\_\sigma \, ] = 2 \, \tau P[ \, \_\text{gleichzeitig zu}\_Q\_\text{sah}\_P\_\sigma,  \_\text{sah}\_Q\_\text{sah}\_P\_\sigma \, ].

    > Warum doppelt?

    Wegen der in der letzter Formel auftretenden Zahl 2, vermute ich. …

    Im Zusammenhang damit steht auch die chronometrische Definition der Messgröße “Distanz”, zwischen (gegenüber einander durchwegs ruhenden) Beteiligten P und Q als:

    PQ := \frac{1}{2} \, c \, \tau P_{\sigma}^{(\text{ping}\_Q)},

    worin c zunachst ein (ausdrücklich von “Null” verschiedenes) rein formales Symbol darstellt, um “Distanz” von “Dauer” zu unterscheiden. Durch die darauf beruhende geeignete Definition der Messgröße “(Durchschnitts-)Geschwindigkeit” ist dieses Symbol anschließend als Wert der Signalfront-(Durchschnitts-)Geschwindigkeit identifiziert.

  328. #328 Alderamin
    24. August 2018

    @Frank Wappler

    […] It is clear that in its short lifetime a muon cannot travel, even at the speed of light, much more than 600 meters.

    Damit wird behauptet und unterstellt, es sei “clear”, dass die im Rahmen der Relativitätstheorie beschriebenen Messgrößen (bzw. Messmethoden zu Feststellung von) Lebensdauer, Geschwindigkeit (bzgl. eines bestimmten geeigneten “Streckensystems”) und Distanz nicht benutzt und zugrundegelegt würden.

    Das ist zu bestreiten; besonders weil Feynman (zumindest im oben vorgelegten Zitat) eine Erklärung schuldig bleibt, wie diese relevanten Messgrößen denn ansonsten anders definiert und deren Messwerte festgestellt werden sollten.

    Ich komme einfach nicht drüber, dass Du mit dem Konzept der Antithese oder des Widerspruchbeweises offenbar nichts anstellen kannst… Wenn gilt A=>B, dann gilt auch ¬B => ¬A. Wenn alle VWs Autos sind, dann kann etwas, das kein Auto ist, kein VW sein.

    Die Chuzpe, einem Feynman in seinem Fach zu widersprechen, muss man auch erst mal haben.

  329. #329 Karl-Heinz
    24. August 2018

    @Frank Wappler

    Verstehe, du verwendest Begrifflichkeiten aus der Sonartechnik.

    Prinzip des aktiven Sonars: Die Zeit vom Aussenden des Impulses bis zum Empfangen des reflektierten Signals ist proportional zur Objektentfernung

    Ich halte mal für später fest. Habe ich die Pingdauer, so kann man daraus die Entfernung berechnen. Habe ich die Entfernung, so kann man daraus die Pingdauer berechnen.

  330. #330 Frank Wappler
    24. August 2018

    Philip schrieb (24. August 2018):
    > Physik muss in sich konsistent [sein]

    Zweifellos.

    > und mit experimentellen Befunden konsistent sein.

    Die Anteile der Physik, die sich damit beschäftigen, wie experimentelle Befunde (Messwerte) überhaupt erst zu erlangen wären, die also bestimmte Messgrößen bzw. bestimmte Messoperationen zur Ermittlung von Messwerten aus (gegebenen, experimentellen) Beobachtungsdaten überhaupt erst definieren,
    sind allerdings zwangsläufig und von vornherein konsistent
    mit den experimentellen Befunden (Messwerten), die ggf. in Anwendung genau dieser definierten Messoperationen ermittelt würden.

    (Der Rest der Physik, so mag man sich als Experimentalphysiker einbilden, ist bloß noch “Modellierung”.)

    > […] um die Formulierung einer eleganten Physik, die allein auf klaren Prinzipien aufgebaut ist

    Das für die Relativitätstheorie wesentliche Prinzip wurde Einstein allerdings offenbar erst im Laufe seiner Arbeiten bewusst, und er hat es erst 1916 so formuliert:

    Alle unsere zeiträumlichen Konstatierungen laufen stets auf die Bestimmung zeiträumlicher Koinzidenzen hinaus.

  331. #331 Captain E.
    24. August 2018

    @Karl-Heinz:

    Das sehe ich auch so. Den Paradigmenwechsel aber zu vollziehen, da kann ich nur sagen „Hut ab“.

    Und dabei hat Einstein dafür nicht einmal einen Nobelpreis erhalten! Er hat natürlich einen erhalten, aber das war, falls ich mich nicht irre, für die Entdeckung des photoelektrischen Effekts. Gehört der eigentlich schon zur Quantenmechanik? Einer derer vielen Väter ist Einstein ja auch noch, wenngleich es für ihn ein eher ungeliebtes Kind gewesen sein dürfte.

  332. #332 Philip
    Wuppertal
    24. August 2018

    Hallo Captain E .,

    Die Relativitätstheorien haben vielleicht nur einen Vater, aber ein paar (Ur-) Großväter und Onkel. Und wohl auch mindestens eine Tante.

    Natürlich. Einer ihrer direkten Urahnen war zweifellos GALILEI. Das gilt sogar für beide, denn der formulierte nicht nur das Relativitätsprinzip, sondern fand auch – im Gedankenexperiment (!) – heraus, dass die Fallbeschleunigung unabhängig von der Masse ist.
    Die SRT hat übrigens ein Halbgeschwister, die LÄT, die dieselben Voraussagen macht und mathematisch im Grunde identisch ist. Nur die naturphilosophische Interpretation ist eine andere.
    LORENTZ und POINCARÉ hielten allerdings an der Vorstellung von einem ruhenden Äther fest, der allerdings seinen Bewegungszustand und die Geschwindigkeit eines relativ zu ihm bewegten Körpers durch genau die Effekte verschleiert, die man heute relativistische Effekte nennt.
    Zu den Ahnen der ART zählen natürlich GAUSS und RIEMANN, welche die Krümmung einer Fläche bzw. ihrer Verallgemeinerung, einer Mannigfaltigkeit, ohne Bezug auf einen Einbettungsraum beschreiben konnten, aber auch The Big MINKOWSKI, ohne dessen Leistung es kaum möglich gewesen wäre, die Raumzeit als Pseudo-RIEMANN’sche Mannigfaltigkeit zu beschreiben.

    Und dabei hat Einstein dafür nicht einmal einen Nobelpreis erhalten!

    Gerade die ART war dazu zu umstritten, da sie auch schwer zu bestätigen ist. Man wollte vielleicht auch diejenigen Physiker nicht vergrätzen, die noch immer NEWTONs Hypothese von der „absoluten, mathematischen Zeit“ anhingen.

    Er hat natürlich einen erhalten, aber das war, falls ich mich nicht irre, für die Entdeckung des photoelektrischen Effekts.

    Für dessen Erklärung. Genauer gesagt, für die Erklärung des Befundes, dass nur Licht oberhalb einer gewissen Frequenz den photoelektrischen Effekt respektive einen Photostrom auslöst.
    Dabei griff er PLANCKs Lichtquantenhypothese auf und nahm sie auch ernst. PLANCK selbst war vorsichtiger gewesen und die Quantisierung der Lichtenergie als Eigenschaft der Wände eines Hohlraums interpretiert.
    Insgesamt kann man wohl sagen, dass EINSTEIN nicht unbedingt cleverer, sondern in erster Linie kühner war als seine Kollegen. Weithin akzeptierte „Alte Zöpfe“ interessierten ihn ebenso wenig wie Autoritäten.

  333. #333 Frank Wappler
    24. August 2018

    Alderamin (#328, 24. August 2018):
    > Konzept der Antithese oder des Widerspruchbeweises […]
    > Wenn gilt A ⇒ B, dann gilt auch ¬B ⇒ ¬A.

    These:
    Falls Messwerte von Messgrößen (bzw. in Anwendung von entsprechenden Messoperatoren) ermittelt wurden,
    die in der Dilatations-Formel der RT auftreten, und für die sich die Dilatations-Formel deshalb als Theorem der RT herleiten lässt,
    und sofern diese Messwerte jeweils einen bestimmten Versuch bzw. eine konsistente Versuchsreihe betreffen,
    dann erfüllen diese Messwerte das Dilatations-Theorem der RT garantiert.

    Antithese:
    Falls Werte in die Dilatations-Formel der RT eingesetzt werden, durch die diese Formel nicht als Gleichung erfüllt ist,
    dann wurden diese Werte nicht als Messwerte der entsprechenden RT-Messgrößen ermittelt, und/oder/bzw. nicht betreffend eine konsistente Versuchsreihe.

  334. #334 Karl-Heinz
    25. August 2018

    @Frank Wappler

    Ich persönlich würde so argumentieren.

    A: SRT-Modell ist richtig und vollständig
    B: Messwerte erfüllen die Voraussagen des SRT-Modells

    A=>B
    Wenn das Modell richtig ist erfüllen die Messwerte die Voraussagen des Modells.

    ¬B => ¬A
    Wenn die Messwerte die Voraussagen des Modells nicht erfüllen, so ist das Modell falsch oder nicht vollständig.

    @Wappler: Wie findest meine Wappelei?
    Sie müsste auch für Dummies verständlich sein. 😉

  335. #335 Karl-Heinz
    25. August 2018

    Ich meinte natürlich für Dummies wie mich. 😉

  336. #336 Karl Mistelberger
    26. August 2018

    > #324 Philip, Wuppertal, 24. August 2018
    > Sch*** was auf die Myonen, um es mal ganz salopp zu sagen.

    Die Leute, die sich mit hochenergetischen Teilchen beschäftigen, z.B. beim CERN, kommen mit dieser Haltung nicht weit. Sie sind pragmatisch und nehmen was Einstein behauptet für wahr. Mit Newton ist auf diesem Gebiet absolut kein Staat zu machen.

    Andererseits funktionierte die Navigation von Cassini auch ohne Relativitätstheorie ganz hervorragend.

  337. #337 Peter
    26. August 2018

    @Alderamin #312
    Die Halbwertszeit von Protonen wird mit mehr als 10³² Jahren angegeben. Bei einer sehr starken Beschleunigung entgegen ihrer Bewegungsrichtung (“Zusammenprall”) zerfallen sie in wenigen milliardstel Sekunden. Welchen weiteren Beleg benötigst Du denn noch? Es ist doch offensichtlich, dass die Beschleunigung/Abbremsung den Zerfall von Strukturen erheblich beschleunigt und damit die Lebensdauer/Halbwertszeit verringert.

  338. #338 Karl-Heinz
    26. August 2018

    @Peter

    Mir scheint, dass du keine Ahnung hast, was eine Proton-Proton Kollision ist.

  339. #339 Peter
    26. August 2018

    @Karl-Heinz
    Wärst du einverstanden damit, dass wir uns auf Inhalte konzentrieren? Inhaltslose Kommentare stören die Kommunikation. Ist nicht böse gemeint.

  340. #340 Karl-Heinz
    26. August 2018

    @Peter

    Ich wollte eigentlich, dass du uns etwas über die Proton-Proton Kollision erzählst. Bei dieser Kollision kann ja alles mögliche passieren. Naja es entstehen halt mal eine Vielzahl von Teilchen.
    Zum Beispiel p +p –> k * p + Vielzahl von Teilchen und Photonen. Ups Proton hat sich vermehrt. Was ist jetzt mit dem Zerfall? 😉

  341. #341 Peter
    26. August 2018

    @Karl-Heinz
    Nach meinem Kenntnisstand sind sämtliche Wechselwirkungen in der Physik mit den vier Grundkräften assoziiert (mal abgesehen von der dunklen Energie/Materie). In Teilchenbeschleunigern sind das insbesondere Elektromagnetismus und starke Kernkraft. Dem kann sich auch der Protonenzerfall bzw. die Protonenkollision nicht entziehen. Oder welche andersartige Physik bzw. welche unbekannten Kräfte vermutest Du hinter den Vorgängen in Teilchenbeschleunigern?

  342. #342 Karl-Heinz
    26. August 2018

    @Peter

    Protonen bestehen aus Quarks, die durch Gluonen gebunden sind. Bei einem Zusammenstoß zwischen zwei Protonen sind es also eigentlich die Gluonen und die Quarks, die miteinander wechselwirken. Bei den hochenergetischen Kollisionen von Protonen am LHC entstehen alle Arten von Teilchen, sowohl solche der gewöhnlichen Materie als auch solche wie sie direkt nach dem Urknall existierten.

    Die hervorgebrachten Teilchen sind wegen der Beziehung E=mc2 für gewöhnlich viel schwerer als die ursprünglich kollidierten Teilchen. Um es mit einfachen Worten zu sagen: Die insgesamt in die Teilchenkollision gesteckte Energie kann nach der Kollision in Masse vorliegen. In Proton-Proton-Kollisionen kann “alles” passieren, solange einige wichtige Prinzipien wie beispielsweise die Energie- und Impulserhaltung erfüllt werden.

    Mit einem Zerfall des Protons im herkömmlichen Sinn hat das Ganze eigentlich nichts zu tun. 😉

  343. #343 Karl-Heinz
    26. August 2018

    @Peter

    Siehst du deinen Irrtum ein?

  344. #344 Peter
    26. August 2018

    @Karl-Heinz
    Die Beschleunigung durch die Magnete sowie die Bremsung durch “Kollision” wird durch dieselben Wechselwirkungen bedingt. Auch der Zerfall von Teilchen kann nur durch die vier bekannten Wechselwirkungen entstehen. Deine Aussage “Mit einem Zerfall des Protons im herkömmlichen Sinn hat das Ganze eigentlich nichts zu tun.” ist also falsch. Falls sie richtig sein sollte, dann erkläre bitte, welche geheimnisvollen Kräfte den Unterschied machen?

  345. #345 Peter
    26. August 2018

    @Karl-Heinz
    Klarer: Die Protonen werden durch ein elektrisches Feld beschleunigt und bei der “Kollision” (was ist das überhaupt?) bremsen sich sich sehr stark (viel stärker als die Beschleunigung) durch ein elektrisches Feld ab. Wo ist also der Unterschied?

  346. #346 Karl-Heinz
    26. August 2018

    @Peter

    Durch die Anregungsenergie entstehen neue Teilchen, die um ein vielfaches schwerer sein können als ein Proton. Ich würde hier nicht vom Zerfall eines Protons zu sprechen.

  347. #347 Karl-Heinz
    26. August 2018

    @Peter

    Ich werde dem Wappler noch entgegen wappeln und noch ein paar Gedanken zum Zerfall bringen.

  348. #348 Karl-Heinz
    26. August 2018

    @Peter

    Was ich noch für meine Ausführungen benötige ist ein Wienfilter. Vielleicht hast du Interesse dieses Dingsbums (Wienfilter) vorzustellen?

  349. #349 Peter
    26. August 2018

    @Karl-Heinz
    Fakt ist, dass durch Aufeinanderstoßen der elektrischen Felder offensichtlich eine Struktur instabil werden kann. Dadurch ändert sich die Lebensdauer (oder Halbwertszeit oder wie wir das auch immer nennen wollen). Nichts anderes habe ich behauptet: Die Myonen werden im Labor durch starkes Abbremsen (mit Elektromagneten) auf 0 km/h gebracht. Dadurch kann sich ihre Lebensdauer ebenfalls ändern. Die SRT spielt bei dieser Betrachtung überhaupt keine Rolle. Also ist die unterschiedliche Lebensdauer von Myonen, welche auf die Erde fallen und welche, die im Labor erzeugt und abgebremst werden, überhaupt kein “Beweis” und auch keine Bestätigung der SRT. Allenfalls widerspricht diese unterschiedliche Lebensdauer der SRT nicht.

  350. #350 Karl-Heinz
    26. August 2018

    @Peter

    Wenden wir uns einstweilen dem Wienfilter zu.
    Hast du keine Lust dazu?

  351. #351 Philip
    Wuppertal
    26. August 2018

    > Sch*** was auf die Myonen, um es mal ganz salopp zu sagen.

    Die Leute, die sich mit hochenergetischen Teilchen beschäftigen, kommen mit dieser Haltung nicht weit.

    So meine ich das überhaupt nicht, ich meine in diesem Zusammenhang.

    Gelegentlich hängen sich Leute zu sehr an den Myonen auf und versuchen alternative Erklärungen für deren (um etwa den Faktor 50, AFAIK) längere längere Halbwertszeit anzubieten, und ich habe schon das sogenannte Kühlschrank-Pseudoargument gelesen, dass niemand auf die Idee komme, die längere Haltbarkeit von Lebensmitteln durch eine verlangsamte Zeit zu erklären usw..

    Die verstehen nicht, dass dies hier etwas komplett anderes ist und die behauptete “Zeitdilatation” (was ich auch nicht für ein glücklich gewähltes Wort halte) nicht etwa dazu herhalten soll, die längere Halbwertszeit der Myonen zu erklären, sondern vielmehr umgekehrt diese längere Halbwertszeit eine Nichtwiderlegung der “Zeitdilatation” darstellt.

  352. #352 Peter
    26. August 2018

    @Philip
    Die Zeitdilatation (ich weiß, der Begriff ist unglücklich) beruht einzig und allein auf der Relativität der Gleichzeitigkeit (bzw. folgt unweigerlich daraus) – richtig?

  353. #353 Philip
    Wuppertal
    27. August 2018

    Hallo Peter,

    richtig. Ich würde sogar eher sagen: Was traditionell als Zeitdilatation oder aber als Längenkontraktion bezeichnet wird, sind Nebeneffekte der Relativität der Gleichzeitigkeit.
    Ich bringe ja gern das Bild mit der Salami und dem Lineal bzw. den beiden Salamis, wobei die Längsrichtung (1z›) die zeitartige Richtung (1t») und die Querrichtung (1x›) eine raumartige Richtung (1x») veranschaulicht – mutatis mutandis, natürlich, denn während ein räumliches Abstandsquadrat

    Δz² + Δx² = Δs²

    ist, ist ein raumzeitliches durch

    Δt² – Δx²/c² = Δτ²

    gegeben.

    Die Länge L steht für die Dauer T eines Vorgangs, der sich in einem umgrenzten Volumen abspielt, dass wiederum durch den Querschnitt gegeben ist. Der Durchmesser d der Salami steht für eine Abmessung des besagten Volumens, zum Beispiel in x-Richtung.

  354. #354 Karl-Heinz
    27. August 2018

    @Peter

    Wenn man Myonen einer dauernden transversalen Beschleunigung von bis zu ∼10^18 g aussetzt, hat das Auswirkungen auf die Zeitdilatation? Was glaubst du? Oder hängt die Zeitdilatation nur von der Relativgeschwindigkeit der Myonen ab?

  355. #355 Peter
    27. August 2018

    @Philip
    Es gibt ja beim ZP eine kontroverse Diskussion darüber, welche Messung von welchem Beobachter “real” bzw. die “richte” ist. Tatsächlich ist es aber doch so, dass alle Messungen real und richtig sind, sich jedoch gegenseitig nicht beeinflussen. D.h. eine im Koordinatensystem des Raumfahrers ruhende Uhr “kümmert” es nicht, wie viele andere, nicht ruhende Uhren etwas anderes messen. Richtig?

  356. #356 Frank Wappler
    27. August 2018

    Karl-Heinz schrieb (#334, 25. August 2018):
    > Wenn das Modell richtig ist erfüllen die Messwerte die Voraussagen des Modells.
    > Wenn die Messwerte die Voraussagen des Modells nicht erfüllen, so ist das Modell falsch oder nicht vollständig.

    Diese beiden Aussagen stimmen weitgehend damit überein, wie auch ich die Begriffe “Modell” und “Messwert” und “Voraussage” verstehe und verwende.

    (Dem disjunktiven Zusatz “oder nicht vollständig” in der zweiten zitierten Aussage scheint allerdings keine direkte, formale Entsprechung in der ersten Aussage gegenüberzustehen. Daher scheint es nicht ganz richtig, genau diese beiden zitierten Aussagen als “These und Antithese” zu bezeichnen. Sicherlich ließen sich dafür noch geeignetere, noch etwas sorgfältiger formulierte Aussagen finden …)

    Aber:
    Im Zusammenhang mit Messwerten geht es nicht nur um die verschiedentlichen Modelle, die bestimmte Messwerte (Versuch für Versuchen) vorhersagen, und die entweder die ggf. schon ermittelten Messwerte richtig zusammenfassen, oder ansonsten schon experimentell falsifiziert sind;

    sondern es geht dabei selbstverständlich vor allem auch um die Definitionen der entsprechenden, zugrundegelegten Messgrößen, also um die (jeweiligen Definitionen der entsprechenden) Messoperationen; also um die Festsetzung(en), wie bestimmte Messwerte überhaupt erst aus den Versuch für Versuch gesammelten Beobachtungsdaten gewonnen werden sollen.

    Es geht nicht nur um Modelle (die bestimmte Wert voraussagen und zusammenfassen);
    sondern es geht auch um die dafür (jeweils) zugrundegelegte Theorie, d.h. jeweils ein bestimmtes System aus selbstverständlichen/axiomatischen Begriffen, den unter Benutzung dieser Begriffe konstruierten bestimmten (Messgrößen-)Definitionen, sowie ggf. um deren logische Konsequenzen/Zusammenhänge/Theoreme.

    > A: SRT-Modell ist richtig und vollständig

    Die Relativitätstheorie ist eine Theorie (im obigen Sinne), und kein Modell (im obigen Sinne).

    Die (S)RT beschäftigt sich u.a. damit,

    wie Versuch für Versuch festgestellt werden soll, ob je zwei gegebene, voneinander getrennte Beteiligte gegenüber einander ruhten (Mitglieder des selben Inertialsystems waren), oder nicht,

    ohne irgendwelche Vorhersagen hinsichtlich der entsprechend ermittelten Messwerte zu machen;
    also ohne irgendwelche Erwartungen, dass z.B. die beiden Beteiligten J und K, die ein einem bestimmten Versuch als gegenüber einander ruhend gemessen wurden, auch im darauffolgenden Versuch gegenüber einander ruhend sein würden;

    und u.a. auch damit,

    wie Versuch für Versuch die Lebensdauer eines gebenen instabilen Teilchens mit der Lebensdauer eines anderen gegebenen instabilen Teilchens verglichen werden kann,

    ohne irgendwelche Vorhersagen hinsichtlich der entsprechend ermittelten Messwerte zu machen …

  357. #357 PS
    27. August 2018

    @Philip #351

    „Zeitdilatation“ ist kein glücklich gewähltes Wort. Das allerdings.
    Ich gehe einen Schritt weiter und meine, es ist der unglücklichste und irreführendste Begriff der Physikgeschichte (und der Philosophiegeschichte, weil er „Zeit“ als Substanz (Newton), statt als Relation (Leibniz) betrachtet).
    „Zeitdilatation“ (“Verlangsamung des Zeitablaufs”) ist irreführend, weil das Zurückbleiben der Anzeige einer bewegten Uhr S‘ gegenüber den Anzeigen der synchronisierten Uhren S, an denen sich die Uhr S‘ vorbeibewegt, nicht auf eine unterschiedliche Ganggeschwindigkeit der Uhren (ein unterschiedliches Vergehen von Zeit), sondern auf die Konstanz der Wirkungsausbreitung c=konst zurückzuführen ist. Weder gehen „alle bewegten Uhren S’“ („die Zeit“) noch auch nur eine einzige bewegte Uhr S‘ langsamer. Der Effekt der SRT erfasst – wie das Philip (mit dem Salamigleichnis) zu Recht immer wieder hervorhebt – ineinandergreifend zeitliche und räumliche Abstände (Relationen), und er ist symmetrisch, gilt also auch für eine Uhr der Uhrenreihe S gegenüber den synchronisierten Uhren S‘.
    Es ist auffällig, dass die Anhänger des Begriffs der “Zeitdilatation” seit längerer Zeit keine Argumente mehr vorbringen, sondern nur an ihr von Kindesbeinen an geglaubtes Dogma weiter glauben möchten, wie an die Dreifaltigkeit nach dem Konzil von Konstantinopel.
    Daran anschließend behaupten sie – unter dem Applaus der “Mehrheit” – weiterhin fest und standhaft, das Zwillingsparadoxon entstünde “wegen der Zeitdilatation”. Die “Ursache” des ZP, wenn man so will, ist aber die Beschleunigung (denkt man sie weg, kommt es nicht zum ZP). Dass zu dem Effekt Geschwindigkeit sowie Reisedauer beitragen, ändert daran nichts.

  358. #358 Alderamin
    27. August 2018

    @PS

    Es ist auffällig, dass die Anhänger des Begriffs der “Zeitdilatation” seit längerer Zeit keine Argumente mehr vorbringen, sondern nur an ihr von Kindesbeinen an geglaubtes Dogma weiter glauben möchten, wie an die Dreifaltigkeit nach dem Konzil von Konstantinopel.

    Verstehe immer noch nicht das Problem. Ein Myon lebt im Labor im Mittel 2,2 µs, eines aus einem Einschlag eines hochenergetischen kosmischen Partikels in 10-15 km Höhe existiert im Schnitt rund zehnmal so lange (Geschwindigkeit/Höhe). Wieso darf ich nicht sagen, dass seine Zeit aus meiner Sicht langsamer als die des Teilchens im Labor verging?

    Die “Ursache” des ZP, wenn man so will, ist aber die Beschleunigung (denkt man sie weg, kommt es nicht zum ZP).

    Kommt darauf an, was man unter “Beschleunigung” versteht. Wenn das einen instantanen Wechsel des Bezugsystems mit einschließt, die über einen Zeitraum von genau 0s auf den betrachteten Zwilling wirksam ist, dann ja, meinetwegen. Mit dem Effekt der ART und der Äquivalenz von Beschleunigung und Gravitation hat es jedoch nichts zu tun, und darum, das zu zeigen, ging’s mir in den Artikeln.

  359. #359 Philip
    Wuppertal
    27. August 2018

    Hallo Peter,
    eine Debatte darüber, welche Messung die richtige sei, wird der Bezeichnung Relativitätstheorie nicht gerecht. Fortbewegung ist ja relativ.

    …eine im Koordinatensystem des Raumfahrers ruhende Uhr „kümmert“ es nicht, wie viele andere, nicht ruhende Uhren etwas anderes messen. Richtig?

    Richtig. Im Ruhesystem des Raumfahrers, würde ich sagen, das ist dasjenige Koordinatensystem Σ’, in dem er ruht.

    In diesem geht die Uhr des nicht Mitreisenden langsamer als die eigene, denn wenn der Raumfahrer dessen Signal mit Zeitstempel empfängt, wird er es als wesentlich jünger interpretieren (also das Signal), wenn er sich selbst als ruhend ansieht, als wenn er Σ als Bezugssystem benutzt, relativ zu dem er sich mit v› bewegt und in dem ihn das Signal gleichsam einholen muss.

    Was beide voneinander sehen, solange sie sich voneinander entfernen, ist ohnehin durch den longitudinalen DOPPLER-Effekt zu beschreiben, und der Faktor ist nicht γ, sondern

    K:=\sqrt{\frac{c+v}{c-v}}=γ(1+\frac{v}{c}

    Sobald der Reisende umkehrt, kehrt sich für ihn allerdings der DOPPLER-Effekt um, d.h. aus Zeitlupe wird Zeitraffer.
    Dies wird umgekehrt auch der Verbleibende beim Reisenden sehen, aber erst viel später.

    Im Endeffekt kommt der Reisende nach weniger Eigenzeit zuhause an als sie der Verbliebene erlebt hat. Ob er dann auch biologisch jünger ist, sei dahingestellt – ionisierende Strahlung macht alt.

  360. #360 Philip
    Wuppertal
    27. August 2018

    Joa Kruzifixoamoal!

    K := √{(c+v)(c-v)} = γ(1+(v/c))

    sollte die Formel heißen.

  361. #361 Peter
    27. August 2018

    Im Endeffekt kommt der Reisende nach weniger Eigenzeit zuhause an als sie der Verbliebene erlebt hat. Ob er dann auch biologisch jünger ist, sei dahingestellt – ionisierende Strahlung macht alt.

    Aber genau in diesem Punkt streiten sich ja die Realtivisten mit den Kritikern.
    Es kann aber gezeigt werden, dass die unterschiedlichen Messungen der bewegten Beobachter keinen Einfluss aufeinander haben, d.h. bewegte Uhren gehen eben nicht schneller!

    Dazu erweitere ich ein berühmtes Gedanken-Experiment zur Relativität der Gleichzeitigkeit
    Vorbedingung:
    Der “Einstein-Zug” wird mit zwei Stoppuhren ausgestattet, eine am Anfang und eine am Ende des Zuges. Der Zug steht vor dem Bahnhof.

    Vorgang:
    Die Stoppuhren werden synchron gestartet und danach der Zug auf 0,5c beschleunigt. Sobald er den Bahnhof passiert, löst der Blitz in der Mitte des Zuges aus. Sobald das Licht des Blitzes die Stoppuhren erreicht, stoppen sie.

    Ergebnis:
    Der Beobachter im Zug sieht die Stoppuhren gleichzeitig stoppen.
    Der Beobachter am Bahnsteig sieht sie nacheinander stoppen.
    Beide haben “Recht”, beide Messungen sind korrekt und real. Bisher nichts Neues.
    Der Zug fährt nun aus dem Bahnhof und rollt aus. Danach kehrt er zurück in den Bahnhof.
    Nun lesen beide Beobachter die Stoppuhren ab. Beide Uhren zeigen dieselbe Stoppzeit an!

    Fazit:
    Damit ist bewiesen, dass beide Messungen korrekt und real waren, jedoch für die Gleichezeitigkeit innerhalb des Zuges nur die Messung der mitgeführten Uhren relevant ist.
    Da aus der Relativität der Gleichzeitigkeit auch die “Zeitdilatation” folgt, gilt für sie dasselbe: jeder Beobachter misst korrekt und real, jedoch hat das keinen Einfluss auf das Altern von bewegten Uhren, Myonen oder Raumfahrern!

  362. #362 Peter
    27. August 2018

    Es sollte natürlich heißen “d.h. bewegte Uhren gehen eben nicht langsamer!”

  363. #363 Karl-Heinz
    27. August 2018

    @Peter

    Ehrlich gesagt verstehe ich dich nicht ganz.
    Du sagst: bewegte Uhren gehen eben nicht langsamer!”
    In Bezug auf was gehen sie nicht langsamer?

  364. #364 Karl-Heinz
    27. August 2018

    @Peter

    Mein Kolleg fliegt mit 99,5% der Lichtgeschwindigkeit einen Kreis mit einem Radius von einem Lichtjahr.
    Nach seiner Rückkehr vergleichen wir wieder unsere Uhren. Ich kann relativ einfach berechnen, um wie viel seine Uhr nach seiner Ankunft nachgeht. Wie sieht es mit dir aus? Sieht so aus, als dass ich im Vorteil bin, denn ich weiß “bewegte Uhren gehen langsamer”.

  365. #365 Karl-Heinz
    27. August 2018

    @Peter

    Anstatt dem Kreis fliegt jetzt mein Kollege eine Ellipse. Der Umfang der Ellipse soll gleich groß sein wie der Umfang des Kreises. Die Bahngeschwindigkeit auf der Ellipse soll ebenfalls konstante 99,5% der Lichtgeschwindigkeit betragen. Durch die Form der Ellipse ist die Beschleunigung meines Kollegen während des Fluges jetzt nicht mehr konstant. Nun lieber Peter, was glaubst du? Hat die unterschiedliche Beschleunigung nun eine Auswirkung auf die Zeitdilatation meines Kollegen?

    Ich hoffe du kannst mir eine Antwort geben.
    Ansonsten schiebe bitte deine Aussage “bewegte Uhren gehen eben nicht langsamer” in den Mülleimer. 😉

  366. #366 Karl-Heinz
    27. August 2018

    @Frank Wappler

    Natürlich macht das SRT-Modell bzw.die SRT-Theorie (ich nehme es mit der Bezeichnung jetzt nicht so genau) eine Vorhersage, die man mit einem Experiment überprüfen kann.

  367. #367 PS
    27. August 2018

    @Alderamin #258

    Verstehe immer noch nicht das Problem. Ein Myon lebt im Labor im Mittel 2,2 µs, eines aus einem Einschlag eines hochenergetischen kosmischen Partikels in 10-15 km Höhe existiert im Schnitt rund zehnmal so lange (Geschwindigkeit/Höhe).

    Die Reise des Myons ist ein aus zwei Ereignissen (Beginn – Ende) bestehender Prozess wie jeder andere. Prozessereignisse können am selben Ort oder an verschiedenen Orten eintreten. In dem System, in dem die Ereignisse am selben Ort eintreten, haben die beiden Ereignisse den kürzesten zeitlichen Abstand voneinander.
    Das Leben, die Existenz, ist auch ein Prozess (Beginn – Ende). Dort, wo gelebt wird, dauert der Prozess im Vergleich zur Sicht anderer Systeme immer am wenigsten lang.
    Die längere Prozessdauer aus Sicht des Systems, in dem die Ereignisse nicht am selben Ort eintreten, bedeutet nicht, der Lebende würde nun länger “existieren”.
    Dies aus zwei Gründen:
    1) So viele Systeme, so viele unterschiedliche “Existenzdauern”.
    2) Ein Leben auf der Erde dauert aus Sicht des Myons länger, ein Leben beim Myon dauert aus Sicht der Erde länger. Wenn eine Zeit wirklich langsamer verginge als die andere, könnte die geschilderte Symmetrie nicht verwirklicht sein.

    Wieso darf ich nicht sagen, dass seine Zeit aus meiner Sicht langsamer als die des Teilchens im Labor verging?

    Du darfst, aber Du solltest nicht. Durch die Einschränkung “aus meiner Sicht” bist Du zwar gegen alle Argumente immun. Die Ausdrucksweise trifft aber nicht den physikalischen Sachverhalt, so wie sie auch bei der Schilderung des Dopplereffekts nicht den wahren physikalischen Sachverhalt trifft. Da wie dort ist die Schilderung zwar unwiderlegbar (“aus meiner Sicht” …). Wie sie aber beim Dopplereffekt nichts erklärt, erklärt sie auch bei der SRT nichts. Sie verwirrt nur und führt das Denken in falsche Geleise. Sie ist nur eine reizvolle Metapher. Wenn man sie Kindern erzählt, machen sie große Augen, aber verstehen tun sie nichts.

    Kommt darauf an, was man unter “Beschleunigung” versteht. Wenn das einen instantanen Wechsel des Bezugsystems mit einschließt, die über einen Zeitraum von genau 0s auf den betrachteten Zwilling wirksam ist, dann ja, meinetwegen.

    Jede Beschleunigung besteht aus einem Integral instantaner Beschleunigungen (instantane Wechsel in das immer infinitesimal “nächstschnellere” Bezugsystem).

  368. #368 Karl-Heinz
    27. August 2018

    @PS

    In dem System, in dem die Ereignisse am selben Ort eintreten, haben die beiden Ereignisse den kürzesten zeitlichen Abstand voneinander.

    Also man lebt länger, wenn man nicht an dem Ort stirbt, an dem man geboren wurde, oder?

  369. #369 Alderamin
    28. August 2018

    @PS

    Die längere Prozessdauer aus Sicht des Systems, in dem die Ereignisse nicht am selben Ort eintreten, bedeutet nicht, der Lebende würde nun länger “existieren”.

    Wieso nicht? Die Myonen kommen mit fast c aus 10 km Höhe. Damit brauchen sie 10 km / (3*10^5 km/s) = ca. 33,3 µs. Das kann ich so messen. Ich könnte theoretisch den Entstehungsprozess des Teilchens mit einer Uhr vor Ort stoppen und den Einschlag am Boden mit einer zweiten, und dann die Uhren vergleichen. In meinem Inertialsystem hat das Teilchen dann genau so lange existiert.

    Es ist ja richtig, dass es aus seiner Sicht anders aussieht und die Zeit kürzer ist. Aber in meinem Inertialsystem (das genau so gut ist wie jedes andere, aber dadurch ausgezeichnet, dass ich und mein Messeuqipment darin ruhen) hat es länger gelebt, als ein Myon, das nicht mit fast c durch die Gegend fliegt. Dann hat für mich objektiv nachmessbar die Zeit zugenommen, die das Teilchen existiert hat.

    Es ist ja eben nicht so, dass es ein übergeordnetes Inertialsystem gäbe, an dem sich alles zu orientieren hätte, und dessen Messungen für alle anderen maßgeblich sind. Sondern dass eben jedes Inertialsystem seine eigene Sicht hat bezogen auf Prozesse, die relativ zu ihm in Bewegung sind.

    1) So viele Systeme, so viele unterschiedliche “Existenzdauern”.

    Ja. Und? So viele, wie es Inertialsysteme gibt. So viele, wie es relative Geschwindigkeiten gibt.

    Was wäre die Alternative? Genau so viele Inertialsysteme, in die ich mich erst hineinversetzen muss, umden dortigen Standpunkt zu erfassen, und ihn dann wieder auf meine Messanzeigen umzurechnen? Was gewinne ich dadurch?

    2) Ein Leben auf der Erde dauert aus Sicht des Myons länger, ein Leben beim Myon dauert aus Sicht der Erde länger. Wenn eine Zeit wirklich langsamer verginge als die andere, könnte die geschilderte Symmetrie nicht verwirklicht sein.

    Doch, und wie das funktioniert, ist ja in meinen Artikeln vorgerechnet. Man kann dies ja auch wirklich ausnutzen. Ich könnte (rein theoretisch) einen Stern in 100.000 LJ Entfernung lebend erreichen, was ich ohne Zeitdilatation niemals hinbekäme (tun die Myonen im Prinzip ja auch). Und wenn ich unterwegs umkehre, kann ich echt jünger sein, als mein zurückgebliebener Zwilling und ihn überleben. Ich könnte wirklich in Jahr 10000 und mir die Welt dort anschauen (wenn es nicht so schwer wäre; für subatomare Teilchen wäre es ein Klacks).

    Und dennoch ist die Situation symmetrisch, solange man das Inertialsystem nicht wechselt.

    Durch die Einschränkung “aus meiner Sicht” bist Du zwar gegen alle Argumente immun.

    Aber darum geht es doch in der SRT. Deswegen heißt sie doch Relativitätstheorie, weil sie Prozesse so beschreibt, wie sie sich relativ zum Beobachter darstellen. “Aus meiner Sicht” heißt immer, aus der Sicht des Experimentators mit seinem Messequipment, so wie alle Naturgesetze gemessen werden. Aus welcher Perspektive sollen die Naturgesetze denn ansonsten gemessen und beschrieben werden?

    Die Ausdrucksweise trifft aber nicht den physikalischen Sachverhalt,

    Ok, dann mal anders rum: wie muss ich den physikalischen Sachverhalt denn beschreiben, wenn ich ausrechnen will, mit welcher Häufigkeit Myonen aus der kosmischen Strahlung den Boden erreichen? Was für Größen muss ich beachten (ohne dass das jetzt im Detail vorgerechnet werden müsste, nur vom Prinzip her)? Mit Worten beschrieben, so, wie Du meinst, dass der Sachverhalt korrekt beschrieben wird?

  370. #370 PS
    28. August 2018

    @Alderamin #369

    Sowohl beim Dopplereffekt als auch bei der Myonenreise stellen sich die Effekte bei oberflächlicher Betrachtung so dar, “DASS die Zeit im anderen System aus meiner Sicht anders vergeht”.

    Eine Zeit, die im anderen System in den verschiedenen Konstellationen der Relativbewegung (Annäherung, Entfernung etc) unterschiedlich vergeht (einmal langsamer, einmal gleich schnell, einmal schneller), widerspricht dem Begriff einer “einheitlich vergehenden Zeit”.

    Man könnte daher sagen, “ALS OB die Zeit im anderen System aus meiner Sicht anders vergeht”. Aber auch so ist und bleibt das eine Kinderversion ohne Erkenntniswert. Ich schau mir an, wie Christian Doppler in Salzburg mit diesem wissenschaftlichen Ansatz seine Theorie entwickeln würde.
    Kannst Du es?

    Wie muss ich den physikalischen Sachverhalt denn beschreiben, wenn ich [aus Sicht der Erde die Reisestrecke, die ein Myon während seines Lebens zurücklegt,] ausrechnen will.

    Um den Sachverhalt nicht in der Kinderversion, sondern physikalisch zutreffend zu beschreiben, benötige ich brauchbare Begriffe, in diesem Fall “Zeit” und “Raum”.

    In Allgemeinsprache: Ein Lichtstrahl legt immer das an Raum zurück, was er an Zeit dafür benötigt (Lichtuhr). Die Zeit ist die Ausbreitung eines Lichtstrahls. Der Raum ist auch die Ausbreitung eines Lichtstrahls. Die mit c begrenzte Wirkungsausbreitung ist die gemeinsame Wurzel von Zeit und Raum.

    Etwas wissenschaftlicher: Die Zeit ist das Verhältnis zwischen zwei Ereignissen, wie es durch die Wirkungsausbreitung von einem zum anderen (die Ausbreitung eines Lichtstrahls) bestimmt wird. Raum ist das Verhältnis zwischen zwei Ereignissen, wie es durch die Wirkungsausbreitung von einem zum anderen (die Ausbreitung eines Lichtstrahls) bestimmt wird. Beide Messgrößen sind durch ein- und dieselbe (frei festgelegte, mit Atomuhren oder Pulsaren geeichte) Vergleichsgröße (Sekunde) definiert.

    Mathematisch: Lorentz-Transformation, mit der Du die Berechnung ausführen kannst (“Eigengeschwindigkeit”).
    Nun kannst Du einem Kind die – für Deine Berechnungen gar nicht erforderlich gewesene – “als ob” Geschichte erzählen, dass die Zeit beim Myon langsamer vergeht (“aus meiner Sicht breitet sich das Licht beim Myon langsamer aus, aber die Lichtgeschwindigkeit ist konstant, das wirst du verstehen, wenn du groß bist” usw).

  371. #371 Karl Mistelberger
    28. August 2018

    > #370 PS, 28. August 2018
    > Sowohl beim Dopplereffekt als auch bei der Myonenreise stellen sich die Effekte bei oberflächlicher Betrachtung so dar, “DASS die Zeit im anderen System aus meiner Sicht anders vergeht”.
    >>Eine Zeit, die im anderen System in den verschiedenen Konstellationen der Relativbewegung (Annäherung, Entfernung etc) unterschiedlich vergeht (einmal langsamer, einmal gleich schnell, einmal schneller), widerspricht dem Begriff einer “einheitlich vergehenden Zeit”.
    > Man könnte daher sagen, “ALS OB die Zeit im anderen System aus meiner Sicht anders vergeht”. Aber auch so ist und bleibt das eine Kinderversion ohne Erkenntniswert. Ich schau mir an, wie Christian Doppler in Salzburg mit diesem wissenschaftlichen Ansatz seine Theorie entwickeln würde.

    Wie real sind die Effekte der Speziellen Relativitätstheorie?

    Vorläufer der Speziellen Relativitätstheorie
    Bis zum Jahr 1905:
    • Relativitätsprinzip („Gleichberechtigung aller
    Inertialsysteme“) -> nur eine Fiktion!
    • Relativität der Gleichzeitigkeit ->nur eine Fiktion!
    • Zeitdehnung und Längenkontraktion, Lorentz – Transformation, wahre und scheinbare Längen und Zeiten -> nur Fiktionen!
    • Abhängigkeit der Elektronenmasse von der Geschwindigkeit -> nur eine Fiktion!
    • Folge dieser Fiktionen: Scheinbare Konstanz der Lichtgeschwindigkeit für alle Beobachter!

    Spezielle Relativitätstheorie
    1905: Spezielle Relativitätstheorie
    • Gemessene Zeiten und Längen hängen vom Beobachter (Inertialsystem) ab. Die Umrechnung zwischen zwei Inertialsystemen wird mit Hilfe der Lorentzschen Formeln bewerkstelligt.
    • Auch was „gleichzeitig“ ist, hängt vom Inertialsystem ab.
    • Energie besitzt Trägheit (E = mc^2).
    • Relativität der Gleichzeitigkeit
    • Zeitdilatation
    • Längenkontraktion

    Diese Kinderversion erklärt die Beobachtungen widerspruchsfrei. Christian Doppler hätte seine Freude damit.

    Aus https://homepage.univie.ac.at/franz.embacher/Rel/UMP2011/100JahreZwillingsparadoxon.pdf

  372. #372 Peter
    28. August 2018

    @Alderamin

    Aber darum geht es doch in der SRT. Deswegen heißt sie doch Relativitätstheorie, weil sie Prozesse so beschreibt, wie sie sich relativ zum Beobachter darstellen. “Aus meiner Sicht” heißt immer, aus der Sicht des Experimentators mit seinem Messequipment, so wie alle Naturgesetze gemessen werden.

    Korrekt, jeder darf und soll aus seiner Sicht messen und rechnen. Jedoch lässt sich nicht jede Messung/Rechnung auf das beobachtete Objekt übertragen! Beispiele:
    – Ich messe die Größe des Mondes aus meiner Sicht mit einem Zollstock und komme auf wenige Zentimeter. Meine Messung ist korrekt, aber ich kann sie nicht auf den Mond übertragen.

    – Ich am Bahnhof sehe eine andere Reihenfolge der Blitzdetektion im Einstein-Zug als der Beobachter im Zug. Ich kann meine Beobachtung aber nicht einfach übertragen auf den Zug (siehe #361).

    Fazit: Messungen/Berechnungen aus unterschiedlichen Sichten beeinflussen sich nicht gegenseitig!

    Nach Deiner Darstellung bzgl. der Myonen misst/berechnet ein Beobachter am Erdboden einen verlangsamten Uhrengang im Myon. Und durch eine “spukhafte Fernwirkung” wird diese Messung/Rechnung nun auf das Myon übertragen und die Uhr geht plötzlich auch im Myon langsamer. Das ist falsch.

  373. #373 Peter
    28. August 2018

    @Karl Mistelberger

    Relativität der Gleichzeitigkeit ->nur eine Fiktion!

    Ungenau ausgedrückt. Jeder Beobachter nimmt eine andere Gleichzeitigkeit wahr, jedoch sind diese Wahrnehmungen unabhängig voneinander. Man kann nicht eine dieser Wahrnehmungen als “die richtige” bezeichen und auf eine andere übertragen! Siehe #361, dort steht der Beweis.

    Zeitdehnung und Längenkontraktion, Lorentz – Transformation, wahre und scheinbare Längen und Zeiten -> nur Fiktionen!

    Falsch verstanden. Jeder Beobachter misst/berechnet eine andere Zeitdehnung und Längenkontraktion. Diese lassen sich aber nicht aufeinander übertragen. Jede Messung/Berechnung ist nur dort gültig, wo sie durchgeführt wird!

    Abhängigkeit der Elektronenmasse von der Geschwindigkeit -> nur eine Fiktion!

    Falsch interpretiert. Die Elektronenmasse kann im Teilchenbeschleuniger gemesen werden, z.B. anhand der Länge des Bremsweges. Jeder unterschiedlich bewegte Beobachter wird aber eine ANDERE Masse für sich messen/berechnen! Keine dieser gemessenen/berechneten Massen lässt sich auf eine der anderen gemessenen/berechneten Massen übertragen!

    Folge dieser Fiktionen: Scheinbare Konstanz der Lichtgeschwindigkeit für alle Beobachter!

    Unsinn. Die Invarianz (nicht Konstanz!) ist keine Folge sondern die Voraussetzung der SRT!

  374. #374 Karl-Heinz
    28. August 2018

    @Peter

    Ich messe die Größe des Mondes aus meiner Sicht mit einem Zollstock und komme auf wenige Zentimeter. Meine Messung ist korrekt, aber ich kann sie nicht auf den Mond übertragen.

    Was du misst, ist eine Projektion. Ich hoffe ich muss dir jetzt nicht erklären was eine Projektion ist?

  375. #375 Frank Wappler
    28. August 2018

    Karl-Heinz schrieb (#366, 27. August 2018):
    > […] das SRT-Modell bzw. die SRT-Theorie (ich nehme es mit der Bezeichnung jetzt nicht so genau) […]

    Denjenigen, die “es mit der Bezeichnung” einigermaßen genau nehmen und genau meinen, dürfte allerdings der Buchstabe “T” in der Abkürzung “SRT” auffallen …

    > Natürlich macht […] die SRT […] eine Vorhersage, die man mit einem Experiment überprüfen kann.

    Wer “es jetzt nicht so genau nimmt“, mag natürlich gar keinen Unterschied zu der (von mir vertretenen) Aussage erkennen,
    dass die SRT einige beweisbare Theoreme beinhaltet, die bestimmte Beziehungen zwischen bestimmten (entsprechend den gedanken-experimentellen Messgrößen-Definitionen der SRT ggf. experimentell zu ermittelnden) Messwerten ausdrücken.
    (Natürlich lassen sich Beweis-Versuche auch überprüfen bzw. ggf. nachvollziehen; und es lässt sich auch überprüfen, ob der betreffende festgesetzte Messoperator zur Ermittlung von Messwerten aus gegebenen experimentellen Beobachtungsdaten jeweils richtig angewendet wurde, oder nicht.)

    Übrigens fällt (mir auch) auf, dass auf dieser gesamten ScienceBlogs-Seite weder im obigen Artikel noch in den (mittlerweile vergleichsweise zahlreichen) Kommentaren bisher von “überprüfen” oder “Überprüfung” oder im Zusammenhang mit der (S)RT von “testen” oder “Test” die Rede gewesen wäre …

    Im Zusammenhang mit dem Begriff “Vorhersage” (und in Unterscheidung zu “Theorem”) würde ich genaugenommen fordern, dass eine (“echte”) Vorhersage “experimentell Ergebnis-offen überprüfbar” sein soll,
    d.h. dass sie nur genau einen (Vorhersage- bzw. Erwartungs-)Wert aus dem Wertebereich einer (bestimmten, nachvollziehbaren, Wett-sicheren) Messgröße auswählen soll, der mindestens zwei (verschiedene) Werte enthält.

    Die (von mir vehement vertretene) Aussage, dass eine Theorie zwar Theoreme beinhalten mag, aber keine echten, experimentell Ergebnis-offen überprüfbaren Vorhersagen macht,
    beruht damit auf der Unterscheidung, dass jeweils eine bestimmte (im Rahmen der betreffenden Theorie definierten) Messgröße entweder nur genau einen Wert in ihrem Wertebereich hat, oder mehrere (verschiedene) Werte, aber keinesfalls “sowohl als auch”.

    (Vorhersagen im Sinne von “echten, experimentell Ergebnis-offen überprüfbaren Vorhersagen” sind dagegen in Modellen beinhaltet und mit den schon ermittelten Messwerten zusammengefasst. Folglich sprechen diejenigen, die “es mit der Bezeichnung” einigermaßen genau nehmen und genau meinen, ggf. jeweils vom “experimentellen Test eines Modells” und ggf. jeweils vom “Standard-Modell” einer bestimmten Theorie.)

  376. #376 Karl-Heinz
    28. August 2018

    @Frank Wappler

    Was gefällt dir am Myon-Experiment nicht?

  377. #377 Frank Wappler
    28. August 2018

    Alderamin schrieb (#369, 28. August 2018):
    > Die Myonen kommen mit fast c aus 10 km Höhe.

    … d.h. mit normalisierten Geschwindigkeit von \beta \lessapprox 1 gegenüber einem bestimmten Detektorsystem, dessen Mitglieder das betreffende Myon beobachten.

    > Damit brauchen sie 10 km / (3*10^5 km/s) = ca. 33,3 µs.

    Nein! — Nicht sie [die Myonen] brauchen ca. 33,3 µs,
    sondern die beiden Enden des betreffenden Detektorsystems (denen eine Entfernung von ca. 10 km voneinander eigen sein soll) brauchen 10 km / (3*10^5 km/s) = ca. 33,3 µs.

    > Das kann ich so messen.

    Der so gemesse Wert ist aber nicht die Lebens- bzw. Existenz- bzw. Reise-Dauer des Myons,
    sondern die Belegungsdauer des betreffenden Detektorsystems bzw. der betreffenden Strecke.

    > In meinem Inertialsystem hat das Teilchen dann genau so lange existiert.

    Das betreffende Inertial- bzw. Detektorsystem hat dann ggf. genau so lange existiert (33,3 µs).

    Das betreffende Myon aber gehört entsprechend der gegebenen Versuchsanordnung ausdrücklich nicht zu diesem System;
    und die Existenz- bzw. Reise-Dauer des Myons hat im beschriebenen Fall stattdessen bekanntlich den Wert
    \frac{10 \, {\rm km}}{\beta \, c} \times \sqrt{1 - \beta^2} \approx 33,3 \times \sqrt{1 - \beta^2} \, {\rm \mu s} \ll 33,3 \, {\rm \mu s}.

  378. #378 Frank Wappler
    28. August 2018

    Karl-Heinz schrieb (#376, 28. August 2018):
    > […]Frank Wappler Was gefällt dir am Myon-Experiment nicht?

    Kannst du bitte konkreter beschreiben, von welchem Myon-Experiment du unterstellst, dass mir daran etwas nicht gefiele ? —
    Denn es gab bzw. gibt etliche Myon-Experimente

    Und hinsichtlich deines vorausgegangenen Kommentars (#366, 27. August 2018) noch relevanter:

    Kannst du bitte konkreter beschreiben, was das betreffende Myon-Experiment (oder überhaupt irgendein Myon-Experiment) mit einer (sogenannten) “von der SRT gemachten Vorhersage, die man mit einem Experiment überprüfen kann ” zu tun hätte ?? —
    Das Wort “Myon” kommt jedenfalls insbesondere in Einsteins relevanten Schriften offenbar überhaupt nicht vor

  379. #379 PS
    28. August 2018

    @Karl Mistelberger #371

    1905: Spezielle Relativitätstheorie
    • Gemessene Zeiten und Längen hängen vom Beobachter (Inertialsystem) ab. Die Umrechnung zwischen zwei Inertialsystemen wird mit Hilfe der Lorentzschen Formeln bewerkstelligt.
    • Auch was „gleichzeitig“ ist, hängt vom Inertialsystem ab.
    • Energie besitzt Trägheit (E = mc^2).
    • Relativität der Gleichzeitigkeit
    • Zeitdilatation
    • Längenkontraktion
    Diese Kinderversion erklärt die Beobachtungen widerspruchsfrei. Christian Doppler hätte seine Freude damit.

    Ein Missverständnis: Du verwendest den Ausdruck “Kinderversion” überschießend: Ich habe mich nur gegen die “Zeitdilatation” im hier gebrauchten Verständnis gewandt (“das Myon lebt länger; dort vergeht die Zeit langsamer”).
    Vgl. zB den Begriff der “Längenkontraktion”: er bezieht sich nicht auf einen Vorgang, sondern auf den räumlichen Abstand zwischen Ereignissen (zu gleichen oder verschiedenen Zeiten). Dieser Begriff ist zutreffend, weil er mit der Definition der räumlichen Länge übereinstimmt.

    Der Begriff “Zeitdilatation” bezieht sich aber bei den meisten, die ihn verwenden (insbesondere den Populärwissenschaftlern), auf ein “langsameres Vergehen von Zeit in der Sphäre des anderen Systems”. Er trifft nicht zu, weil er mit der Definition der zeitlichen Länge nicht übereinstimmt.

    Unsere Aufgabe ist es, geduldig dafür zu werben, dass der Begriff “Zeitdilatation”, wenn er schon nicht durch einen besseren ersetzt werden kann, in Hinkunft – analog zum Begriff der Längenkontraktion – nur auf den zeitlichen Abstand (an gleichen oder an verschiedenen Orten) bezogen wird. Ansätze zu einer auf zeitliche und räumliche Abstände konzentrierte Betrachtungsweise gibt es (Artikel “Lichtuhr” in Schulentwicklung NRW, Handbuch zu 25 Schlüsselexperimenten; https://www.schulentwicklung.nrw.de/lehrplaene/lehrplannavigator-s-ii/gymnasiale-oberstufe/physik/hinweise-und-beispiele/se2.html). In dieser erstklassigen Fundgrube übrigens auch das Myonenexperiment sehr umsichtig behandelt, wenngleich die alte Begrifflichkeit “Zeitdilatation” immer wieder durchschimmert.

    Zu Embacher und ähnlichen populärwissenschaftliche (irreführenden) Darstellungen siehe oben #104.

  380. #380 Alderamin
    28. August 2018

    @PS

    In Allgemeinsprache: Ein Lichtstrahl legt immer das an Raum zurück, was er an Zeit dafür benötigt (Lichtuhr).

    Das tut ein Auto auch, aber das Problem ist ja, dass in einem System der Lichtstrahl die Strecke a zwischen den Spiegeln zurücklegt und in einem anderen die Strecke a / √(1-cos²(v/c)). Welche Strecke ist jetzt die richtige, oder hängt sie vom Inertialsystem ab? Davon hängt dann auch die Zeit ab.

    Meine Frage drehte sich aber ursprünglich darum, wie man die Häufigkeit einer Myon-Detektion am Erdboden ausrechnen kann, ohne einen Term mit verlangsamter Zeit oder verkürzter Strecke darin zu benutzen.

    “aus meiner Sicht breitet sich das Licht beim Myon langsamer aus, aber die Lichtgeschwindigkeit ist konstant, […]”

    Das ist nicht die Aussage. Die Aussage ist: in allen Bezugssystemen bewegt sich Licht gleich schnell, daraus folgt, dass eine relativ zu mir bewegte Lichtuhr aus meiner Sicht langsamer ticken muss und da diese einen objektiven Zeitraum misst, die Zeit im anderen System also messbar langsamer vergeht. Also existieren bewegte Myonen in meinem Inertialsystem länger als in einem zu ihnen ruhenden (oder langsamer bewegten).

    Da das für alle Inertialsysteme gilt, muss im bewegten System des Myons die Strecke verkürzt erscheinen. Betrachtet man den Vorgang aus dem System des Myons, erhält man keinen Widerspruch dazu, ob das Myon den Boden erreicht oder nicht, d.h. das Ergebnis ist über alle Systeme hinaus konsistent. Nicht jedoch die gemessenen Zeiten und Abstände. Warum das so ist, sieht man z.B. in Kommentar #182 an einem Beispiel.

  381. #381 Alderamin
    28. August 2018

    @Peter

    – Ich messe die Größe des Mondes aus meiner Sicht mit einem Zollstock und komme auf wenige Zentimeter. Meine Messung ist korrekt, aber ich kann sie nicht auf den Mond übertragen.

    Deine Messung ist nicht korrekt, weil man einen Sehwinkel nicht mit einem Zollstock messen kann. Ich rede hier immer von korrekt ausgeführten Messungen mit geeigneten Messmitteln.

    – Ich am Bahnhof sehe eine andere Reihenfolge der Blitzdetektion im Einstein-Zug als der Beobachter im Zug. Ich kann meine Beobachtung aber nicht einfach übertragen auf den Zug (siehe #361).

    Müsste ich erst ausrechnen, wie es sich da verhält, Deine Aussage bzgl. des Myons mit dem Pulsgeber in der Mitte der Strecke war jedenfalls oben nicht korrekt, wie ich in #182 gezeigt hatte.

    Fazit: Messungen/Berechnungen aus unterschiedlichen Sichten beeinflussen sich nicht gegenseitig!

    Hat ja auch niemand behauptet.

    Nach Deiner Darstellung bzgl. der Myonen misst/berechnet ein Beobachter am Erdboden einen verlangsamten Uhrengang im Myon.

    Ja. Alleine die Tatsache, dass ein signifikanter Anteil von ihnen den Boden erreicht, ist ein Beleg dafür, dass ihre Zeit langsamer abläuft. Die Korrektheit von GPS, wo die gravitative Zeitdilatation UND die durch die Geschwindigkeit gegebene beide berücksichtigt werden, ist ein anderer Beleg.

    Und durch eine “spukhafte Fernwirkung” wird diese Messung/Rechnung nun auf das Myon übertragen und die Uhr geht plötzlich auch im Myon langsamer.

    Bitte? Wo soll ich das gesagt haben? “Spukhafte Fernwirkung” war Einsteins Spottbegriff für die (zigfach bewiesene) Nichlokalität der Quantenwelt, die aber mit der Relativitätstheorie überhaupt nichts zu tun hat.

    Das ist falsch.

    Wohl wahr, aber eben nichts, was ich annähernd irgendwo behauptet hätte.

  382. #382 Karl Mistelberger
    28. August 2018

    > #379 PS, 28. August 2018
    > Der Begriff “Zeitdilatation” bezieht sich aber bei den meisten, die ihn verwenden (insbesondere den Populärwissenschaftlern), auf ein “langsameres Vergehen von Zeit in der Sphäre des anderen Systems”. Er trifft nicht zu, weil er mit der Definition der zeitlichen Länge nicht übereinstimmt.

    Embacher erläutert, was Albert Einstein gefunden hat:

    Ein starrer Körper welcher in ruhendem Zustand ausgemessen die Gestalt einer Kugel hat, hat also in bewegtem Zustande – vom ruhenden System aus betrachtet – die Gestalt eines Rotationsellipsoids …

    Man schließt daraus, daß eine am Erdäquator befindliche Unruhuhr um eine sehr kleinen Betrag langsamer laufen muß als eine genau gleich beschaffene, sonst gleichen Bedingungen unterworfene, an einem Erdpole befindliche Uhr.

    Aus #104 und #379 folgt Ihrer Ansicht nach, dass es sich bei Einstein um einen dilettierenden Populärwissenschaftler handelt.

    Zur Elektrodynamik bewegter Körper

  383. #383 Karl Mistelberger
    28. August 2018
  384. #384 Karl Mistelberger
    28. August 2018

    Ein guter Rat von einem, der es wissen muss:

    So what I’m saying there is that when you publish, if you don’t understand some of your data you should leave that out of the table.

    Ohterwise someone will read your data and they will win the Nobel Prize without doing any work.

    And I can tell you people who did that. the most famous one of course is Einstein.

    https://www.mediatheque.lindau-nobel.org/videos/33641/2014-man-helicobacter

  385. #385 Peter
    28. August 2018

    @Alderamin

    Ja. Alleine die Tatsache, dass ein signifikanter Anteil von ihnen den Boden erreicht, ist ein Beleg dafür, dass ihre Zeit langsamer abläuft.

    Falsch. Es kann auch andere Gründe haben, warum so viele Myonen den Erdboden erreichen. Korrekterweise muss es heißen: Die hohe Anzahl der Myonen, die den Erdboden ereicht, widerspricht der SRT nicht.

    Die Korrektheit von GPS, wo die gravitative Zeitdilatation UND die durch die Geschwindigkeit gegebene beide berücksichtigt werden, ist ein anderer Beleg.

    Falsch. Es sind viele störende Einfüsse vorhanden sind (z.B. Sonnenwind, Hintergrundstrahlung, Luftdichte etc. Jeder “Experte” schreibt was anderes dazu). Fakt ist: die Frequenz in den GPS-Satelliten wird herabgesetzt, da sie zu schnell (!!) laufen. Korrekt muss es heißen: Die Korrektheit von GPS widerspricht nicht der ART. (Die SRT kommt hier nicht vor).

  386. #386 Peter
    28. August 2018

    @Alderamin

    Fazit: Messungen/Berechnungen aus unterschiedlichen Sichten beeinflussen sich nicht gegenseitig!

    Hat ja auch niemand behauptet.

    Doch, Du hast gesagt, dass im ZP der auf der Erde gebliebene Zwilling die Uhr des reisenden Zwillings langsamer gehen sieht/misst/berechnet. Und dann plötzlich nach der Rückkehr geht die Uhr des reisenden Zwillings auch tatsächlich langsamer als die des auf der Erde verbliebenden. Das Widerspricht der Aussage, dass die Messungen der Zwlillinge sich nicht beeinflussen. Du widersprichst Dir also selbst.
    Durch welche “spukhafte Fernwirkung” (ich gebrauche diese Worte nicht, weil sie Eintein bzgl. der Verschränkung von Teilchen sagte, sondern weil sie passend sind!) soll denn nun die Uhr des Reisenden umgestellt worden sein? Welche der vier Grundkräfte war es? Bitte ankreuzen:
    1. Gravitation
    2. Schwache Kernkraft
    3. Starke Kernkraft
    4. Elektromagnetismus

  387. #387 Karl-Heinz
    28. August 2018

    @Peter

    Du hast ein Feld zum Ankreuzen vergessen.
    5. Zeitdilatation

  388. #388 Peter
    28. August 2018

    @Karl-Heinz
    Nein, die Zeitdilatation ist keine der Grundkräfte der Physik. Jegliche Wirkung MUSS assoziiert sein mit mindestens einer der vier Grundkräfte.

  389. #389 Karl-Heinz
    28. August 2018

    @Peter

    Ich weiß nicht ob dir der Begriff Eigenzeit was sagt. Eigenzeit ist die in einem mit dem mitbewegten Bezugssystem ablaufende Zeit. Die Eigenzeit ist ein fundamentaler Begriff der Relativitätstheorie.

  390. #390 Peter
    28. August 2018

    @Karl-Heinz
    Ich weiß nicht, ob dir der Begriff “Logik” erwas sagt. “Logik” ist ein fundamentaler Begriff in der wissenschaftlichen Arbeit.

    Nochmal: Wo ist die Verbindung der “Zeitdilatation” zur Physik? Welche Wechselwirkung verursacht den unterschiedlichen Gang der Uhren, wenn lediglich unabhängig gemessen wird?

  391. #391 Alderamin
    28. August 2018

    @Peter

    Falsch. Es kann auch andere Gründe haben, warum so viele Myonen den Erdboden erreichen. Korrekterweise muss es heißen: Die hohe Anzahl der Myonen, die den Erdboden ereicht, widerspricht der SRT nicht.

    Es kommen so viele an, wie gemäß SRT und Zerfallsdauer vorhergesagt werden. Das ist eine stärkere Aussage. Andersrum wäre ohne SRT nicht erklärbar, warum es so viele sind. Es gibt keine alternative Erklärung.

    Falsch. Es sind viele störende Einfüsse vorhanden sind (z.B. Sonnenwind, Hintergrundstrahlung, Luftdichte etc. Jeder “Experte” schreibt was anderes dazu).

    Habe noch nirgends war anderes von einem Experten (ohne Anführungszeichen) gelesen. Welcher äußere Effekt sol eine Atomuhr dazu bringen, mit der Präzision einer Atomuhr verstimmt zu sein? Auch hier gilt: die Uhr geht genau so, wie SRT und ART es vorhersagen. Mit entsprechend modifizierter Taktgeschwindigkeit bringt man sie wieder in Einklang mit irdischen Uhren.

    Die Korrektheit von GPS widerspricht nicht der ART. (Die SRT kommt hier nicht vor).

    Selbstverständlich kommt die SRT vor, die Satelliten bewegen sich doch mit einigen km/s. Aufgrund der SRT gehen sie 7 µs pro Tag nach. Aufgrund der ART 45 µs pro Tag vor. Daher lässt man sie um 38 µs pro Tag langsamer gehen, damit sie synchron zu Uhren am Erdboden sind.

    Doch, Du hast gesagt, dass im ZP der auf der Erde gebliebene Zwilling die Uhr des reisenden Zwillings langsamer gehen sieht/misst/berechnet. Und dann plötzlich nach der Rückkehr geht die Uhr des reisenden Zwillings auch tatsächlich langsamer als die des auf der Erde verbliebenden. Das Widerspricht der Aussage, dass die Messungen der Zwlillinge sich nicht beeinflussen.

    Ich kann da keinerlei Zusammenhang erkennen. Der rückkehrende Zwilling wechselt das Bezugssystem, deswegen bleibt seine Uhr gegenüber dem auf der Erde gebliebenen zurück. Die Zeitmessungen sind doch vollkommen unabhängig voneinander, am Ende wird nur der Stand beider Uhren verglichen.

    Durch welche “spukhafte Fernwirkung” (ich gebrauche diese Worte nicht, weil sie Eintein bzgl. der Verschränkung von Teilchen sagte, sondern weil sie passend sind!) soll denn nun die Uhr des Reisenden umgestellt worden sein? Welche der vier Grundkräfte war es? Bitte ankreuzen:
    1. Gravitation
    2. Schwache Kernkraft
    3. Starke Kernkraft
    4. Elektromagnetismus

    Keine? Der Wechsel des Bezugssystems war’s. Hat doch nichts mit den Grundkräften zu tun. (Die Gravitation hat was mit der Raumkrümmung und der gravitativen Zeitdilatation zu tun, aber die spielt beim Zwillingsparadoxon keine Rolle, wie ich nicht müde werde, zu wiederholen).

    Ich weiß auch nicht mehr, wie ich Dich überzeugen kann, ich habe in den Artikeln wirklich klein-klein vorgerechnet, warum die Zeit langsamer zu vergehen erscheint, warum das wechselseitig der Fall ist und warum es trotzdem kein Paradoxon gibt. Das war alles mathematisch zwingend. Dass das schwer mental überschaubar ist, kann ich nachvollziehen, aber es ist nunmal das, was rauskommt, und Prozesse wie die Funktion des GPS; die Myonen der kosmischen Strahlung oder Zerfälle in Beschleunigern bestätigen jeden Tag, dass die Formeln zutreffen. Wir können uns darüber streiten, wie die Begrifflichkeiten sind (wie mit Philip) oder ob der Formalismus korrekt angewendet wurde (wie mit Frank), aber an der RT und ihren Auswirkungen gibt es eigentlich nichts zu diskutieren.

  392. #392 PS
    28. August 2018

    @Alderamin #380

    “Ein Lichtstrahl legt immer das an Raum zurück, was er an Zeit dafür benötigt (Lichtuhr).”
    Das tut ein Auto auch.

    Ein Auto legt nicht einmal aus der Sicht eines einzigen Systems (geschweige denn – wie das Licht – aus der Sicht aller Systeme) eine Einheit Raum in der dazugehörigen einer Einheit Zeit zurück, zB eine Lichtsekunde Raum in einer Sekunde Zeit. Ein Lichtstrahl definiert Zeit und Raum (und den Zusammenhang dieser Größen), ein bewegter Materiepunkt nicht. Seine Koordinatengeschwindigkeit ergibt sich vielmehr erst aus dieser Definition (als Bruchteil von c).

    Wie kann man die Häufigkeit einer Myon-Detektion am Erdboden ausrechnen, ohne einen Term mit verlangsamter Zeit oder verkürzter Strecke darin zu benutzen.

    ZB mit dem Term V (Eigengeschwindigkeit). Auch die Rechnung mit verkürzter Strecke oder mit der Zeitdilatation (die das richtige Verhältnis der Prozessdauer aus Sicht des Myons und der Prozessdauer aus Erdensicht beinhaltet) führt zu richtigen Ergebnissen. Beim Rechnen muss man ja nicht mitdenken.

    “aus meiner Sicht breitet sich das Licht beim Myon langsamer aus, aber die Lichtgeschwindigkeit ist konstant, […]”
    Das ist nicht die Aussage.

    Nicht? Wie ist das bei der Betrachtung eines senkrecht zur Bewegungsrichtung ausgesendeten Lichtstrahls in einer bewegten Lichtuhr. Der erscheint dem Beobachter doch langsamer, als der schräge Lichtstrahl, den der Beobachter bei sich wahrnimmt (was dann mit “Zeitdilatation” korrigiert wird).
    Man handelt sich einen Widerspruch ein, wenn man Lichtstrahlen betrachtet, die in einer bewegten Lichtuhr in andere Richtungen ausgesendet werden (zB in oder gegen die Bewegungsrichtung). Die sind jetzt einmal schneller, einmal langsamer als der eigene Lichtstrahl. Soll das auch mit einer anders vergehenden Zeit korrigiert werden, damit das Licht immer “gleich schnell” bleibt?

    Die Aussage ist: in allen Bezugssystemen bewegt sich Licht gleich schnell, daraus folgt, dass eine relativ zu mir bewegte Lichtuhr aus meiner Sicht langsamer ticken muss, und da diese einen objektiven Zeitraum misst, die Zeit im anderen System also messbar langsamer vergeht.

    Aus Sicht der bewegten Lichtuhr finden die (Tick-)Ereignisse am selben Ort statt, aus Sicht des Beobachters nicht. Wegen der L-T (die wiederum auf der Definition von Zeit und Raum beruht) muss die Prozessdauer aus Sicht des Beobachters länger sein. Das symmetrische Ineinandergreifen von Zeit und Raum ist wird aber mit einem “langsameren Vergehen von Zeit” wo auch immer und aus wessen Sicht nicht richtig beschrieben.

    Wir bewegen uns im Kreis. Vielleicht gelingt uns einmal ein konstruktiver Blick auf das Denken des anderen, es muss ja nicht hier und jetzt sein.

  393. #393 Alderamin
    28. August 2018

    @PS

    Ein Auto legt nicht einmal aus der Sicht eines einzigen Systems (geschweige denn – wie das Licht – aus der Sicht aller Systeme) eine Einheit Raum in der dazugehörigen einer Einheit Zeit zurück, zB eine Lichtsekunde Raum in einer Sekunde Zeit.

    Ach, so war das gemeint, gleichwertige Einheiten, war missverständlich.

    ZB mit dem Term V (Eigengeschwindigkeit). Auch die Rechnung mit verkürzter Strecke oder mit der Zeitdilatation (die das richtige Verhältnis der Prozessdauer aus Sicht des Myons und der Prozessdauer aus Erdensicht beinhaltet) führt zu richtigen Ergebnissen.

    Mit Eigengeschwindigkeit habe ich die Rechnung noch nicht gesehen, aber jedenfalls ist die Betrachtung einer verlangsamten Prozessdauer offenbar zulässig, wenn die entsprechenden Formeln das richtige Ergebnis auswerfen.

    Nicht? Wie ist das bei der Betrachtung eines senkrecht zur Bewegungsrichtung ausgesendeten Lichtstrahls in einer bewegten Lichtuhr. Der erscheint dem Beobachter doch langsamer, als der schräge Lichtstrahl, den der Beobachter bei sich wahrnimmt (was dann mit “Zeitdilatation” korrigiert wird).

    Nö, der Lichtstrahl ist in beiden Fällen genau gleich schnell, nur beim schrägen Verlauf ist sein Weg länger, deswegen dauert ein Tick der Uhr länger. Dass die Lichtgeschwindigkeit immer konstant erscheint, ist doch die Kernaussage der SRT.

    Man handelt sich einen Widerspruch ein, wenn man Lichtstrahlen betrachtet, die in einer bewegten Lichtuhr in andere Richtungen ausgesendet werden (zB in oder gegen die Bewegungsrichtung). Die sind jetzt einmal schneller, einmal langsamer als der eigene Lichtstrahl.

    Nein, die sind immer gleich schnell! Wenn der Strahl dem Raumschiff, das fast mit c fliegt, vorauseilt, tut er das aus Sicht des ruhenden Beobachters nur mit geringem Geschwindigkeitsvorsprung. Wird er in Gegenrichtung ausgesendet, entfernt er sich mit fast 2c vom Raumschiff. Das habe ich oben im Artikel doch auch verwendet, als ich ausrechnete, wie lange aus System H“ der Flug von H‘ von der Erde zu Alpha Centauri dauert.

    Das kann auch gar nicht anders sein, denn c=const. Und aus Sicht des Raumschiffs verlässt der Strahl das Schiff auch immer mit c, auch wenn ein anderer Beobachter es mit 0,999c durch die Gegend fliegen sieht. Wie andere Beobachter sein Bezugssystem sehen, kann ihm vollkommen schnurz sein, in ihm gelten dieselben physikalischen Gesetze, und dazu gehört, dass Licht immer mit c unterwegs ist.

    Und deswegen kann man auch nie c erreichen, der Lichtstrahl eilt einem in jedem Bezugsystem voraus. Es kann nicht sein, dass man in einem Bezugssystem das Licht überholt und in einem anderen nicht. Es gibt kein „langsames Licht“.

    Das symmetrische Ineinandergreifen von Zeit und Raum ist wird aber mit einem “langsameren Vergehen von Zeit” wo auch immer und aus wessen Sicht nicht richtig beschrieben.

    Nur, wenn man partout einen absolut langsameren Zeitverlauf da hinein interpretieren will. Der besteht aber nicht. Trotzdem ist der Effekt real – hole ich die langsamer gelaufene Uhr in mein Bezugssystem, zeigt sie weniger Zeit an (Zwillingsparadoxon). Gehe ich mit meiner Uhr in ihr Bezugssystem, zeigt meine weniger an. Eigentlich ganz simpel.

    Wir bewegen uns im Kreis. Vielleicht gelingt uns einmal ein konstruktiver Blick auf das Denken des anderen, es muss ja nicht hier und jetzt sein.

    Ich verteidige nur den von mir in den Artikeln vertretenen Standpunkt gegen die Behauptung, er sei falsch. Ist er nicht. Vielleicht gibt‘s noch andere, mir nicht geläufige Betrachtungsweisen, die ich (noch) nicht nachvollziehen kann (bei Martin gab‘s einen Artikel mit Vierervektoren, der ist jedenfalls auch richtig), mag sein, aber das macht meinen hier vertretenen Standpunkt nicht falscher.

    Meinetwegen können wir‘s hier abbrechen. Ist eh‘ schon der längste Kommentarstrang auf Alpha Cephei, soweit ich das sehe.

  394. #394 Frank Wappler
    29. August 2018

    Alderamin schrieb (#393, 28. August 2018):
    > […] die Betrachtung einer verlangsamten Prozessdauer […]

    Dauer ist eine extensive Größe.
    Der Vergleich zweier ungleicher Dauern ist deshalb dadurch auszudrücken, dass gesagt wird,
    die eine Dauer sei kleiner (oder genauer, da es sich um ein eindimensionales Maß handelt, kürzer) als die andere Dauer;
    bzw. umgekehrt: die eine größer (oder genauer: länger) als die andere.

    Von “Verlangsamung” usw. spricht man dagegen beim Vergleich von Raten, die übrigens “intensive Größen” (bzw. deren jeweilige Werte) sind.

    Alderamin schrieb (#391, 28. August 2018):
    > [… » Anzahl der Myonen, die den Erdboden ereicht &171; …] Es kommen so viele an, wie gemäß SRT und Zerfallsdauer vorhergesagt werden.

    Ah! (Fortschritt !?) — Es lässt sich also offenbar trennen zwischen:

    – (S)RT einerseits, und

    – andererseits gewissen Modellen, die (z.B. aufgrund von Zerfalls- bzw. Lebensdauer-Messungen aus vorausgegangenen Versuchen) bestimmte Vorhersagen machen.

    (Modelle können Versuch für Versuch experimentell getestet und ggf. für falsifiziert befunden und verworfen werden.
    Aber nicht die Definition der betreffenden Messgröße, hier insbesondere “Lebensdauer”; bzw. nicht das beinhaltende System aus Begriffen, Definitionen und Theorem, hier insbesondere die SRT.)

  395. #395 Peter
    29. August 2018

    @Alderamin

    Der Wechsel des Bezugssystems war’s. Hat doch nichts mit den Grundkräften zu tun.

    Du sagst gerade, dass die SRT nichts mit Physik zu tun hat – und das ist korrekt. Es ist ein rein mathematisches Konstrukt. Es ist elegant und mathematisch korrekt, wird aber falsch angewendet!

    Welcher äußere Effekt sol eine Atomuhr dazu bringen, mit der Präzision einer Atomuhr verstimmt zu sein?

    Ich habe Dir doch welche aufgezählt, wie so ignorierst Du das einfach? Ich hatte vor ca. 10 Jahren mit einem Mitarbeiter bei der PTB Kontakt. Grund war eine Falschaussage in der Sendung “Quarks” bzgl. der ART (Eine Atomuhr auf Kölner Dom, eine am Boden). Damals teilte er mir mit, man habe das Experiment auf Wunsch des WDR in Köln durchgeführt, es hatte aber keine wissenschaftliche Relevanz bzw. das “Ergebnis” war tatsächlich erheblich geschönt. Dieser Mitarbeiter war und ist noch immer für den präzisen Gang von Atomuhren zuständig. Er erklärte mir sehr viel über die verschiedenen Störeinflüsse von Lufrdruck, Luftfeuchtigkeit, Strahlung, Temperatur etc, welche minimale Gangunterschiede hervorrufen können. Was sagst Du nun? Willst Du weiterhin etwas anderes behaupten?

    Selbstverständlich kommt die SRT vor, die Satelliten bewegen sich doch mit einigen km/s. Aufgrund der SRT gehen sie 7 µs pro Tag nach. Aufgrund der ART 45 µs pro Tag vor. Daher lässt man sie um 38 µs pro Tag langsamer gehen, damit sie synchron zu Uhren am Erdboden sind.

    Bevor Du einen Text kopierst solltest Du ihn vorher lesen und verstehen. In Deinem verlinkten Text wird behauptet, dass Positionen um 10 km pro Tag falsch gingen, würde man die RT nicht berücksichtigen. Das ist Blödsinn, oder hat Dein Navigationssystem eine Atomuhr eingebaut, die diesen Gangunterschied überhaupt bemerken würde? Nein, hat es nicht! Es berechnet seine Position aus der Position von 4 Satelliten. Die RT spielt hierbei keine Rolle! Ebenso steht dort, dass die Uhren in den Satelliten um 45-7=38 µs verstellt werden: auch das ist falsch! Sie werden von 10,23 MHz auf 10,229999995453 MHz gestellt. Das sind genau 45 µs, was allein der ART entspricht! Und dies wird nicht gemacht, um die Positionsbestimmung zu korrigieren, sondern um Zeitbestimmung zu ermöglichen. Die SRT hat hier keinen Platz!

  396. #396 Frank Wappler
    https://scienceblogs.de/sic/2014/01/27/science-of-science-communication/#ScienceBlogs.SandBox
    29. August 2018

    Philip schrieb (#359, 27. August 2018):
    > […] Im Endeffekt kommt der Reisende nach weniger Eigenzeit

    … Dauer! (Reisedauer des Reisenden!) …

    > zuhause an als sie

    … “sie“: die Verbleibensdauer des Verbliebenen! …

    > der Verbliebene erlebt hat.

    So weit, so gut.
    (Es ist wohl eine Konvention, denjenigen der beiden genannten Beteiligten “den Reisenden” zu nennen, dessen Dauer von seiner Anzeige des Getrenntwerdens der beiden bis zu seiner Anzeige des Wiederzusammentreffens kleiner als die entsprechende Dauer des anderen, “Verbliebenen” ist.)

    > Ob er dann auch biologisch jünger ist, sei dahingestellt

    Sehr richtig und wichtig!:
    Die Bewertung jeweils einer bestimmten Anzeige eines bestimmten Beteiligten durch einen bestimmten Wert von “Alter” (sei es “biologisches”, “forensisches”, “radiologisches”, “geologisches” usw. usf.),
    bzw. jegliche Zuordnung t_{\mathfrak A} : \mathcal A \rightarrow \mathbb R eines (reellen) Wertes (als “Ablesewert”, “Koordinate”, “Timestamp” …) jeweils zu einer bestimmten Anzeige (A_J \in \mathcal A) eines bestimmten Beteiligten (A) überhaupt,
    ist sorgfältig von dessen jeweiliger Anzeige an sich zu unterscheiden;

    und entsprechend sorgfältig ist jemandes Alterszuwachs \Delta t_{\mathfrak A} := t_{\mathfrak A}[ \, A_K \, ] - t_{\mathfrak A}[ \, A_J \, ]
    von der entsprechenden Dauer des Betreffenden \tau A[ \, \_J, \_K \, ] zu unterscheiden.

    Erst auf dieser Grundlage lässt sich ja überhaupt von “Alterungs-Raten” (bzw. zunächst -Durchschnittsraten) sprechen, d.h. jeweils vom Verhältniswert
    \frac{\Delta t_{\mathfrak A} := t_{\mathfrak A}[ \, A_K \, ] - t_{\mathfrak A}[ \, A_J \, ]}{\tau A[ \, \_J, \_K \, ]}.

    p.s.
    Philip schrieb (#359, 27. August 2018):
    > Joa Kruzifix[n]oamoal!

    > K := √{(c+v)(c-v)} = γ(1+(v/c)) sollte die Formel heißen.

    Wohl eher: … √{(c+v) / (c-v)} = γ(1+(v/c)).

  397. #397 Frank Wappler
    29. August 2018

    p.s. — Formel-Korrektur zu #396 nach “instant preview”:

    … Durchschnittsrate als Verhältniswert \frac{t_{\mathfrak A}[ \, A_K \, ] - t_{\mathfrak A}[ \, A_J \, ]}{\tau A[ \, \_J, \_K \, ]}.

  398. #398 Alderamin
    29. August 2018

    @Peter

    Du sagst gerade, dass die SRT nichts mit Physik zu tun hat – und das ist korrekt.

    Nur weil etwas nicht unmittelbar auf die 4 Grundkräfte zurückgeht, ist es noch lange nicht unphysikalisch. Meinetwegen kann man auf den Elektromagnetismus verweisen, aus den Maxwellschen Gleichungen folgt eine konstante Lichtgeschwindigkeit unabhängig vom Bezugssystem 1/√(ε0μ0).

    Auf welche der vier Grundkräfte führst Du denn die Hauptsätze der Thermodynamik zurück? Oder ist das auch keine Physik?

    Es ist ein rein mathematisches Konstrukt. Es ist elegant und mathematisch korrekt, wird aber falsch angewendet!

    Ist es nicht, die RT ist experimentell tausendfach belegt. Und darüber zu diskutieren habe ich wirklich keine Lust.

    Er erklärte mir sehr viel über die verschiedenen Störeinflüsse von Lufrdruck, Luftfeuchtigkeit, Strahlung, Temperatur etc, welche minimale Gangunterschiede hervorrufen können. Was sagst Du nun? Willst Du weiterhin etwas anderes behaupten?

    Hat der Mann auch an Hafele-Keating gezweifelt? Kannst Du Größenordnungen nennen?

    Natürlich bleibe ich bei meiner Behauptung. Die Leute, die das GPS gebaut haben, sind doch keine Idioten, wenn sie die Uhren um 38µs pro Tag langsamer laufen lassen – und es funktioniert. Das ist ein systematischer Gangunterschied. Du redest von zufälligen Einflüssen. Die Uhren gehen nicht schneller, wenn gerade ein Sonnensturm vorherrscht. Die Satelliten regeln selbstverständlich ihre Temperatur intern (Hier für Galileo: “The clock’s electronics includes circuitry for the control of the frequency plus the thermal control system to maintain the resonant cavity at the correct temperature. “).Und Luftfeuchtigkeit ist wohl nicht das Problem im All.

    Die RT spielt hierbei keine Rolle! Ebenso steht dort, dass die Uhren in den Satelliten um 45-7=38 µs verstellt werden: auch das ist falsch! Sie werden von 10,23 MHz auf 10,229999995453 MHz gestellt. Das sind genau 45 µs, was allein der ART entspricht!

    Gangabweichung also (10,23-10,229999995453)/10,23 = 4,4477*10-10. Mal 86400 Sekunden / Tag = 38,4*10-6 s / Tag.

    Du behauptest allen Ernstes, die Ingenieure, die solche Satelliten bauen, wüssten das nicht und wären nicht in der Lage, die technischen Herausforderungen einer konstant laufenden Atomuhr in den Griff zu bekommen – und dass der resultierende Fehler eine konstante Abweichung von 38 µs pro Tag produziert, die rein zufällig genau dem völlig inkorrekterweise von SRT und ART in Kombination vorhergesagten Wert entspricht?

    Dann danke ich an dieser Stelle für das Gespräch, dessen Fortführung ich nicht als fruchtbar oder zielführend erachte.

  399. #399 Karl Mistelberger
    29. August 2018

    > #398 Alderamin, 29. August 2018
    > Hat der Mann auch an Hafele-Keating gezweifelt?

    I want to mention one of my current favorite … my head is getting older than my feet, but I am not to worried about it:

    https://youtu.be/dufZysUK0eM?list=PLgxD9DiwxLGrVAQfOmzTDARafMJf47sux&t=824

  400. #400 Karl Mistelberger
    29. August 2018

    Eigentlich sollte es dieses Video sein:

  401. #401 Philip
    Wuppertal
    29. August 2018

    Hallo PS,
    ich kann nicht wirklich zustimmen:

    Dieser Begriff [„Längenkontraktion“] ist zutreffend, weil er mit der Definition der räumlichen Länge übereinstimmt.

    Trotzdem nicht. Da wird ja nichts kontrahiert, sondern der Abstand zweier Σ-gleichzeitiger Ereignisse an den Enden des Körpers ist einfach kürzer als die Eigen-Ausdehnung d des Körpers. In Formeln: Der raumartige Abstand

    Δς = √{Δs² – Δt²} ≡√{Δs’² – Δt²}

    konkretisiert sich dabei zu

    Δς = Δs = √{d² – v²d²/c²} = √{1 – v²/c²}.

    Das ist von vornherein so.

    Der Begriff „Zeitdilatation“ bezieht sich aber bei den meisten, die ihn verwenden … auf ein „langsameres Vergehen von Zeit in der Sphäre des anderen Systems“. Er trifft nicht zu, weil er mit der Definition der zeitlichen Länge nicht übereinstimmt.

    Der trifft genauso viel oder wenig zu wie der der „Längenkontraktion“, denn auch dieser Effekt ist ein Nebeneffekt der Relativität der Gleichzeitigkeit.
    Ich würde es als Zeitprojektion bezeichnen.

  402. #402 Philip
    Wuppertal
    30. August 2018

    Hallo Peter,

    Das sind genau 45 µs, was allein der ART entspricht! Und dies wird nicht gemacht, um die Positionsbestimmung zu korrigieren, sondern um Zeitbestimmung zu ermöglichen. Die SRT hat hier keinen Platz!

    Du kannst nicht die ART gegen die SRT ausspielen, denn die ist in der ART enthalten. In der SRT ist die Raumzeit in einer ganz bestimmten Weise beschaffen, nämlich MINKOWSKIsch bzw. Pseudo-EUKLIDisch.

    Dies folgt zwingend aus der Anwendung des Relativitätsprinzips von GALILEI auf die Elektrodynamik von MAXWELL.

    In der ART ist die Raumzeit eine Pseudo-RIEMANNsche Mannigfaltigkeit, das ist eine Art verallgemeinerte Fläche, die lokal MINKOWSKIsch ist.

  403. #403 Karl-Heinz
    30. August 2018

    @Peter

    Ich verstehe dich nicht.
    Jedes kleine Kind weiß, dass die SRT ein Spezialfall der ART ist. Wie kann dir so ein Schnitzer unterlaufen. Das mit den 45 µs finde ich auch unschön von dir. Für mich stellt sich jetzt die Frage: darf ich dich noch ernst nehmen?
    Was sagst du dazu? 😉

  404. #404 Philip
    30. August 2018

    Hallo Frank,
    ich hoffe, Du hast mich nicht missverstanden.

    > Ob er dann auch biologisch jünger ist, sei dahingestellt.

    Sehr richtig und wichtig!:
    Die Bewertung jeweils einer bestimmten Anzeige eines bestimmten Beteiligten durch einen bestimmten Wert von „Alter“ (sei es „biologisches“, „forensisches“, „radiologisches“, „geologisches“ usw. usf.),

    ist sorgfältig von dessen jeweiliger Anzeige an sich zu unterscheiden;…

    Leider drückst Du Dich nicht besonders leicht verständlich aus, weshalb ich nicht ganz genau weiß, was Du meinst.

    Sozusagen vorsorglich möchte ich in aller Deutlichkeit darauf hinweisen, dass ich mit der Bemerkung über das biologische Alter mitnichten konstatieren will, biologische Uhren würden irgendwie anders durch die Bewegung beeinflusst als mechanische oder elektronische. Das verstieße gegen das Relativitätsprinzip.

    Nein, Bewegung ist relativ, und das impliziert, dass die Bewegung selbst alle Uhren in der gleichen Weise beeinflusst, die man als exakte Uhren bezeichnen kann – was biologische Uhren nicht sind.
    Die Bemerkung über die biologische Alterung bezieht sich allein auf die Auswirkungen von Strahlung und Ähnlichem auf den menschlichen Körper, von der ich annehme, dass sie einen Weltraumreisenden in wesentlich höherem Maße trifft als jemanden, der im Schutz der Erdatmosphäre lebt.

  405. #405 Frank Wappler
    https://scilogs.spektrum.de/menschen-bilder/zum-verhaeltnis-von-glauben-philosophie-und-naturwissenschaft/#comment-36313
    30. August 2018

    Philip schrieb (#401, 29. August 2018):
    > der Abstand zweier Σ-gleichzeitiger Ereignisse an den Enden des Körpers ist einfach kürzer als die Eigen-Ausdehnung d des Körpers.

    Solcher Jargon erlaubt wohl, sich besonders knapp auszudrücken;
    dem möchte ich aber die folgende Formulierung gegenüberstellen, worin der (Messgrößen-)Begriff “gleichzeitig” schlicht im Sinne der Einsteinschen Definition (1916/17) benutzt wird:

    Die Distanz zweier gegenüber einander ruhender Beteiligter voneinander, von denen der eine die Passage eines Endes des Körpers anzeigte, und der andere dazu gleichzeitig die Passage des anderen Körperendes anzeigte, ist (beweisbar) kleiner als die Distanz der beiden Körperenden voneinander.

    (Wobei vorauszusetzen ist, dass auch die beiden Körperenden gegenüber einander ruhten; so dass auch ihnen überhaupt eine bestimmte Distanz voneinander zugeschrieben werden kann.)

    > Der raumartige Abstand [zwischen den beiden beschriebenen Ereignissen …] konkretisiert sich dabei zu […]

    Im Flachen besteht (natürlich) ein direkter (definitiver?) Zusammenhang zwischen dem Raum-Zeit-Intervall-Wert (s^2) eines Paares von Raum-artig voneinander getrennten Ereignissen und dem (ggf. mit Konventions-bedingtem Minuszeichen versehenen Quadrat der) geringsten Distanz zweier Beteiligter/Enden voneinander, von denen jeweils der eine an dem einen Ereignis und der andere an dem anderen Ereignis teilgenommen hatte.

    Ein (geeignet spezifischer und kompatibler) Abstands-Begriff für Paare Raum-artig voneinander getrennten Ereignisse im Allgemeinen (und insbesondere im Krummen) erscheint … auch wünschenswert.
    (Wer die Syngesche “Weltfunktion” untauglich findet, weil dafür Kenntnis von “Raum-artigen Geodäten” voraussetzt wird, kann ja auf “chronometrischen Abstand” ausweichen … ;)

    Zwischen dem s^2-Wert eines Paares von Zeit-artig voneinander getrennten Ereignissen und der (ggf. mit Konventions-bedingtem Minuszeichen versehenen Quadrat der) maximalen Dauer eines Beteiligten, jeweils von dessen Anzeige der Teilnahme in dem einen Ereignis, bis zu dessen Anzeige der Teilnahme im anderen, besteht ja ebenso ein direkter Zusammenhang;
    wobei ein (geeignet spezifischer und kompatibler) Abstands-Begriff für Paare Zeit-artig voneinander getrennten Ereignisse im Allgemeinen (und insbesondere im Krummen) bekanntlich durch die sogenannte “Lorentzsche Distanz” gegeben ist.

    > [… »Der Begriff „Zeitdilatation“« …] trifft genauso viel oder wenig zu wie der der „Längenkontraktion“. […] Ich würde es als Zeitprojektion bezeichnen.

    Nicht ganz unähnlich, aber etwas umfassender (und ein klein wenig eher) wurde (anderswo) schon die Bezeichnung “Einstein-Projektion” vorgeschlagen.

    p.s.
    Habe gerade erst Kommentar #404 gesehen …

  406. #406 Frank Wappler
    31. August 2018

    Philip schrieb (#404, 30. August 2018):
    > […] alle Uhren in der gleichen Weise beeinflusst, die man als exakte Uhren bezeichnen kann […]

    Vielleicht dient es ja unserem (gegnseitigen) Verständnis, erst einmal zu vereinbaren, was mit einer “Uhr” im allgemeinen und zur Diskussion von Geometrie/Kinematik/RT geeigneten Sinne gemeint sein soll;
    um uns anschließend (im Falle des Einverständnisses) mit den zusätzlichen Bedingungen zu beschäftigen, unter denen eine gegebene Uhr als “exakte Uhr” zu bezeichnen wäre (bzw. wie MTW es ausdrückten: a “good” clock).

    Ich halte die folgende allgemeine Definition einer Uhr dafür geeignet:

    – etwas bzw. jemand bestimmtes Beobachtbares und Identifizierbares, nennen wir’s A,

    – das eine (geordnete) Menge \mathcal A einzelner beobachtbarer unterscheidbarer Anzeigen darstellte (Einstein schrieb bekanntlich etwas weniger allgemein von “Stellung[en] des kleinen Zeigers meiner Uhr”),

    – zusammen mit einer bestimmten Zuordnung von (reellen) Ablese- bzw. Alterswerten:
    t_{\mathfrak A} : \mathcal A \rightarrow \mathbb R.

    Meinst Du nicht auch? …

  407. #407 Philip
    Wuppertal
    31. August 2018

    Hallo Frank,
    eines hast Du m.E. noch nicht explizit genannt:

    Der zeitliche Abstand zwischen den unterscheidbaren Anzeigen muss gewissermaßen genormt sein, d.h., ein relativ zu A ruhender Beobachter müsste immer denselben Zeittakt feststellen können.
    Umgekehrt muss z.B. ein relativ zu A ruhender Oszillator (z.B. mechanisch: bekannte Masse an Feder mit bekannter Härte) nach A immer dieselbe Schwingungsdauer haben.

    Ein Wort zur Form: Du verwendest gern LaTeX – Code und eine Menge unterschiedlicher Indizes, richtig? Die Exaktheit in allen Ehren, aber leider geht das manchmal auf Kosten der Lesbarkeit, eine Kritik, die ich mir auch schon oft gefallen lassen musste.
    Ich verwende auch gern LaTeX – Code – wo er einwandfrei funktioniert. Dazu zählt für mich auch, dass ich eine Vorschau oder eine Editierungsmöglichkeit habe (oder beides). Was hier leider nicht der Fall ist. Auch die hier ordentlich aussehenden Formeln werden in den E-Mails grausam entstellt wiedergegeben.

    Ein bisschen habe ich mir LaTeX – Code in gutefrage.net abgewöhnt, weil das da lange gar nicht ging und in Kommentaren bis heute nicht geht.
    So habe ich mir ein relativ breites Repertoire an Sonderzeichen angeeignet, um Formeln trotzdem leserlich schreiben zu können.

    Ich versuche auch nicht zu viele zu “exotische” Zeichen zu verwenden, was man der Exaktheit wegen wohl vor allem als Mathematiker gern tut.
    Und einfach zu formulieren. Das ist manchmal mühsam und dauert länger als eine mathematisch völlig korrekte, für den Leser aber unleserliche Herleitung oder Formel hinschreiben, aber die Mühe lohnt sich.

  408. #408 Frank Wappler
    31. August 2018

    Philip schrieb (#407, 31. August 2018):
    > Hallo Frank, eines hast Du m.E. noch nicht explizit genannt: […]

    … ich wollte meinen vorausgegangenen Kommentar ja nicht überladen, sondern eher behutsam versuchen, nur ausgewählte allererste wesentliche Begriffe zu vereinbaren …

    > Der zeitliche Abstand zwischen den unterscheidbaren Anzeigen

    … Ganz recht; bzw. etwas genauer unter Verwendung der wenigen, offenbar nun schon vereinbarten Begriffe:

    der zeitliche Abstand jeweils eines bestimmten Beteiligten, zwischen jeweils zwei von dessen (identifizierbaren, unterscheidbaren, i.A. verschiedenen) Anzeigen.
    Kurz: die Dauer dieses Beteiligten, jeweils zwischen einem bestimmten Paar seiner Anzeigen; symbolisch:
    \tau A : \mathcal A \times \mathcal A \rightarrow \mathbb R^{\ge 0} \times \tau_{\text{unit}}.

    (Das Symbol \tau_{\text{unit}} soll dabei darauf hinweisen, dass Dauern an sich keine reellen Zahlen sind, sondern “dimensionsbehaftet”;
    aber dass sich Dauern miteinander vergleichen lassen, also reell-wertige Verhältnisse haben.)

    Worauf es (mir) dabei im Zusammenhang mit der Diskussion von “Uhren” und “Alterung” besonders ankommt:

    Die Dauer eines bestimmten Beteiligten zwischen zwei bestimmten Anzeigen
    sorgfältigst
    von der Differenz der beiden Werte (Ablesewerte, “Alter”), die der einen bzw. der anderen Anzeige zugeordnet wurden,
    zu unterscheiden.

    Konkret zum Beispiel: die Dauer As zwischen As beiden Anzeigen A_J und A_K
    sorgfältigst
    von der Differenz der beiden Werte t_{\mathfrak A}[ \, A_K \, ] und t_{\mathfrak A}[ \, A_J \, ]
    zu unterscheiden;
    d.h. symbolisch ausgedrückt: die Dauer \tau A[ \, \_J, \_K \, ]
    vom entsprechenden “Fortgang” bzw. der entsprechenden “Alters”-Differenz t_{\mathfrak A}[ \, A_K \, ] - t_{\mathfrak A}[ \, A_J \, ]
    zu unterscheiden;
    was wiederum ermöglicht, die entsprechende (durchschnittliche) “Gang”-Rate bzw. “Alterungs”-Rate genau dieser Uhr \mathfrak A \equiv (\mathcal A, t_{\mathfrak A})
    als das Verhältnis
    \frac{t_{\mathfrak A}[ \, A_K \, ] - t_{\mathfrak A}[ \, A_J \, ]}{\tau A[ \, \_J, \_K \, ]}
    auszudrücken.

    > d.h., ein relativ zu A ruhender Beobachter müsste immer denselben Zeittakt feststellen können. […]

    Erstens: Wurde etwa unterstellt, dass der/die/das Beteiligte A (“an sic
    h”) kein Beobachter wäre ??
    Ich betrachte im Rahmen der Diskussion von Geometrie/Kinematik/RT die Begriffe
    – “(identifizierbarer) Beteiligter” oder
    – “(bestimmtes) Ende (eines Körpers)” oder
    – “(bestimmter) materieller Punkt” oder
    – “principal identifiable point” oder
    – “(bestimmter) Beobachter”
    jedenfalls als weitgehend synonym.

    Zweitens geht es nun wohl (auch) darum, was schon in Kommentar #404 auftrat; nämlich:
    welche Uhren als “exakte Uhren” bezeichnet werden (sollen), bzw. welche nicht.

    Ich lasse Dir dahingehend (bis auf Weiteres) gern den Vortritt:
    Kannst Du/möchtest Du/würdest Du mit den nun offenbar zur Verfügung stehenden (vereinbarten) Begriffen Dauer, Ablese- bzw. Alterungs-Wert und Gang- bzw. Alterungs-Rate bitte definieren, welche Uhren als “exakte Uhren” bezeichnet werden sollen ?

    p.s.
    > Ein Wort zur Form: […]

    Form folgt Funktion. …

    Jedenfalls strebe ich insbesondere im Einsatz von Symbolen und Formeln weitgehende Eindeutigkeit und Konsistenz an (vergleichbar mit dem Verfassen von kompilierbarem Computer-Code). Darüberhinaus bin ich unbedingt dafür, dass jedem Leser weitgehende [[User:Option]]en zur Auswahl stehen, um die betreffenden Symbole und Formeln dabei so individuell lesbar wie möglich darzustellen. (D.h. für meinen Geschmack, bis auf Weiteres/Besseres: \LaTeX; auch wenn die Vorschau- und Editieroptionen hier leider (noch) bei Weitem nicht so komfortabel sind wie z.B. bei physics.stackexchange).

  409. #409 PS
    31. August 2018

    @Philip #401
    Wir meinen dasselbe:
    Der Begriff “Längenkontraktion” verweist für mich auf ein Ergebnis: die Länge (Zeit der Lichtausbreitung zwischen den beiden räumlich getrennten Ereignissen) ist in S kleiner (“der Abstand … ist einfach kürzer”).
    Sollten ihn manche so verstehen, dass ein “Vorgang der Kontraktion” die Ursache für die Verkürzung sein sollte, wäre er genauso verfehlt wie der Begriff der “Zeitdilatation”, mit dem ein langsameres Vergehen von Zeit als Ursache für eine längere Prozessdauer angesprochen wird.

  410. #410 PS
    31. August 2018

    @Frank W. #408

    Kannst Du/möchtest Du/würdest Du mit den nun offenbar zur Verfügung stehenden (vereinbarten) Begriffen Dauer, Ablese- bzw. Alterungs-Wert und Gang- bzw. Alterungs-Rate bitte definieren, welche Uhren als “exakte Uhren” bezeichnet werden sollen?

    Verzeih, dass ich mich einmische. Man kann in dem Bemühen, immer exakter und noch exakter und noch exakter zu definieren, was man meint, über das Ziel hinausschießen bzw. dieses aus den Augen verlieren. Loriot erzählt die Geschichte, dass eine Polizistin einem Delinquenten auf der Straße immer genauer erklärt, gegen welche Vorschriften er verstoßen hat, und darüber schließlich selbst in polizeiliche Gewahrsam genommen werden muss.

    Es gibt für mich nur eine “exakte Uhr”, die Lichtuhr. Die absolut begrenzte, aber auch überall gleiche Wirkungsausbreitung ist das einzige, woran wir uns festhalten können.

  411. #411 Karl-Heinz
    31. August 2018

    @PS

    War es nicht eine Politesse, die dann von der Polizei weggebracht wurde? 😉

  412. #412 Philip
    Wuppertal
    1. September 2018

    Hallo Karl-Heinz,

    Eigenzeit ist die in einem mit dem mitbewegten Bezugssystem ablaufende Zeit.

    Würde ich anders formulieren, denn das Bezugssystem gilt definitionsgemäß als ruhend – schließlich ist es das Koordinatensystem, auf das Orte und Geschwindigkeiten etc. bezogen werden.
    Wenn in der räumlichen Umgebung eines Punktes P ein Vorgang stattfindet, so ist die Eigenzeit

    Δτ = ∫_[begin]^{end} dτ

    dieses Vorgangs die von einer mitgeführten Uhr gemessene Dauer und damit (chronogeometrisch ausgedrückt) die absolute Länge des Weltlinienabschnittes, der ihn repräsentiert.

  413. #413 Philip
    1. September 2018

    Hallo Karl Mistelberger,

    Ihre/Deine Aufstellung aus #371 würde ich gern modifizieren. Ich nehme an, dass Du mit…

    Vorläufer der Speziellen Relativitätstheorie
    Bis zum Jahr 1905:

    …die LÄT meinst, die bereits die LORENTZ-Transformation verwendet.

    • Relativitätsprinzip („Gleichberechtigung aller
    Inertialsysteme“) -> nur eine Fiktion!

    „Illusion“ würde besser passen. Man hatte ja schon 1881/87 keine Abweichung vom Relativitätsprinzip feststellen können.

    • Zeitdehnung und Längenkontraktion, Lorentz – Transformation, wahre und scheinbare Längen und Zeiten -> nur Fiktionen

    Genau die gerade nicht. Die Verkürzung musste LORENTZ als real ansehen. So abwegig ist die Idee auch nicht, haben doch bewegte geladene Körper auch transversalere Felder und zudem Magnetfelder. Auch die Verlangsamung von Vorgängen durch Bewegung relativ zum Äther müssen als real angenommen worden sein. Allerdings unterschieden LORENTZ und auch POINCARÉ noch die „Ortszeit“ von der „richtigen Zeit“.

    1905: Spezielle Relativitätstheorie

    • Auch was „gleichzeitig“ ist, hängt vom Inertialsystem ab.

    Hundertpro.

    • Zeitdilatation
    • Längenkontraktion

    Diese Effekte werden immer noch so genannt, aber diese Termini sind gerade in einer Theorie ohne bevorzugtes Bezugssystem besonders deplaziert.
    Die „Zeitdilatation“ sollte lieber „Zeitprojektion“ heißen, und die „Längenkontraktion“ eines bewegten Körpers ist eigentlich ein Schrägschnitt durch dessen „Weltwurst“. Auch eine Salami verändert ja nicht wirklich ihre Breite, wenn man einen Messschieber schief anlegt.

    Es ist übrigens nicht so, dass die Relativitätstheorie die Physik absoluter Größen beraubt bzw. diese relativiert hätte. Zeitspannen und Distanzen sind nur nicht mehr für sich genommen absolut. Eine Kombination davon ist es.

  414. #414 Karl-Heinz
    1. September 2018

    @Philip

    “Eigenzeit ist die in einem mit dem mitbewegten Bezugssystem ablaufende Zeit.“

    Na ja, so falsch ist die Aussage nicht, wie du behauptest. Nehmen wir zum Beispiel ein Schwerion, welches im Speicherring seine Runden dreht. Nehmen wir weiters an, dass das Schwerion eine Uhr mitführe. Bei jeder Umrundung wird die Zeit gemessen. Die Zeitdifferenz, die ich messe ist die Dauer einer Umrundung aus meiner Sicht. Die Zeitdifferenz, die das Schwerion mit seiner Uhr misst, ist seine Eigenzeit für eine Umrundung. Will ich aber berechnen, welche Eigenzeit für das Teilchen vergangen ist, so hilft mir das in meiner Überlegung, dass das Teilchen jetzt eine Uhr mit führt. Ich könnte jetzt die Länge seiner Weltenlinie berechnen und durch c dividieren um zu seiner Eigenzeit zu kommen und unter Umständen nicht ganz verstehen, was ich da mache, oder alternativ wie folgt vorgehen, was im Prinzip natürlich das gleiche ist.
    Man mache ein Bezugssystem an dem Teilchen fest. Das Teilche wechselt wohl andauernd sein Bezugssystem, aber halb so schlimm, ich summiere einfach die berechneten Zeitabschnitte infinitesimal auf, die für das Teilchen vergehen und komme so zu seiner Eigenzeit. Ich hoffe, es ist nicht zu konfus geschrieben, so das jeder folgen kann, worauf ich hinaus will.

  415. #415 Philip
    1. September 2018

    Hallo Karl-Heinz,
    ich habe gar nicht behauptet, die Aussage sei falsch, sondern schlecht formuliert.
    Wenn ich darauf hingewiesen werde, dass mir eine Formulierung misslungen sei, und sehe, dass das stimmt, sehe ich es nicht als Frage der Ehre an, meine ursprüngliche Formulierung beizubehalten und zu verteidigen.

    Bei Deiner ursprünglichen Formulierung hast Du jedoch nicht von einer Uhr, sondern von etwas Abstraktem geschrieben, nämlich einem Bezugssystem.
    Besser wäre Koordinatensystem gewesen, denn das ist allgemeiner. Beschreibt man ein Koordinatensystem allerdings als bewegt, behandelt man es nun gerade nicht als Bezugssystem.

    Ohnehin tue ich mich schwer damit, von einem bewegten Koordinatensystem zu reden, denn wenn ich von einem solchen schreibe, gehört die Zeit dazu – ein Koordinatensystem kartographiert die Raumzeit.
    3D- Koordinatensysteme zu verwenden, die sich mit der Zeit irgendwie entwickeln, halte ich für sinnfrei, weil es die Zeit als separate Größe nicht gibt. Gleichzeitigkeit ist ja relativ.

    Will ich aber berechnen, welche Eigenzeit für das Teilchen vergangen ist, so hilft mir das in meiner Überlegung, dass das Teilchen jetzt eine Uhr mit führt.

    Nichts anderes habe ich geschrieben, wenn auch nicht konkret auf ein Schwerion bezogen.

    Man mache ein Bezugssystem an dem Teilchen fest.

    Wenn man das für mehr als einen „infinitesimalen“ Zeitabschnitt tut, hat man ein Nichtinertialsystem als Bezugssystem und betreibt somit ART. Kann man machen, ist aber vermutlich unnötiger Aufwand.

    Das Teilchen wechselt wohl andauernd sein Bezugssystem,…

    Es ist fast schon konventionell geworden, das Wort „Bezugssystem“ synonym zu dem Wort „Ruhesystem (eines Körpers)“ zu verwenden. Letzteres ist ein Koordinatensystem, in dem der Körper zumindest momentan ruht.
    Das Bezugssystem ist hingegen ein Koordinatensystem, dessen Anker (z.B. ein Körper) man als ruhend betrachtet. Letzteres ist frei wählbar. Ein Beobachter kann sich als ruhend betrachten, aber das muss er nicht. Wenn ich mit dem Fahrrad irgendwo entlang fahre, werde ich auch normalerweise immer die Erde und nicht ein momentanes Ruhesystem von mir als Bezugssystem verwenden.
    Ein Bezugssystemwechsel ist eine Umrechnung, also eine LORENTZ-Transformation, und damit eine Uminterpretation, weil etwas, das man zuvor als ruhend interpretiert hat, nunmehr als mit einer Geschwindigkeit –v› bewegt interpretiert wird.

  416. #416 Karl-Heinz
    1. September 2018

    @Philip

    Wenn wir schon dabei sind.
    Man verwendet nicht den Begriff „exakte Uhr“ sondern den Begriff „ideale Uhr“.
    Eine ideale Uhr wird definiert als eine Uhr, die von Beschleunigungen vollständig unbeeinflusst bleibt; das heißt eine Uhr, deren momentane Rate allein von ihrer momentanen Geschwindigkeit nach abhängt. Dann kann man eine Größe einführen, nämlich die Eigenzeit τ, die von einer beliebig bewegten, idealen Uhr angezeigt wird.
    Siehste jetzt sind wir schon ein Stückchen weiter. 😉

  417. #417 Karl-Heinz
    1. September 2018

    @Philip

    Wenn ich darauf hingewiesen werde, dass mir eine Formulierung misslungen sei, und sehe, dass das stimmt, sehe ich es nicht als Frage der Ehre an, meine ursprüngliche Formulierung beizubehalten und zu verteidigen.

    Na ja, hätte ich nicht’s gesagt und geschrieben, dass du vollkommen Recht hast, dann wäre es zu keiner Diskussion gekommen. Ausserdem finde ich es lustig, wenn man seinen Standpunkt auch dann verteidigt, falls dieser nicht ganz korrekt ist. Durch die dadurch entstehende Diskussion bleibt man dadurch im Gedächtnis der anderen eher haften. Wenn ich merke, dass ich total auf dem Holzweg bin, da mache ich natürlich einen Rückzieher.
    Du könntest aber natürlich auch denken, der will um jeden Preis Kompetenz zeigen. 😉

    In der Praxis rechne ich die Beispiele durch. Dann sehe ich zumeist, wie was gemeint ist.
    Es gibt natürlich auch Ausnahmen. Frank Wappler ist so einer. Er bringt mich mit seinen Ausführungen und Erläuterungen zum Verzweifeln. 😉

  418. #418 Frank Wappler
    3. September 2018

    PS schrieb (#410, 31. August 2018):
    > […] Es gibt für mich nur eine “exakte Uhr”, die Lichtuhr.

    Die Lichtuhr — sehr wohl!:

    – Wünschen Sie “die Lichtuhr” lediglich im Sinne zweier unterscheidbarer und voneinander getrennter Enden, die einander durchwegs beobachteten ?

    – Oder wünschen Sie zusätzlich die Identifikation eines weiteren Beteiligten als “Mitte zwischen” zwei solchen Enden, der durchwegs koinzidente Pings bzgl. der beiden Uhren-Enden fand ?

    – Oder wünschen Sie zusätzlich eine “affine Parametrisierung” der Anzeigen dieser drei Beteiligten; etwa im Sinne von Schild’s ladder ?

    – Oder wünschen Sie ein System mehrerer solcher (parameterfreier, oder z.B. Schildscher) Leitern ?

    – Und wünschen Sie insbesondere, dass die die gegenseitigen Beobachtungen (Ping-Koinzidenzen) der Mitglieder eines solchen Leiter-Systems ein flaches Gitter bilden; etwa ein tetrahedral-oktahedralen Wabengitter bzw. eine Sierpinski-Pyramide ? …

    – Oder wünschen Sie die Identifizierung der Uhren-Enden unter Voraussetzung von “freien Partikeln” nach Marzke-Wheeler ? …

    (Der genaue Ursprung des Butler-Sketches, an den ich die obige Präsentation eines Überangebots von Möglichkeiten angelehnt habe, ließe sich sicherlich recherchieren …)

  419. #419 Karl-Heinz
    3. September 2018

    @Frank Wappler

    exakte Uhr … = Blödsinn
    Warum?
    Diese Uhr muss sich nur in einem Gravitationsfeld befinden und schon ist ihre Exaktheit dahin.
    Es gibt nur eine “ideale Uhr”.

  420. #420 Frank Wappler
    3. September 2018

    Karl-Heinz (#416, 1. September 2018):
    > […] eine Uhr, deren momentane Rate

    … bloß gut, dass der Begriff der “(durchschnittlichen Gang- bzw. Alterungs-)Rate einer Uhr” schon aus #396 (bzw. korrigiert #397) und nochmals #408 zur Verfügung steht! …

    > allein von ihrer momentanen Geschwindigkeit nach abhängt.

    Ihrer momentanen Geschwindigkeit” — gegenüber wem ?? …

    Im Übrigen halte ich (auch) jene Uhren für besondere erwähnens- und beachtenswert, deren Durchschnittsraten jeweils konstant sind; d.h. jede Uhr \mathfrak A \equiv (\mathcal A, t_{\mathfrak A}), deren (geordneter) Anzeigenmenge \mathcal A reelle Ablesewerte t_{\mathfrak A} : \mathcal A rightarrow \mathbb R insbesondere so zugeordnet wären,
    dass für je drei verschiedene ihrer Anzeigen, A_J, A_K, A_P \in \mathcal A erfüllt wäre:

    t[ \, A_P \, ] := t[ \, A_J \, ] + (t[ \, A_K \, ] - t[ \, A_J \, ]) \, \left( \frac{\tau A[ \, \_J, \_P \, ]}{\tau A[ \, \_J, \_K \, ]} \right).

    Was (offensichtlich) die Messgröße “Dauer, \tau” (jeweils eines bestimmten Beteiligten, jeweils zwischen zwei seiner Anzeigen) voraussetzt; bzw. zumindest jeweils das Verhältnis zweier solcher Dauern, als Messwert.

  421. #421 Karl-Heinz
    3. September 2018

    @Frank Wappler

    “Ihrer momentanen Geschwindigkeit” — gegenüber wem ?? …

    Wenn ich von eine bewegten Uhr spreche, die natürlich auch unbewegt sein kann, was glaubst worauf sich dann ihre Geschwindigkeit beziehen wird?
    Bin gespannt, ob du das durchschaust.

  422. #422 Frank Wappler
    3. September 2018

    Karl-Heinz (#416, 1. September 2018):
    > […] eine Uhr, deren momentane Rate

    … bloß gut, dass der Begriff der “(durchschnittlichen Gang- bzw. Alterungs-)Rate einer Uhr” schon aus #396 (bzw. korrigiert #397) und nochmals #408 zur Verfügung steht! …

    > allein von ihrer momentanen Geschwindigkeit nach abhängt.

    Ihrer momentanen Geschwindigkeit” — gegenüber wem ?? …

    Im Übrigen halte ich (auch) jene Uhren für besondere erwähnens- und beachtenswert, deren Durchschnittsraten jeweils konstant sind; d.h. jede Uhr \mathfrak A \equiv (\mathcal A, t_{\mathfrak A}), deren (geordneter) Anzeigenmenge \mathcal A reelle Ablesewerte t_{\mathfrak A} : \mathcal A \rightarrow \mathbb R insbesondere so zugeordnet wären,
    dass für je drei verschiedene ihrer Anzeigen, A_J, A_K, A_P \in \mathcal A erfüllt wäre:

    t_{\mathfrak A}[ \, A_P \, ] := t_{\mathfrak A}[ \, A_J \, ] + (t_{\mathfrak A}[ \, A_K \, ] - t_{\mathfrak A}[ \, A_J \, ]) \, \left( \frac{\tau A[ \, \_J, \_P \, ]}{\tau A[ \, \_J, \_K \, ]} \right).

    Was (offensichtlich) die Messgröße “Dauer, \tau” (jeweils eines bestimmten Beteiligten, jeweils zwischen zwei seiner Anzeigen) voraussetzt; bzw. zumindest jeweils das Verhältnis zweier solcher Dauern, als Messwert.

  423. #423 Frank Wappler
    3. September 2018

    Karl-Heinz schrieb (#421, 3. September 2018):
    > Wenn ich von eine[r] bewegten Uhr spreche, die natürlich auch unbewegt sein kann […]

    … dann passt das meines Erachtens bestens zu den vorausgegangenen Karl-Heinzschen Kommentaren auf dieser ScienceBlogs-Seite;
    und erspart mir jeden weiteren Kommentar dazu.

    p.s.
    Karl-Heinz schrieb (#419, 3. September 2018):
    > exakte Uhr … = Blödsinn

    Die Bezeichnung “exakte Uhr” wurde auf dieser ScienceBlogs-Seite zuerst von Philip (#404, 30. August 2018) benutzt;
    woraufhin ich versucht habe, ihn zunächst (#406) zu ermutigen und anschließend (#408) direkt aufzufordern, eine nachvollziehbare Definition dieser Bezeichnung abzuliefern.
    Bis es dazu kommt (falls überhaupt), kann und will ich weder bestätigen noch bestreiten, dass Philip “exakte Uhr” im Sinne von “Blödsinn” gemeint hat.

    Dass Kommentator PS (#410) diese Bezeichnung offenbar ohne Weiteres/Nachfragen selbst benutzte — kann ich auch nicht verantworten.

    Im Übrigen entsprechen die in Kommentar #422 (als Korrektur von #420) beschriebenen Uhren nach meinem Verständnis denen, die MTW als “good clocks” bezeichnen (vgl. #406).

  424. #424 PS
    3. September 2018

    @Frank W. #418
    Es geht hier um das Zwillingsparadoxon auf der Grundlage der SRT. Darauf bezieht sich die Lichtuhr als exakte Uhr. Es gibt bei ihr nur einen Beobachter, dessen Lichtstrahl zu ihm zurück reflektiert wird. Der zeitliche Abstand der Ereignisse des Absenders und der Rückkehr des Lichtstrahls ist in allen Bezugssystemen das doppelte des zeitlichen Abstands zum Reflexionsereignis (“Halbzeitreflexion”). Das ist die einzigartige Eigenschaft dieser Uhr.

    Wodurch in der ART eine exakte Uhr repräsentiert werden könnte, weiß ich nicht, ich vermute aber, dass auch dort die Wirkungsausbreitung beschränkt und in allen Bezugssystemen gleich ist.

  425. #425 Frank Wappler
    4. September 2018

    PS schrieb (#424, 3. September 2018):
    > […] die Lichtuhr als exakte Uhr. […] Der zeitliche Abstand der Ereignisse des Absenders und der Rückkehr des Lichtstrahls ist in allen Bezugssystemen das doppelte des zeitlichen Abstands zum Reflexionsereignis (“Halbzeitreflexion”). Das ist die einzigartige Eigenschaft dieser Uhr.

    Na schön — inwiefern diese Beschreibung dem entspricht, was Philip oben (#404, 30. August 2018) als “exakte Uhren” bezeichnet hat, müsst Ihr wohl unter Euch ausmachen.

    Immerhin besteht ja ein gewisser Zusammenhang zwischen “zeitlichem Abstand” zwischen je zwei bestimmten (geeigneten, d.h. zueinander “zeitartigen”) Ereignissen, nämlich (im Flachen) dem entsprechenden Raum-Zeit-Interval-Wert “s^2“, und der Dauer “\tau” (jeweils eines bestimmten Beteiligten, der an beiden Ereignissen teilgenommen hatte, von dessen Anzeige der Teilnahme in dem einen Ereignis, bis zu dessen Anzeige der Teilnahme im anderen); vgl. “p.s.” und “p.p.s.” unten.

    Um aber überhaupt von einer “Uhr” sprechen zu können, verlange ich allerdings außerdem eine bestimmte Zuordnung von Ablesewerten “$latex t” (die in Deiner Beschreibung offenbar nicht ausdrücklich vorkommt).

    > Es gibt bei ihr nur einen Beobachter, dessen Lichtstrahl zu ihm zurück reflektiert wird.

    Im Zusammenhang mit “Reflexion” (und auch dem, was ich oben, #418, als verschiedene mögliche Auffassungen von “Lichtuhren” unterbreitet habe), ist allerdings nicht nur von einem bestimmten Ende/Beteiligten/Beobachter die Rede, sondern von zwei unterscheidbaren und voneinander getrennten Enden/Beteiligten/Beobachtern; bei Einstein z.B. von sowohl A als auch B.

    p.s.
    Skizze der Beziehung zwischen Verhältnissen von Raum-Zeit-Intervallen und dem Verhältnis zweier Dauern eines bestimmten Beteiligten, A:

    \left( \frac{\tau A[ \, \_J, \_P \, ]}{\tau A[ \, \_J, \_Q \, ]} \right) := \underset{ \varepsilon_{AJ} \equiv \varepsilon_{A \Upsilon \_ (0)} ... \ll \varepsilon_{A \Upsilon \_ (q)} \ll \varepsilon_{A \Upsilon \_ (q+1)} ... \ll \varepsilon_{AQ} \equiv \varepsilon_{A \Upsilon \_ (n \_ Q)} }{\hbox{Sup}} \! \Big[

    \, \qquad \frac{ \underset{ \varepsilon_{AJ} \equiv \varepsilon_{A \Xi \_ (0)} ... \ll \varepsilon_{A \Xi \_ (p)} \ll \varepsilon_{A \Xi \_ (p+1)} ... \ll \varepsilon_{AP} \equiv \varepsilon_{A \Xi \_ (n \_ P)} }{\hbox{Inf}} \! \Large[ \, \sum_{p = 0}^{n \_ P} \sqrt{\left(\frac{s^2[ \, \varepsilon_{A \Xi \_ (p)}, \varepsilon_{A \Xi \_ (p+1)} \, ]}{s^2[ \, \varepsilon_{AJ}, \varepsilon_{AP} \, ]} \right)} \, \Large] }{ \sum_{q = 0}^{n \_ Q} \sqrt{ \left( \frac{s^2[ \, \varepsilon_{A \Upsilon \_ (q)}, \varepsilon_{A \Upsilon \_ (q+1)} \, ]}{s^2[ \, \varepsilon_{AJ}, \varepsilon_{AQ} \, ]} \right)} } \, \Big].

    p.p.s.
    Verallgemeinerung der oben gezeigten Beziehung “im Krummen”, d.h. zwischen Verhältnissen Lorentzscher Distanzen und dem Verhältnis zweier Dauern eines bestimmten Beteiligten, A:

    \left( \frac{\tau A[ \, \_J, \_P \, ]}{\tau A[ \, \_J, \_Q \, ]} \right) := \underset{ \varepsilon_{AJ} \equiv \varepsilon_{A \Upsilon \_ (0)} ... \ll \varepsilon_{A \Upsilon \_ (q)} \ll \varepsilon_{A \Upsilon \_ (q+1)} ... \ll \varepsilon_{AQ} \equiv \varepsilon_{A \Upsilon \_ (n \_ Q)} }{\hbox{Sup}} \! \Big[

    \, \qquad \frac{ \underset{ \varepsilon_{AJ} \equiv \varepsilon_{A \Xi \_ (0)} ... \ll \varepsilon_{A \Xi \_ (p)} \ll \varepsilon_{A \Xi \_ (p+1)} ... \ll \varepsilon_{AP} \equiv \varepsilon_{A \Xi \_ (n \_ P)} }{\hbox{Inf}} \! \Large[ \, \sum_{p = 0}^{n \_ P} \left( \frac{\ell[ \, \varepsilon_{A \Xi \_ (p)}, \varepsilon_{A \Xi \_ (p+1)} \, ]}{\ell[ \, \varepsilon_{AJ}, \varepsilon_{AP} \, ]} \right) \, \Large] }{ \sum_{q = 0}^{n \_ Q} \left(\frac{\ell[ \, \varepsilon_{A \Upsilon \_ (q)}, \varepsilon_{A \Upsilon \_ (q+1)} \, ]}{\ell[ \, \varepsilon_{AJ}, \varepsilon_{AQ} \, ]} \right) } \, \Big].

  426. #426 Frank Wappler
    4. September 2018

    In Korrektur des vorausgegangenen Kommentars #425:

    PS schrieb (#424, 3. September 2018):
    > […] die Lichtuhr als exakte Uhr. […] Der zeitliche Abstand der Ereignisse des Absenders und der Rückkehr des Lichtstrahls ist in allen Bezugssystemen das doppelte des zeitlichen Abstands zum Reflexionsereignis (“Halbzeitreflexion”). Das ist die einzigartige Eigenschaft dieser Uhr.

    Na schön — inwiefern diese Beschreibung dem entspricht, was Philip oben (#404, 30. August 2018) als “exakte Uhren” bezeichnet hat, müsst Ihr wohl unter Euch ausmachen.

    Immerhin besteht ja ein gewisser Zusammenhang zwischen “zeitlichem Abstand” zwischen je zwei bestimmten (geeigneten, d.h. zueinander “zeitartigen”) Ereignissen, nämlich (im Flachen) dem entsprechenden Raum-Zeit-Interval-Wert “s^2“, und der Dauer “\tau” (jeweils eines bestimmten Beteiligten, der an beiden Ereignissen teilgenommen hatte, von dessen Anzeige der Teilnahme in dem einen Ereignis, bis zu dessen Anzeige der Teilnahme im anderen); vgl. “p.s.” und “p.p.s.” unten.

    Um aber überhaupt von einer “Uhr” sprechen zu können, verlange ich allerdings außerdem eine bestimmte Zuordnung von Ablesewerten “t” (die in Deiner Beschreibung offenbar nicht ausdrücklich vorkommt).

    > Es gibt bei ihr nur einen Beobachter, dessen Lichtstrahl zu ihm zurück reflektiert wird.

    Im Zusammenhang mit “Reflexion” (und auch dem, was ich oben, #418, als verschiedene mögliche Auffassungen von “Lichtuhren” unterbreitet habe), ist allerdings nicht nur von einem bestimmten Ende/Beteiligten/Beobachter die Rede, sondern von zwei unterscheidbaren und voneinander getrennten Enden/Beteiligten/Beobachtern; bei Einstein z.B. von sowohl A als auch B.

    p.s.
    Skizze der Beziehung zwischen Verhältnissen von Raum-Zeit-Intervallen und dem Verhältnis zweier Dauern eines bestimmten Beteiligten, A:

    \left( \frac{\tau A[ \, \_J, \_P \, ]}{\tau A[ \, \_J, \_Q \, ]} \right) := \underset{ \varepsilon_{AJ} \equiv \varepsilon_{A \Upsilon \_ (0)} ... \ll \varepsilon_{A \Upsilon \_ (q)} \ll \varepsilon_{A \Upsilon \_ (q+1)} ... \ll \varepsilon_{AQ} \equiv \varepsilon_{A \Upsilon \_ (n \_ Q)} }{\hbox{Sup}} \! \Big[

    \, \qquad \frac{ \underset{ \varepsilon_{AJ} \equiv \varepsilon_{A \Xi \_ (0)} ... \ll \varepsilon_{A \Xi \_ (p)} \ll \varepsilon_{A \Xi \_ (p+1)} ... \ll \varepsilon_{AP} \equiv \varepsilon_{A \Xi \_ (n \_ P)} }{\hbox{Inf}} \! \Large[ \, \sum_{p = 0}^{n \_ P} \sqrt{\left(\frac{s^2[ \, \varepsilon_{A \Xi \_ (p)}, \varepsilon_{A \Xi \_ (p+1)} \, ]}{s^2[ \, \varepsilon_{AJ}, \varepsilon_{AP} \, ]} \right)} \, \Large] }{ \sum_{q = 0}^{n \_ Q} \sqrt{ \left( \frac{s^2[ \, \varepsilon_{A \Upsilon \_ (q)}, \varepsilon_{A \Upsilon \_ (q+1)} \, ]}{s^2[ \, \varepsilon_{AJ}, \varepsilon_{AQ} \, ]} \right)} } \, \Big].

    p.p.s.
    Verallgemeinerung der oben gezeigten Beziehung “im Krummen”, d.h. zwischen Verhältnissen Lorentzscher Distanzen und dem Verhältnis zweier Dauern eines bestimmten Beteiligten, A:

    \left( \frac{\tau A[ \, \_J, \_P \, ]}{\tau A[ \, \_J, \_Q \, ]} \right) := \underset{ \varepsilon_{AJ} \equiv \varepsilon_{A \Upsilon \_ (0)} ... \ll \varepsilon_{A \Upsilon \_ (q)} \ll \varepsilon_{A \Upsilon \_ (q+1)} ... \ll \varepsilon_{AQ} \equiv \varepsilon_{A \Upsilon \_ (n \_ Q)} }{\hbox{Sup}} \! \Big[

    \, \qquad \frac{ \underset{ \varepsilon_{AJ} \equiv \varepsilon_{A \Xi \_ (0)} ... \ll \varepsilon_{A \Xi \_ (p)} \ll \varepsilon_{A \Xi \_ (p+1)} ... \ll \varepsilon_{AP} \equiv \varepsilon_{A \Xi \_ (n \_ P)} }{\hbox{Inf}} \! \Large[ \, \sum_{p = 0}^{n \_ P} \left( \frac{\ell[ \, \varepsilon_{A \Xi \_ (p)}, \varepsilon_{A \Xi \_ (p+1)} \, ]}{\ell[ \, \varepsilon_{AJ}, \varepsilon_{AP} \, ]} \right) \, \Large] }{ \sum_{q = 0}^{n \_ Q} \left(\frac{\ell[ \, \varepsilon_{A \Upsilon \_ (q)}, \varepsilon_{A \Upsilon \_ (q+1)} \, ]}{\ell[ \, \varepsilon_{AJ}, \varepsilon_{AQ} \, ]} \right) } \, \Big].

  427. #427 Frank Wappler
    4. September 2018

    In Korrektur des vorausgegangenen Kommentars #426:

    … p.s.
    Skizze der Beziehung zwischen Verhältnissen von Raum-Zeit-Intervallen und dem Verhältnis zweier Dauern eines bestimmten Beteiligten, A:

    \left( \frac{\tau A[ \, \_J, \_P \, ]}{\tau A[ \, \_J, \_Q \, ]} \right) := \sqrt{\left(\frac{s^2[ \, \varepsilon_{AJ}, \varepsilon_{AP} \, ]}{s^2[ \, \varepsilon_{AJ}, \varepsilon_{AQ} \, ]} \right)} \times

    \Bigg( \, \underset{ \varepsilon_{AJ} \equiv \varepsilon_{A \Upsilon \_ (0)} ... \ll \varepsilon_{A \Upsilon \_ (q)} \ll \varepsilon_{A \Upsilon \_ (q+1)} ... \ll \varepsilon_{AQ} \equiv \varepsilon_{A \Upsilon \_ (n \_ Q)} }{\hbox{Sup}^{(n \_ Q \in \mathbb N)}} \! \bigg[

    \, \qquad \frac{ \underset{ \varepsilon_{AJ} \equiv \varepsilon_{A \Xi \_ (0)} ... \ll \varepsilon_{A \Xi \_ (p)} \ll \varepsilon_{A \Xi \_ (p+1)} ... \ll \varepsilon_{AP} \equiv \varepsilon_{A \Xi \_ (n \_ P)} }{\hbox{Inf}^{(n \_ P \in \mathbb N)}} \! \Big[ \, \sum_{p = 0}^{(n \_ P - 1)} \sqrt{\left(\frac{s^2[ \, \varepsilon_{A \Xi \_ (p)}, \varepsilon_{A \Xi \_ (p+1)} \, ]}{s^2[ \, \varepsilon_{AJ}, \varepsilon_{AP} \, ]} \right)} \, \Big] }{ \sum_{q = 0}^{(n \_ Q - 1)} \sqrt{ \left( \frac{s^2[ \, \varepsilon_{A \Upsilon \_ (q)}, \varepsilon_{A \Upsilon \_ (q+1)} \, ]}{s^2[ \, \varepsilon_{AJ}, \varepsilon_{AQ} \, ]} \right)} } \, \bigg] \, \Bigg).

    p.p.s.
    Verallgemeinerung der oben gezeigten Beziehung “im Krummen”, d.h. zwischen Verhältnissen Lorentzscher Distanzen und dem Verhältnis zweier Dauern eines bestimmten Beteiligten, A:

    \left( \frac{\tau A[ \, \_J, \_P \, ]}{\tau A[ \, \_J, \_Q \, ]} \right) := \left(\frac{\ell[ \, \varepsilon_{AJ}, \varepsilon_{AP} \, ]}{\ell[ \, \varepsilon_{AJ}, \varepsilon_{AQ} \, ]} \right) \times

    \Bigg( \, \underset{ \varepsilon_{AJ} \equiv \varepsilon_{A \Upsilon \_ (0)} ... \ll \varepsilon_{A \Upsilon \_ (q)} \ll \varepsilon_{A \Upsilon \_ (q+1)} ... \ll \varepsilon_{AQ} \equiv \varepsilon_{A \Upsilon \_ (n \_ Q)} }{\hbox{Sup}^{(n \_ Q \in \mathbb N)}} \! \bigg[

    \, \qquad \frac{ \underset{ \varepsilon_{AJ} \equiv \varepsilon_{A \Xi \_ (0)} ... \ll \varepsilon_{A \Xi \_ (p)} \ll \varepsilon_{A \Xi \_ (p+1)} ... \ll \varepsilon_{AP} \equiv \varepsilon_{A \Xi \_ (n \_ P)} }{\hbox{Inf}^{(n \_ P \in \mathbb N)}} \! \Big[ \, \sum_{p = 0}^{(n \_ P - 1)} \left( \frac{\ell[ \, \varepsilon_{A \Xi \_ (p)}, \varepsilon_{A \Xi \_ (p+1)} \, ]}{\ell[ \, \varepsilon_{AJ}, \varepsilon_{AP} \, ]} \right) \, \Big] }{ \sum_{q = 0}^{(n \_ Q - 1)} \left(\frac{\ell[ \, \varepsilon_{A \Upsilon \_ (q)}, \varepsilon_{A \Upsilon \_ (q+1)} \, ]}{\ell[ \, \varepsilon_{AJ}, \varepsilon_{AQ} \, ]} \right) } \, \bigg] \, \Bigg).

  428. #428 PS
    4. September 2018

    @Frank W.

    Ich vermag Deinen mathematisch-logischen Ausführungen – aus eigenem Unvermögen – nicht zu folgen und weiß auch nicht, was Du mit ihnen in Beziehung auf die Lichtuhr unter Beweis stellen willst.
    Ich kann mich nur darauf zurückziehen, dass es für das Ablesen der Lichtuhr nur eines Beobachters bedarf. Der Ablesewert ist das “ping” bei der Rückkehr des Lichtstrahls bzw. bei mehreren “pings” deren Zahl. Ich habe keine Ahnung, was A und B in deinem Modell anstellen müssen, um eine Lichtuhr korrekt abzulesen, verwirren die sich nicht gegenseitig?

  429. #429 Karl-Heinz
    4. September 2018

    @Frank W.
    Na Frank
    Erkläre uns das mal mit einfachen Worten, was du da eigentlich machst. Denk daran, dass wir diejenigen sind, die es verstehen müssen.

  430. #430 Frank Wappler
    5. September 2018

    PS schrieb (#428, 4. September 2018):
    > […] dass es für das Ablesen der Lichtuhr nur eines Beobachters bedarf.

    Dann sind wir uns wohl doch einig, dass eine Lichtuhr (sehr allgemein) aus zwei unterscheidbaren, voneinander getrennten Enden bestehen soll.

    Bedarf es etwa noch eines (wahrnehmenden, erkennenden, ablesenden, sich erinnernden, zählenden) Beobachters darüberhinaus ?? — Wohl kaum, denn im Rahmen von Gedankenexperimenten sind solche beobachterischen Fähigkeiten allen Beteiligten zugestanden.

    Eines gebe aber ausdrücklich zu (und habe das nie bestritten, sondern bei Gelegenheit eher versucht hervorzuheben):
    Die relevanten Wahrnehmungen werden jeweils von einem der Beteiligten/Enden gemacht;
    die entsprechenden Anzeigen “gehören” jeweils einem bzw. “konstituieren” jeweils einen der Beteiligten/Enden;
    es lassen sich aufeinanderfolgende Ping-Anzeigen jeweils eines der beiden Enden (bzgl. des anderen) zählen.

    (Es besteht allerdings zwangsläufig ein enger Zusammenhang zur “entsprechenden” Anzahl aufeinanderfolgender Ping-Anzeigen des anderen Endes, bzgl. des erstgenannten.)

    > Der Ablesewert ist das “ping” bei der Rückkehr des Lichtstrahls bzw. bei mehreren “pings” deren Zahl.

    Wesentlich ist aber die Identifikation, das Erkennen, wessen Reflexions-Anzeige jeweils wahrgenommen wurde. Jeder bestimmte einzelne Beteiligte (z.B. “A”) kann ja zumindest im Prinzip nicht nur Reflexions-Anzeigen nur eines bestimmten anderen Beteiligten wahrnehmen un erkennen (also z.B. “B”s Reflexions-Anzeigen der jeweiligen Signal-Anzeigen As), sondern die entsprechenden Reflexions-Anzeigen zahlreicher verschiedener Beteiligter (d.h. auch von “J”, “K”, …).

    Zur Beschreibung einer bestimmten Lichtuhr gehört deshalb sicherlich die Identifikation beider Enden.

  431. #431 PS
    5. September 2018

    @Frank W.
    Ich stimme in allem zu, würde eher nicht von anderen Beteiligten sprechen, sondern den Schwerpunkt auf die Identifikation des anderen Endes (die Festlegung der Länge der Lichtuhr) legen. Das ist nun Vereinbarungssache, wir nehmen die Sekunde.

  432. #432 Karl-Heinz
    5. September 2018

    @PS, @Frank Wappler

    Die Lichtuhr dient eigentlich ja nur dazu um die Zeitdilatation einigermaßen verständlich zu machen. Ansonsten verwendet man für die Betrachtung eine punktförmige ideale Uhr.
    Bei eurer Lichtuhr müsst ihr aufpassen, dass sie nicht umfällt (um 90° kippt). 😉

  433. #433 PS
    5. September 2018

    @Karl Heinz # 432
    Was ist eine punktförmige Uhr konkret? Oder meinst Du bestimmte Ortskoordinaten eines Bezugssystems, denen Du eine bestimmte (mit der Lichtuhr gemessen) Zeit zuordnest.
    Die Lichtuhr kannst Du – bei beliebigen Relativbewegungen – um die eigene Achse drehen. In Bezug auf ein bestimmtes Ereignis zeigt sie sowohl in S als auch in jedem S’ auf Grund der überall geltenden Halbzeitreflexion die (nach der L-T) richtige Zeit für ein ausgewähltes Ereignis an.

  434. #434 Frank Wappler
    6. September 2018

    PS schrieb (#431, 5. September 2018):
    > […] die Identifikation des anderen Endes

    … also doch beider Enden (nicht wahr?); so dass von einer geometrischen Beziehung jeweils dieser betreffenden beiden Enden gegenüber einander die Rede sein kann. Wobei jedes der beiden Enden wiederum etliche einzeln identifizierbare Bestandteile haben mag: bisweilen ist im Zusammenhang mit Lichtuhren ja z.B. von zwei ganzen Spiegeln die Rede …

    > (die Festlegung der Länge der Lichtuhr)

    Sofern den beiden betreffenden Enden eine bestimmte “Länge” (“Abstand voneinander”, oder sogar “Distanz voneinander”) zugeschrieben werden soll, ist damit wohl die Forderung und Messung verbunden, dass diese beiden

    – zumindest gegenüber einander (durchwegs) starr waren und blieben (selbst dafür reichen die in Kommentar #424 gesetzten bestimmten Bedingungen an “zeitliche[n] Abstand der Ereignisse“, also an bestimmte Werte bzw. zumindest Verhältnis-Werte von Raumzeit-Intervallen, s^2, nicht aus),

    – oder sogar gegenüber einander (durchwegs) ruhten (was noch erheblich schwieriger zu definieren bzw. zu messen wäre).

    > Das ist nun Vereinbarungssache

    Sehr richtig!
    Mich interessieren dabei die ganz konkret zugrundeliegenden Vereinbarungen;
    und hinsichtlich der Vereinbarung (der Definitionen und entsprechenden “Konstatierung” von Messwerten) von geometrischen bzw. “zeiträumlichen” Messgrößen insbesondere solche, die, wie von Einstein im Rahmen der RT gefordert,

    “stets auf die Bestimmung zeiträumlicher Koinzidenzen hinauslaufen”

    .

    > wir nehmen die Sekunde.

    Wie liefe denn “die [Definition bzw. Messung der] Sekunde” konkret auf Koinzidenz-Bestimmungen hinaus ?

    Und konkret betreffend die (gegenwärtige) SI-Definition:
    Anhand welcher Koinzidenz-Bestimmungen ließe sich denn zumindest im Prinzip Versuch für Versuch feststellen und quantifizieren, in wie fern ein gegebenes Cs133-Atom

    […]perturbed by black body radiation

    bzw. (noch allgemeiner)

    [… subject to] any perturbation

    gewesen wäre ?? …

    Daher mein obiger (#418) Hinweis auf (flache) Ping-Koinzidenz-Gitter im Zusammenhang mit Lichtuhren (und damit verbundenen Messungen von Dauern bzw. von chronometrischen Distanzen).

  435. #435 PS
    6. September 2018

    @Frank W.
    Die praktischen Schwierigkeiten beim Aufbau einer Lichtuhr sind mir bewusst. Grundsätzlich dürften sie zu bewältigen sein, wie LIGO zeigt. Das berechtigt, in Gedankenexperimenten zur SRT die Lichtuhr vorauszusetzen.
    Ob das Prinzip einer solchen Uhr unter Gravitationsbedingungen aufrecht erhalten werden kann, kann ich nicht sagen.
    Bei der Lichtuhr geht es an sich nur um einen reflektierten Lichtstrahl. Koinzidenzen kommen erst ins Spiel, wenn zwei zueinander bewegte Beobachter ihre Lichtuhren koordinieren. Da stellen sie fest, dass sich die Laufzeiten ihrer koordinierten Lichtstrahlen nach der L-T richten. Die Koinzidenz bildet die gemeinsame Reflexion dieser beiden Lichtstrahlen.

  436. #436 Philip
    Wuppertal
    7. September 2018

    Mal was ganz anderes: Was zum Henker ist mit ‘www.relativ-kritisch.net’ los?

    Im Moment hatte sich dort wieder eine rege Diskussion entwickelt, und auf einmal sehe ich gestern die gesamte Domain zum Verkauf angeboten.

  437. #437 Karl-Heinz
    7. September 2018

    @Philip

    Ich tippe auf technische Probleme mit dem Domain Name Registrar. 😉

  438. #438 Albrecht Storz
    19. September 2018

    Sehr schönes Verständnis von Kritikfähigkeit: der Moderator hat hier eine ganze Reihe von Posts von mir wegzensiert:

    Herzlichen Glückwunsch, willkommen im “Safe Space”. Fühlen SIe sich wohl ohne jede Irritationen. Der Moderator sorgt für Ihren und seinen guten Schlaf. Danke für Nichts.

  439. #439 Captain E.
    19. September 2018

    @Albrecht Storz:

    Dann war es entweder etwas wirklich unerträgliches oder sogar strafbares, oder der Spamfilter hat gnadenlos zugeschlagen. Bekanntlich führt der bei ScienceBlogs ein skurriles Eigenleben.

    Merke: Der Admin ist nur in den seltensten Fällen Schuld.

  440. #440 Alderamin
    19. September 2018

    @Albrecht Storz

    Ich war den ganzen Morgen unterwegs, habe im Moment gerade andere Sorgen, als irgendwen zu moderieren.

    Siehe FAQ.

    Gesperrt bist Du nicht, sonst wärest Du nicht hier. Wenn ich etwas lösche oder jemand sperre, dann tue ich das kund, z.B. in dem gelöschten Post (@Captain E.: auch wenn’s unerträglich oder strafbar wäre).

    Alles andere läuft vollautomatisch ab, da habe ich keinerlei Einfluss drauf.

    Eine Benachrichtigung über eine zu moderierende Nachricht habe ich nicht bekommen – bei Dir hätte dann “Ihr Beitrag wartet auf Moderation” gestanden.

    Der Spamordner ist auch leer.

    Also war das anscheinend ein Fall von Akismet-Filter Level 3 (siehe Link). Der Inhalt geht ins Nirwana, ich kann ihn auch nicht sehen oder zurückholen. Tut mir leid.

  441. #441 Philip
    Wuppertal
    27. März 2019

    Angesichts dessen, was ich unlängst in einer “unzensierten Diskussion” erleben durfte, finde ich Albrechts “Zensur”vorwürfe einfach nur “Mimimi”.

  442. #442 Spritkopf
    28. März 2019

    @Philip

    Angesichts dessen, was ich unlängst in einer “unzensierten Diskussion” erleben durfte, finde ich Albrechts “Zensur”vorwürfe einfach nur “Mimimi”.

    Genau bei dieser von dir erwähnten “unzensierten” Diskussion lese ich gerade mit (beteilige mich aber nicht, da ich keine Lust darauf habe, dass die für ihre Klagewütigkeit bekannte Bloganbieterin irgendwelche Daten wie Mail- oder IP-Adresse von mir erhält).

    Du hast Recht. Was in jener Diskussion geboten wird, ist ein mehr als armseliges Schauspiel. Werden die Kommentare der Beteiligten andernorts gelöscht (oder erscheinen auch nur verspätet wegen den Software-Hickups bei WordPress), dann wird laut aufgejault. Aber selber wird flächendeckend zur Löschtaste gegriffen. The pot calling the kettle black…

  443. #443 Alderamin
    28. März 2019

    Wer sich hier benimmt, wird nicht gesperrt. Egal, was für eine Meinung er vertritt (na gut, menschenverachtenden Mist oder Ungesetzliches würde natürlich gelöscht und zur Sperrung führen; Trollerei und Derailing will ich auch keines).

    In Bezug auf #438 hatte ich überhaupt nichts gelöscht, aber wir haben hier drei Ebenen von Spam-Filtern und bei Ebene 3 habe ich keine Chance, die Posts wieder herzustellen, ich werde auch nicht informiert. Ebene 2 geht in den Müll (ohne Meldung an mich), die kann ich noch retten. Und Ebene 1 geht in die Moderation, darüber werde ich informiert und die kann ich vom Handy aus freischalten.

    Albrecht wurde dann später trotzdem *eine Zeitlang* suspendiert, weil er sich lieber mit einem anderen Kommentator streiten wollte, als über das Sachthema zu reden.

    Im Großen und Ganzen ist es aber ziemlich friedlich hier. Dank an die Kommentatoren!